You are on page 1of 294

ANECO REALTY AND DEVELOPMENT CORPORATION, Petitioner, vs. LANDEX DEVELOPMENT CORPORATION, Respondent. DECISION REYES, R.T., J.

: THIS is a simple case of a neighbor seeking to restrain the landowner from fencing his own property. The right to fence flows from the right of ownership. Absent a clear legal and enforceable right, We will not unduly restrain the landowner from exercising an inherent proprietary right. Before Us is a petition for review on certiorari of the Decision1 of the Court of Appeals (CA) affirming the Order2 of the Regional Trial Court (RTC) dismissing the complaint for injunction filed by petitioner Aneco Realty and Development Corporation (Aneco) against respondent Landex Development Corporation (Landex). Facts Fernandez Hermanos Development, Inc. (FHDI) is the original owner of a tract of land in San Francisco Del Monte, Quezon City. FHDI subdivided the land into thirty-nine (39) lots.3 It later sold twenty-two (22) lots to petitioner Aneco and the remaining seventeen (17) lots to respondent Landex.4 The dispute arose when Landex started the construction of a concrete wall on one of its lots. To restrain construction of the wall, Aneco filed a complaint for injunction5 with the RTC in Quezon City. Aneco later filed two (2) supplemental complaints seeking to demolish the newly-built wall and to hold Landex liable for two million pesos in damages.6 Landex filed its Answer7 alleging, among others, that Aneco was not deprived access to its lots due to the construction of the concrete wall. Landex claimed that Aneco has its own entrance to its property along Miller Street, Resthaven Street, and San Francisco del Monte Street. The Resthaven access, however, was rendered inaccessible when Aneco constructed a building on said street. Landex also claimed that FHDI sold

ordinary lots, not subdivision lots, to Aneco based on the express stipulation in the deed of sale that FHDI was not interested in pursuing its own subdivision project. RTC Disposition On June 19, 1996, the RTC rendered a Decision8 granting the complaint for injunction, disposing as follows: Wherefore, premises considered, and in the light aforecited decision of the Supreme Court judgment is hereby rendered in favor of the plaintiff and the defendant is hereby ordered: 1. To stop the completion of the concrete wall and excavation of the road lot in question and if the same is already completed, to remove the same and to return the lot to its original situation; 2. To pay actual and compensatory damage to the plaintiff in the total amount of P50,000.00; 3. To pay attorney s fees in the amount of P20,000.00; 4. To pay the cost. SO ORDERED.9 Landex moved for reconsideration.10 Records reveal that Landex failed to include a notice of hearing in its motion for reconsideration as required under Section 5, Rule 15 of the 1997 Rules of Civil Procedure. Realizing the defect, Landex later filed a motion11 setting a hearing for its motion for reconsideration. Aneco countered with a motion for execution12 claiming that the RTC decision is already final and executory. Acting on the motion of Landex, the RTC set a hearing on the motion for reconsideration on August 28, 1996. Aneco failed to attend the slated hearing. The RTC gave Aneco additional time to file a comment on the motion for reconsideration.13 On March 13, 1997, the RTC issued an order14 denying the motion for execution of Aneco.

On March 31, 1997, the RTC issued an order granting the motion for reconsideration of Landex and dismissing the complaint of Aneco. In granting reconsideration, the RTC stated: In previously ruling for the plaintiff, this Court anchored its decision on the ruling of the Supreme Court in the case of "White Plains Association vs. Legaspi, 193 SCRA 765," wherein the issue involved was the ownership of a road lot, in an existing, fully developed and authorized subdivision, which after a second look, is apparently inapplicable to the instant case at bar, simply because the property in question never did exist as a subdivision. Since, the property in question never did exist as a subdivision, the limitations imposed by Section 1 of Republic Act No. 440, that no portion of a subdivision road lot shall be closed without the approval of the Court is clearly in appropriate to the case at bar. The records show that the plaintiff s property has access to a public road as it has its own ingress and egress along Miller St.; That plaintiff s property is not isolated as it is bounded by Miller St. and Resthaven St. in San Francisco del Monte, Quezon City; that plaintiff could easily make an access to a public road within the bounds and limits of its own property; and that the defendant has not yet been indemnified whatsoever for the use of his property, as mandated by the Bill of rights. The foregoing circumstances, negates the alleged plaintiffs right of way.15 Aneco appealed to the CA.16 CA Disposition On March 31, 2003, the CA rendered a Decision17 affirming the RTC order, disposing as follows: WHEREFORE, in consideration of the foregoing, the instant appeal is perforce dismissed. Accordingly, the order dated 31 March 1996 is hereby affirmed. SO ORDERED.18 In affirming the RTC dismissal of the complaint for injunction, the CA held that Aneco knew at the time of the sale that the lots sold by FHDI were not

subdivision units based on the express stipulation in the deed of sale that FHDI, the seller, was no longer interested in pursuing its subdivision project, thus: The subject property ceased to be a road lot when its former owner (Fernandez Hermanos, Inc.) sold it to appellant Aneco not as subdivision lots and without the intention of pursuing the subdivision project. The law in point is Article 624 of the New Civil Code, which provides: Art. 624. The existence of an apparent sign of easement between two estates, established or maintained by the owner of both, shall be considered, should either of them be alienated, as a title in order that the easement may continue actively and passively, unless, at the time the ownership of the two estates is divided, the contrary should be provided in the title of conveyance of either of them, or the sign aforesaid should be removed before the execution of the deed. This provision shall also apply in case of the division of a thing owned in common by two or more persons. Viewed from the aforesaid law, there is no question that the law allows the continued use of an apparent easement should the owner alienate the property to different persons. It is noteworthy to emphasize that the lot in question was provided by the previous owner (Fernandez Hermanos, Inc.) as a road lot because of its intention to convert it into a subdivision project. The previous owner even applied for a development permit over the subject property. However, when the twenty-two (22) lots were sold to appellant Aneco, it was very clear from the seller s deed of sale that the lots sold ceased to be subdivision lots. The seller even warranted that it shall undertake to extend all the necessary assistance for the consolidation of the subdivided lots, including the execution of the requisite manifestation before the appropriate government agencies that the seller is no longer interested in pursuing the subdivision project. In fine, appellant Aneco knew from the very start that at the time of the sale, the 22 lots sold to it were not intended as subdivision units, although the titles to the different lots have yet to be consolidated. Consequently, the easement that used to exist on the subject lot ceased when appellant Aneco and the former owner agreed that the lots would be consolidated and would no longer be intended as a subdivision project.

Appellant Aneco insists that it has the intention of continuing the subdivision project earlier commenced by the former owner. It also holds on to the previous development permit granted to Fernandez Hermanos, Inc. The insistence is futile. Appellant Aneco did not acquire any right from the said previous owner since the latter itself expressly stated in their agreement that it has no more intention of continuing the subdivision project. If appellant desires to convert its property into a subdivision project, it has to apply in its own name, and must have its own provisions for a road lot.19 Anent the issue of compulsory easement of right of way, the CA held that Aneco failed to prove the essential requisites to avail of such right, thus: An easement involves an abnormal restriction on the property of the servient owner and is regarded as a charge or encumbrance on the servient owner and is regarded as a charge or encumbrance on the servient estate (Cristobal v. CA, 291 SCRA 122). The essential requisites to be entitled to a compulsory easement of way are: 1) that the dominant estate is surrounded by other immovables and has no adequate outlet to a public highway; 2) that proper indemnity has been paid; 3) that the isolation was not due to acts of the proprietor of the dominant estate; 4) that the right of way claimed is at a point least prejudicial to the servient estate and in so far as consistent with this rule, where the distance from the dominant estate to a public highway may be the shortest (Cristobal v. Court of Appeals, 291 SCRA 122). An in depth examination of the evidence adduced and offered by appellant Aneco, showed that it had failed to prove the existence of the aforementioned requisites, as the burden thereof lies upon the appellant Aneco.20 Aneco moved for reconsideration but its motion was denied.21 Hence, the present petition or appeal by certiorari under Rule 45. Issues Petitioner Aneco assigns quadruple errors to the CA in the following tenor: A.

THE COURT OF APPEALS GRAVELY ERRED IN DISMISSING PETITIONER S APPEAL AND SUSTAINING THE TRIAL COURT S ORDER DATED 31 MARCH 1997 GRANTING RESPONDENT S MOTION FOR RECONSIDERATION WHICH IS FATALLY DEFECTIVE FOR LACK OF NOTICE OF HEARING. B. THE COURT OF APPEALS GRAVELY ERRED IN AFFIRMING THE TRIAL COURT S ORDER WHICH GAVE FULL WEIGHT AND CREDIT TO THE MISLEADING AND ERRONEOUS CERTIFICATION ISSUED BY GILDA E. ESTILO WHICH SHE LATER EXPRESSLY AND CATEGORICALLY RECANTED BY WAY OF HER AFFIDAVIT. C. THE COURT OF APPEALS GRAVELY ERRED IN APPLYING THE LIBERAL CONSTRUCTION OF THE RULES IN ORDER TO SUSTAIN THE TRIAL COURT S ORDER DATED 31 MARCH 1997. D. THE COURT OF APPEALS GRAVELY ERRED IN AFFIRMING THE TRIAL COURT S ORDER THAT MADE NO PRONOUNCEMENTS AS TO COSTS, AND IN DISREGARDING THE MERIT OF THE PETITIONER S CAUSE OF ACTION.22 Our Ruling The petition is without merit. Essentially, two (2) issues are raised in this petition. The first is the procedural issue of whether or not the RTC and the CA erred in liberally applying the rule on notice of hearing under Section 5, Rule 15 of the 1997 Rules of Civil Procedure. The second is the substantive issue of whether or not Aneco may enjoin Landex from constructing a concrete wall on its own property. We shall discuss the twin issues sequentially.

Strict vs. Liberal Construction of Procedural Rules; Defective motion was cured when Aneco was given an opportunity to comment on the motion for reconsideration. Section 5, Rule 15 of the 1997 Rules of Civil Procedure23 requires a notice of hearing for a contested motion filed in court. Records disclose that the motion for reconsideration filed by Landex of the RTC decision did not contain a notice of hearing. There is no dispute that the motion for reconsideration is defective. The RTC and the CA ignored the procedural defect and ruled on the substantive issues raised by Landex in its motion for reconsideration. The issue before Us is whether or not the RTC and the CA correctly exercised its discretion in ignoring the procedural defect. Simply put, the issue is whether or not the requirement of notice of hearing should be strictly or liberally applied under the circumstances. Aneco bats for strict construction. It cites a litany of cases which held that notice of hearing is mandatory. A motion without the required notice of hearing is a mere scrap of paper. It does not toll the running of the period to file an appeal or a motion for reconsideration. It is argued that the original RTC decision is already final and executory because of the defective motion.24 Landex counters for liberal construction. It similarly cites a catena of cases which held that procedural rules may be relaxed in the interest of substantial justice. Landex asserts that the procedural defect was cured when it filed a motion setting a hearing for its motion for reconsideration. It is claimed that Aneco was properly informed of the pending motion for reconsideration and it was not deprived of an opportunity to be heard.25 It is true that appeals are mere statutory privileges which should be exercised only in the manner required by law. Procedural rules serve a vital function in our judicial system. They promote the orderly resolution of cases. Without procedure, there will be chaos. It thus behooves upon a litigant to follow basic procedural rules. Dire consequences may flow from procedural lapses. Nonetheless, it is also true that procedural rules are mere tools designed to facilitate the attainment of justice. Their strict and rigid application should be relaxed when they hinder rather than promote substantial justice. Public

policy dictates that court cases should, as much as possible, be resolved on the merits not on mere technicalities. Substantive justice trumps procedural rules. In Barnes v. Padilla,26 this Court held: Let it be emphasized that the rules of procedure should be viewed as mere tools designed to facilitate the attainment of justice. Their strict and rigid application, which would result in technicalities that tend to frustrate rather than promote substantial justice, must always be eschewed. Even the Rules of Court reflect this principle. The power to suspend or even disregard rules can be so pervasive and compelling as to alter even that which this Court itself has already declared to be final x x x.lawph!l The emerging trend in the rulings of this Court is to afford every party litigant the amplest opportunity for the proper and just determination of his cause, free from the constraints of technicalities. Time and again, this Court has consistently held that rules must not be applied rigidly so as not to override substantial justice.27 Here, We find that the RTC and the CA soundly exercised their discretion in opting for a liberal rather than a strict application of the rules on notice of hearing. It must be stressed that there are no vested right to technicalities. It is within the court s sound discretion to relax procedural rules in order to fully adjudicate the merits of a case. This Court will not interfere with the exercise of that discretion absent grave abuse or palpable error. Section 6, Rule 1 of the 1997 Rules of Civil Procedure even mandates a liberal construction of the rules to promote their objectives of securing a just, speedy, and inexpensive disposition of every action and proceeding. To be sure, the requirement of a notice of hearing in every contested motion is part of due process of law. The notice alerts the opposing party of a pending motion in court and gives him an opportunity to oppose it. What the rule forbids is not the mere absence of a notice of hearing in a contested motion but the unfair surprise caused by the lack of notice. It is the dire consequences which flow from the procedural error which is proscribed. If the opposing party is given a sufficient opportunity to oppose a defective motion, the procedural lapse is deemed cured and the intent of the rule is substantially complied. In E & L Mercantile, Inc. v. Intermediate Appellate Court,28 this Court held:

Procedural due process is not based solely on a mechanistic and literal application of a rule such that any deviation is inexorably fatal. Rules of procedure, and this includes the three (3) days notice requirement, are liberally construed in order to promote their object and to assist the parties in obtaining just, speedy, and inexpensive determination of every action and proceeding (Section 2, Rule 1, Rules of Court). In Case and Nantz v. Jugo (77 Phil. 517), this Court made it clear that lapses in the literal observance of a rule of procedure may be overlooked when they have not prejudiced the adverse party and have not deprived the court of its authority. A party cannot ignore a more than sufficient opportunity to exercise its right to be heard and once the court performs its duty and the outcome happens to be against that negligent party, suddenly interpose a procedural violation already cured, insisting that everybody should again go back to square one. Dilatory tactics cannot be the guiding principle. The rule in De Borja v. Tan (93 Phil. 167), that "what the law prohibits is not the absence of previous notice, but the absolute absence thereof and lack of opportunity to be heard," is the applicable doctrine. (See also Aguilar v. Tan, 31 SCRA 205; Omico v. Vallejos, 63 SCRA 285; Sumadchat v. Court of Appeals, 111 SCRA 488.) x x x29 We also find that the procedural lapse committed by Landex was sufficiently cured when it filed another motion setting a hearing for its defective motion for reconsideration. Records reveal that the RTC set a hearing for the motion for reconsideration but Aneco s counsel failed to appear. The RTC then gave Aneco additional time to file comment on the motion for reconsideration.30 Aneco was afforded procedural due process when it was given an opportunity to oppose the motion for reconsideration. It cannot argue unfair surprise because it was afforded ample time to file a comment, as it did comment, on the motion for reconsideration. There being no substantial injury or unfair prejudice, the RTC and the CA correctly ignored the procedural defect.

The RTC and the CA did not err in dismissing the complaint for injunction; factual findings and conclusions of law of the RTC and the CA are afforded great weight and respect. Anent the substantive issue, We agree with the RTC and the CA that the complaint for injunction against Landex should be dismissed for lack of merit. What is involved here is an undue interference on the property rights of a landowner to build a concrete wall on his own property. It is a simple case of a neighbor, petitioner Aneco, seeking to restrain a landowner, respondent Landex, from fencing his own land. Article 430 of the Civil Code gives every owner the right to enclose or fence his land or tenement by means of walls, ditches, hedges or any other means. The right to fence flows from the right of ownership. As owner of the land, Landex may fence his property subject only to the limitations and restrictions provided by law. Absent a clear legal and enforceable right, as here, We will not interfere with the exercise of an essential attribute of ownership. Well-settled is the rule that factual findings and conclusions of law of the trial court when affirmed by the CA are accorded great weight and respect. Here, We find no cogent reason to deviate from the factual findings and conclusion of law of the trial court and the appellate court. We have meticulously reviewed the records and agree that Aneco failed to prove any clear legal right to prevent, much less restrain, Landex from fencing its own property. Aneco cannot rely on the road lot under the old subdivision project of FHDI because it knew at the time of the sale that it was buying ordinary lots, not subdivision lots, from FHDI. This is clear from the deed of sale between FHDI and Aneco where FHDI manifested that it was no longer interested in pursuing its own subdivision project. If Aneco wants to transform its own lots into a subdivision project, it must make its own provision for road lots. It certainly cannot piggy back on the road lot of the defunct subdivision project of FHDI to the detriment of the new owner Landex. The RTC and the CA correctly dismissed the complaint for injunction of Aneco for lack of merit. WHEREFORE, the petition is DENIED and the appealed Decision AFFIRMED.

SO ORDERED.

PALALI VS AWISAN Property; ownership of land. We hold that as between the petitioner and the respondent, it is the petitioner who has the better claim or title to the subject property. While the respondent merely relied on her tax declaration, petitioner was able to prove actual possession of the subject property coupled with his tax declaration. We have ruled in several cases that possession, when coupled with a tax declaration, is a weighty evidence of ownership. It certainly is more weighty and preponderant than a tax declaration alone. The preponderance of evidence is therefore clearly in favor of petitioner, particularly considering that, as the actual possessor under claim of ownership, he enjoys the presumption of ownership. Moreover, settled is the principle that a party seeking to recover real property must rely on the strength of her case rather than on the weakness of the defense. The burden of proof rests on the party who asserts the affirmative of an issue. For he who relies upon the existence of a fact should be called upon to prove that fact. Having failed to discharge her burden to prove her affirmative allegations, we find that the trial court rightfully dismissed respondent s complaint. Modesto Palali vs. Juliet Awisan, represented by her Attorney-in-fact Gregorio Awisan, G.R. No. 158385, February 12, 2010 NELSON LAGAZO, Petitioner, vs. GERALD B. SORIANO and GALILEO B. SORIANO, Respondents. DECISION PERALTA, J.: This resolves the Petition for Review on Certiorari under Rule 45 of the Rules of Court, praying that the Decision1cralaw of the Court of Appeals (CA) in CA-G.R. SP No. 80709, promulgated on October 28, 2005, granting herein respondents' petition for review, and the CA Resolution2cralaw promulgated on December 20, 2005, denying herein petitioner's motion for reconsideration, be reversed and set aside.

The undisputed facts are as follows. On January 16, 2001, respondents filed with the Municipal Trial Court of Tabuk, Kalinga (MTC), a complaint for Forcible Entry with Application for Termporary Restraining Order and a Writ of Preliminary Injunction and Damages against petitioner. Respondents claimed they were the owners of a parcel of land covered by Original Certificate of Title No. P-665, Lot No. 816, Pls-93 with an area of 58,171 square meters. They allegedly acquired the same by purchase from their grandfather, Arsenio Baac, on September 10, 1998, but even prior thereto, they were already allowed by Arsenio Baac to cultivate said land. They paid real property taxes for said property from 1990 to 1998 and had been in actual possession from that time. However, on January 6, 2001, herein petitioner allegedly unlawfully entered the property by means of force, stealth, and strategy and began cultivating the land for himself. On the other hand, petitioner insisted in his Answer that he, together with his mother, brothers, and sisters, were the lawful owners of the land in question, being the legal heirs of Alfredo Lagazo, the registered owner thereof. They denied that the subject land was sold to Arsenio Baac, alleging instead that the agreement between Alfredo Lagazo and Arsenio Baac was merely one of mortgage. Petitioner, likewise maintained that he and his co-heirs had always been in possession of the disputed land. They allegedly tried several times to redeem the property, but Baac increased the redemption price from P10,000.00 to P100,000.00. This prompted them to bring the matter before the Barangay Lupon of Balong, Tabuk, Kalinga, but no agreement was reached. On November 23, 2001, the MTC rendered a Decision, the dispositive portion of which reads as follows: WHEREFORE, judgment is hereby rendered as follows: 1. Dismissing the complaint of Forcible Entry filed against defendant Nelson Lagazo; 2. Ordering the plaintiffs, Gerald B. Soriano and Galileo B. Soriano to surrender Original Certificate of Title No. P-665 in the name of Alfredo Lagazo to the heirs of Lagazo which was given to Arsenio

Baac by Alfredo Lagazo when the Deed of Mortgage was executed between them; 3. Ordering the heirs of Alfredo Lagazo to execute the deed of conveyance in favor of the plaintiffs covering the one (1) hectare portion subject of the mortgage between Alfredo Lagazo and Arsenio Baac and to segregate the same from property covered by OCT P665; 4. Plaintiffs to pay the costs of suit. SO ORDERED.3cralaw The foregoing Decision was appealed to the Regional Trial Court (RTC) of Tabuk, Kalinga. Said appellate court ruled that herein respondents failed to prove prior physical possession, thus, it reversed the MTC Decision and dismissed the complaint against herein petitioner. Respondents then filed with the CA a Petition for Review under Rule 42 of the Rules of Court and on October 28, 2005, the CA promulgated the assailed Decision which disposed thus: WHEREFORE, premises considered, the petition is GRANTED. Physical possession is hereby ordered returned to the petitioners, without prejudice to the respondent's right to take recourse to remedies provided for under the law, if he is so inclined. Actual, moral and exemplary damages cannot be granted because of lack of substantive evidence to prove the same. However, we grant the amount of P10,000.00 in attorney's fees plus P500.00 per appearance of petitioners' counsel, as well as another P10,000.00 in litigation expenses as prayed for in their complaint, conformably to par. 11 of Art. 2208 of the Civil Code, i.e. it is just and equitable under the circumstances, and considering that the award is well deserved by the petitioners who had shown evident good faith in, and respect for, the judicial system. SO ORDERED.4cralaw

Petitioner moved for reconsideration, but the same was denied per CA Resolution dated December 20, 2005. Hence, this petition where the following issues are raised: WHETHER THE TRIAL COURT GRAVELY ERRED IN FINDING THAT THERE WAS IMPLIED ADMISSION ON THE PART OF THE PETITIONER THAT RESPONDENTS HAD BEEN IN ACTUAL PHYSICAL POSSESSION OF THE LOT IN CONTROVERSY SINCE 1979. WHETHER THE TRIAL COURT GRAVELY ERRED IN NOT GIVING CREDENCE TO THE EVIDENCE ADDUCED BY PETITIONER SUBSTANTIATING HIS PRIORITY IN POSSESSION OVER THE LOT IN CONTROVERSY. WHETHER THE TRIAL COURT GRAVELY ERRED IN FINDING THAT THE RESPONDENTS HAVE BETTER RIGHT OF POSSESSION OVER THE LOT IN CONTROVERSY.5cralaw The Court finds the petition unmeritorious. Prior physical possession is an indispensable element in forcible entry cases.6cralaw Thus, the ultimate question here is who had prior physical possession of the disputed land. Ordinarily, in a Petition for Review on Certiorari , this Court only considers questions of law, as it is not a trier of facts. However, there are exceptions to this general rule, such as, when the findings of fact of the appellate court are contrary to those of the trial court.7cralaw Such circumstance exists in this case, hence, the Court is compelled to take a closer look at the records. In Sudaria v. Quiambao ,8cralaw the Court held that: Ejectment proceedings are summary proceedings intended to provide an expeditious means of protecting actual possession or right to possession of property. Title is not involved.The sole issue to be resolved is who is entitled to the physical or material possession of the premises or possession de facto. On this point, the pronouncements in Pajuyo v. Court of Appeals are enlightening, thus:

xxxx x x x Regardless of the actual condition of the title to the property, the party in peaceable quiet possession shall not be thrown out by a strong hand, violence or terror. Neither is the unlawful withholding of property allowed. Courts will always uphold respect for prior possession. Thus, a party who can prove prior possession can recover such possession even against the owner himself.Whatever may be the character of his possession, if he has in his favor prior possession in time, he has the security that entitles him to remain on the property until a person with a better right lawfully ejects him. To repeat, the only issue that the court has to settle in an ejectment suit is the right to physical possession.9cralaw (Emphasis supplied.) Moreover, in De Grano v. Lacaba ,10cralaw it was explained that: x x x the word "possession," as used in forcible entry and unlawful detainer cases, means nothing more than physical possession, not legal possession in the sense contemplated in civil law. When the law speaks of possession, the reference is to prior physical possession or possession de facto, as contra-distinguished from possession de jure. Only prior physical possession, not title, is the issue. Issues as to the right of possession or ownership are not involved in the action; evidence thereon is not admissible, except only for the purpose of determining the issue of possession.11cralaw (Emphasis supplied.) Bearing the foregoing in mind, a thorough examination of the evidence revealed that, indeed, the parties in last peaceable quiet possession of the property in question were herein respondents. The most important evidence for respondents was the testimony of Brgy. Capt. Artemio Fontanilla, who stated that he was born and had continuously resided in Balong, Tabuk, Kalinga; that the disputed land was only about three kilometers from his house; that for the longest time, he had always known that it was Arsenio Baac who was cultivating and occupying said property; and that it was only sometime in January 2001, when the police asked him to accompany them to the subject land, that he saw petitioner with some other men working said land. 12cralaw

On the other hand, what petitioner's evidence sought to establish was that he and his co-heirs continued to be the owners of the land, as his predecessor never intended to sell the property to Arsenio Baac, the true agreement being only one of a mortgage. Petitioner never established the fact of his physical possession over the disputed land. Ironically, the most telling pieces of evidence that doomed petitioner's case were the testimonies of petitioner himself and his sister, Marina Nialga. Their own admissions on the witness stand proved that respondents were indeed the ones in physical possession of the subject property. Petitioner Lagazo himself testified as follows: Q: So, at that time that you were at Alicia, Isabela and at that time that you staying thereat, you have no knowledge to what is happening to the land which is now the subject of this case, Am I correct? A: I was only hearing stories from my father and my mother that they want to regain back the land which was mortgaged, sir. xxxx Q: It is when only on January of 2001 that you allegedly claimed over the parcel of land in question, am I correct Mr. Witness? A: Was not only during that time but that was only the time we entered into the land, sir. Q: So, you are now admitting Mr. Witness, its only on January 6, 2001, you entered the land in question? A: Yes, sir. Q: And, prior to January 6 of 2001, you never possessed or cultivated the land in question, Am I correct? xxxx Q: Who was an apparent heir of spouses Alfredo Lagaso, you never personally cultivated or possessed the land in question prior to January 6, 2001, am I correct?

A: No, sir because according to them it was mortgaged, Your Honor. Q: But you never personally cultivated the land prior to January 6, 2001? A: No, sir.13cralaw Meanwhile, Marina Nialga also recounted that in 1979, they left the subject property out of fear because Arsenio Baac allegedly wanted to grab the land for himself. She testified that after they left in 1979, it was already Arsenio Baac who cultivated said land. Despite such claim that Arsenio Baac took their land with force and intimidation, Marina said they never reported the matter to the police, and never filed any criminal action in court against Arsenio Baac.14cralaw Verily, the foregoing leaves no doubt in our mind that it was only on January 6, 2001 that petitioner, believing himself to be the lawful owner of the disputed land, entered the same, thereby disturbing respondents' peaceful possession thereof. IN VIEW OF THE FOREGOING, the instant petition is dismissed. The Decision and Resolution of the Court of Appeals dated October 28, 2005 and December 20, 2005, respectively, in CA G.R. SP No. 80709 are AFFIRMED. SO ORDERED. HEIRS OF PEDRO LAURORA and LEONORA LAURORA, petitioners, vs. STERLING TECHNOPARK III and S.P. PROPERTIES, INC., respondents. DECISION PANGANIBAN, J.: The owners of a property have no authority to use force and violence to eject alleged usurpers who were in prior physical possession of it. They must file the appropriate action in court and should not take the law into their own hands. The Case

Before us is a Petition for Review[1] under Rule 45 of the Rules of Court, seeking to set aside the June 27, 2000 Decision[2] and the January 22, 2001 Resolution[3] of the Court of Appeals[4] (CA) in CA-GR SP No. 54667. The dispositive part of the Decision reads: WHEREFORE, the [P]etition is GRANTED and the RTC [D]ecision dated 06 May 1999 and the RTC [O]rder dated 03 August 1999 are hereby REVERSED and SET ASIDE, and corollarily, the MCTC [D]ecision is AFFIRMED. [5] The assailed Resolution denied petitoners Motion for Reconsideration. The Facts The factual antecedents are summarized by the CA as follows: In a [C]omplaint for Forcible Entry with Damages filed on 27 September 1997 before the Fifth Municipal Circuit Trial Court of Carmona and Gen. Mariano Alvarez, plaintiffs therein, x x x Pedro Laurora and Leonora Laurora [herein petitioners] alleged that they [were] the owners of Lot 1315-G, SWD-40763 of the Yaptinchay Estate with an area of 39,771 sq. meters and located in Carmona, Cavite. Pedro Laurora planted trees and has possessed the land up to the present. On 15 September 1997, [respondents] Sterling Technopark III and S.P. Properties, Inc. x x x through their Engr. Bernie Gatchalian bulldozed and uprooted the trees and plants, and with the use of armed men and by means of threats and intimidation, succeeded in forcibly ejecting [petitioners]. As a result of their dispossession, [petitioners] suffered actual damages in the amount of P3,000,000.00 and P10,000.00 as attorney s fees. In their [A]nswer to the [C]omplaint, [respondents] averred that [petitioners were] not the owners of the land because they disposed of it sometime in 1976 as shown by legal documents. On 02 April 1969, the Land Authority issued an order of award in favor of [petitioners], approving the application of Pedro Laurora to buy the subject Lot 1315-G from the government. On 01 March 1974, [petitioners] requested the Department of Agrarian Reform for the transfer of the lot to Juan Manaig. Favorably acted upon, the DAR issued a permit to transfer dated 03 June 1975 through its Regional Director Benjamin R. Estrellado. On 03 July 1975,

Juan Manaig, as transferee and buyer, paid the required amount of P10,643.65 under Official Receipt No. 8304707 to the government as full payment for the transfer of said lot to him. On 26 March 1976, the [petitioners] as sellers and witnessed by their sons, Efren Laurora and Dominador Laurora, executed a Kasulatan ng Paglilipatan ng Lupa transferring the land to Juan Manaig as buyer. On 11 June 1976, the [petitioners] again witnessed by their sons, Efren and Dominador, executed a Kasulatan ng Bilihang Tuluyan or Deed of Sale wherein they sold Lot 1315-G including all improvements therein, in favor of Juan Manaig. The Deed of Absolute Sale was approved by the Department of Agrarian Reform on 14 June 1976 in DAR Approval of Transfer of Rights signed by DAR Regional Director, Benjamin R. Estrellado. After the approval of the sale from the [petitioners] to Juan Manaig, the latter paid its real estate taxes. The tax declarations of the land in the name of its previous owners, Yaptinchays, were cancelled and transferred in the name of [petitioner] Pedro Laurora as owner-transferee. Thereupon, the heirs of the late JUAN MANAIG sold the land to Golden Mile Resources Development Corporation which likewise sold it to [respondent] S. P. Properties, Inc. After summary proceedings in the MCTC, x x x, a judgment was rendered dismissing the complaint. The case was elevated to the Regional Trial Court. In due course, the said court rendered a decision reversing the MCTC judgment. x x x [6] Ruling of the Court of Appeals The CA reversed the Regional Trial Court (RTC) and reinstated the Order of dismissal issued by the Municipal Circuit Trial Court (MCTC). It held that there was no evidence to support the claim of petitioners to the prior physical possession of the property. The evidence allegedly showed that they had already sold the land with the approval of the Department of Agrarian Reform (DAR). Accordingly, their subsequent entry into and possession of the land constituted plain usurpation, which could not be the source of any right to occupy it. Being planters in bad faith, they had no right to be reimbursed for improvements on the land, in accordance with Article 449 of the New Civil Code. Hence, this Petition.[7]

The Issue In their Memorandum,[8] petitioners raise this sole issue for our consideration: x x x [W]hether [p]rivate [r]espondent[s] ha[ve] a valid and legal right to forcibly eject petitioners from the premises despite their resistance and objection, through the use of arm[ed] men and by bulldozing, cutting, and destroying trees and plants planted by petitioners, without court order, to the damage and prejudice of the latter. [9] The Court s Ruling The Petition is meritorious. Main Issue: Physical Possession of the Land The only issue in forcible entry cases is the physical or material possession of real property -- possession de facto, not possession de jure.[10] Only prior physical possession, not title, is the issue.[11] If ownership is raised in the pleadings, the court may pass upon such question, but only to determine the question of possession.[12] The ownership claim of respondents upon the land is based on the evidence they presented. Their evidence, however, did not squarely address the issue of prior possession. Even if they succeed in proving that they are the owners of the land,[13] the fact remains that they have not alleged or proved that they physically possess it by virtue of such ownership. On the other hand, petitioners prior possession of the land was not disputed by the CA, which merely described it as usurpation.[14] We stress that the issue of ownership in ejectment cases is to be resolved only when it is intimately intertwined with the issue of possession,[15] to such an extent that the question of who had prior possession cannot be determined without ruling on the question of who the owner of the land is.[16] No such intertwinement has been shown in the case before us. Since respondents claim of ownership is not being made in order to prove

prior possession, the ejectment court cannot intrude or dwell upon the issue of ownership.[17] Notwithstanding the actual condition of the title to the property, a person in possession cannot be ejected by force, violence or terror -- not even by the owners.[18] If such illegal manner of ejectment is employed, as it was in the present case, the party who proves prior possession -- in this case, petitioners -- can recover possession even from the owners themselves. [19] Granting arguendo that petitioners illegally entered into and occupied the property in question, respondents had no right to take the law into their own hands and summarily or forcibly eject the occupants therefrom. Verily, even if petitioners were mere usurpers of the land owned by respondents, still they are entitled to remain on it until they are lawfully ejected therefrom. Under appropriate circumstances, respondents may file, other than an ejectment suit, an accion publiciana -- a plenary action intended to recover the better right to possess;[20] or an accion reivindicatoria -- an action to recover ownership of real property.[21] The availment of the aforementioned remedies is the legal alternative to prevent breaches of peace and criminal disorder resulting from the use of force by claimants out to gain possession.[22] The rule of law does not allow the mighty and the privileged to take the law into their own hands to enforce their alleged rights. They should go to court and seek judicial vindication. WHEREFORE, the Petition is GRANTED and the assailed Decision REVERSED and SET ASIDE. No costs. SO ORDERED. PHILIPPINE NATIONAL BANK, petitioner, vs. THE COURT OF APPEALS and ERNESTO AUSTRIA and LORETO Q. QUINTANA, respondents. DECISION YNARES-SANTIAGO, J.:

Before us is a petition for review under Rule 45 of the Rules of Court, seeking a reversal of the Court of Appeals resolution in CA-G.R. SP No. 48660 dated August 25, 1998, which affirmed the order of the Regional Trial Court of Makati, Branch 60 in LRC Case No. M-2635. Sometime during the late 70 s, the spouses Godofredo and Wilma Monsod obtained a loan in the amount of P120,000.00 from petitioner Philippine National Bank (PNB). To secure their loan, the Monsods mortgaged to PNB a parcel of land covered by TCT No. S-84843, located within the Monte Villa de Monsod Subdivision in Paraaque, Rizal. Due to Monsods failure to pay their loan obligation, PNB extrajudicially foreclosed the mortgage. At the auction sale of the subject real property, PNB was declared the highest bidder. On December 21, 1981, a certificate of sale was issued in favor of PNB, and was registered on July 11, 1984.[1] Upon expiration of the redemption period on July 12, 1985, ownership of the property was consolidated in PNB. Thereafter, TCT No. S-84843 was cancelled and TCT No. 99480 was issued in PNB s name.[2] On June 23, 1992, PNB filed an Ex-Parte Petition for the Issuance of Writ of Possession with Branch 60 of the Regional Trial Court of Makati City, docketed as LRC Case No. M-2635. Pursuant to the provisions of Act No. 3135, as amended, the trial court conducted an ex parte hearing. PNB s representative testified that the foreclosed property is occupied by one Ernesto Austria. According to PNB, Mr. Austria was invited by the bank to a conference to discuss the ownership of the foreclosed lot, however, he did not honor the bank s invitation.[3] On August 28, 1992, the trial court granted PNB s petition and a writ of possession was issued on October 26, 1992.[4] On December 11, 1992, respondents Ernesto and Loreto Quintana Austria filed a Motion for Intervention and to Recall and/or Stop the Enforcement of the Writ of Possession. The Austrias alleged that they are the actual occupants of the subject lot, which they purportedly bought from the Monsods as early as 1974. They claimed that the foreclosed property was enclosed within a concrete fence and formed part of their family

compound. PNB allegedly knew of this fact even before it granted the loan to the Monsods, because the bank s credit investigators were advised of the same when they inspected the property in the summer of 1976. Consequently, the Austrias maintained that the issuance of the possessory writ ex parte was improper, since it will deprive them of their property without due process.[5] Due to the Austrias refusal to vacate the premises, the sheriff failed to enforce the challenged writ. On July 27, 1993, on motion of PNB, the trial court issued an alias writ of possession. Again, the writ was not implemented.[6] On September 17, 1993, the sheriff sought to enforce the first alias writ of possession for the second time. The Austrias filed a Second Motion for Intervention seeking to restrain the enforcement of the writ of possession issued on October 26, 1992.[7] PNB then filed an Urgent Ex-Parte Motion for Issuance of Break Open Order [8] and, subsequently, an Opposition to the Austrias Second Motion for Intervention.[9] On January 31, 1994, the trial court denied the Austrias second motion and granted PNB s Motion for Issuance of Break Open Order. The trial court ruled that the Austrias can no longer be permitted to intervene in the case during said stage of the proceedings and that the remedy of the Austrias was to file an ordinary civil action to assert their claim of ownership over the property.[10] In the meantime, the first alias writ of possession lapsed. PNB thus filed an Ex-Parte Motion for Issuance of Second Alias Writ of Possession, [11] and on November 29, 1994, a second alias writ was issued.[12] Unfazed, the Austrias filed an Omnibus Motion on January 25, 1995, seeking a recall of the second alias writ and a reconsideration of the trial court s order denying their motion to intervene.[13] Meanwhile, the second alias writ had likewise expired. PNB filed a Manifestation and Motion for Issuance of Third Alias Writ of Possession, which the trial court granted anew in an order dated October 10, 1995.[14]

However, on December 12, 1995, the Austrias again filed a motion to set aside the trial court s order dated October 10, 1995 and to recall the third alias writ.[15] Consequent to the filing of this fourth motion, the sheriff again failed to implement the third alias writ, which also lapsed. Thus, on February 15, 1996, PNB filed another Motion for Issuance of a Fourth Alias Writ, [16] which was granted on March 26, 1996. The trial court, after hearing the Austrias fourth motion, issued an order on October 4, 1996, denying the same, on the ground that the issuance of a possessory writ for a property sold at public auction pursuant to an extrajudicial foreclosure proceeding was a ministerial duty on its part. The Austrias failed to establish any legal ground for recalling the writs, even as they claimed a superior right to the subject property.[17] On February 19, 1997, the fourth alias writ was issued by the trial court. The writ was partially implemented with the posting of PNB security guards within the premises of the foreclosed lot.[18] On April 17, 1997, the Austrias, for the fifth time, filed a motion to stop the enforcement of the fourth alias writ and to set aside all prior writs issued by the trial court.[19] In the meantime, the Austrias filed before the Regional Trial Court of Paraaque, an action for cancellation of PNB s title to the property, docketed as Civil Case No. 97-0184.[20] On October 28, 1997, the trial court denied the Austrias fifth motion but ruled that: any writ of possession that may be issued in this case, is declared unenforceable against the MOVANTS ERNESTO AUSTRIA and the HEIRS OF LORETO AUSTRIA, until the Court declares otherwise. [21] PNB filed a motion for reconsideration, which was denied on May 20, 1998.[22] A petition for certiorari under Rule 65 of the Rules of Court was filed by PNB before the Court of Appeals. However, the Court of Appeals dismissed the petition, stating:

There is no prima facie showing of grave abuse of discretion on the part of respondent Judge in issuing his assailed Order which the Court finds to be in accord with law, the pertinent rules and jurisprudence cited therein. Hence, PNB filed the instant petition, contending that: I THE COURT OF APPEALS COMMITTED A SERIOUS ERROR BY SIMPLY ADOPTING THE FINDINGS OF THE TRIAL COURT THAT WRIT OF POSSESSION CANNOT BE ENFORCED AGAINST RESPONDENT AUSTRIA. SAID FINDINGS ARE UNPROVEN AND UNSUPPORTED BY EVIDENCE. II THE COURT OF APPEALS COMMITTED SERIOUS MISAPPREHENSION OF FACTS IN: A) SUPPORTING THE JURISPRUDENCE CITED BY THE TRIAL COURT IN THE OCTOBER 28, 1997 ORDER. THE RULINGS DO NOT JUSTIFY THE NON-ENFORCEMENT OF THE WRIT OF POSSESSION AGAINST RESPONDENTS. RESPONDENTS WERE GIVEN THE OPPORTUNITY TO BE HEARD BUT NO EVIDENCE WAS PRESENTED TO SUPPORT THEIR CLAIM; B) NOT GIVING DUE CONSIDERATION TO THE FACT THAT PNB HAS THE LEGAL RIGHT TO POSSESS THE PROPERTY AS ITS REGISTERED OWNER; C) LOSING SIGHT OF THE FACT THAT THE TRIAL COURT BELATEDLY ISSUED THE OCTOBER 28, 1997 ORDER DIRECTING THAT THE WRIT OF POSSESSION CANNOT BE ENFORCED AGAINST THE RESPONDENTS. THE TRIAL COURT HAD EARLIER ISSUED FOUR (4) POSSESSORY WRITS ALL OF WHICH WERE DIRECTED AGAINST RESPONDENTS AUSTRIA & QUINTANA.[23] The basic issue to be resolved in this case is whether or not an ex-parte writ of possession issued pursuant to Act No. 3135, as amended, can be enforced against a third person who is in actual possession of the

foreclosed property and who is not in privity with the debtor/ mortgagor.[24] Petitioner PNB maintains that the trial court s order was based on the unproven allegation that respondents had purchased the property from the Monsods before the latter mortgaged it to PNB. According to petitioner PNB, respondents did not adduce any proof to support their claim of ownership, even as they were repeatedly given the opportunity to do so during the hearings on the numerous motions filed by respondents themselves. Petitioner PNB also submits that since it is the registered owner of the property, it is entitled to a writ of possession as a matter of right. The bank insists that it could rely on the title of the registered land which does not have any annotation of respondents supposed rights. Petitioner PNB likewise avers that the trial court could not now belatedly refuse to enforce the writ of possession against respondents. The trial court had already issued a total of four possessory writs directing the ouster of all occupants of the lot, including respondents herein. On the other hand, respondents assert that the trial court correctly held that the writ of possession can only be implemented against the debtor/mortgagor and his successors-in-interest. Since respondents acquired their rights as owners of the property by virtue of a sale made to them by the Monsods prior to the bank s mortgage lien, respondents can not be dispossessed therefrom without due notice and hearing, through the simple expedient of an ex-parte possessory writ. We agree with respondents. Under applicable laws and jurisprudence, they can not be ejected from the property by means of an ex-parte writ of possession. The operative provision under Act No. 3135, as amended,[25] is Section 6, which states: Sec. 6. Redemption. In all cases in which an extrajudicial sale is made under the special power hereinbefore referred to, the debtor, his successors in interest or any person having a lien on the property

subsequent to the mortgage or deed of trust under which the property is sold, may redeem the same at any time within the term of one year from and after the date of the sale; and such redemption shall be governed by the provisions of section four hundred and sixty-four to four hundred and sixty-six, inclusive, of the Code of Civil Procedure, in so far as these are not inconsistent with the provisions of this Act. (Italics ours) Despite the evolutionary development of our procedural laws throughout the years, the pertinent rule in the Code of Civil Procedure[26] remains practically unchanged. Particularly, Rule 39, Section 33, second paragraph, which relates to the right of possession of a purchaser of property in an extrajudicial foreclosure sale: Sec. 33. x x x Upon the expiration of the right of redemption, the purchaser or redemptioner shall be substituted to and acquire all the rights, title, interest and claim of the judgment obligor to the property at the time of levy. The possession of the property shall be given to the purchaser or last redemptioner by the same officer unless a third party is actually holding the property adversely to the judgment obligor. (Italics ours) Thus, in Barican v. Intermediate Appellate Court,[27] we held that the obligation of a court to issue an ex-parte writ of possession in favor of the purchaser in an extrajudicial foreclosure sale ceases to be ministerial once it appears that there is a third party in possession of the property who is claiming a right adverse to that of the debtor/mortgagor. The same principle was inversely applied in a more recent case,[28] where we ruled that a writ of possession may be issued in an extrajudicial foreclosure of real estate mortgage, only if the debtor is in possession and no third party had intervened. Although the factual nuances of this case may slightly differ from the aforecited cases, the availing circumstances are undeniably similar a party in possession of the foreclosed property is asserting a right adverse to the debtor/mortgagor and is a stranger to the foreclosure proceedings in which the ex-parte writ of possession was applied for.

It should be stressed that the foregoing doctrinal pronouncements are not without support in substantive law. Notably, the Civil Code protects the actual possessor of a property, to wit: Art. 433. Actual possession under claim of ownership raises a disputable presumption of ownership. The true owner must resort to judicial process for the recovery of the property. Under the aforequoted provision, one who claims to be the owner of a property possessed by another must bring the appropriate judicial action for its physical recovery. The term judicial process could mean no less than an ejectment suit or reinvindicatory action, in which the ownership claims of the contending parties may be properly heard and adjudicated. An ex-parte petition for issuance of a possessory writ under Section 7 of Act No. 3135 is not, strictly speaking, a judicial process as contemplated above. Even if the same may be considered a judicial proceeding for the enforcement of one s right of possession as purchaser in a foreclosure sale, it is not an ordinary suit filed in court, by which one party sues another for the enforcement or protection of a right, or the prevention or redress of a wrong. [29] It should be emphasized that an ex-parte petition for issuance of a writ of possession is a non-litigious proceeding authorized in an extrajudicial foreclosure of mortgage pursuant to Act 3135, as amended. Unlike a judicial foreclosure of real estate mortgage under Rule 68 of the Rules of Court, any property brought within the ambit of the act is foreclosed by the filing of a petition, not with any court of justice, but with the office of the sheriff of the province where the sale is to be made.[30] As such, a third person in possession of an extrajudicially foreclosed realty, who claims a right superior to that of the original mortgagor, will have no opportunity to be heard on his claim in a proceeding of this nature. It stands to reason, therefore, that such third person may not be dispossessed on the strength of a mere ex-parte possessory writ, since to do so would be tantamount to his summary ejectment, in violation of the basic tenets of due process.

Besides, as earlier stressed, Article 433 of the Civil Code, cited above, requires nothing less than an action for ejectment to be brought even by the true owner. After all, the actual possessor of a property enjoys a legal presumption of just title in his favor,[31] which must be overcome by the party claiming otherwise. In the case at bar, petitioner PNB admitted that as early as 1990, it was aware that the subject lot was occupied by the Austrias. Yet, instead of bringing an action in court for the ejectment of respondents, it chose to simply file an ex-parte petition for a writ of possession pursuant to its alleged right as purchaser in the extra-judicial foreclosure sale. We cannot sanction this procedural shortcut. To enforce the writ against an unwitting third party possessor, who took no part in the foreclosure proceedings, would be tantamount to the taking of real property without the benefit of proper judicial intervention. Consequently, it was not a ministerial duty of the trial court under Act No. 3135 to issue a writ of possession for the ouster of respondents from the lot subject of this instant case. The trial court was without authority to grant the ex-parte writ, since petitioner PNB s right of possession under said Act could be rightfully recognized only against the Monsods and the latter s successors-in-interest, but not against respondents who assert a right adverse to the Monsods. Hence, the trial court cannot be precluded from correcting itself by refusing to enforce the writs it had previously issued. Its lack of authority to direct issuance of the writs against respondents assured that its earlier orders would never attain finality in the first place. In the same vein, respondents are not obliged to prove their ownership of the foreclosed lot in the ex-parte proceedings conducted below. The trial court has no jurisdiction to determine who between the parties is entitled to ownership and possession of the foreclosed lot. Likewise, registration of the lot in petitioner PNB s name does not automatically entitle the latter to possession thereof. As discussed earlier, petitioner PNB must resort to the appropriate judicial process for recovery of the property and cannot simply invoke its title in an ex-parte proceeding to justify the ouster of respondents.

WHEREFORE, the instant petition is DENIED and the resolution of the Court of Appeals in CA G.R. SP No. 48660 is AFFIRMED. SO ORDERED. SPS. RONALD HUTCHISON and VALENTINE NAVALLE-HUTCHISON, Petitioners, vs. ENRIQUE M. BUSCAS, respondent. DECISION PUNO, J.: The case at bar concerns a boundary dispute involving 6,471 square meters of land in San Juan, Lubao, Pampanga. Petitioner spouses RONALD and VALENTINE HUTCHISON seek the reversal of the Decision of the Court of Appeals in CA-G.R. CV No. 66077, dated February 19, 2003, holding that respondent ENRIQUE M. BUSCAS is entitled to the possession of the disputed area. The records show that on October 1, 1987, petitioner spouses purchased from V.A. Development Enterprises, Inc. a 76,207-sq. m. land (designated as Lot No. 7216) in San Juan, Lubao, Pampanga. They occupied the land after a title was issued in their names. On August 22, 1989, one Juanita Arrastia, the owner of a lot adjacent to that of petitioner spouses, sold a portion of her land to respondent. The transaction, covering 7,581 sq. m. (designated as Lot No. 7047-A), was evidenced by a Quitclaim Deed in favor of respondent. Respondent occupied 1,100 sq. m. of his land. However, he failed to register the portion of the lot in his name and title to the property remained in Arrastia's name. On January 10, 1995, respondent commissioned geodetic engineer Narciso Manansala to survey his property. Manansala prepared a sketch/subdivision plan of respondent's lot. His survey revealed that 6,471 sq. m. thereof was occupied by petitioner spouses. Respondent sent a demand letter to petitioner spouses to vacate the encroached area. Petitioner spouses refused and insisted that it was part of

their land. Thus, respondent filed a complaint for unlawful detainer (Civil Case No. 1329) against petitioner spouses before the Municipal Trial Court (MTC) of Lubao, Pampanga. After trial, the MTC ruled in favor of respondent. However, on appeal, the Regional Trial Court (RTC) dismissed the case. It ruled that MTC had no jurisdiction over the subject matter as it is a boundary dispute and the proper action should have been an accion reinvindicatoria before the RTC. Consequently, respondent filed a case for accion reinvindicatoria against petitioner spouses with the RTC of Guagua, Pampanga.[1] At the trial, respondent adduced in evidence the Quitclaim Deed to prove his title over the disputed area. He likewise testified on the survey conducted by Manansala. Another geodetic engineer, Angelito H. Nicdao, testified that in the unlawful detainer case earlier filed by the respondent, he was directed by the MTC judge hearing the case to conduct a verification survey of the parties' lots. In compliance with the order, he surveyed the two (2) lots using the title of petitioner spouses and the records of the Bureau of Lands.[2] His survey revealed that petitioner spouses encroached on 6,471 sq. m. of the adjacent land claimed by respondent. Respondent offered in evidence the verification plan and report of Nicdao relative to his survey. On the part of petitioner spouses, petitioner Valentine Hutchison testified that she purchased Lot No. 7216 in Lubao, Pampanga, covering an area of 76,207 sq. m., and title thereto was duly issued in her name and that of her spouse. After trial, the RTC dismissed[3] the complaint for lack of merit. It ruled that respondent's Quitclaim Deed was not sufficient proof of ownership; that respondent failed to clearly identify the property claimed as it was only marked with an 'X sign, and; that petitioner spouses, as registered owners, are entitled to possession of the disputed lot. On appeal, the Court of Appeals reversed the decision of the trial court. [4] It ruled that respondent is entitled to possession of the disputed area as he was able to prove his claim of ownership and the identity of the subject land. Hence, this appeal where petitioner spouses assign the following errors:

I THE COURT OF APPEALS ERRED IN ITS CONCLUSION THAT THE RESPONDENT SUFFICIENTLY IDENTIFIED THE PROPERTY HE SEEKS TO RECOVER. II THE COURT OF APPEALS ERRED IN ITS LEGAL CONCLUSION OF LAW THAT THE TITLE OF THE RESPONDENT TO THE SUBJECT PROPERTY IS THE QUITCLAIM DEED OVER A PORTION OF LAND. III THE COURT OF APPEALS ERRED IN ITS LEGAL CONCLUSION THAT THE RESPONDENT STRENGTHENED HIS 'TITLE BY THE SURVEY HE CAUSED TO BE PREPARED. IV THE COURT OF APPEALS ERRED IN ITS CONCLUSION OF LAW THAT THE RESPONDENT PROVED BY A PREPONDERANCE OF EVIDENCE THAT HIS PROPERTY WAS ENCROACHED UPON BY THE PETITIONERS. V THE COURT OF APPEALS ERRED IN ITS CONCLUSION OF LAW THAT THE RESPONDENT 'IS DECLARED OWNER OF THE 6,471 SQUARE-METERS DISPUTED LOT, AND THE PETITIONERS ARE THUS ORDERED TO VACATE THE SAME. Petitioner spouses contend that there was a gross misapprehension of facts by the Court of Appeals and its legal conclusions were contrary to law and jurisprudence. They assert that respondent failed to identify the portion of land he was claiming and prove his ownership thereof. They allege that: (a) respondent's identification of his 7,581 sq. m. property with a mere 'X mark on the Annex 'A of the Quirclaim Deed is insufficient as the attached Annex 'A was not presented at the trial, and; (b) the surveys conducted by the geodetic engineers cannot be used to identify respondent's lot as they

were based on the records of the Bureau of Lands and not on the document of title of respondent. We find for the petitioner spouses. In civil cases, the law requires that the party who alleges a fact and substantially asserts the affirmative of the issue has the burden of proving it.[5] This evidentiary rule is based on the principle that the suitor who relies upon the existence of a fact should be called upon to prove it.[6]chanroblesvirtuallawlibrary Article 434 of the New Civil Code[7] provides that to successfully maintain an action to recover the ownership of a real property, the person who claims a better right to it must prove two (2) things: first, the identity of the land claimed, and; second, his title thereto. In the case at bar, we find that respondent failed to establish these two (2) legal requirements. The first requisite: the identity of the land. In an accion reinvindicatoria, the person who claims that he has a better right to the property must first fix the identity of the land he is claiming by describing the location, area and boundaries thereof.[8] Anent the second requisite, i.e., the claimant's title over the disputed area, the rule is that a party can claim a right of ownership only over the parcel of land that was the object of the deed.[9] Respondent sought to prove these legal requisites by anchoring his claim on the Quitclaim Deed over a portion of land which was executed by Arrastia in his favor. However, a cursory reading of the Quitclaim Deed shows that the subject land was described, thus: x x x a portion of that property situated at San Juan, Lubao, Pampanga which portion subject of this sale consists of 7,581 square meters more or less, as indicated particularly in the herein attached plan marked as Annex 'A and made an integral part hereof, and the subject property with an 'X sign. Thus, the Quitclaim Deed specified only the extent of the area sold, i.e., 7,581 sq. m. of Arrastia's land. Annex 'A of the Deed, where the entire lot of Arrastia was particularly described and where the specific portion of the property sold to respondent was marked, was not presented by respondent at the trial. As the Deed itself failed to mention the metes and bounds of

the land subject of the sale, it cannot be successfully used by respondent to identify the area he was claiming and prove his ownership thereof. Indeed, the presentation of the Annex 'A is essential as what defines a piece of land is not the size mentioned in the instrument but the boundaries thereof which enclose the land and indicate its exact limits.[10]chanroblesvirtuallawlibrary Neither can the surveys of the lots of petitioner spouses and respondent prove the identity of the contested area and respondent's ownership thereof. The records show that when geodetic engineers Manansala and Nicdao surveyed the lands, they merely relied on the self-serving statement of respondent that he owns the portion of the lot adjacent to petitioner spouses. They were not shown the Deed of Quitclaim and its Annex 'A or any other document of title which described the specific portion of the land allegedly conveyed to respondent.[11] Thus, the surveys cannot be given evidentiary weight to prove the identity of the land sold to respondent and his ownership thereof. Moreover, the rules on evidence provide that where the contents of the document are the facts in issue, the best evidence is the instrument itself.[12] In the case at bar, the identity of the land claimed and respondent's ownership thereof are the very facts in issue. The best evidence to prove these facts is the Quitclaim Deed and its Annex 'A where respondent derives his title and where the land from which he purchased a part was described with particularity, indicating the metes and bounds thereof. Respondent's failure to adduce in evidence Annex 'A of the Quitclaim Deed or produce secondary evidence, after proof of its loss, destruction or unavailability,[13] is fatal to his cause. Finally, it bears stress that in an action to recover real property, the settled rule is that the plaintiff must rely on the strength of his title, not on the weakness of the defendant's title.[14] This requirement is based on two (2) reasons: first, it is possible that neither the plaintiff nor the defendant is the true owner of the property in dispute,[15] and second, the burden of proof lies on the party who substantially asserts the affirmative of an issue for he who relies upon the existence of a fact should be called upon to prove that fact.[16] In the case at bar, as respondent failed to prove his

title to and identity of the contested land, there exists no legal ground upon which to turn over the possession of the disputed area to him. IN VIEW WHEREOF, the petition is GRANTED. The Decision of the Court of Appeals in CA-G.R. CV No. 66077, dated February 19, 2003, is hereby reversed and set aside. The Decision of the Regional Trial Court of Guagua, Pampanga, dismissing the complaint for accion reinvindicatoria in Civil Case No. G-3183, is reinstated. No pronouncement as to costs. SO ORDERED.

RUDY LAO, petitioner, vs. JAIME LAO, respondent. DECISION CALLEJO, SR., J.: As early as 1956, the spouses Julian Lao and Anita Lao had constructed a building on a parcel of land in Balasan, Iloilo City, owned by Alfredo Alava and covered by Transfer Certificate of Title (TCT) No. 28382. They then occupied and leased the same without any written agreement thereon. Anita Lao also put up her business in the premises. On May 12, 1982, Alfredo Alava, as lessor, and Anita Lao, as lessee, executed a Contract of Lease[1] over the said property. The parties agreed that the lease of the property was to be for a period of 35 years, at an annual rental of P120.00. However, the contract of lease was not filed with the Office of the Register of Deeds; hence, was not annotated at the dorsal portion of the said title. Aside from Anita Lao, petitioner Rudy Lao also leased another portion of the same property where he put up his business.[2] In fact, Anita Lao s building was adjacent to where the petitioner conducted his business. At that time, the petitioner knew that Anita Lao and her husband were the

owners of the said building. He also knew that she had leased that portion of the property, and that respondent Jaime Lao, their son, managed and maintained the building, as well as the business thereon. In the meantime, on March 21, 1995, the petitioner purchased the property from Alava, and was later issued TCT No. 152,097 in his name. By then, the property had been classified as commercial, but the yearly rental of P120.00 in the contract of lease between Alava and Anita Lao subsisted. On July 14, 1997, the petitioner filed a Complaint for Unlawful Detainer against the respondent with the 1st Municipal Circuit Trial Court (MCTC) of Carles-Balasan, Iloilo City. The petitioner alleged, inter alia, that the respondent had occupied a portion of his property without any lease agreement and without paying any rentals therefor, and that the same was only through his tolerance and generosity. The petitioner prayed that, after due proceedings, judgment be rendered in his favor as follows: 1. Ordering the defendant, his agents and/or representatives and all persons claiming under him, to vacate the premises he occupies, remove all improvements thereon and restore possession thereof to the plaintiff; 2. Directing the defendant, his agents and/or representatives and all persons claiming under him, when proper, jointly and severally, to pay plaintiff the sums of: P50,000.00 as attorney s fees; at least P15,000.00 as miscellaneous litigation and necessary expenses; such compensation for use of the portion she (sic) occupies, at the rate of P5,000.00 a month from January 24, 1997, until the full and complete surrender thereof to the plaintiff; and 3. The costs of this suit.[3]

In his answer to the complaint, the respondent alleged that the petitioner had no cause of action against him, the truth being that the lessee of the property was his mother, Anita Lao, as evidenced by a contract of lease executed by Alava, the former owner thereof. He further alleged that she had been paying the annual rentals therefor, the last of which was on July 16, 1997 and evidenced by a receipt.[4] He further alleged that she had designated him as manager to maintain the building, pay rentals and operate the business. He then prayed for the dismissal of the complaint.

During the preliminary conference, the respondent admitted that he was in actual possession of the property. For his part, the petitioner admitted that he had been renting another portion of the same property from Alava for years, and that his business establishment and that of Anita Lao s were adjacent to each other. He also admitted that Anita Lao had been renting the said portion of the property for years before he bought it. The respondent adduced in evidence the contract of lease[5] between his mother, Anita Lao, and Alava. On March 4, 1999, the MCTC rendered judgment in favor of the petitioner and against the respondent. The fallo of the decision reads: WHEREFORE, based on the foregoing circumstances, JUDGMENT is hereby rendered in favor of the Plaintiff, Rudy Lao and as against defendant, Jaime Lao, as follows: 1. Ordering defendant, Jaime Lao, his successors-in-interest, agents, members of his family, privies or any person or persons claiming under his name to vacate the portion of Lot No. 3 occupied by him, and to deliver the physical possession thereof to plaintiff, Rudy Lao; 2. Ordering defendant to pay plaintiff, Rudy Lao, the sum of P3,000.00 representing as the monthly rentals of the premises occupied by defendant on Lot No. 3 starting the month of January 1997, until the possession thereof is actually delivered and turned over to the plaintiff; 3. Ordering defendant, Jaime Lao, to pay plaintiff the amount of P20,000.00 as attorney s fees; 4. Ordering defendant, Jaime Lao, to pay Plaintiff, Rudy Lao, the sum of P10,000.00 representing as litigation expenses; and to pay the costs of this suit. SO ORDERED.[6] The respondent appealed the decision to the Regional Trial Court (RTC) of Barotac Viejo, Iloilo City, Branch 66, which rendered judgment on January

28, 2000 affirming the said decision with modification. The fallo of the decision reads: WHEREFORE, the decision appealed from this court is hereby affirmed with a modification that defendant-appellant Jaime Lao is ordered to pay plaintiff-appellee Rudy Lao the sum of P1,000.00 per month as reasonable use of the land subject of the case from January 1997 until possession is turned over to the plaintiff; to pay Rudy Lao the sum of P10,000.00 attorney s fees and P5,000.00 litigation expenses. With cost against the defendant-appellant. SO ORDERED.[7] The RTC ruled that under Article 1676 of the New Civil Code, the petitioner was the purchaser of the property and had the right to terminate the lease between Alava and Anita Lao, it appearing that the lease contract was not registered with the Office of the Register of Deeds. Not being the lessee, the respondent could not invoke the same provision. The trial court also held that the respondent, not his mother, was the real party as defendant in the MCTC, since it was he who was in actual possession of the property. The RTC maintained that if Anita Lao was sued as defendant and was ordered evicted, the decision would not be binding on the respondent since he was not impleaded as defendant. The respondent filed a petition for review with the Court of Appeals (CA), asserting that I. THE HONORABLE REGIONAL TRIAL COURT SERIOUSLY ERRED IN AFFIRMING THE ERRONEOUS FINDING OF THE MCTC THAT THIS CASE WAS PROPERLY BROUGHT AGAINST THE DEFENDANT WHEN HE IS ONLY AN AGENT OF THE REAL PARTY-IN-INTEREST, ANITA LAO. II. THE HONORABLE REGIONAL TRIAL COURT SERIOUSLY ERRED IN AFFIRMING THE ERRONEOUS FINDING OF THE MCTC THAT THERE IS NO AGENCY BETWEEN ANITA LAO AND THE DEFENDANT-APPELLANT BECAUSE THERE WAS NO DOCUMENTARY EVIDENCE PRESENTED TO SHOW THE FACT OF AGENCY.

III. THE HONORABLE REGIONAL TRIAL COURT SERIOUSLY ERRED IN IGNORING THE FACT THAT THE MCTC BLATANTLY DISREGARDING (sic) THE PRE-TRIAL CONFERENCE ORDER IT ISSUED, ISSUING A DECISION CONTRARY TO THE FACTS ADMITTED BY [THE] PARTIES THEMSELVES ESPECIALLY THE ADMISSION OF THE PLAINTIFF-APPELLEE THAT HE KNOWS OF THE EXISTENCE OF THE LEASE.[8] On February 27, 2001, the CA rendered judgment setting aside and reversing the decision of the RTC. The CA ruled that the real party-ininterest as defendant in the MCTC was Anita Lao, the lessee of the property, and not the respondent who was merely the administrator/manager of Anita Lao s building and the occupant of the property. The petitioner s motion for the reconsideration of the decision having been denied by the appellate court, he now comes to this Court for relief via a petition for review on certiorari, claiming that: The Hon. Court of Appeals committed a reversible error when it converted petitioner s cause of action against respondent into a cause of action against respondent s mother; and on the basis thereof, dismissed petitioner s complaint for ejectment against respondent under the mistaken finding that said ejectment case should have been filed against respondent s mother.[9] The petitioner avers that the respondent was the real party-in-interest as defendant in the complaint for unlawful detainer because the respondent s possession of the property was in his personal capacity, and not as the caretaker of the property and the business in the building owned by Anita Lao, the lessee thereon. The petitioner argues that, in an ejectment suit, the threshold issue is who has the right to the material or de facto possession of the subject property as distinguished from the de jure possession thereof; hence, the defendant in an ejectment case is the person in actual physical possession of the property. The petitioner insists that the respondent, having admitted in the MCTC that he was in actual possession of the property and that in fact, Anita Lao was no longer staying in the property after her husband died, is the real party-in-interest, as defendant. He posits that if he filed a complaint for

ejectment against Anita Lao, it would be dismissed because it was the respondent, and not his mother, who was in actual possession of the property. The petition has no merit. We agree with the petitioner that, in ejectment cases, the word possession means nothing more than actual physical possession, not legal possession, in the sense contemplated in civil law.[10] The only issue in such cases is who is entitled to the physical or material possession of the property involved, independent of any claim of ownership set forth by any of the party-litigants.[11] We, likewise, conform to the petitioner s contention that in an action for unlawful detainer, the real party-in-interest as party-defendant is the person who is in possession of the property without the benefit of any contract of lease and only upon the tolerance and generosity of its owner. Such occupant is bound by an implied promise that he will vacate the premises upon demand. This situation is analogous to that of a lessee or tenant whose term has expired, but whose occupancy continued by mere tolerance of the owner.[12] He is the real party-in-interest as defendant.[13] However, the records in this case show that the respondent has been in possession of the property subject of the complaint not by mere tolerance or generosity of the petitioner, but as the manager of his mother, Anita Lao, who conducted her business in the building/warehouse which stood on a portion of the property leased from Alava, the former owner. Contrary to the petitioner s claim, the respondent s possession of the property was in behalf of his mother, the lessee thereof, and not in his own right, independently of that of his mother. The petitioner cannot feign ignorance of the existence of the lease of the subject property by Anita Lao, the existence of the building and her business thereon, and the fact that the respondent managed his mother s building and business. It must be stressed that during the preliminary conference of the parties before the MCTC, the petitioner admitted his knowledge of the foregoing facts.

While it is true that the contract of lease between Alava and Anita Lao was not filed in the Office of the Register of Deeds and annotated at the dorsal portion of the petitioner s title over the property, nevertheless, the petitioner was bound by the terms and conditions of the said contract of lease. The lease, in effect, became a part of the contract of sale.[14] Under Section 2, Rule 70 of the Rules of Court, the petitioner, as the vendee of the property, had the right to file an action for unlawful detainer against Anita Lao upon demand, but for breach of the contract of lease: SEC. 2. Lessor to proceed against lessee only after demand. Unless otherwise stipulated, such action by the lessor shall be commenced only after demand to pay or comply with the conditions of the lease and to vacate is made upon the lessee, or by serving written notice of such demand upon the person found on the premises, or by posting such notice on the premises if no person be found thereon, and the lessee fails to comply therewith after fifteen (15) days in the case of land or five (5) days in the case of buildings. If the petitioner had done so and judgment was rendered in his favor, ordering Anita Lao to vacate the property, the respondent herein, who is in possession of the property for and in her behalf, would then have to abide by the decision and vacate the same. This was the ruling of the Court in Oro Cam Enterprises, Inc. v. Court of Appeals,[15] thus: It is well-settled that a judgment in an ejectment suit is binding not only upon the defendants in the suit but also against those not made parties thereto, if they are: a) trespassers, squatters or agents of the defendant fraudulently occupying the property to frustrate the judgment; b) guests or other occupants of the premises with the permission of the defendant; c) transferees pendente lite; d) sublessee; e) co-lessees; or f) members of the family, relatives and other privies of the defendant.[16]

Apparently, the petitioner believed that it was unfair for Anita Lao to be paying an annual rental of only P120.00 for the portion of the property leased by her, considering that the said lot had already been classified as commercial property. Moreover, it was not Anita Lao who stayed in the leased premises; it was her son. The petitioner had no cause of action for unlawful detainer against Anita Lao because of the subsisting contract of lease; hence, he could not file the complaint against her. What the petitioner had no right to do directly, he did indirectly by filing a complaint for unlawful detainer against her son, the respondent, believing that by so doing, he will be rid of Anita Lao s lease contract. The Court, thus, rules that the CA acted in accord with law when it ordered the dismissal of the complaint. IN LIGHT OF ALL THE FOREGOING, the petition is DENIED for lack of merit. Costs against the petitioner. SO ORDERED. RENE GANILA,* EDUARDO DUMADA-OG, SR., RAFAEL GANILA, JOSE PASTRANA, LOURDES GANILA, FLORENTINO GANILA, SERAFIN GANILA, LORETO ARELLANO, CONRADO GANILA, VIVENCIO ALVIOR, EDUARDO GANTALA, AMPARO VILLANUEVA, ELEUTERIO SILVA, ADELINA GANILA, FELIZARDO GANILA, SR., ENRIQUE GANILA, ABRAHAM TANONG, EMILIO ALFARAS, JR., BAPTIST CHRISTIAN LEARNING CENTER, petitioners, vs. HON. COURT OF APPEALS AND VIOLETA C. HERRERA, respondents. DECISION QUISUMBING, J.: For review on certiorari are the Decision[1] dated March 30, 2001 of the Court of Appeals in CA-G.R. SP No. 58191, and its Resolution[2] dated October 18, 2001 denying the motion for reconsideration. The assailed decision denied the petition to set aside the Resolution[3] of the Regional Trial Court (RTC) of San Miguel, Jordan, Guimaras, Branch 65, affirming the Order of the Municipal Circuit Trial Court (MCTC) for the 19 petitioners to vacate the contested parcel of land.

The facts are as follows: On March 19, 1997, private respondent Violeta Herrera filed 21 ejectment Complaints[4] before the 16thbarangaycomplaints. MCTC, JordanBuenavista-Nueva Valencia, Jordan, Guimaras. Private respondent alleged that she owns Lot 1227 of the Cadastral Survey of Jordan, Guimaras, with an area of 43,210 square meters; that she inherited the lot from her parents; and that she only tolerated petitioners to construct residential houses or other improvements on certain portions of the lot without rental. Sometime in September or October 1996, private respondent demanded that the petitioners vacate the lot and remove their houses and other improvements thereon. Petitioners refused, despite offer of money by way of assistance to them. After the conciliation failed, private respondent filed the In their Answers,[5] eight[6] of the petitioners claimed that Lot 1227 was formerly a shoreline which they developed when they constructed their respective houses. Another eight[7] maintained that their houses stood on Lot 1229 of the Cadastral Survey of Jordan, Guimaras. The other three[8] asserted that Lot 1227 is a social forest area. At the preliminary conference, the parties agreed to designate two geodetic engineers as commissioners of the MCTC to conduct a relocation survey of Lot 1227 and to identify who among the petitioners have houses within the lot.[9] The commissioners reported that: (1) the house of Henry Gabasa, defendant in Civil Case No. 288-J, is almost outside Lot 1227; (2) the house of Ludovico Amatorio, defendant in Civil Case No. 289-J, diagonally traversed the boundary; and (3) the houses of the 19 petitioners are inside Lot 1227.[10] Eight months after herein petitioners failure to comment on the manifestation of private respondent to terminate the preliminary conference, the MCTC terminated the preliminary conference.[11] Thereafter, petitioners counsel Atty. Nelia Jesusa L. Gonzales failed to file her clients position papers and affidavits, even after they sought a 30-day extension to file the same.[12]

Consequently, the MCTC decided the cases as follows: WHEREFORE, premises considered, judgment is hereby rendered in favor of the plaintiff whereby each of the twenty-one (21) defendants are hereby ordered: 1. To vacate Lot 1227 of the Cadastral Survey of Jordan, Guimaras;

2. To pay Two Hundred Pesos (P200.00) per month from October, 1996 as compensation for the use of the property until the same is vacated; and 3. To pay Two Thousand Pesos (P2,000.00) as attorney s fees and litigation expenses. SO ORDERED.[13] Petitioners appealed to the RTC, Branch 65, at Jordan, Guimaras, which decided as follows: WHEREFORE, premises considered, the decision in Civil Cases Nos. 0270-J, 0272-J, 0273-J, 0274-J, 0275-J, 0276-J, 0277-J, 0278-J, 0279-J, 0280-J, 0281-J, 0282-J, 0283-J, 0284-J, 0285-J, 0286-J, 0287-J, 0291-J and 0292-J are hereby affirmed. The decision of the court below in Civil Cases Nos. 0288-J and 0289-J are set aside. Civil Cases Nos. 0288-J and 0289-J are hereby DISMISSED. SO ORDERED.[14] The RTC ruled that the evidence showed the better right of private respondent to possess Lot 1227. Private respondent s position paper, affidavit and tax declaration supported her allegations. In addition, the commissioners report and sketch plan showed that indeed petitioners occupy Lot 1227. On the other hand, according to the RTC, the petitioners failed to present evidence which would show that they are entitled to possess the lot. Based on the sketch plan, the RTC dismissed the cases against Gabasa and Amatorio since their houses occupy only a small area of Lot 1227. It

declared that Gabasa and Amatorio believed in good faith that the whole area they occupied was part of the seashore. The 19 petitioners, who were ordered to vacate the lot, filed a joint petition for review with the Court of Appeals. The appellate court denied the petition. Petitioners moved for reconsideration and filed an amended petition. The Court of Appeals, however, affirmed the factual findings and conclusions arrived at by the trial courts and denied the amended petition for lack of merit.[15] It also denied the motion for reconsideration. Petitioners are now before us, on a petition for review, alleging that: The Honorable Court of Appeals, with due respect and deference, committed a reversible error in the interpretation/application of the law in the instant case and in the appreciation of the facts and evidence presented. The Court of Appeals gravely abused its discretion when it denied and dismissed the petition filed by the petitioners.[16] After considering the parties submissions, we find three basic issues: (1) Did the MCTC err in taking jurisdiction over and deciding the cases? (2) Did the RTC err in sustaining the MCTC s judgment? (3) Did the CA err in denying the petition for review filed by the 19 petitioners ordered to be ejected? Petitioners insist that private respondent should have filed an action to recover possession de jure, not a mere complaint for ejectment, for two reasons. One, they possessed Lot 1227 in good faith for more than 30 years in the concept of owners. And two, there was no withholding of possession since private respondent was not in prior possession of the lot. Private respondent states in her Comment before us that the allegations in her Complaints make out a clear case of unlawful detainer which is cognizable by the MCTC. We are in agreement with her stance. There was no error in the choice of the complainant s remedy, a matter left to her determination as the suitor. And the complaint itself is defined by the allegations therein, not the allegations of the defendants. At the outset, we note that petitioners question the MCTC s jurisdiction yet they admit in their preliminary statement that the Complaints filed are

indeed for unlawful detainer, and that the only issue to be determined is mere physical possession (possession de facto) and not juridical possession (possession de jure), much less ownership.[17] While petitioners assert that this case involves only deprivation of possession, they confuse the remedy of an action for forcible entry with that of unlawful detainer. In unlawful detainer, prior physical possession by the plaintiff is not necessary. It is enough that plaintiff has a better right of possession. Actual, prior physical possession of a property by a party is indispensable only in forcible entry cases. In unlawful detainer cases, the defendant is necessarily in prior lawful possession of the property but his possession eventually becomes unlawful upon termination or expiration of his right to possess.[18] Thus, the fact that petitioners are in possession of the lot does not automatically entitle them to remain in possession. And the issue of prior lawful possession by the defendants does not arise at all in a suit for unlawful detainer, simply because prior lawful possession by virtue of contract or other reasons is given or admitted. Unlike in forcible entry where defendants, by force, intimidation, threat, strategy or stealth, deprive the plaintiff or the prior physical possessor of possession. Here there is no evidence to show that petitioners entered the lot by any of these acts. If only to stress the fundamental principles related to present controversy, jurisdiction over unlawful detainer suits is vested in municipal trial courts.[19] And in ejectment cases, the jurisdiction of the court is determined by the allegations of the complaint.[20] In this case for ejectment, private respondent s allegations sufficiently present a case of unlawful detainer. She alleged that (1) she owns Lot 1227; (2) she tolerated petitioners to construct their houses thereon; (3) she withdrew her tolerance; and (4) petitioners refused to heed her demand to vacate the lot. The Complaints were also filed within one year from the date of her demand. The cause of action for unlawful detainer between the parties springs from the failure of petitioners to vacate the lot upon lawful demand of the private respondent. When they refused to vacate the lot after her demand, petitioners continued possession became unlawful. Her complaint for ejectment against respondent, to put it simply, is not without sufficient basis.

Petitioners contention that private respondent should have filed an action to recover possession de jure with the RTC is not supported by law or jurisprudence. The distinction between a summary action of ejectment and a plenary action for recovery of possession and/or ownership of the land is settled in our jurisprudence. What really distinguishes an action for unlawful detainer from a possessory action (accion publiciana) and from a reinvindicatory action (accion reinvindicatoria) is that the first is limited to the question of possession de facto. An unlawful detainer suit (accion interdictal) together with forcible entry are the two forms of an ejectment suit that may be filed to recover possession of real property. Aside from the summary action of ejectment, accion publiciana or the plenary action to recover the right of possession and accion reinvindicatoria[21] or the action to recover ownership which includes recovery of possession, make up the three kinds of actions to judicially recover possession. It is not up to defendants, now petitioners herein, to dictate upon plaintiff, now the private respondent, what her initial recourse should be. Her choice of an action for ejectment against so-called squatters is well within her rights. Petitioners cite the case of Bayubay v. Court of Appeals,[22] and argue that the MCTC s decision was without jurisdictional or legal basis because the MCTC did not issue a preliminary conference order. They assert that the 10-day period to file position papers and affidavits only starts after the parties had received a preliminary conference order. They insist they were denied due process when the MCTC decided the cases based merely on private respondent s Complaints and affidavit, without considering their Answers. For her part, private respondent maintains that there was substantial compliance with the rules in the MCTC s conduct of the preliminary conference, hence there was no violation of due process nor disregard of its proper jurisdiction. Petitioners present contention was first raised only in their appeal to the RTC. Raising it before the appellate tribunal is barred by estoppel.[23] They should have raised it in the proceedings before the MCTC. In our

view, this issue is a mere afterthought, when the MCTC decided against them. Basic rules of fair play, justice and due process require that as a rule an issue cannot be raised by the petitioners for the first time on appeal.[24] Besides, petitioners did not question initially the MCTC s Order dated February 19, 1999, when they moved for an extension of time to file their position papers and affidavits. They wanted another 30 days on top of the 30 days set by the MCTC, which strictly should have been 10 days only. In this regard, petitioners could not claim that they were denied sufficient time to file their position papers and affidavits before the trial court. Further, they cannot validly invoke our ruling[25] in Bayubay, for in that case there was no order at all terminating the preliminary conference and requiring the parties to submit position papers and affidavits. We note with dismay petitioners insistence that we order the MCTC to conduct the requisite preliminary conference. The summary character of ejectment suits will be disregarded if we allow petitioners to further delay this case by allowing a second preliminary conference. Ejectment by way of forcible entry and unlawful detainer cases are summary proceedings, designed to provide an expeditious means of protecting actual possession or the right to possession over the property involved. It is a timely procedure designed to remedy the delay in the resolution of such cases.[26] Lastly, petitioners aver that private respondent failed to allegation of ownership of Lot 1227 as it is only based on a tax which is not an evidence of ownership. They also claim possession of the lot was not and could not be by mere However, this is a factual matter best left to the trial courts. prove her declaration that their tolerance.

What we have now is sufficient evidence showing that private respondent has a better right to possess Lot 1227. The commissioners report and sketch plan show that the 19 petitioners occupy the lot, which corroborate private respondent s allegation and disprove petitioners defense that Lot 1227 is a shoreline; or that Lot 1227 is a social forest area. While not a conclusive evidence of ownership, private respondent s tax declaration constitutes proof that she has a claim of title over the lot. It has been held that:

Although tax declarations or realty tax payment of property are not conclusive evidence of ownership, nevertheless, they are good indicia of possession in the concept of owner for no one in his right mind would be paying taxes for a property that is not in his actual or at least constructive possession. They constitute at least proof that the holder has a claim of title over the property. The voluntary declaration of a piece of property for taxation purposes manifests not only one s sincere and honest desire to obtain title to the property and announces his adverse claim against the State and all other interested parties, but also the intention to contribute needed revenues to the Government. Such an act strengthens one s bona fide claim of acquisition of ownership.[27] The lower courts did not err in adjudicating the issue of possession. Mere absence of title over the lot is not a ground for the courts to withhold relief from the parties in an ejectment case. Plainly stated, the trial court has validly exercised its jurisdiction over the ejectment cases below. The policy behind ejectment suits is to prevent breaches of the peace and criminal disorder, and to compel the party out of possession to respect and resort to the law alone to obtain what she claims is hers. The party deprived of possession must not take the law into his or her own hands.[28] For their part, herein petitioners could not be barred from defending themselves before the court adequately, as a matter of law and right. However, petitioners in their defense should show that they are entitled to possess Lot 1227. If they had any evidence to prove their defenses, they should have presented it to the MCTC with their position papers and affidavits. But they ignored the court s order and missed the given opportunity to have their defenses heard, the very essence of due process.[29] Their allegations were not only unsubstantiated but were also disproved by the plaintiff s evidence. In sum, we find no reversible error much less any grave abuse of discretion committed by the Court of Appeals. A person who occupies the land of another at the latter s tolerance or permission, without any contract between them, is necessarily bound by an implied promise that he will vacate upon demand, failing which a summary action for ejectment is the proper remedy against him.[30] His status is analogous to that of a lessee or tenant whose term of lease has expired but whose occupancy continued

by tolerance of the owner. In such a case, the date of unlawful deprivation or withholding of possession is to be counted from the date of the demand to vacate.[31] WHEREFORE, the instant petition is DENIED for lack of merit. The Decision of the Court of Appeals dated March 30, 2001 and its Resolution dated October 18, 2001 are AFFIRMED. Costs against petitioners. SO ORDERED. RUBEN SANTOS, petitioner, vs. SPOUSES TONY AYON and MERCY AYON, respondents. DECISION SANDOVAL-GUTIERREZ, J.: For our resolution is the petition for review on certiorari assailing the Decision[1] of the Court of Appeals dated October 5, 1998 in CA-G.R. SP No. 4735 and its Resolution[2] dated December 11, 1998 denying the motion for reconsideration. The petition alleges that on November 6, 1996, Ruben Santos, petitioner, filed with the Municipal Trial Court in Cities (MTCC), Branch 2, Davao City a complaint for illegal detainer against spouses Tony and Mercy Ayon, respondents, docketed as Civil Case No. 3506-B-96. In his complaint, petitioner averred that he is the registered owner of three lots situated at Lanzona Subdivision, Matina, Davao City, covered by Transfer Certificates of Title (TCT) Nos. 108174, 108175, and 108176. Respondent spouses are the registered owners of an adjacent parcel of land covered by TCT No. T-247792. The previous occupant of this property built a building which straddled both the lots of the herein parties. Respondents have been using the building as a warehouse. Petitioner further alleged in his complaint that in 1985, when he bought the three lots, he informed respondents that the building occupies a portion of his land. However, he allowed them to continue using the building. But in

1996, he needed the entire portion of his lot, hence, he demanded that respondents demolish and remove the part of the building encroaching his property and turn over to him their possession. But they refused. Instead, they continued occupying the contested portion and even made improvements on the building. The dispute was then referred to the barangay lupon, but the parties failed to reach an amicable settlement. Accordingly, on March 27, 1996, a certification to file action was issued. In their answer, respondents sought a dismissal of this case on the ground that the court has no jurisdiction over it since there is no lessor-lessee relationship between the parties. Respondents denied they were occupying petitioner s property by mere tolerance, claiming they own the contested portion and have been occupying the same long before petitioner acquired his lots in 1985. On July 31, 1997, the MTCC rendered its Decision in favor of petitioner, thus: WHEREFORE, judgment is rendered in favor of the plaintiff and against the defendants ordering the latter, their successors-in-interest and other persons acting in their behalf to vacate the portion of the subject properties and peacefully surrender possession thereof to plaintiff as well as dismantle/remove the structures found thereon. Defendants are further ordered to pay reasonable value for the use and occupation of the encroached area in the amount of One Thousand Pesos (P1,000.00) a month beginning September 1996 and the subsequent months thereafter until premises are vacated; to pay attorney s fees of Ten Thousand Pesos (P10,000.00); and to pay the costs of suit. SO ORDERED. [3] On appeal, the Regional Trial Court (RTC), Branch 11, Davao City, in its Decision dated February 12, 1998 in Civil Case No. 25, 654-97, affirmed in toto the MTCC judgment.[4] The RTC upheld the finding of the MTCC that respondents occupation of the contested portion was by mere tolerance. Hence, when petitioner needed the same, he has the right to eject them through court action.

Respondents then elevated the case to the Court of Appeals through a petition for review. In its Decision dated October 5, 1988 now being challenged by petitioner, the Court of Appeals held that petitioner s proper remedy should have been an accion publiciana before the RTC, not an action for unlawful detainer, thus: In this case, petitioners were already in possession of the premises in question at the time private respondent bought three (3) lots at the Lanzona Subdivision in 1985, a portion of which is occupied by a building being used by the former as a bodega. Apart from private respondent s bare claim, no evidence was alluded to show that petitioners possession was tolerated by (his) predecessor-in-interest. The fact that respondent might have tolerated petitioners possession is not decisive. What matters for purposes of determining the proper cause of action is the nature of petitioners possession from its inception. And in this regard, the Court notes that the complaint itself merely alleges that defendants-petitioners have been occupying a portion of the above properties of the plaintiff for the past several years by virtue of the tolerance of the plaintiff. Nowhere is it alleged that his predecessor likewise tolerated petitioners possession of the premises. x x x. Consequently, x x x, respondent should present his claim before the Regional Trial Court in an accion publiciana and not before the Municipal Trial Court in a summary proceeding of unlawful detainer. WHEREFORE, the decision under review is hereby REVERSED and SET ASIDE. Accordingly, the complaint for unlawful detainer is ordered DISMISSED. [5] Petitioner filed a motion for reconsideration, but was denied by the Appellate Court in its Resolution dated December 11, 1998. Hence, the instant petition for review on certiorari ascribing to the Court of Appeals the following errors: I THE HONORABLE COURT OF APPEALS MISAPPLIED THE LAW IN DISMISSING THE INSTANT CASE ON THE GROUND THAT PETITIONER

SHOULD PRESENT HIS CLAIM BEFORE THE REGIONAL TRIAL COURT IN AN ACCION PUBLICIANA. II THE FINDINGS OF THE HONORABLE COURT OF APPEALS IS NOT IN CONSONANCE WITH EXISTING LAWS AND JURISPRUDENCE. The sole issue here is whether the Court of Appeals committed a reversible error of law in holding that petitioner s complaint is within the competence of the RTC, not the MTCC. Petitioner contends that it is not necessary that he has prior physical possession of the questioned property before he could file an action for unlawful detainer. He stresses that he tolerated respondents occupancy of the portion in controversy until he needed it. After his demand that they vacate, their continued possession became illegal. Hence, his action for unlawful detainer before the MTCC is proper. Respondents, in their comment, insisted that they have been in possession of the disputed property even before petitioner purchased the same on April 10, 1985. Hence, he cannot claim that they were occupying the property by mere tolerance because they were ahead in time in physical possession. We sustain the petition. It is an elementary rule that the jurisdiction of a court over the subject matter is determined by the allegations of the complaint and cannot be made to depend upon the defenses set up in the answer or pleadings filed by the defendant.[6] This rule is no different in an action for forcible entry or unlawful detainer.[7] All actions for forcible entry or unlawful detainer shall be filed with the proper Metropolitan Trial Courts, the Municipal Trial Courts and the Municipal Circuit Trial Courts, which actions shall include not only the plea for restoration of possession but also all claims for damages and costs arising therefrom.[8] The said courts are not divested of jurisdiction over such cases even if the defendants therein raises the question of ownership over the litigated property in his pleadings and the question of possession cannot be resolved without deciding the issue of ownership.[9]

Section 1, Rule 70 on forcible entry and unlawful detainer of the 1997 Rules of Civil Procedure, as amended, reads: Section 1. Who may institute proceedings, and when. Subject to the provisions of the next succeeding section, a person deprived of the possession of any land or building by force, intimidation, threat, strategy, or stealth, or a lessor, vendor, vendee, or other person against whom the possession of any land or building is unlawfully withheld after the expiration or termination of the right to hold possession, by virtue of any contract, express or implied, or the legal representatives or assigns of any such lessor, vendor, vendee or other person may, at any time within one (1) year after such unlawful deprivation or withholding of possession, bring an action in the proper Municipal Trial Court against the person or persons unlawfully withholding or depriving of possession, or any person or persons claiming under them, for the restitution of such possession, together with damages and costs. Under the above provision, there are two entirely distinct and different causes of action, to wit: (1) a case for forcible entry, which is an action to recover possession of a property from the defendant whose occupation thereof is illegal from the beginning as he acquired possession by force, intimidation, threat, strategy or stealth; and (2) a case for unlawful detainer, which is an action for recovery of possession from defendant whose possession of the property was inceptively lawful by virtue of a contract (express or implied) with the plaintiff, but became illegal when he continued his possession despite the termination of his right thereunder.[10] Petitioner s complaint for unlawful detainer in Civil Case No. 3506-B-96 is properly within the competence of the MTCC. His pertinent allegations in the complaint read: 4. That defendants (spouses) have constructed an extension of their residential house as well as other structures and have been occupying a portion of the above PROPERTIES of the plaintiff for the past several years by virtue of the tolerance of the plaintiff since at the time he has no need of the property;

5. That plaintiff needed the property in the early part of 1996 and made demands to the defendants to vacate and turn over the premises as well as the removal (of) their structures found inside the PROPERTIES of plaintiff; that without any justifiable reasons, defendants refused to vacate the portion of the PROPERTIES occupied by them to the damage and prejudice of the plaintiff. 6. Hence, plaintiff referred the matter to the Office of the Barangay Captain of Matina Crossing 74-A, Davao City for a possible settlement sometime in the latter part of February 1996. The barangay case reached the Pangkat but no settlement was had. Thereafter, a Certification To File Action dated March 27, 1996 was issued x x x; x x x. [11] (underscoring ours) Verily, petitioner s allegations in his complaint clearly make a case for an unlawful detainer. We find no error in the MTCC assuming jurisdiction over petitioner s complaint. A complaint for unlawful detainer is sufficient if it alleges that the withholding of the possession or the refusal to vacate is unlawful without necessarily employing the terminology of the law.[12] Here, there is an allegation in petitioner s complaint that respondents occupancy on the portion of his property is by virtue of his tolerance. Petitioner s cause of action for unlawful detainer springs from respondents failure to vacate the questioned premises upon his demand sometime in 1996. Within one (1) year therefrom, or on November 6, 1996, petitioner filed the instant complaint. It bears stressing that possession by tolerance is lawful, but such possession becomes unlawful when the possessor by tolerance refuses to vacate upon demand made by the owner. Our ruling in Roxas vs. Court of Appeals[13] is applicable in this case: A person who occupies the land of another at the latter s tolerance or permission, without any contract between them, is necessarily bound by an implied promise that he will vacate upon demand, failing which, a summary action for ejectment is the proper remedy against him. WHEREFORE, the petition is GRANTED. The assailed Decision and Resolution of the Court of Appeals in CA-G.R. SP No. 47435 are hereby REVERSED and SET ASIDE. The Decision dated February 12, 1998 of the

Regional Trial Court, Branch 11, Davao City in Civil Case No. 25, 654-97, affirming the Decision dated July 31, 1997 of the Municipal Trial Court in Cities, Branch 2, Davao City in Civil Case No. 3506-B-96, is hereby REINSTATED. SO ORDERED. HEIRS OF DEMETRIO MELCHOR, represented by CLETO MELCHOR, petitioners, vs. JULIO MELCHOR, respondent. DECISION PANGANIBAN, J.: The Municipal Trial Court would not have jurisdiction over a purported unlawful detainer suit, if the complaint fails to allege jurisdictional facts. The Case Before us is a Petition for Review on Certiorari under Rule 45 of the Rules of Court, seeking to nullify the August 16, 2001 Decision and the October 18, 2001 Resolution of the Court of Appeals (CA) in CA-GR SP No. 63465. The dispositive portion of the assailed Decision is as follows: WHEREFORE, premises considered, the present petition is hereby DENIED DUE COURSE and accordingly DISMISSED, for lack of merit. The Joint Decision dated February 5, 2001 of the Regional Trial Court, Branch 20 of Cauayan, Isabela which embodied the assailed judgment in Civil Case No. 20-1125 and affirmed the Decision dated September 1, 2000 of the Municipal Trial Court of Cauayan, Isabela, dismissing the complaint for ejectment of the petitioners in Civil Case No. 2325, entitled Heirs of Demetrio Melchor represented by Cleto Melchor v. Julio Melchor, is hereby AFFIRMED and REITERATED. Costs against the petitioners. The assailed Resolution denied petitioners Motion for Reconsideration. The Facts

The facts of the case are narrated by the CA as follows: Petitioners, who are the heirs of DEMETRIO MELCHOR, claim to be the owners, by way of succession, of the subject property allegedly in possession of respondent JULIO MELCHOR. The subject property is a portion of the twenty (20) hectares of land registered in the name of PEDRO MELCHOR, evidenced by Original Certificate of Title No.I-6020 of the Registry of Deeds for Isabela. The said property was purchased by the late DEMETRIO MELCHOR from PEDRO MELCHOR, the deceased father of herein respondent JULIO MELCHOR. During the lifetime of the late DEMETRIO MELCHOR, a request for the approval of the Deed of Sale dated February 14, 1947 between DEMETRIO MELCHOR and PEDRO MELCHOR was made to the Secretary of Agriculture and Natural Resources on September 4, 1953, which was subsequently approved. Since February 14, 1947 up to the present, petitioners further allege that respondent has been occupying the subject property and has been harvesting crops thereon and using it for grassing cows and carabaos. A demand letter dated August 21, 1999 was allegedly sent by the petitioners to the respondent, demanding him to vacate and surrender the said property, but the latter refused. The disagreement reached the barangay authorities, which case was not amicably settled, resulting in the issuance of a certification to file action. Petitioners filed against respondent a complaint for ejectment before the MTC of Cauayan, Isabela which they subsequently refiled in their Second Amended Complaint, docketed as Civil Case No. 2325 and dated May 31, 2001, to accommodate additional allegations therein. For his part, the defendant (now respondent) in Civil Case No. 2325 principally raised the matter of ownership by alleging affirmative/special defenses, among others, that the parcel of land in possession of the defendant is registered in the name of ANTONIA QUITERAS, the deceased mother of the defendant, as per Transfer of Certificate of Title No. T274828 of the Registry of Deeds for Isabela, and that the same property is now owned by the defendant and his three (3) sisters and one (1) brother, having inherited the same from their late mother, ANTONIA QUITERAS.

The Decision dated September 1, 2000, which was penned by acting MTC Judge BERNABE B. MENDOZA, was rendered in favor of the respondent, the pertinent portions of which read: There is no allegation that plaintiffs have been deprived of the possession of the land by force, intimidation, threat, strategy or stealth. The dispossession was made in 1947. As such, ejectment is not the proper remedy. WHEREFORE, a judgment is hereby rendered dismissing the case. No pronouncement as to costs. SO ORDERED. On appeal, the Regional Trial Court, Branch 20 of Cauayan, Isabela, presided over by Executive Judge HENEDINO P. EDUARTE, rendered, together with another related complaint for ejectment, i.e., Civil Case No. 20-1126, the Joint Decision dated February 5, 2001, the decretal portion of which reads: WHEREFORE, judgment is hereby rendered: 1. Affirming the decision in Civil Case No. Br. 20-1126 entitled, Heirs of Liberato Lumelay, et al. vs. Heirs of Julio Melchor. Costs against the appellants. 2. Affirming the decision in Civil Case No. 201-1125, entitled, Heirs of Demetrio Melchor, et al. vs. Julio Melchor. Costs against the appellants. SO ORDERED. Ruling of the Court of Appeals Sustaining the Regional Trial Court (RTC), the CA ruled that petitioners had failed to make a case for unlawful detainer. It opined that the MTC had never acquired jurisdiction over the case, because there was no allegation

that the parties had entered into a contract -- express or implied -- or that there was possession by tolerance. Furthermore, the appellate court held that the proper remedy should have been a plenary action for recovery of possession, not a summary action for ejectment. Hence, this Petition. The Issue In their Memorandum, petitioners raised only one alleged error: The Court of Appeals committed a grave error when it ruled that the Second Amended Complaint does not allege a sufficient cause of action for x x x unlawful detainer. The Court s Ruling The Petition has no merit. Lone Issue: Sufficiency of the Complaint for Ejectment Petitioners filed a summary action for ejectment based on Rule 70 of the Rules of Court. Under Section 1 of the Rule, two separate remedies are available -- one for forcible entry and another for unlawful detainer. Petitioners maintain that while the Complaint does not support a cause of action for forcible entry, the allegations therein certainly indicate one for unlawful detainer. They add that they did not commit any jurisdictional infirmity in failing to allege prior physical possession, because that fact is not an element of unlawful detainer. We do not agree. Even if petitioners may be correct in saying that prior physical possession by the plaintiff need not be alleged in an action for unlawful detainer, the absence of such possession does not ipso facto make their Complaint sufficient to confer jurisdiction on the MTC.

In ejectment cases, the jurisdiction of the court is determined by the allegations of the complaint. The test for determining the sufficiency of those allegations is whether, admitting the facts alleged, the court can render a valid judgment in accordance with the prayer of the plaintiff. A review of the Second Amended Complaint of petitioners discloses these pertinent allegations: the absolute owner of the subject land was their father, Demetrio Melchor, who bought it on February 14, 1947 from respondent s father, Pedro Melchor; being the heirs of Demetrio Melchor, petitioners became the owners of the property by reason of succession; as such, they sent a formal demand letter to respondent, who had been using the property since February 14, 1947, for grazing cows and carabaos and for planting crops; and in that letter, they asked him to vacate and surrender the property, but he failed to do so. Accordingly, petitioners prayed for judgment ordering respondent to vacate the property and to pay P500,000, which represented the income earned from February 14, 1947 to the present, as well the costs of the suit. It is clear from the foregoing that the allegations in the Complaint failed to constitute a case for either forcible entry or unlawful detainer. These actions, which deal with physical or de facto possession, may be distinguished as follows: (1) In an action for forcible entry, the plaintiff must allege and prove that he was in prior physical possession of the premises until deprived thereof, while in illegal detainer, the plaintiff need not have been in prior physical possession; and (2) in forcible entry, the possession by the defendant is unlawful ab initio because he acquires possession by force, intimidation, threat, strategy, or stealth, while in unlawful detainer, possession is originally lawful but becomes illegal by reason of the termination of his right of possession under his contract with the plaintiff. In pleadings filed in courts of special jurisdiction, the special facts giving the court jurisdiction must be specially alleged and set out. Otherwise, the complaint is demurrable. As correctly held by the appellate court, [f]orcible entry must be ruled out as there was no allegation that the petitioners were denied possession of

the subject property through any of the means stated in Section 1, Rule 70 [of the Rules of Court]. Neither was unlawful detainer satisfactorily alleged. In determining the sufficiency of a complaint therefor, it is not necessary to employ the terminology of the law. Not averred in this case, however, were certain essential facts such as how entry was effected, or how and when dispossession started. Petitioners merely alleged their ownership of the land, which had supposedly been possessed by respondent since 1947. There was no allegation showing that his possession of it was initially legal -- by virtue of a contract, express or implied -- and that it became illegal after the expiration of his right to possess. Neither did the Complaint claim as a fact any overt act on the part of petitioners showing that they had permitted or tolerated respondent s occupancy of the subject property. It is a settled rule that in order to justify an action for unlawful detainer, the owner s permission or tolerance must be present at the beginning of the possession. Furthermore, the complaint must aver the facts showing that the inferior court has jurisdiction to try the case; for example, by describing how defendant s possession started or continued. The prayer of petitioners contradicts, however, the existence of possession by tolerance. It must be noted that they seek to be paid P500,000 as payment for the use of the property by respondent from 1947 to the present. This allegation implies that they never permitted him to possess the land. Since the Complaint did not satisfy the jurisdictional requirements of a valid cause for forcible entry or unlawful detainer, the appellate court was correct in holding that the MTC had no jurisdiction to hear the case. Verily, the failure of petitioners to properly allege a case for ejectment does not leave them without any other remedy. Under the proper circumstances, what may be filed is a case either for accion publiciana, which is a plenary action intended to recover the better right to possess; or an accion reivindicatoria, a suit to recover ownership of real property. This principle was laid down in Ong v. Parel as follows:

The jurisdictional facts must appear on the face of the complaint. When the complaint fails to aver facts constitutive of forcible entry or unlawful detainer, as where it does not state how entry was effected or how and when dispossession started, as in the case at bar, the remedy should either be an accion publiciana or an accion reivindicatoria in the proper regional trial court. If private respondent is indeed the owner of the premises subject of this suit and she was unlawfully deprived of the real right of possession or the ownership thereof, she should present her claim before the regional trial court in an accion publiciana or an accion reivindicatoria, and not before the municipal trial court in a summary proceeding of unlawful detainer or forcible entry. For even if one is the owner of the property, the possession thereof cannot be wrested from another who had been in the physical or material possession of the same for more than one year by resorting to a summary action for ejectment. This is especially true where his possession thereof was not obtained through the means or held under the circumstances contemplated by the rules on summary ejectment. WHEREFORE, the Petition is DENIED, and the assailed Decision AFFIRMED. Costs against petitioners. SO ORDERED. [G.R. No. 176995, July 30, 2008] PABLO D. ACAYLAR, JR., PETITIONER, VS. DANILO G. HARAYO, RESPONDENT. DECISION CHICO-NAZARIO, J.: Before this Court is a Petition for Review on Certiorari[1] under Rule 45 of the Revised Rules of Court filed by petitioner Pablo D. Acaylar, Jr., seeking the reversal and the setting aside of the Resolutions[2] dated 28 July 2006 and 30 January 2007 of the Court of Appeals in CA-G.R. SP No. 01077MIN. The appellate court, in its assailed Resolution dated 28 July 2006, dismissed petitioner's Petition for Review on Certiorari therein on technical

grounds; thus, it affirmed the Decision dated 20 January 2006 of the Regional Trial Court (RTC) of Dipolog City, Branch 9, in Civil Case No. 6087, which, in turn, affirmed the Decision[3] dated 28 March 2005 of the Municipal Trial Court in Cities (MTCC) of Dapitan City, in Civil Case No. 622, awarding possession of the subject property to respondent Danilo G. Harayo on the ground that he is the lawful possessor thereof. In its assailed Resolution dated 30 January 2007, the Court of Appeals refused to reconsider its earlier Resolution of 28 July 2006. The subject property is a parcel of land designated as Lot 741-B-1 situated in Tolon, Potungan, Dapitan City, with an area of 30,000 square meters, described and bounded as follows: Lot 741-B-1 of the Sketch Plan, situated at Tolon, Potungan, Dapitan City, containing an area of 30,000 square meters, bounded on the N., by Tolon River; on the South by Lot 741-A; on the E by Lot 741-B-2; and on the West by the Municipal Road, and embraced in OCT No. - (P-14969)-1119.[4] In his Complaint filed with the MTCC, and docketed as Civil Case No. 622, respondent alleged that he acquired the subject property from the spouses Pablo Acaylar, Sr., and Zoila Dangcalan Acaylar (the spouses Acaylar) by virtue of a Deed of Sale executed on 14 September 2004. On the same day, respondent took possession of the subject property. On 19 September 2004, one of the spouses Acaylar's sons, the petitioner, using strategy, intimidation, threats and stealth, entered the subject property, cut the tall grasses in the coconut plantation therein, gathered the fallen coconuts and other fruits, and pastured his cows and other animals thereon.[5] In his Answer, petitioner countered that the subject property claimed by respondent is a portion of the entire property owned by petitioner's parents, the spouses Acaylar, with a total area of 59,775 square meters. Petitioner is in possession of his parents' entire property since 1979 as administrator thereof. He built his house on the property and farmed the land. Respondent cannot definitively claim which portion of the entire property he was able to buy from the spouses Acaylar since the same was not clearly delineated.[6] In addition, petitioner, together with his sisters, Rosario Acaylar Herrera and Asteria Acaylar, already filed against respondent and his spouse Beatriz Harayo a case for annulment of the Deed of Sale dated 14 September 2004, with prayer for preliminary injunction and damages, presently pending before the RTC, Branch 6.

During the Pre-Trial Conference held before the MTCC on 17 February 2005, the parties stipulated that the spouses Acaylar sold to respondent only a 30,000-square-meter portion of their entire property; and that there is a pending civil case before the RTC on the validity of the sale of the subject property. Among the pieces of evidence presented by respondent before the MTCC was an Affidavit of Zoila Acaylar (First Affidavit) attesting that she sold the subject property to respondent for consideration and she did not give petitioner authority to either administer or remain on her and her husband's property. After trial, the MTCC rendered a Decision[7] on 28 March 2005, awarding to respondent the possession of the subject property. The MTCC gave credence to respondent's claim that he took immediate possession of the subject property after the execution of the Deed of Sale but was ousted therefrom by petitioner who invoked the alleged authority granted to him by Zoila Acaylar as the administrator of the unsold portion of her and her husband's property. The MTCC referred to the First Affidavit executed by Zoila Acaylar wherein she refuted that she gave petitioner authority or designated him as the administrator of her and her husband's property. Zoila Acaylar further admitted therein that the subject property was already sold to respondent. For lack of any legal right to remain on the subject property, the MTCC adjudged that petitioner's possession of the same was illegal. The dispositive portion of the MTCC Decision reads: WHEREFORE, judgment is hereby rendered, by preponderance of evidence in favor of the [herein respondent] as against the [herein petitioner], and hereby orders: (1) For [petitioner] and all other persons who may have derived rights from him to vacate lot 741-B-1 containing an area of 30,000 square meters as shown in the sketch plan prepared by Christopher Palpagan and turn over peaceful possession thereof to [herein respondent]; (2) For [petitioner] to pay [respondent] the amount of P5,000.00 as attorney's fees and P 1,591.25 as costs of the suit.

All other claims and counterclaims are hereby dismissed for lack of merit.[8] On appeal, docketed as Civil Case No. 6087, the RTC promulgated its Decision[9] dated 20 January 2006 affirming the award of possession in favor of respondent after finding that the appealed MTCC Decision was based on facts and law on the matter. The RTC declared that the sale of the subject property by the spouses Acaylar to respondent vested ownership and possession of said property in the latter. Thus, petitioner's acts of entering the subject property, cutting the tall grasses and gathering the agricultural products therein, constitute forcible entry, which gave rise to an action for ejectment. The RTC decreed: WHEREFORE, premises considered, [the RTC] finds by preponderance of evidence that [herein respondent] is in physical possession of the [subject property] that is on September 14, 2004 prior to the [herein petitioner] on September 19, 2004 and therefore affirms the decision of the Municipal Trial Court in the City of Dapitan without modification.[10] Banking on another Affidavit (Second Affidavit) executed by Zoila Acaylar, in which she recanted the statements she made in her First Affidavit denying that she designated petitioner as the administrator of her and her husband's property, petitioner moved for the reconsideration of the 20 January 2006 Decision of the RTC. The RTC, however, issued a Resolution[11] dated 18 April 2006 denying petitioner's Motion for Reconsideration. Consequently, petitioner filed a Petition for Review on Certiorari[12] with the Court of Appeals where it was docketed as CA-G.R. SP No. 01077-MIN. Petitioner argued in his Petition that the RTC gravely erred in ruling that respondent was in prior possession of the subject property based solely on the Deed of Sale executed by the spouses Acaylar in respondent's favor. Petitioner also asserted therein that the RTC gravely abused its discretion when it did not give credence to the Second Affidavit executed by Zoila Acaylar.[13] On 28 July 2006, the Court of Appeals issued a Resolution[14] dismissing outright CA-G.R. SP No. 01077-MIN for failure of petitioner to avail himself of the correct remedy under the law. Petitioner should have filed a Petition for Review under Rule 42 of the Revised Rules of Court, the proper remedy to appeal the adverse decisions rendered by the RTC in its appellate

capacity. Instead, petitioner erroneously filed a Petition for Review on Certiorari[15] to assail the 20 January 2006 Decision and 8 April 2006 Resolution of the RTC in Civil Case No. 6087. The Court of Appeals also noted non-compliance by petitioner and his counsel with several more requirements for filing a petition with the Court of Appeals, namely: (a) shortage in the payment of the docket fees; (b) failure of petitioner's counsel to indicate the place of issue of his Integrated Bar of the Philippines (IBP) number and his complete address; (3) failure of petitioner to furnish the appellate court which rendered the assailed decision, in this case the RTC, a copy of the Petition; and (4) failure of the Petition to state the material dates. The Court of Appeals, in a Resolution[16] dated 30 January 2007, denied for lack of merit the Motion for Reconsideration interposed by petitioner. The appellate court, however, excused the mistake of petitioner in the designation of the pleading as a Petition for Review on Certiorari, since it was clear from petitioner's Motion for Extension to file Petition for Review that he wished to avail himself of the remedy provided under Rule 42 of the Revised Rules of Court. Petitioner is now before this Court via the Petition at bar, making the following assignment of errors: I. THE HONORABLE COURT OF APPEALS GRAVELY ERRED IN DENYING THE PETITION DESPITE ADEQUATE EXPLANATION SUBMITTED BY THE PETITIONER ON THE TECHNICALITIES ASSIGNED TO THE PETITIONER; II. THE HONORABLE COURT OF APPEALS GRAVELY ERRED IN READING SHORT THE GIST OF THE PETITION WHEN IT RULED THAT SPECIFIC MATTERS INVOLVED IN THE CASE WERE INDICATED IN THE PETITION; III. THE HONORABLE COURT OF APPEALS GRAVELY ERRED IN RULING THAT ANNEXES WERE NOT ATTACHED WHEN THEY ARE DULY ATTACHED;

IV. THE HONORABLE COURT OF APPEALS GRAVELY ERRED IN FAILING TO EVALUATE THE PROPRIETY (SIC) FORCIBLE ENTRY CASE WHICH IS THE ORIGINAL ACTION INVOLVED IN THIS CASE VIS- -VIS UNLAWFUL DETAINER.[17] The Court first addresses the procedural issues involved in the present case. The Court of Appeals pointed several procedural defects of petitioner's Petition for Review therein. Petitioner's payment of docket fees was short of P500.00. It is also evident after a perusal of the records that petitioner failed to indicate in his Petition with the Court of Appeals the material dates to establish when he received notice of the assailed RTC Decision and when he filed his motion for reconsideration thereof with the RTC, as required by Section 2, Rule 42[18] of the Revised Rules of Court. Petitioner further failed to set forth concisely a statement of the matters involved in the case in accordance with the same provision. Finally, petitioner did not furnish the RTC, the court which rendered the assailed decision, a copy of the Petition he filed with the Court of Appeals.[19] Petitioner, however, submits that he raised meritorious arguments in his Petition with the Court of Appeals and, thus, the dismissal thereof on a mere technicality would cause a miscarriage of justice. The petitioner invokes considerations of substantial justice and prays that this Court give his Petition due course and set aside the Court of Appeals Resolutions dated 28 July 2006 and 30 January 2007 in CA-G.R. SP No. 01077-MIN. Respondent counters that the Court of Appeals did not commit any reversible error in dismissing the Petition in CA-G.R. SP No. 01077-MIN and adopted the discussion of the appellate court in his Memorandum. In appealed cases, failure to pay the docketing fees does not automatically result in the dismissal of the appeal; the dismissal is discretionary on the part of the appellate court.[20] Section 5, Rule 141 of the Revised Rules of Court provides that "If the fees are not paid, the court may refuse to proceed with the action until they are paid and may dismiss the appeal or the action or proceedings." Petitioner explained in his Motion for

Reconsideration before the Court of Appeals that he relied in good faith on the computation provided by the Clerk of Court of Zamboanga with whom he inquired as regards the amount of docket fees due. He had previously paid P4,030.00 and was short of only P500.00, which he also immediately paid upon being informed of the deficiency. Given the circumstances, petitioner should have been granted leniency by the Court of Appeals on this matter. We also agree with the petitioner that failure to state the material dates is not fatal to his cause of action, provided the date of his receipt, i.e., 9 May 2006, of the RTC Resolution dated 18 April 2006 denying his Motion for Reconsideration is duly alleged in his Petition.[21] In the recent case of Great Southern Maritime Services Corporation v. Acua,[22] we held that "the failure to comply with the rule on a statement of material dates in the petition may be excused since the dates are evident from the records." The more material date for purposes of appeal to the Court of Appeals is the date of receipt of the trial court's order denying the motion for reconsideration.[23] The other material dates may be gleaned from the records of the case if reasonably evident.[24] Likewise excusable is petitioner's failure to strictly follow the required form for presenting the facts and law of his case before the Court of Appeals. His Petition before the appellate court consists of only five pages, presenting concisely enough the facts and law supporting his case. With respect to petitioner's failure to furnish the RTC a copy of his Petition with the Court of Appeals, this Court found upon examination of the records that petitioner had already complied with such requirement.[25] Accordingly, the parties are now given the amplest opportunity to fully ventilate their claims and defenses brushing aside technicalities in order to truly ascertain the merits of this case. Indeed, judicial cases do not come and go through the portals of a court of law by the mere mandate of technicalities.[26] Where a rigid application of the rules will result in a manifest failure or miscarriage of justice, technicalities should be disregarded in order to resolve the case. In Aguam v. Court of Appeals,[27] we ruled that:

The court has [the] discretion to dismiss or not to dismiss an appellant's appeal. It is a power conferred on the court, not a duty. The "discretion must be a sound one, to be exercised in accordance with the tenets of justice and fair play, having in mind the circumstances obtaining in each case." Technicalities, however, must be avoided. The law abhors technicalities that impede the cause of justice. The court's primary duty is to render or dispense justice. "A litigation is not a game of technicalities." "Law suits, unlike duels, are not to be won by a rapier's thrust. Technicality, when it deserts its proper office as an aid to justice and becomes its great hindrance and chief enemy, deserves scant consideration from courts." Litigations must be decided on their merits and not on technicality. Every party litigant must be afforded the amplest opportunity for the proper and just determination of his cause, free from the unacceptable plea of technicalities. Thus, dismissal of appeals purely on technical grounds is frowned upon where the policy of the court is to encourage hearings of appeals on their merits and the rules of procedure ought not to be applied in a very rigid, technical sense; rules of procedure are used only to help secure, not override substantial justice. It is a far better and more prudent course of action for the court to excuse a technical lapse and afford the parties a review of the case on appeal to attain the ends of justice rather than dispose of the case on technicality and cause a grave injustice to the parties, giving a false impression of speedy disposal of cases while actually resulting in more delay, if not a miscarriage of justice. In this case, the Court finds that petitioner's procedural lapses are forgivable and opts to dispose the instant Petition on its merits rather than remand the case to the appellate court, a remand not being necessary where, as in the instant case, the ends of justice would not be served thereby and we are already in a position to resolve the dispute based on the records before us. We now proceed to discuss the merits of the case. Relevant in the case at bar is Section 1, Rule 70 of the Revised Rules of Court which provides: SECTION 1. Who may institute proceedings, and when. - Subject to the provisions of the next succeeding section, a person deprived of the possession of any land or building by force, intimidation, threat, strategy,

or stealth, or a lessor, vendor, vendee, or other person against whom the possession of any land or building is unlawfully withheld after the expiration or termination of the right to hold possession, by virtue of any contract, express or implied, or the legal representatives or assigns of any such lessor, vendor, vendee, or other person, may, at any time within one (1) year after such unlawful deprivation or withholding of possession, bring an action in the proper Municipal Trial Court against the person or persons unlawfully withholding or depriving of possession, or any person or persons claiming under them, for the restitution of such possession, together with damages and costs. Under the above provision, there are two entirely distinct and different causes of action, to wit: (1) a case for forcible entry, which is an action to recover possession of a property from the defendant whose occupation thereof is illegal from the beginning as he acquired possession by force, intimidation, threat, strategy or stealth; and (2) a case for unlawful detainer, which is an action for recovery of possession from defendant whose possession of the property was inceptively lawful by virtue of a contract (express or implied) with the plaintiff, but became illegal when he continued his possession despite the termination of his right thereunder.[28] The distinctions between the two forms of ejectment suits, are: first, in forcible entry, the plaintiff must prove that he was in prior physical possession of the premises until he was deprived thereof by the defendant, whereas, in unlawful detainer, the plaintiff need not have been in prior physical possession; second, in forcible entry, the possession of the land by the defendant is unlawful from the beginning as he acquires possession thereof by force, intimidation, threat, strategy or stealth, while in unlawful detainer, the possession of the defendant is inceptively lawful but it becomes illegal by reason of the termination of his right to the possession of the property under his contract with the plaintiff; third, in forcible entry, the law does not require a previous demand for the defendant to vacate the premises, but in unlawful detainer, the plaintiff must first make such demand, which is jurisdictional in nature.[29] The above distinctions, more importantly the nature of defendant's entry into the property, are material to the present case in order to ascertain the propriety of respondent's action for forcible entry filed before the MTCC. It bears to stress that it is the nature of defendant's entry into the land which

determines the cause of action, whether it is forcible entry or unlawful detainer. If the entry is illegal, then the action which may be filed against the intruder is forcible entry. If, however, the entry is legal but the possession thereafter becomes illegal, the case is unlawful detainer.[30] In the case at bar, respondent filed an action for forcible entry before the MTCC. Respondent alleged that he took possession of the subject property immediately after the spouses Acaylar executed a Deed of Sale thereof in his favor on 14 September 2004, but was forcibly deprived thereof by petitioner. A case for forcible entry, therefore, is proper since petitioner's entry into the subject property is already illegal at its incipience. Petitioner, on the other hand, harps on the fact that he was in possession of the subject property since 1979, having built his house thereon and farmed the land, and it was impossible for him to wrest possession of the subject property from respondent, for he was already occupying the same way before its alleged sale to respondent. Petitioner, thus, maintains that his possession over the subject property is lawful from the start, as he was authorized by Zoila Acaylar to administer the same, making respondent's suit for forcible entry before the MTCC the wrong remedy. In a long line of cases,[31] this Court reiterated that the fact of prior physical possession is an indispensable element in forcible entry cases. The plaintiff must prove that he was in prior physical possession of the premises long before he was deprived thereof by the defendant.[32] It must be stressed that plaintiff cannot succeed where it appears that, as between himself and the defendant, the latter had possession antedating his own. To ascertain this, it is proper to look at the situation as it existed long before the first act of spoliation occurred in order to intelligibly determine whose position is more in accord with the surrounding circumstances of the case and the applicable legal principles. Such determination in this case requires a review of factual evidence, generally proscribed in a petition like this. However, where the factual findings of the courts a quo are contrary to each other, this Court may intervene to resolve the conflict and settle the factual issues raised by the parties.[33] In the instant Petition, the MTCC cited Zoila Acaylar's First Affidavit in which she attested that she did not appoint or designate petitioner as

administrator of her and her husband's property, and that she gathered the coconuts and harvested other crops from the property by employing farm workers. Since petitioner was never in possession of the subject property, then the MTCC concluded that respondent had taken possession of the same from the spouses Acaylar right after its purchase. The RTC, on the other hand, expressly recognized that petitioner possessed the subject property, but his possession was merely tolerated by his parents, and that respondent, as purchaser of the subject property from the parents, the spouses Acaylar, had better right to the possession of the same. Thus, as to whether petitioner had actual or physical possession of the subject property prior to respondent is a factual issue which we are called upon to resolve, considering that the courts below had contradicting findings. After careful and thorough recalibration and re-examination of the evidence available on record, we find that petitioner had physical possession of the subject property prior to and at the time of its sale by the spouses Acaylar to respondent. It is actually irrelevant whether petitioner possessed the subject property as the administrator thereof. As the son of the spouses Acaylar, he could very well enter into possession of the subject property either with the express permission or at the tolerance of his parents who owned the property. Petitioner alleged, and respondent did not dispute, that petitioner had entered into possession of his parents' property as early as 1979, and he even built his house thereon. Although Zoila Acaylar may have attested in her First Affidavit that she did not appoint or designate petitioner as the administrator of her and her husband's property, she never claimed that petitioner unlawfully or illegally entered her property when he built his house thereon. We are not persuaded by respondent's assertion that after he took possession of the subject property from the Zoila spouses, petitioner entered the subject property on a whim, for not only does such postulation lack clear, positive, and convincing evidentiary support, but also because it is illogical and contrary to common human experience. A person would not, for a reason so shallow as a whim, encroach upon another's property and gather fruits and other agricultural products therefrom, thereby risking criminal prosecution and civil liabilities. The more plausible and logical scenario would be that petitioner was already occupying the subject property prior to the sale. Petitioner, in gathering the coconut fruits and

other crops, cutting grasses, and domesticating animals on the subject property, even after its sale to respondent on 14 September 2004, was only continuing to exercise acts of possession over the subject property as he had done in years before. Moreover, we note that the subject property was sold to respondent and he supposedly took possession thereof on 14 September 2004; and that petitioner allegedly forced his way into the property on 19 September 2004. This would mean that respondent, after taking over possession of the subject property from petitioner's parents, possessed the subject property for only five days before being deprived thereof by the petitioner. The very short period when respondent purportedly possessed the subject property renders said possession suspect. It is not clear to us how petitioner took actual possession of the subject property on 14 September 2004. Neither are we enlightened on the manner in which respondent exercised or demonstrated his physical or material possession over the subject property for the five days before he was reputedly ousted therefrom by petitioner. Both the MTCC and the RTC decided in favor of petitioner since they considered him to have been vested with possession of the subject property by virtue of the execution of the Deed of Sale on 14 September 2004. However, such a ruling violates one of the most basic doctrines in resolving ejectment cases. We had long settled that the only question that the courts must resolve in ejectment proceedings is - who is entitled to the physical or material possession of the property, that is, possession de facto; and they should not involve the question of ownership or of possession de jure, which is to be settled in the proper court and in a proper action.[34] As we elucidated in the recent case of Sudaria v. Quiambao[35]: Regardless of the actual condition of the title to the property, the party in peaceable quiet possession shall not be thrown out by a strong hand, violence or terror. Neither is the unlawful withholding of property allowed. Courts will always uphold respect for prior possession. Thus, a party who can prove prior possession can recover such possession even against the owner himself. Whatever may be the character of his possession, if he has in his favor prior possession in time, he has the

security that entitles him to remain on the property until a person with a better right lawfully ejects him. To repeat, the only issue that the court has to settle in an ejectment suit is the right to physical possession. Hence, the Deed of Sale conferring ownership of the subject property upon respondent is clearly irrelevant in the case presently before us. The Deed of Sale did not automatically place respondent in physical possession of the subject property. It is thus incumbent upon respondent to establish by evidence that he took physical possession of the subject property from the spouses Acaylar on 14 September 2004 and he was in actual possession of the said property when petitioner forcibly entered the same five days later. The conflicting Affidavits of Zoila Acaylar, notwithstanding, we find that petitioner was in peaceful possession of the subject property prior to its sale to respondent. Even if petitioner was not authorized by Zoila Acaylar to possess the subject property as administrator, his possession was not opposed and was, thus, tolerated by his parents. As we ruled in Arcal v. Court of Appeals[36]: The rule is that possession by tolerance is lawful, but such possession becomes unlawful upon demand to vacate made by the owner and the possessor by torelance refuses to comply with such demand. A person who occupies the land of another at the latter's tolerance or permission, without any contract between them, is necessarily bound by an implied promise that he will vacate upon demand, failing which, a summary action for ejectment is the proper remedy against him. The status of the possessor is analogous to that of a lessee or tenant whose term of lease has expired but whose occupancy continued by tolerance of the owner. In such case, the unlawful deprivation or withholding of possession is to be counted from the date of the demand to vacate. In the instant case, there is no showing that either Zoila Acaylar or respondent made an express demand upon petitioner to vacate the subject property. In the absence of an oral or written demand, petitioner's possession of the subject property has yet to become unlawful. The absence of demand to vacate precludes us from treating this case, originally instituted as one for forcible entry, as one of unlawful detainer, since demand to vacate is jurisdictional in an action for unlawful detainer.[37] In conclusion, since petitioner was in prior physical possession of the

subject property, respondent has no cause of action against petitioner for forcible entry. Neither can we treat respondent's case against petitioner as one for unlawful detainer absent the jurisdictional requirement of demand to vacate made upon petitioner. However, our dismissal of respondent's Complaint herein against petitioner is without prejudice to respondent's filing of the appropriate remedy under the law to acquire possession of the subject property, as well as to the resolution of the civil case pending with the RTC, Branch 6, for the annulment of the Deed of Sale dated 14 September 2004. WHEREFORE, premises considered, the instant Petition is GRANTED. The Decision dated 28 July 2006 of the Court of Appeals and its Resolution dated 30 January 2007 in CA-G.R. SP No. 01077-MIN are REVERSED and SET ASIDE, and the Complaint of respondent Danilo G. Harayo against petitioner Pablo D. Acaylar before the Municipal Trial Court in Cities of Dapitan City, in Civil Case No. 622, is DISMISSED, without prejudice. No costs. SO ORDERED. HEIRS OF ANACLETOG.R. No. 150654 B. NIETO, namely, SIXTA P. NIETO, EULALIO P.Present: NIETO, GAUDENCIO P. NIETO, and CORAZON P.YNARES-SANTIAGO, J., NIETO-IGNACIO, represented by Chairperson, EULALIO P. NIETO, Petitioners, - versus AUSTRIA-MARTINEZ, CHICO-NAZARIO,

NACHURA, and MUNICIPALITY OF MEYCAUAYAN, BULACAN,REYES, JJ. represented by MAYOR EDUARDO ALARILLA, Promulgated: December 13, 2007

Respondent. x--------------------------------------------------------------x DECISION

NACHURA, J.: This is a petition for review on certiorari of the Decision[1] of the Court of Appeals, dated October 30, 2001, which dismissed the petition for review of the Decision of the Regional Trial Court (RTC) of Malolos, Bulacan. The latter dismissed a complaint to recover possession of a registered land on the ground of prescription and laches. The antecedents are as follows: Anacleto Nieto was the registered owner of a parcel of land, consisting of 3,882 square meters, situated at Poblacion, Meycauayan, Bulacan and covered by TCT No. T-24.055 (M). The property is being used by respondent, Municipality of Meycauayan, Bulacan, which constructed an extension of the public market therein. Upon Anacleto s death on July 26, 1993, his wife, Sixta P. Nieto, and their three children, namely, Eulalio P. Nieto, Gaudencio Nieto and Corazon Nieto-Ignacio, herein petitioners, collated all the documents pertaining to his estate. When petitioners failed to locate the owner s duplicate copy of TCT No. T-24.055 (M), they filed a petition for the issuance of a second owner s copy with the RTC, Malolos, Bulacan. In that case, petitioners discovered that the missing copy of the title was in the possession of the respondent. Consequently, petitioners withdrew the petition and demanded from respondent the return of property and the certificate of title. On February 23, 1994, petitioners formally demanded from respondent the return of the possession and full control of the property, and payment of a monthly rent with interest from January 1964. Respondent did not comply with petitioners demand.[2]

On December 28, 1994, petitioners filed a complaint[3] for recovery of possession and damages against respondent alleging that the latter was in possession of the owner s copy of TCT No. T-24.055 (M). They averred that, in 1966, respondent occupied the subject property by making it appear that it would expropriate the same. Respondent then used the land as a public market site and leased the stalls therein to several persons without paying Anacleto Nieto the value of the land or rent therefor. Petitioners prayed that respondent be ordered to surrender to them the owner s copy of TCT No. T-24.055 (M), vacate the property, and pay them the rents thereon from 1966 until the date of the filing of the complaint for the total of P1,716,000.00, and P10,000.00 a month thereafter, as well as P300,000.00 as moral damages, and P100,000.00 as attorney s fees. In its Answer,[4] respondent alleged that the property was donated to it and that the action was already time-barred because 32 years had elapsed since it possessed the property. Respondent and counsel failed to appear during the scheduled pre-trial conference.[5] Upon petitioners motion, respondent was declared as in default and petitioners were allowed to present evidence ex parte. Respondent filed a motion for reconsideration which the RTC granted. Respondent was then allowed to cross-examine petitioners lone witness and present its own evidence. However, despite notice, respondent failed again to appear during the scheduled hearing. Hence, the RTC considered respondent to have waived its right to cross-examine petitioners witness and present its own evidence. The case was then submitted for decision. On August 1, 1995, the RTC rendered a Decision dismissing the complaint as well as respondent s counterclaims for damages. For lack of proof, the RTC disregarded respondent s claim that Anacleto Nieto donated the property to it in light of the fact that the title remained in the name of Anacleto. Nonetheless, the RTC did not rule in favor of petitioners because of its finding that the case was already barred by prescription. It held that the imprescriptibility of actions to recover land covered by the Torrens System could only be invoked by the registered owner, Anacleto Nieto, and that the action was also barred by laches.

Petitioners appealed the case to the Court of Appeals (CA). On October 30, 2001, the CA rendered a Decision dismissing the case for lack of jurisdiction. According to the CA, the petition involved a pure question of law; hence, petitioners should have filed a petition directly with this Court.[6] Accordingly, petitioners elevated the case to this Court through a petition for review on certiorari, raising the following issues: A. Are lands covered by the Torrens System subject to prescription? B. May the defense of [l]aches be invoked in this specific case? C. May the defense of imprescriptibility only be invoked by the registered owner to the exclusion of his legitimate heirs?[7] The petition is meritorious. Respondent argues that the action of petitioner to recover possession of the property is already barred by prescription. We do not agree. An action to recover possession of a registered land never prescribes in view of the provision of Section 44 of Act No. 496 to the effect that no title to registered land in derogation of that of a registered owner shall be acquired by prescription or adverse possession.[8] It follows that an action by the registered owner to recover a real property registered under the Torrens System does not prescribe.

Despite knowledge of this avowed doctrine, the trial court ruled that petitioners cause of action had already prescribed on the ground that the imprescriptibility to recover lands registered under the Torrens System can only be invoked by the person under whose name the land is registered. Again, we do not agree. It is well settled that the rule on imprescriptibility of registered lands not only applies to the registered owner but extends to the heirs of the registered owner as well.[9] Recently in Mateo v. Diaz,[10]

the Court held that prescription is unavailing not only against the registered owner, but also against his hereditary successors because the latter step into the shoes of the decedent by operation of law and are the continuation of the personality of their predecessor-in-interest. Hence, petitioners, as heirs of Anacleto Nieto, the registered owner, cannot be barred by prescription from claiming the property. Aside from finding that petitioners cause of action was barred by prescription, the trial court reinforced its dismissal of the case by holding that the action was likewise barred by laches.

Laches has been defined as the failure or neglect, for an unreasonable and unexplained length of time, to do that which, by exercising due diligence could or should have been done earlier. It is negligence or omission to assert a right within a reasonable time, warranting the presumption that the party entitled to assert his right has either abandoned or declined to assert it.[11]

In a number of cases, the Court has held that an action to recover registered land covered by the Torrens System may not be barred by laches.[12] Laches cannot be set up to resist the enforcement of an imprescriptible legal right.[13] Laches, which is a principle based on equity, may not prevail against a specific provision of law, because equity, which has been defined as justice outside legality, is applied in the absence of and not against statutory law or rules of procedure.[14] In recent cases, [15] however, the Court held that while it is true that a Torrens title is indefeasible and imprescriptible, the registered landowner may lose his right to recover possession of his registered property by reason of laches. Yet, even if we apply the doctrine of laches to registered lands, it would still not bar petitioners claim. It should be stressed that laches is not concerned only with the mere lapse of time.[16] The following elements must be present in order to constitute laches:

(1) conduct on the part of the defendant, or of one under whom he claims, giving rise to the situation of which complaint is made for which the complaint seeks a remedy; (2) delay in asserting the complainant s rights, the complainant having had knowledge or notice, of the defendant s conduct and having been afforded an opportunity to institute a suit; (3) lack of knowledge or notice on the part of the defendant that the complainant would assert the right on which he bases his suit; and (4) injury or prejudice to the defendant in the event relief is accorded to the complainant, or the suit is not held to be barred.[17] We note that the certificate of title in the name of Anacleto Nieto was found in respondent s possession but there was no evidence that ownership of the property was transferred to the municipality either through a donation or by expropriation, or that any compensation was paid by respondent for the use of the property. Anacleto allegedly surrendered the certificate of title to respondent upon the belief that the property would be expropriated. Absent any showing that this certificate of title was fraudulently obtained by respondent, it can be presumed that Anacleto voluntarily delivered the same to respondent. Anacleto s delivery of the certificate of title to respondent could, therefore, be taken to mean acquiescence to respondent s plan to expropriate the property, or a tacit consent to the use of the property pending its expropriation. This Court has consistently held that those who occupy the land of another at the latter s tolerance or permission, without any contract between them, are necessarily bound by an implied promise that the occupants will vacate the property upon demand.[18] The status of the possessor is analogous to that of a lessee or tenant whose term of lease has expired but whose occupancy continues by tolerance of the owner. In such case, the unlawful deprivation or withholding of possession is to be counted from the date of the demand to vacate.[19] Upon the refusal to vacate the property, the owner s cause of action accrues. In this case, the first element of laches occurred the moment respondent refused to vacate the property, upon petitioners demand, on February 23,

1994. The filing of the complaint on December 28, 1994, after the lapse of a period of only ten months, cannot be considered as unreasonable delay amounting to laches.

Moreover, case law teaches that if the claimant s possession of the land is merely tolerated by its lawful owner, the latter s right to recover possession is never barred by laches. Even if it be supposed that petitioners were aware of respondent s occupation of the property, and regardless of the length of that possession, the lawful owners have a right to demand the return of their property at any time as long as the possession was unauthorized or merely tolerated, if at all.[20] Furthermore, the doctrine of laches cannot be invoked to defeat justice or to perpetrate fraud and injustice. It is the better rule that courts, under the principle of equity, will not be guided or bound strictly by the statute of limitations or the doctrine of laches when by doing so, manifest wrong or injustice would result.[21] Finally, we find that the rentals being prayed for by petitioners are reasonable considering the size and location of the subject property. Accordingly, the award of rentals is warranted. WHEREFORE, premises considered, the petition is GRANTED. The Decision of the Regional Trial Court of Malolos, Bulacan, dated August 1, 1995, is REVERSED and SET ASIDE. Respondent is ORDERED (a) to vacate and surrender peaceful possession of the property to petitioners, or pay the reasonable value of the property; (b) to pay P1,716,000.00 as reasonable compensation for the use of the property from 1966 until the filing of the complaint and P10,000.00 monthly rental thereafter until it vacates the property, with 12% interest from the filing of the complaint until fully paid; and (c) to return to petitioners the duplicate copy of TCT No. T-24.055 (M).

SO ORDERED.

EQUATORIAL REALTY DEVELOPMENT, INC., petitioner, vs. MAYFAIR THEATER, INC., respondent. DECISION PARDO, J.: Ky-calr Before us is an appeal from the decision of the Court of Appeals dismissing the petition for certiorari and prohibition initiated by petitioner and the resolution modifying the aforementioned decision, thus: "WHEREFORE, the Decision of this Court of March 24, 1998 is hereby modified, thus"1. Mayfair to deposit with the Clerk of Court of Manila the amount of P847,000.00 in addition to the P10,452,500.00 already deposited therein, within ten (10) days from receipt hereof; "2. The Clerk of Court of Manila, to turn over to petitioner Equatorial the full amount of P11,300,000.00, within ten (10) days from completion of said amount by Mayfair. "SO ORDERED." The facts are as follows: On November 21, 1996, the Supreme Court promulgated its decision in Equatorial Realty Development, Inc. and Carmelo and Bauermann, Inc. vs. Mayfair Theater, Inc., the decretal portion of which reads: "WHEREFORE, the petition for review of the decision of the Court of Appeals, dated June 23, 1992, in CA-G. R. CV No. 32918, is HEREBY DENIED. The Deed of Absolute Sale between petitioners Equatorial Realty Development, Inc. and Carmelo & Bauermann, Inc. is hereby deemed rescinded; petitioner Carmelo and Bauermann is ordered to return to petitioner Equatorial Realty Development the purchase price. The latter is directed to execute the deeds and documents necessary to return ownership to Carmelo & Bauermann of the disputed lots.

Carmelo & Bauermann is ordered to allow Mayfair Theater, Inc. to buy the lots for P11,300,000.00. "SO ORDERED." In due time, Equatorial filed a motion for reconsideration of the above decision. On January 28, 1997, the Court denied the motion with finality. A second motion for reconsideration filed by Equatorial was denied on March 4, 1997. Calr-ky On March 17, 1997, the decision became final and executory. Equatorial filed a third motion for reconsideration, and on April 22, 1997, the Court noted the same without action since the decision had become final and executory. To stress the finality of the decision, on June 17, 1997, the Court issued a resolution emphasizing its finality and warning the parties that no further pleadings or motions regarding the matter would be entertained. On April 25, 1997, Mayfair Theater, Inc. (hereinafter Mayfair) filed with the Regional Trial Court, Manila, Branch 07 a motion for execution. On May 20, 1997, the trial court granted respondent s motion for execution, as follows: "WHEREFORE, acting on Plaintiff s Motion dated, April 24, 1997, let a Writ of Execution be issued "1. ORDERING defendant CARMELO and BAUERMANN, INC. to return within 10 days to Defendant EQUATORIAL REALTY and DEVELOPMENT the amount of P11,300,000.00 the total purchase price of properties covered by: a.......T.C.T. No. 130410, formerly T.C.T. No. 17350; b.......T.C.T. No. 130407, formerly T.C.T. No. 118612; c.......T.C.T. No. 130408, formerly T.C.T. No. 60936; and d.......T.C.T. No. 130409, formerly T.C.T. No. 52571;

subject of sale date, July 30, 1978 which the Court of Appeals in CA-GR CV No. 32918 ordered rescinded, and to execute another Deed of transfer over said properties in favor of Plaintiff, MAYFAIR THEATER, INC.; "2. ORDERING MAYFAIR THEATER, INC. to pay Defendant CARMELO BAUERMANN INC. the amount of Eleven Million Three Hundred Thousand Pesos (P11,300,000.00), Philippine Currency upon the latter s execution of such Deed of Transfer; "3. ORDERING Defendant EQUATORIAL REALTY DEVELOPMENT to accept this Eleven Million Three Hundred Thousand Pesos (P11,300,000.00), Philippine Currency from defendant CARMELO and BAUERMANN, INC. and to execute also in 10 days time "the deeds and documents necessary to return ownership of these properties to CARMELO and BAUERMANN, INC." Me-sm "4. ORDERING both Plaintiff and Defendants to submit thereafter their corresponding manifestations of compliance. "SO ORDERED." On the same date, the trial court issued a writ of execution. On May 21, 1997, Sheriff IV Manuelito P. Viloria of the trial court issued to Carmelo and Bauermann a notice to comply with the trial court s order and writ of execution. However, Carmelo did not receive both the writ and notice to comply since it could not be located. On June 9, 1997, Equatorial filed a motion for reconsideration of the order dated May 20, 1997, the recall and/or quashal of the writ of execution and the notice to comply, on the ground that the order of execution did not conform to the dispositive portion of the Supreme Court s decision dated November 21, 1996. Equatorial specifically averred the following variance: "a. The ten (10)- day period given by the Court to defendant Carmelo & Bauermann, Inc. (Carmelo, for brevity) within which to return the purchase price to Equatorial, and the same period

given to Equatorial to execute the documents necessary to return ownership of the subject properties to Carmelo, do not appear in the dispositive portion of the Supreme Court Decision; "b. The order of execution makes reference to TCT No. 130410, TCT No. 130407, TCT No. 130408, and TCT No. 130409, which are not mentioned in the Supreme Court Decision; "c. The order of execution directs defendant Carmelo to execute a deed of transfer over the subject properties in favor of plaintiff, while the Supreme Court decision merely orders Carmelo to allow plaintiff to buy the same for P11,300,000.00; "d. The order of execution orders plaintiff to pay Carmelo the amount of P11,300,000.00 upon the latter s execution of such deed of transfer, but no such directive appears in the dispositive portion of the Supreme Court Decision; "e. The order of execution directs Equatorial to accept the amount of P11,300,000.00 from Carmelo, but this directive is not contained in the dispositive portion of the Supreme Court Decision; and S-l-x "f. The dispositive portion of the Supreme Court decision refers to disputed lots , while the order of execution refers to properties ." On August 25, 1997, the trial court denied Equatorial s motion for reconsideration, the dispositive portion of the order reads: "WHEREFORE, the Motion for Reconsideration filed by defendant Equatorial Realty Development, Inc. is hereby denied for lack of merit. Pursuant to Section 10 (a), Rule 39 of the 1997 Rules of Civil Procedure, Acting Clerk of Court, Atty. Jennifer N. dela Cruz-Buendia, who was directed by the Hon. Roberto A. Barrios, Executive Judge to immediately assume and perform the duties and functions of Atty. Jesusa Maningas, Clerk of Court and Ex-Officio Sheriff of the Regional Trial Court

of Manila in view of the latter s indefinite leave of absence, is hereby appointed and directed to execute in behalf of defendant Equatorial Realty and Development, Inc. all the deeds and documents necessary to return ownership of the subject properties to Carmelo & Bauermann, Inc., and thereupon to execute on behalf of defendant Carmelo & Bauermann, Inc. a deed of transfer over the same properties in favor of plaintiff, upon receipt from plaintiff of the purchase price of P11,300,000.00 which the Office of the Clerk of Court shall thereafter hold in trust for defendant Carmelo or its order. All costs incidental to the execution of these conveyances shall be borne by defendants. Finally, the Register of Deeds for the City of Manila is hereby ordered to register the aforementioned deeds and documents of transfer executed by acting Clerk of Court Atty. Jennifer N. dela Cruz-Buendia of the regional Trial Court of Manila upon proof of payment of such fees and taxes as may be due, including documentary stamp and transfer taxes; to cancel in its registry TCT No. 130410, TCT No. 130407, TCT No. 130408 and TCT No. 130409, as well as the owner s duplicates thereof in the possession of defendant Equatorial; and to issue in lieu thereof corresponding new titles and owner s duplicates in the name of plaintiff Mayfair Theater, Inc." Thereafter, Mayfair deposited with the Clerk of Court, Regional Trial Court, Manila its payment to Carmelo, amounting to P10,452,500.00 (P11,300,000.00 less P847,000.00, as withholding tax). Sc-slx On August 27, 1997, pursuant to the aforesaid order, the acting clerk of court, Regional Trial Court, Manila executed a deed of re-conveyance of the subject property in favor of Carmelo, and a deed of absolute sale in favor of Mayfair. On August 28, 1997, both the deed of re-conveyance and deed of absolute sale were submitted to the Register of Deeds of Manila for registration. On the same date, the Register of Deeds of Manila registered the documents, cancelled the transfer certificates of title (TCTs) of Equatorial over the subject property, and issued new TCTs in the name of Mayfair.

On September 5, 1997, Equatorial filed with the trial court an urgent motion for reconsideration of the denial. Equatorial contended that the trial court erred in applying Section 10 (a) of Rule 39 since both Carmelo and Equatorial had not yet failed to comply with the order of execution, hence it was erroneous to designate the acting clerk of court to execute the deed of re-conveyance. In fact, Carmelo had not received the notice to comply. Equatorial maintained that the order of execution should not be implemented because it varied with the Supreme Court s November 21, 1996 decision. Equatorial stressed that since Carmelo had not returned the "purchase price" as ordered by the Court, it could not be compelled to execute the deed of re-conveyance in favor of Carmelo. On November 6, 1997, the trial court denied the motion for lack of merit. On December 16, 1997, Equatorial filed with the Court of Appeals, a petition for certiorari and prohibition seeking the annulment of the trial court s orders dated May 20, August 25, and November 6, 1997, the writ of execution dated May 20, 1997, the sheriff s notice to comply dated May 21, 1997, the deeds of re-conveyance and transfer, sale and the certificates of title in favor of Mayfair. Equatorial contended that the trial court acted with grave of discretion in issuing a writ of execution, which was at variance with the dispositive portion of the Court s decision. Equatorial averred that Carmelo s failure to receive the writ of execution and notice to comply was tantamount to denial of due process. Equatorial further stated that the trial court erred in applying Section 10 (a), Rule 39, 1997 Rules of Civil Procedure in issuing the writ of execution, since the parties did not fail to comply within the time specified. Equatorial also submitted that it would be grossly and unconscionably unjust, unfair and inequitable to compel it to accept P11,300,000.00 for land which was worth more than P400 million, due to the appreciation of the property or the depreciation of the peso. On March 24, 1998, the Court of Appeals rendered decision dismissing the petition, to wit: Sl-xsc

"WHEREFORE, the petition is hereby DISMISSED for being dilatory and for lack of merit. The challenged orders and acts of the public respondents and the questioned notices and processes, writ of execution, deeds of re-conveyance and absolute sale, and transfer certificates of title are validated and AFFIRMED. "SO ORDERED." On April 8, 1998, petitioner filed with the Court of Appeals a motion for reconsideration of the decision. On November 20, 1998, the Court of Appeals issued a resolution as follows: "WHEREFORE, the Decision of this Court of March 24, 1998 is hereby modified, thus "1. Mayfair to deposit with the Clerk of Court of Manila the amount of P847,000.00 in addition to the P10,452,500.00 already deposited therein, within ten (10) days from receipt hereof; "2. The Clerk of Court of Manila, to turn over to petitioner Equatorial the full amount of P11,300,000.00, within ten (10) days from completion of said amount by Mayfair. "SO ORDERED." The Court of Appeals explained that Mayfair had no right to deduct P847,000.00 as withholding tax which must be paid by Carmelo, as the seller. Carmelo must return the amount of P11,300,000.00 to Equatorial as a consequence of the rescission. However, since Carmelo could not be located, Mayfair must complete the full amount of P11,300,000.00 deposited with the Clerk of Court, Manila by depositing an additional P847,000.00. Thereafter, the full amount of P11,300,000.00 must be turned over to Equatorial. The Court of Appeals explained that the judge could not stand idly while Carmelo and Bauermann took time accepting the

P11,300,000.00 from Mayfair and returning the amount to Equatorial. Slxmis Hence, this petition for review assailing the decision of the Court of Appeals dated March 24, 1998, and its resolution dated November 20, 1998. Petitioner Equatorial contends that the Court of Appeals erred in upholding the applicability of Rule 39, Section 10 (a), 1997 Rules of Civil Procedure, since it did not fail to comply with the order, as it did not receive the writ of execution and notice to comply. Petitioner submits that the application of Rule 39, Section 10 (a) violated Carmelo and Bauermann s constitutional right to due process. Petitioner maintains that the writ of execution varies with the dispositive portion of the Supreme Court decision and cannot be implemented. Petitioner stresses that the Register of Deeds erred in cancelling its certificates of titles and issuing new titles to Mayfair, since Carmelo and Bauermann has not returned the purchase price. The money deposited by Mayfair with the Office of the Clerk of Court was for the account of Carmelo and cannot be considered as a sufficient tender of payment to Equatorial. The Supreme Court s decision in G. R. No. 106063 is clear. Having attained finality, there is nothing left for the parties to do but adhere to the mandates of the decision. It is a fundamental rule that when a judgment becomes final and executory, it thereby becomes immutable and unalterable and any amendment or alteration, which substantially affects a final and executory judgment, is null and void for lack of jurisdiction, including the entire proceedings held for that purpose. A writ of execution must conform to the judgment to be executed and adhere strictly to the very essential particulars. An order of execution, which varies the tenor of the judgment or exceeds the terms thereof is a nullity. The writ of execution "may not vary the terms of the judgment it seeks to enforce. Nor may it go beyond the terms of the judgment sought to be executed. Where the execution is not in harmony with the judgment

which gives it life, and in fact exceeds it, has pro tanto no validity. To maintain otherwise would be to ignore the constitutional provision against depriving a person of his property without due process of law." Having attained finality, the decision in G. R. No. 106063 is beyond review or modification even by the Supreme Court. M-issdaa In issuing the questioned orders, the trial court exceeded its authority by altering the essential portions of the Supreme Court decision. Thus, the proceedings held below and the orders issued therein inconsistent with the Supreme Court decision are null and void for want of jurisdiction. We agree that Carmelo and Bauermann is obliged to return the entire amount of eleven million three hundred thousand pesos (P11,300,000.00) to Equatorial. On the other hand, Mayfair may not deduct from the purchase price the amount of eight hundred forty-seven thousand pesos (P 847,000.00) as withholding tax. The duty to withhold taxes due, if any, is imposed on the seller, Carmelo and Bauermann, Inc. WHEREFORE, the petition is partially GRANTED; decision is hereby rendered setting aside the decision and resolution of the Court of Appeals and the orders of execution of the trial court inconsistent and at variance with the dispositive portion of our decision in G. R. No. 106036. Let the trial court carry out the execution following strictly the terms of the aforesaid Supreme Court decision. No costs. SO ORDERED. Sd-aad-sc

THIRD DIVISION

ELVIRA T. ARANGOTE,

G.R. No. 178906

Petitioner, Present:

QUISUMBING, J.,* - versus AUSTRIA-MARTINEZ, Acting Chairperson, CHICO-NAZARIO, NACHURA, and SPS. MARTIN MAGLUNOB and LOURDES S. MAGLUNOB, and ROMEO SALIDO, Respondents. PERALTA, JJ.

Promulgated:

February 18, 2009 x- - - - - - - - - - - - - - - - - - - - - - - - - - - - - - - - - - - - - - - - - - - - - - - - - -x

DECISION

CHICO-NAZARIO, J.:

Before this Court is a Petition for Review on Certiorari under Rule 45 of the 1997 Revised Rules of Civil Procedure seeking to reverse and set aside the Decision dated 27 October 2006 and Resolution dated 29 June 2007 of the Court of Appeals in CA-G.R. SP No. 64970. In its assailed Decision, the appellate court affirmed the Decision dated 12 September 2000 of the Regional Trial Court (RTC), 6th Judicial Region, Branch 1, Kalibo, Aklan, in Civil Case No. 5511, which reversed the Decision dated 6 April 1998 of the 7th Municipal Circuit Trial Court (MCTC) of Ibajay-Nabas, Ibajay, Aklan, in Civil Case No. 156; and declared the herein respondentSpouses Martin and Lourdes Maglunob (Spouses Maglunob) and respondent Romeo Salido (Romeo) as the lawful owners and possessors of Lot 12897 with an area of 982 square meters, more or less, located in Maloco, Ibajay, Aklan (subject property). In its assailed Resolution, the appellate court denied herein petitioner Elvira T. Arangote s Motion for Reconsideration.

Elvira T. Arangote, herein petitioner married to Ray Mars E. Arangote, is the registered owner of the subject property, as evidenced by Original Certificate of Title (OCT) No. CLOA-1748. Respondents Martin (Martin II) and Romeo are first cousins and the grandnephews of

Esperanza Maglunob-Dailisan (Esperanza), from whom petitioner acquired the subject property.

The Petition stems from a Complaint filed by petitioner and her husband against the respondents for Quieting of Title, Declaration of Ownership and Possession, Damages with Preliminary Injunction, and Issuance of Temporary Restraining Order before the MCTC, docketed as Civil Case No. 156.

The Complaint alleged that Esperanza inherited the subject property from her uncle Victorino Sorrosa by virtue of a notarized Partition Agreement dated 29 April 1985, executed by the latter s heirs. Thereafter, Esperanza declared the subject property in her name for real property tax purposes, as evidenced by Tax Declaration No. 16218 (1985).

The Complaint further stated that on 24 June 1985, Esperanza executed a Last Will and Testament bequeathing the subject property to petitioner and her husband, but it was never probated. On 9 June 1986, Esperanza executed another document, an Affidavit, in which she renounced, relinquished, waived and quitclaimed all her rights, share, interest and participation whatsoever in the subject property in favor of petitioner and her husband. On the basis thereof, Tax Declaration No.

16218 in the name of Esperanza was cancelled and Tax Declaration No. 16666 (1987) was issued in the name of the petitioner and her husband.

In 1989, petitioner and her husband constructed a house on the subject property. On 26 March 1993, OCT No. CLOA-1748 was issued by the Secretary of the Department of Agrarian Reform (DAR) in the name of petitioner, married to Ray Mars E. Arangote. However, respondents, together with some hired persons, entered the subject property on 3 June 1994 and built a hollow block wall behind and in front of petitioner s house, which effectively blocked the entrance to its main door.

As a consequence thereof, petitioner and her husband were compelled to institute Civil Case No. 156.

In their Answer with Counterclaim in Civil Case No. 156, respondents averred that they co-owned the subject property with Esperanza. Esperanza and her siblings, Tomas and Inocencia, inherited the subject property, in equal shares, from their father Martin Maglunob (Martin I). When Tomas and Inocencia passed away, their shares passed on by inheritance to respondents Martin II and Romeo, respectively. Hence, the subject property was co-owned by Esperanza, respondent Martin II (together with his wife Lourdes), and respondent Romeo, each

holding a one-third pro-indiviso share therein. Thus, Esperanza could not validly waive her rights and interest over the entire subject property in favor of the petitioner.

Respondents also asserted in their Counterclaim that petitioner and her husband, by means of fraud, undue influence and deceit were able to make Esperanza, who was already old and illiterate, affix her thumbmark to the Affidavit dated 9 June 1986, wherein she renounced all her rights and interest over the subject property in favor of petitioner and her husband. Respondents thus prayed that the OCT issued in petitioner s name be declared null and void insofar as their two-thirds shares are concerned.

After trial, the MCTC rendered its Decision dated 6 April 1998 in Civil Case No. 156, declaring petitioner and her husband as the true and lawful owners of the subject property. The decretal portion of the MCTC Decision reads:

WHEREFORE, judgment is hereby rendered:

A. husband] the true,

Declaring the [herein petitioner and her lawful and exclusive owners and entitled to

the possession of the [subject property] described and referred to under paragraph 2 of the [C]omplaint and covered by Tax Declaration No. 16666 in the names of the [petitioner and her husband];

B. Ordering the [herein respondents] and anyone hired by, acting or working for them, to cease and desist from asserting or claiming any right or interest in, or exercising any act of ownership or possession over the [subject property];

C. Ordering the [respondents] to pay the [petitioner and her husband] the amount of P10,000.00 as attorney s fee. With cost against the [respondents].

The respondents appealed the aforesaid MCTC Decision to the RTC. Their appeal was docketed as Civil Case No. 5511.

Respondents argued in their appeal that the MCTC erred in not dismissing the Complaint filed by the petitioner and her husband for failure to identify the subject property therein. Respondents further faulted the MCTC for not declaring Esperanza s Affidavit dated 9 June 1986 -relinquishing all her rights and interest over the subject property in favor of

petitioner and her husband -- as null and void insofar as respondents twothirds share in the subject property is concerned.

On 12 September 2000, the RTC rendered its Decision reversing the MCTC Decision dated 6 April 1998. The RTC adjudged respondents, as well as the other heirs of Martin Maglunob, as the lawful owners and possessors of the entire subject property. The RTC decreed:

WHEREFORE, judgment is hereby rendered as follows:

1) The appealed [D]ecision is REVERSED;

2) [Herein respondents] and the other heirs of Martin Maglunob are declared the lawful owners and possessors of the whole [subject property] as described in Paragraph 2 of the [C]omplaint, as against the [herein petitioner and her husband].

3) [Petitioner and her husband] are ordered to immediately turn over possession of the [subject property] to the [respondents] and the other heirs of Martin Maglunob; and

4) [Petitioner and her husband] are ordered to pay [respondents] attorney s fees of P5,000.00, other litigation expenses of P5,000.00, moral damages of P10,000.00 and exemplary damages of P5,000.00.

Petitioner and her husband filed before the RTC, on 26 September 2000, a Motion for New Trial or Reconsideration on the ground of newly discovered evidence consisting of a Deed of Acceptance dated 23 September 2000, and notice of the same, which were both made by the petitioner, for herself and in behalf of her husband, during the lifetime of Esperanza. In the RTC Order dated 2 May 2001, however, the RTC denied the aforesaid Motion for New Trial or Reconsideration.

The petitioner and her husband then filed a Petition for Review, under Rule 42 of the 1997 Revised Rules of Civil Procedure, before the Court of Appeals, where the Petition was docketed as CA-G.R. SP No. 64970.

In their Petition before the appellate court, petitioner and her husband raised the following errors committed by the RTC in its 12 September 2000 Decision:

I. of the [MCTC];

It erred in reversing the [D]ecision

II. It erred in declaring the [herein respondents] and the other heirs of Martin Maglunob as the lawful owners and possessors of the whole [subject property];

III. It erred in declaring [OCT] No. CLOA1748 in the name of [herein petitioner] Elvie T. Arangote as null and void;

IV. It erred in denying [petitioner and her husband s] [M]otion for [N]ew [T]rial or [R]econsideration dated [26 September 2000; and

V. It erred in not declaring the [petitioner and her husband] as possessors in good faith.

On 27 October 2006, the Court of Appeals rendered a Decision denying the Petition for Review of petitioner and her husband and affirming the RTC Decision dated 12 September 2000. Petitioner and her

husband s subsequent Motion for Reconsideration was similarly denied by the Court of Appeals in its Resolution dated 29 June 2007.

Hence, petitioner now comes before this Court raising in her Petition the following issues:

I. Whether the [RTC] acted with grave abuse of discretion amounting to lack or excess of jurisdiction when it declared the [petitioner and her husband s title to the subject property] null and void;

II. Whether the [RTC] acted with grave abuse of discretion amounting to lack of jurisdiction when it declared the Affidavit of Quitclaim null and void; and

III. Whether the [RTC] and the Honorable Court of Appeals acted with grave abuse of discretion amounting to lack or excess of jurisdiction when it rejected petitioner s claim as possessors (sic) in good faith, hence, entitled to the rights provided in [Article] 448 and [Article] 546 of the Civil Code.

Petitioner contends that the aforesaid OCT No. CLOA-1748 was issued in her name on 26 March 1993 and was registered in the Registry of Deeds of Aklan on 20 April 1993. From 20 April 1993 until the institution of Civil Case No. 156 on 10 June 1994 before the MCTC, more than one year had already elapsed. Considering that a Torrens title can only be attacked within one year after the date of the issuance of the decree of registration on the ground of fraud and that such attack must be through a direct proceeding, it was an error on the part of the RTC and the Court of Appeals to declare OCT No. CLOA-1748 null and void.

Petitioner additionally posits that both the RTC and the Court of Appeals committed a mistake in declaring null and void the Affidavit dated 9 June 1986 executed by Esperanza, waiving all her rights and interest over the subject property in favor of petitioner and her husband. Esperanza s Affidavit is a valid and binding proof of the transfer of ownership of the subject property in petitioner s name, as it was also coupled with actual delivery of possession of the subject property to petitioner and her husband. The Affidavit is also proof of good faith on the part of petitioner and her husband.

Finally, petitioner argues that, assuming for the sake of argument, that Esperanza s Affidavit is null and void, petitioner and her husband had no knowledge of any flaw in Esperanza s title when the latter relinquished

her rights to and interest in the subject property in their favor. Hence, petitioner and her husband can be considered as possessors in good faith and entitled to the rights provided under Articles 448 and 546 of the Civil Code.

This present Petition is devoid of merit.

It is a hornbook doctrine that the findings of fact of the trial court are entitled to great weight on appeal and should not be disturbed except for strong and valid reasons, because the trial court is in a better position to examine the demeanor of the witnesses while testifying. It is not a function of this Court to analyze and weigh evidence by the parties all over again. This Court s jurisdiction is, in principle, limited to reviewing errors of law that might have been committed by the Court of Appeals. This rule, however, is subject to several exceptions, one of which is present in this case, i.e., when the factual findings of the Court of Appeals and the trial court are contradictory.

In this case, the findings of fact of the MCTC as regards the origin of the subject property are in conflict with the findings of fact of both the RTC and the Court of Appeals. Hence, this Court will have to examine the records to determine first the true origin of the subject property and to

settle whether the respondents have the right over the same for being coheirs and co-owners, together with their grand aunt, Esperanza, before this Court can resolve the issues raised by the petitioner in her Petition.

After a careful scrutiny of the records, this Court affirms the findings of both the RTC and the Court of Appeals as regards the origin of the subject property and the fact that respondents, with their grand aunt Esperanza, were co-heirs and co-owners of the subject property.

The records disclosed that the subject property was part of a parcel of land situated in Maloco, Ibajay, Aklan, consisting of 7,176 square meters and commonly owned in equal shares by the siblings Pantaleon Maglunob (Pantaleon) and Placida Maglunob-Sorrosa (Placida). Upon the death of Pantaleon and Placida, their surviving and legal heirs executed a Deed of Extrajudicial Settlement and Partition of Estate in July 1981, however, the Deed was not notarized. Considering that Pantaleon died without issue, his one-half share in the parcel of land he co-owned with Placida passed on to his four siblings (or their respective heirs, if already deceased), namely: Placida, Luis, Martin I, and Victoria, in equal shares.

According to the aforementioned Deed of Extrajudicial Settlement and Partition of Estate, the surviving and legal heirs of Pantaleon and

Placida agreed to have the parcel of land commonly owned by the siblings declared for real property tax purposes in the name of Victorino Sorrosa (Victorino), Placida s husband. Thus, Tax Declarations No. 5988 (1942), No. 6200 (1945) and No. 7233 (1953) were all issued in the name of Victorino.

Since Martin I already passed away when the Deed of Extrajudicial Settlement and Partition of Estate was executed, his heirs were represented therein by Esperanza. By virtue of the said Deed, Martin I received as inheritance a portion of the parcel of land measuring 897 square meters.

After the death of Victorino, his heirs executed another Partition Agreement on 29 April 1985, which was notarized on the same date. The Partition Agreement mentioned four parcels of land. The subject property, consisting of a portion of the consolidated parcels 1, 2, and 3, and measuring around 982 square meters, was allocated to Esperanza. In comparison, the property given to Esperanza under the Partition Agreement is bigger than the one originally allocated to her earlier under the Deed of Extrajudicial Settlement and Partition of Estate dated July 1981, which had an area of only 897 square meters. It may be reasonably assumed, however, that the subject property, measuring 982 square meters, allocated to Esperanza under the Partition Agreement dated 29 April 1985, is already inclusive of the smaller parcel of 897 square meters assigned to her under the Deed of Extrajudicial Settlement and Partition of Estate dated July 1981. As explained by the RTC in its 12 September 2000 Decision:

The [subject property] which is claimed by the [herein petitioner and her husband] and that which is claimed by the [herein respondents] are one and the same, the difference in area and technical description being due to the repartition and re-allocation of the parcel of land originally co-owned by Pantaleon Maglunob and his sister Placida Maglunob and subsequently declared in the name of [Victorino] under Tax Declaration No. 5988 of 1949.

It is clear from the records that the subject property was not Esperanza s exclusive share, but also that of the other heirs of her father, Martin I. Esperanza expressly affixed her thumbmark to the Deed of Extrajudicial Settlement of July 1981 not only for herself, but also on behalf of the other heirs of Martin I. Though in the Partition Agreement dated 29 April 1985 Esperanza affixed her thumbmark without stating that she was doing so not only for herself, but also on behalf of the other heirs of Martin I, this does not mean that Esperanza was already the exclusive owner thereof. The evidence shows that the subject property is the share of the heirs of Martin I. This is clear from the sketch attached to the Partition Agreement dated 29 April 1985, which reveals the proportionate areas given to the heirs of the two siblings, Pantaleon and Placida, who were the original owners of the whole parcel of land from which the subject property was taken.

Further, it bears emphasis that the Partition Agreement was executed by and among the son, grandsons, granddaughters and cousins of Victorino. Esperanza was neither the granddaughter nor the cousin of Victorino, as she was only Victorino s grandniece. The cousin of Victorino is Martin I, Esperanza s father. In effect, therefore, the subject property allotted to Esperanza in the Partition Agreement was not her exclusive share, as she holds the same for and on behalf of the other heirs of Martin I, who was already deceased at the time the Partition Agreement was made.

To further bolster the truth that the subject property was not exclusively owned by Esperanza, the Affidavit she executed in favor of petitioner and her husband on 6 June 1985 was worded as follows:

That I hereby renounce, relinquish, waive and quitclaim all my rights, share, interest and participation whatsoever in the [subject property] unto the said Sps. Ray Mars Arangote and Elvira T. Arangote, their heirs, successors, and assigns including the improvement found thereon;

Logically, if Esperanza fully owned the subject property, she would have simply waived her rights to and interest in the subject property,

without mentioning her

share

and

participation

in the same.

By

including such words in her Affidavit, Esperanza was aware of and was limiting her waiver, renunciation, and quitclaim to her one-third share and participation in the subject property.

Going to the issues raised by the petitioner in this Petition, this Court will resolve the same concurrently as they are interrelated.

In this case, the petitioner derived her title to the subject property from the notarized Affidavit executed by Esperanza, wherein the latter relinquished her rights, share, interest and participation over the same in favor of the petitioner and her husband.

A careful perusal of the said Affidavit reveals that it is not what it purports to be. Esperanza s Affidavit is, in fact, a Donation. Esperanza s real intent in executing the said Affidavit was to donate her share in the subject property to petitioner and her husband.

As no onerous undertaking is required of petitioner and her husband under the said Affidavit, the donation is regarded as a pure donation of an

interest in a real property covered by Article 749 of the Civil Code. Article 749 of the Civil Code provides:

Art. 749. In order that the donation of an immovable may be valid, it must be made in a public document, specifying therein the property donated and the value of the charges which the donee must satisfy. The acceptance may be made in the same deed of donation or in a separate public document, but it shall not take effect unless it is done during the lifetime of the donor.

If the acceptance is made in a separate instrument, the donor shall be notified thereof in an authentic form, and this step shall be noted in both instruments.

From the aforesaid provision, there are three requisites for the validity of a simple donation of a real property, to wit: (1) it must be made in a public instrument; (2) it must be accepted, which acceptance may be made either in the same Deed of Donation or in a separate public instrument; and (3) if the acceptance is made in a separate instrument, the donor must be notified in an authentic form, and the same must be noted in both instruments.

This Court agrees with the RTC and the Court of Appeals that the Affidavit executed by Esperanza relinquishing her rights, share, interest and participation over the subject property in favor of the petitioner and her husband suffered from legal infirmities, as it failed to comply with the aforesaid requisites of the law.

In Sumipat v. Banga, this Court declared that title to immovable property does not pass from the donor to the donee by virtue of a Deed of Donation until and unless it has been accepted in a public instrument and the donor duly notified thereof. The acceptance may be made in the very same instrument of donation. If the acceptance does not appear in the same document, it must be made in another. Where the Deed of Donation fails to show the acceptance, or where the formal notice of the acceptance, made in a separate instrument, is either not given to the donor or else not noted in the Deed of Donation and in the separate acceptance, the donation is null and void.

In the present case, the said Affidavit, which is tantamount to a Deed of Donation, met the first requisite, as it was notarized; thus, it became a public instrument. Nevertheless, it failed to meet the aforesaid second and third requisites. The acceptance of the said donation was not made by the petitioner and her husband either in the same Affidavit or in a separate public instrument. As there was no acceptance made of the said

donation, there was also no notice of the said acceptance given to the donor, Esperanza. Therefore, the Affidavit executed by Esperanza in favor of petitioner and her husband is null and void.

The subsequent notarized Deed of Acceptance dated 23 September 2000, as well as the notice of such acceptance, executed by the petitioner did not cure the defect. Moreover, it was only made by the petitioner several years after the Complaint was filed in court, or when the RTC had already rendered its Decision dated 12 September 2000, although it was still during Esperanza s lifetime. Evidently, its execution was a mere afterthought, a belated attempt to cure what was a defective donation.

It is true that the acceptance of a donation may be made at any time during the lifetime of the donor. And granting arguendo that such acceptance may still be admitted in evidence on appeal, there is still need for proof that a formal notice of such acceptance was received by the donor and noted in both the Deed of Donation and the separate instrument embodying the acceptance. At the very least, this last legal requisite of annotation in both instruments of donation and acceptance was not fulfilled by the petitioner. Neither the Affidavit nor the Deed of Acceptance bears the fact that Esperanza received notice of the acceptance of the donation by petitioner. For this reason, even Esperanza s one-third share in the subject property cannot be adjudicated to the petitioner.

With the foregoing, this Court holds that the RTC and the Court of Appeals did not err in declaring null and void Esperanza s Affidavit.

The next issue to be resolved then is whether the RTC, as well as the Court of Appeals, erred in declaring OCT No. CLOA-1748 in the name of petitioner and her husband null and void.

Again, this Court answers the said issue in the negative.

Section 48 of Presidential decree No. 1529 states:

SEC. 48. Certificate not subject to collateral attack. A certificate of title shall not be subject to collateral attack. It cannot be altered, modified, or cancelled except in a direct proceeding in accordance with law.

Such

proscription

has

long

been

enshrined

in

Philippine

jurisprudence. The judicial action required to challenge the validity of title is a direct attack, not a collateral attack.

The attack is considered direct when the object of an action is to annul or set aside such proceeding, or enjoin its enforcement. Conversely, an attack is indirect or collateral when, in an action to obtain a different relief, an attack on the proceeding is nevertheless made as an incident thereof. Such action to attack a certificate of title may be an original action or a counterclaim, in which a certificate of title is assailed as void.

A counterclaim is considered a new suit in which the defendant is the plaintiff and the plaintiff in the complaint becomes the defendant. It stands on the same footing as, and is to be tested by the same rules as if it were, an independent action.

In their Answer to the Complaint for Quieting of Title filed by the petitioner and her husband before the MCTC, respondents included therein a Counterclaim wherein they repleaded all the material allegations in their affirmative defenses, the most essential of which was their claim that petitioner and her husband -- by means of fraud, undue influence and deceit -- were able to make their grand aunt, Esperanza, who was already old and illiterate, affix her thumbmark to the Affidavit, wherein she renounced, waived, and quitclaimed all her rights and interest over the subject property in favor of petitioner and her husband. In addition,

respondents maintained in their Answer that as petitioner and her husband were not tenants either of Esperanza or of the respondents, the DAR could not have validly issued in favor of petitioner and her husband OCT No. CLOA-1748. Thus, the respondents prayed, in their counterclaim in Civil Case No. 156 before the MCTC, that OCT No. CLOA-1748 issued in the name of petitioner, married to Ray Mars E. Arangote, be declared null and void, insofar as their two-thirds shares in the subject property are concerned.

It is clear, thus, that respondents Answer with Counterclaim was a direct attack on petitioner s certificate of title. Furthermore, since all the essential facts of the case for the determination of the validity of the title are now before this Court, to require respondents to institute a separate cancellation proceeding would be pointlessly circuitous and against the best interest of justice.

Esperanza s Affidavit, which was the sole basis of petitioner s claim to the subject property, has been declared null and void. Moreover, petitioner and her husband were not tenants of the subject property. In fact, petitioner herself admitted in her Complaint filed before the MCTC that her husband is out of the country, rendering it impossible for him to work on the subject property as a tenant. Instead of cultivating the subject property, petitioner and her husband possessed the same by constructing a house thereon. Thus, it is highly suspicious how the petitioner was able to secure from the DAR a Certificate of Land Ownership Award (CLOA) over the subject property. The DAR awards such certificates to the grantees

only if they fulfill the requirements of Republic Act No. 6657, otherwise known as the Comprehensive Agrarian Reform Program (CARP). Hence, the RTC and the Court of Appeals did not err in declaring null and void OCT No. CLOA-1748 in the name of the petitioner, married to Ray Mars E. Arangote.

Considering that Esperanza died without any compulsory heirs and that the supposed donation of her one-third share in the subject property per her Affidavit dated 9 June 1985 was already declared null and void, Esperanza s one-third share in the subject property passed on to her legal heirs, the respondents.

As petitioner s last-ditch effort, she claims that she is a possessor in good faith and, thus, entitled to the rights provided for under Articles 448 and 546 of the Civil Code.

This claim is untenable.

The Civil Code describes a possessor in good faith as follows:

Art. 526. He is deemed a possessor in good faith who is not aware that there exists in his title or mode of acquisition any flaw which invalidates it.

He is deemed a possessor in bad faith who possesses in any case contrary to the foregoing.

Mistake upon a doubtful or difficult question of law may be the basis of good faith.

Art. 1127. The good faith of the possessor consists in the reasonable belief that the person from whom he received the thing was the owner thereof, and could transmit his ownership.

Possession in good faith ceases from the moment defects in the title are made known to the possessor by extraneous evidence or by a suit for recovery of the property by the true owner. Every possessor in good faith becomes a possessor in bad faith from the moment he becomes aware that what he believed to be true is not so.

In the present case, when respondents came to know that an OCT over the subject property was issued and registered in petitioner s name on

26 March 1993, respondents brought a Complaint on 7 August 1993 before the Lupon of Barangay Maloco, Ibajay, Aklan, challenging the title of petitioner to the subject property on the basis that said property constitutes the inheritance of respondent, together with their grandaunt Esperanza, so Esperanza had no authority to relinquish the entire subject property to petitioner. From that moment, the good faith of the petitioner had ceased.

Petitioner cannot be entitled to the rights under Articles 448 and 546 of the Civil Code, because the rights mentioned therein are applicable only to builders in good faith and not to possessors in good faith.

Moreover, the petitioner cannot be considered a builder in good faith of the house on the subject property. In the context that such term is used in particular reference to Article 448 of the Civil Code, a builder in good faith is one who, not being the owner of the land, builds on that land, believing himself to be its owner and unaware of any defect in his title or mode of acquisition.

The various provisions of the Civil Code, pertinent to the subject, read:

Article 448. The owner of the land on which anything has been built, sown, or planted in good faith, shall have the right to appropriate as his own the works, sowing or planting, after payment of the indemnity provided for in Articles 546 and 548, or to oblige the one who built or planted to pay the price of the land, and the one who sowed, the proper rent. However, the builder or planter cannot be obliged to buy the land if its value is considerably more than that of the building or trees. In such a case, he shall pay reasonable rent, if the owner of the land does not choose to appropriate the building or trees after proper indemnity. The parties shall agree upon the terms of the lease and in case of disagreement, the court shall fix the terms thereof.

Article 449. He who builds, plants, or sows in bad faith on the land of another, loses what is built, planted or sown without right to indemnity.

Article 450. The owner of the land on which anything has been built, planted or sown in bad faith may demand the demolition of the work, or that the planting or sowing be removed, in order to replace things in their former condition at the expense of the person who built, planted or sowed; or he may compel the builder or planter to pay the price of the land, and the sower the proper rent.

Under the foregoing provisions, the builder in good faith can compel the landowner to make a choice between appropriating the building by

paying the proper indemnity or obliging the builder to pay the price of the land. The choice belongs to the owner of the land, a rule that accords with the principle of accession, i.e., that the accessory follows the principal and not the other way around. Even as the option lies with the landowner, the grant to him, nevertheless, is preclusive. He must choose one. He cannot, for instance, compel the owner of the building to instead remove it from the land. In order, however, that the builder can invoke that accruing benefit and enjoy his corresponding right to demand that a choice be made by the landowner, he should be able to prove good faith on his part.

Good faith, here understood, is an intangible and abstract quality with no technical meaning or statutory definition, and it encompasses, among other things, an honest belief, the absence of malice and the absence of design to defraud or to seek an unconscionable advantage. An individual s personal good faith is a concept of his own mind and, therefore, may not conclusively be determined by his protestations alone. It implies honesty of intention, and freedom from knowledge of circumstances which ought to put the holder upon inquiry. The essence of good faith lies in an honest belief in the validity of one s right, ignorance of a superior claim, and absence of intention to overreach another. Applied to possession, one is considered in good faith if he is not aware that there exists in his title or mode of acquisition any flaw which invalidates it.

In this case, the subject property waived and quitclaimed by Esperanza to the petitioner and her husband in the Affidavit was only covered by a tax declaration in the name of Esperanza. Petitioner did not even bother to look into the origin of the subject property and to probe into the right of Esperanza to relinquish the same. Thus, when petitioner and her husband built a house thereon in 1989 they cannot be considered to have acted in good faith as they were fully aware that when Esperanza executed an Affidavit relinquishing in their favor the subject property the only proof of Esperanza s ownership over the same was a mere tax declaration. This fact or circumstance alone was enough to put the It is merely an indicium of a claim petitioner and her husband under inquiry. Settled is the rule that a tax declaration does not prove ownership. of ownership. Payment of taxes is not proof of ownership; it is, at best, an indicium of possession in the concept of ownership. Neither tax receipts nor a declaration of ownership for taxation purposes is evidence of ownership or of a right to possess realty when not supported by other effective proofs. With the foregoing, the petitioner is not entitled to the rights under Article 448 and 546 as the petitioner is not a builder and possessor in good faith. WHEREFORE, premises considered, the instant Petition is hereby DENIED. The Decision and Resolution of the Court of Appeals in CA-G.R. SP No. 64970, dated 27 October 2006 and 29 June 2007, respectively, affirming the RTC Decision dated 12 September 2000 in Civil Case No.

5511 and declaring the respondents the lawful owners and possessors of the subject property are hereby AFFIRMED. No costs. SO ORDERED. SPOUSES JUAN NUGUID AND ERLINDA T. NUGUID, petitioners, vs. HON. COURT OF APPEALS AND PEDRO P. PECSON, respondents. DECISION QUISUMBING, J.: This is a petition for review on certiorari of the Decision dated May 21, 2001, of the Court of Appeals in CA-G.R. CV No. 64295, which modified the Order dated July 31, 1998 of the Regional Trial Court (RTC) of Quezon City, Branch 101 in Civil Case No. Q-41470. The trial court ordered the defendants, among them petitioner herein Juan Nuguid, to pay respondent herein Pedro P. Pecson, the sum of P1,344,000 as reimbursement of unrealized income for the period beginning November 22, 1993 to December 1997. The appellate court, however, reduced the trial court s award in favor of Pecson from the said P1,344,000 to P280,000. Equally assailed by the petitioners is the appellate court s Resolution dated January 10, 2002, denying the motion for reconsideration. It may be recalled that relatedly in our Decision dated May 26, 1995, in G.R. No. 115814, entitled Pecson v. Court of Appeals, we set aside the decision of the Court of Appeals in CA-G.R. SP No. 32679 and the Order dated November 15, 1993, of the RTC of Quezon City, Branch 101 and remanded the case to the trial court for the determination of the current market value of the four-door two-storey apartment building on the 256square meter commercial lot. The antecedent facts in this case are as follows: Pedro P. Pecson owned a commercial lot located at 27 Kamias Road, Quezon City, on which he built a four-door two-storey apartment building. For failure to pay realty taxes, the lot was sold at public auction by the City

Treasurer of Quezon City to Mamerto Nepomuceno, who in turn sold it for P103,000 to the spouses Juan and Erlinda Nuguid. Pecson challenged the validity of the auction sale before the RTC of Quezon City in Civil Case No. Q-41470. In its Decision, dated February 8, 1989, the RTC upheld the spouses title but declared that the four-door two-storey apartment building was not included in the auction sale. This was affirmed in toto by the Court of Appeals and thereafter by this Court, in its Decision dated May 25, 1993, in G.R. No. 105360 entitled Pecson v. Court of Appeals. On June 23, 1993, by virtue of the Entry of Judgment of the aforesaid decision in G.R. No. 105360, the Nuguids became the uncontested owners of the 256-square meter commercial lot. As a result, the Nuguid spouses moved for delivery of possession of the lot and the apartment building. In its Order of November 15, 1993, the trial court, relying upon Article 546 of the Civil Code, ruled that the Spouses Nuguid were to reimburse Pecson for his construction cost of P53,000, following which, the spouses Nuguid were entitled to immediate issuance of a writ of possession over the lot and improvements. In the same order the RTC also directed Pecson to pay the same amount of monthly rentals to the Nuguids as paid by the tenants occupying the apartment units or P21,000 per month from June 23, 1993, and allowed the offset of the amount of P53,000 due from the Nuguids against the amount of rents collected by Pecson from June 23, 1993 to September 23, 1993 from the tenants of the apartment. Pecson duly moved for reconsideration, but on November 8, 1993, the RTC issued a Writ of Possession, directing the deputy sheriff to put the spouses Nuguid in possession of the subject property with all the improvements thereon and to eject all the occupants therein. Aggrieved, Pecson then filed a special civil action for certiorari and prohibition docketed as CA-G.R. SP No. 32679 with the Court of Appeals.

In its decision of June 7, 1994, the appellate court, relying upon Article 448 of the Civil Code, affirmed the order of payment of construction costs but rendered the issue of possession moot on appeal, thus: WHEREFORE, while it appears that private respondents [spouses Nuguid] have not yet indemnified petitioner [Pecson] with the cost of the improvements, since Annex I shows that the Deputy Sheriff has enforced the Writ of Possession and the premises have been turned over to the possession of private respondents, the quest of petitioner that he be restored in possession of the premises is rendered moot and academic, although it is but fair and just that private respondents pay petitioner the construction cost of P53,000.00; and that petitioner be ordered to account for any and all fruits of the improvements received by him starting on June 23, 1993, with the amount of P53,000.00 to be offset therefrom. IT IS SO ORDERED. [Underscoring supplied.] Frustrated by this turn of events, Pecson filed a petition for review docketed as G.R. No. 115814 before this Court. On May 26, 1995, the Court handed down the decision in G.R. No 115814, to wit: WHEREFORE, the decision of the Court of Appeals in CA-G.R. SP No. 32679 and the Order of 15 November 1993 of the Regional Trial Court, Branch 101, Quezon City in Civil Case No. Q-41470 are hereby SET ASIDE. The case is hereby remanded to the trial court for it to determine the current market value of the apartment building on the lot. For this purpose, the parties shall be allowed to adduce evidence on the current market value of the apartment building. The value so determined shall be forthwith paid by the private respondents [Spouses Juan and Erlinda Nuguid] to the petitioner [Pedro Pecson] otherwise the petitioner shall be restored to the possession of the apartment building until payment of the required indemnity. No costs. SO ORDERED. [Emphasis supplied.]

In so ruling, this Court pointed out that: (1) Article 448 of the Civil Code is not apposite to the case at bar where the owner of the land is the builder, sower, or planter who then later lost ownership of the land by sale, but may, however, be applied by analogy; (2) the current market value of the improvements should be made as the basis of reimbursement; (3) Pecson was entitled to retain ownership of the building and, necessarily, the income therefrom; (4) the Court of Appeals erred not only in upholding the trial court s determination of the indemnity, but also in ordering Pecson to account for the rentals of the apartment building from June 23, 1993 to September 23, 1993. On the basis of this Court s decision in G.R. No. 115814, Pecson filed a Motion to Restore Possession and a Motion to Render Accounting, praying respectively for restoration of his possession over the subject 256-square meter commercial lot and for the spouses Nuguid to be directed to render an accounting under oath, of the income derived from the subject fourdoor apartment from November 22, 1993 until possession of the same was restored to him. In an Order dated January 26, 1996, the RTC denied the Motion to Restore Possession to the plaintiff averring that the current market value of the building should first be determined. Pending the said determination, the resolution of the Motion for Accounting was likewise held in abeyance. With the submission of the parties assessment and the reports of the subject realty, and the reports of the Quezon City Assessor, as well as the members of the duly constituted assessment committee, the trial court issued the following Order dated October 7, 1997, to wit: On November 21, 1996, the parties manifested that they have arrived at a compromise agreement that the value of the said improvement/building is P400,000.00 The Court notes that the plaintiff has already received P300,000.00. However, when defendant was ready to pay the balance of P100,000.00, the plaintiff now insists that there should be a rental to be paid by defendants. Whether or not this should be paid by defendants, incident is hereby scheduled for hearing on November 12, 1997 at 8:30 a.m.

Meantime, defendants are directed to pay plaintiff the balance of P100,000.00. SO ORDERED. On December 1997, after paying the said P100,000 balance to Pedro Pecson the spouses Nuguid prayed for the closure and termination of the case, as well as the cancellation of the notice of lis pendens on the title of the property on the ground that Pedro Pecson s claim for rentals was devoid of factual and legal bases. After conducting a hearing, the lower court issued an Order dated July 31, 1998, directing the spouses to pay the sum of P1,344,000 as reimbursement of the unrealized income of Pecson for the period beginning November 22, 1993 up to December 1997. The sum was based on the computation of P28,000/month rentals of the four-door apartment, thus: The Court finds plaintiff s motion valid and meritorious. The decision of the Supreme Court in the aforesaid case [Pecson vs. Court of Appeals, 244 SCRA 407] which set aside the Order of this Court of November 15, 1993 has in effect upheld plaintiff s right of possession of the building for as long as he is not fully paid the value thereof. It follows, as declared by the Supreme Court in said decision that the plaintiff is entitled to the income derived therefrom, thus . . . Records show that the plaintiff was dispossessed of the premises on November 22, 1993 and that he was fully paid the value of his building in December 1997. Therefore, he is entitled to the income thereof beginning on November 22, 1993, the time he was dispossessed, up to the time of said full payment, in December 1997, or a total of 48 months. The only question left is the determination of income of the four units of apartments per month. But as correctly pointed out by plaintiff, the defendants have themselves submitted their affidavits attesting that the income derived from three of the four units of the apartment building is P21,000.00 or P7,000.00 each per month, or P28,000.00 per month for the whole four units. Hence, at P28,000.00 per month, multiplied by 48

months, plaintiff is entitled to be paid by defendants the amount of P1,344,000.00. The Nuguid spouses filed a motion for reconsideration but this was denied for lack of merit. The Nuguid couple then appealed the trial court s ruling to the Court of Appeals, their action docketed as CA-G.R. CV No. 64295. In the Court of Appeals, the order appealed from in CA-G.R. CV No. 64295, was modified. The CA reduced the rentals from P1,344,000 to P280,000 in favor of the appellee. The said amount represents accrued rentals from the determination of the current market value on January 31, 1997 until its full payment on December 12, 1997. Hence, petitioners state the sole assignment of error now before us as follows: THE COURT OF APPEALS ERRED IN HOLDING PETITIONERS LIABLE TO PAY RENT OVER AND ABOVE THE CURRENT MARKET VALUE OF THE IMPROVEMENT WHEN SUCH WAS NOT PROVIDED FOR IN THE DISPOSITIVE PORTION OF THE SUPREME COURT S RULING IN G.R. No. 115814. Petitioners call our attention to the fact that after reaching an agreed price of P400,000 for the improvements, they only made a partial payment of P300,000. Thus, they contend that their failure to pay the full price for the improvements will, at most, entitle respondent to be restored to possession, but not to collect any rentals. Petitioners insist that this is the proper interpretation of the dispositive portion of the decision in G.R. No. 115814, which states in part that [t]he value so determined shall be forthwith paid by the private respondents [Spouses Juan and Erlinda Nuguid] to the petitioner [Pedro Pecson] otherwise the petitioner shall be restored to the possession of the apartment building until payment of the required indemnity. Now herein respondent, Pecson, disagrees with herein petitioners contention. He argues that petitioners are wrong in claiming that inasmuch as his claim for rentals was not determined in the dispositive

portion of the decision in G.R. No. 115814, it could not be the subject of execution. He points out that in moving for an accounting, all he asked was that the value of the fruits of the property during the period he was dispossessed be accounted for, since this Court explicitly recognized in G.R. No. 115814, he was entitled to the property. He points out that this Court ruled that [t]he petitioner [Pecson] not having been so paid, he was entitled to retain ownership of the building and, necessarily, the income therefrom. In other words, says respondent, accounting was necessary. For accordingly, he was entitled to rental income from the property. This should be given effect. The Court could have very well specifically included rent (as fruit or income of the property), but could not have done so at the time the Court pronounced judgment because its value had yet to be determined, according to him. Additionally, he faults the appellate court for modifying the order of the RTC, thus defeating his right as a builder in good faith entitled to rental from the period of his dispossession to full payment of the price of his improvements, which spans from November 22, 1993 to December 1997, or a period of more than four years. It is not disputed that the construction of the four-door two-storey apartment, subject of this dispute, was undertaken at the time when Pecson was still the owner of the lot. When the Nuguids became the uncontested owner of the lot on June 23, 1993, by virtue of entry of judgment of the Court s decision, dated May 25, 1993, in G.R. No. 105360, the apartment building was already in existence and occupied by tenants. In its decision dated May 26, 1995 in G.R. No. 115814, the Court declared the rights and obligations of the litigants in accordance with Articles 448 and 546 of the Civil Code. These provisions of the Code are directly applicable to the instant case. Under Article 448, the landowner is given the option, either to appropriate the improvement as his own upon payment of the proper amount of indemnity or to sell the land to the possessor in good faith. Relatedly, Article 546 provides that a builder in good faith is entitled to full reimbursement for all the necessary and useful expenses incurred; it also gives him right of retention until full reimbursement is made. While the law aims to concentrate in one person the ownership of the land and the improvements thereon in view of the impracticability of creating a

state of forced co-ownership, it guards against unjust enrichment insofar as the good-faith builder s improvements are concerned. The right of retention is considered as one of the measures devised by the law for the protection of builders in good faith. Its object is to guarantee full and prompt reimbursement as it permits the actual possessor to remain in possession while he has not been reimbursed (by the person who defeated him in the case for possession of the property) for those necessary expenses and useful improvements made by him on the thing possessed. Accordingly, a builder in good faith cannot be compelled to pay rentals during the period of retention nor be disturbed in his possession by ordering him to vacate. In addition, as in this case, the owner of the land is prohibited from offsetting or compensating the necessary and useful expenses with the fruits received by the builder-possessor in good faith. Otherwise, the security provided by law would be impaired. This is so because the right to the expenses and the right to the fruits both pertain to the possessor, making compensation juridically impossible; and one cannot be used to reduce the other. As we earlier held, since petitioners opted to appropriate the improvement for themselves as early as June 1993, when they applied for a writ of execution despite knowledge that the auction sale did not include the apartment building, they could not benefit from the lot s improvement, until they reimbursed the improver in full, based on the current market value of the property. Despite the Court s recognition of Pecson s right of ownership over the apartment building, the petitioners still insisted on dispossessing Pecson by filing for a Writ of Possession to cover both the lot and the building. Clearly, this resulted in a violation of respondent s right of retention. Worse, petitioners took advantage of the situation to benefit from the highly valued, income-yielding, four-unit apartment building by collecting rentals thereon, before they paid for the cost of the apartment building. It was only four years later that they finally paid its full value to the respondent. Petitioners interpretation of our holding in G.R. No. 115814 has neither factual nor legal basis. The decision of May 26, 1995, should be construed in connection with the legal principles which form the basis of the decision,

guided by the precept that judgments are to have a reasonable intendment to do justice and avoid wrong. The text of the decision in G.R. No. 115814 expressly exempted Pecson from liability to pay rentals, for we found that the Court of Appeals erred not only in upholding the trial court s determination of the indemnity, but also in ordering him to account for the rentals of the apartment building from June 23, 1993 to September 23, 1993, the period from entry of judgment until Pecson s dispossession. As pointed out by Pecson, the dispositive portion of our decision in G.R. No. 115814 need not specifically include the income derived from the improvement in order to entitle him, as a builder in good faith, to such income. The right of retention, which entitles the builder in good faith to the possession as well as the income derived therefrom, is already provided for under Article 546 of the Civil Code. Given the circumstances of the instant case where the builder in good faith has been clearly denied his right of retention for almost half a decade, we find that the increased award of rentals by the RTC was reasonable and equitable. The petitioners had reaped all the benefits from the improvement introduced by the respondent during said period, without paying any amount to the latter as reimbursement for his construction costs and expenses. They should account and pay for such benefits. We need not belabor now the appellate court s recognition of herein respondent s entitlement to rentals from the date of the determination of the current market value until its full payment. Respondent is clearly entitled to payment by virtue of his right of retention over the said improvement. WHEREFORE, the instant petition is DENIED for lack of merit. The Decision dated May 21, 2001 of the Court of Appeals in CA-G.R. CV No. 64295 is SET ASIDE and the Order dated July 31, 1998, of the Regional Trial Court, Branch 101, Quezon City, in Civil Case No. Q-41470 ordering the herein petitioners, Spouses Juan and Erlinda Nuguid, to account for the rental income of the four-door two-storey apartment building from November 1993 until December 1997, in the amount of P1,344,000, computed on the basis of Twenty-eight Thousand (P28,000.00) pesos monthly, for a period of 48 months, is hereby REINSTATED. Until fully paid, said amount of

rentals should bear the legal rate of interest set at six percent (6%) per annum computed from the date of RTC judgment. If any portion thereof shall thereafter remain unpaid, despite notice of finality of this Court s judgment, said remaining unpaid amount shall bear the rate of interest set at twelve percent (12%) per annum computed from the date of said notice. Costs against petitioners. SO ORDERED.

PHILIPPINE NATIONAL BANK, petitioner, vs. GENEROSO DE JESUS, represented by his Attorney-in-Fact, CHRISTIAN DE JESUS, respondent. DECISION VITUG, J.: Petitioner Philippine National Bank disputes the decision handed down by the Court of Appeals promulgated on 23 March 2001 in CA-G.R. CV No. 56001, entitled Generoso De Jesus, represented by his Attorney-in-Fact, Christian De Jesus, versus Philippine National Bank. The assailed decision has affirmed the judgment rendered by the Regional Trial Court, Branch 44, of Mamburao, Occidental Mindoro, declaring respondent Generoso de Jesus as being the true and lawful owner of the 124-square-meter portion of the land covered by Transfer Certificate of Title (TCT) No. T-17197 and ordering petitioner bank to vacate the premises, to deliver possession thereof to respondent, and to remove the improvement thereon. It would appear that on 10 June 1995, respondent filed a complaint against petitioner before the Regional Trial Court of Occidental Mindoro for recovery of ownership and possession, with damages, over the questioned property. In his complaint, respondent stated that he had acquired a parcel of land situated in Mamburao, Occidental Mindoro, with an area of 1,144 square meters covered by TCT No. T-17197, and that on 26 March 1993, he had caused a verification survey of the property and discovered that the northern portion of the lot was being encroached upon by a building of petitioner to the extent of 124 square meters. Despite two

letters of demand sent by respondent, petitioner failed and refused to vacate the area. Petitioner, in its answer, asserted that when it acquired the lot and the building sometime in 1981 from then Mayor Bienvenido Ignacio, the encroachment already was in existence and to remedy the situation, Mayor Ignacio offered to sell the area in question (which then also belonged to Ignacio) to petitioner at P100.00 per square meter which offer the latter claimed to have accepted. The sale, however, did not materialize when, without the knowledge and consent of petitioner, Mayor Ignacio later mortgaged the lot to the Development Bank of the Philippines. The trial court decided the case in favor of respondent declaring him to be the rightful owner of the disputed 124-square-meter portion of the lot and ordering petitioner to surrender possession of the property to respondent and to cause, at its expense, the removal of any improvement thereon. The Court of Appeals, on appeal, sustained the trial court but it ordered to be deleted the award to respondent of attorney s fees, as well as moral and exemplary damages, and litigation expenses. Petitioner went to this Court, via a petition for review, after the appellate court had denied the bank s motion for reconsideration, here now contending that 1. THE COURT OF APPEALS GRAVELY ERRED IN LAW IN ADJUDGING PNB A BUILDER IN BAD FAITH OVER THE ENCROACHED PROPERTY IN QUESTION; 2. THE COURT OF APPEALS GRAVELY ERRED IN LAW IN NOT APPLYING IN FAVOR OF PNB THE PROVISION OF ARTICLE 448 OF THE CIVIL CODE AND THE RULING IN TECNOGAS PHILIPPINES MANUFACTURING CORP. VS. COURT OF APPEALS, G.R. No. 108894, February 10, 1997, 268 SCRA 7. The Regional Trial Court and the Court of Appeals have both rejected the idea that petitioner can be considered a builder in good faith. In the context that such term is used in particular reference to Article 448, et seq., of the Civil Code, a builder in good faith is one who, not being the

owner of the land, builds on that land believing himself to be its owner and unaware of any defect in his title or mode of acquisition. The various provisions of the Civil Code, pertinent to the subject, read: Article 448. The owner of the land on which anything has been built, sown, or planted in good faith, shall have the right to appropriate as his own the works, sowing or planting, after payment of the indemnity provided for in Articles 546 and 548, or to oblige the one who built or planted to pay the price of the land, and the one who sowed, the proper rent. However, the builder or planter cannot be obliged to buy the land if its value is considerably more than that of the building or trees. In such a case, he shall pay reasonable rent, if the owner of the land does not choose to appropriate the building or trees after proper indemnity. The parties shall agree upon the terms of the lease and in case of disagreement, the court shall fix the terms thereof. Article 449. He who builds, plants, or sows in bad faith on the land of another, loses what is built, planted or sown without right to indemnity. Article 450. The owner of the land on which anything has been built, planted or sown in bad faith may demand the demolition of the work, or that the planting or sowing be removed, in order to replace things in their former condition at the expense of the person who built, planted or sowed; or he may compel the builder or planter to pay the price of the land, and the sower the proper rent. A builder in good faith can, under the foregoing provisions, compel the landowner to make a choice between appropriating the building by paying the proper indemnity or obliging the builder to pay the price of the land. The choice belongs to the owner of the land, a rule that accords with the principle of accession, i.e., that the accessory follows the principal and not the other way around. Even as the option lies with the landowner, the grant to him, nevertheless, is preclusive. He much choose one. He cannot, for instance, compel the owner of the building to instead remove it from the land. In order, however, that the builder can invoke that accruing benefit and enjoy his corresponding right to demand that a choice be made by the landowner, he should be able to prove good faith on his part.

Good faith, here understood, is an intangible and abstract quality with no technical meaning or statutory definition, and it encompasses, among other things, an honest belief, the absence of malice and the absence of design to defraud or to seek an unconscionable advantage. An individual s personal good faith is a concept of his own mind and, therefore, may not conclusively be determined by his protestations alone. It implies honesty of intention, and freedom from knowledge of circumstances which ought to put the holder upon inquiry. The essence of good faith lies in an honest belief in the validity of one s right, ignorance of a superior claim, and absence of intention to overreach another. Applied to possession, one is considered in good faith if he is not aware that there exists in his title or mode of acquisition any flaw which invalidates it. Given the findings of both the trial court and the appellate court, it should be evident enough that petitioner would fall much too short from its claim of good faith. Evidently, petitioner was quite aware, and indeed advised, prior to its acquisition of the land and building from Ignacio that a part of the building sold to it stood on the land not covered by the land conveyed to it. Equally significant is the fact that the building, constructed on the land by Ignacio, has in actuality been part of the property transferred to petitioner. Article 448, of the Civil Code refers to a piece of land whose ownership is claimed by two or more parties, one of whom has built some works (or sown or planted something) and not to a case where the owner of the land is the builder, sower, or planter who then later loses ownership of the land by sale or otherwise for, elsewise stated, where the true owner himself is the builder of works on his own land, the issue of good faith or bad faith is entirely irrelevant. In fine, petitioner is not in a valid position to invoke the provisions of Article 448 of the Civil Code. The Court commiserates with petitioner in its present predicament; upon the other hand, respondent, too, is entitled to his rights under the law, particularly after having long been deprived of the enjoyment of his property. Nevertheless, the Court expresses hope that the parties will still be able to come up with an arrangement that can be mutually suitable and acceptable to them.

WHEREFORE, the decision of the Court of Appeals in CA-G.R. CV No. 56001 is AFFIRMED. No costs. SO ORDERED. Pleasantville Development v. CA [G.R. No. 79688. February 1, 1996.] Third Division, Panganiban (J): 4 concur, 1 took no part Facts: Edith Robillo purchased from Pleasantville Development Corporation a parcel of land designated as Lot 9, Phase II and located at Taculing Road, Pleasantville Subdivision, Bacolod City. Eldred Jardinico bought the rights to the lot from Robillo. At that time, Lot 9 was vacant. On the other hand, on 26 March 1974, Kee bought on installment Lot 8 of the same subdivision from C.T. Torres Enterprises, Inc. (CTTEI), the exclusive real estate agent of Pleasantville Development. Under the Contract to Sell on Installment, Kee could possess the lot even before the completion of all installment payments. On 20 January 1975, Kee paid CTTEI the relocation fee of P50.00 and another P50.00 on 27 January 1975, for the preparation of the lot plan. These amounts were paid prior to Kee's taking actual possession of Lot 8. After the preparation of the lot plan and a copy thereof to Kee, CTTEI through its employee, Zenaida Octaviano, accompanied Kee's wife, Donabelle Kee, to inspect Lot 8. Unfortunately, the parcel of land pointed by Octaviano was Lot 9. Thereafter, Kee proceeded to construct his residence, a store, an auto repair shop and other improvements on the lot. Upon completing all payments, Jardinico secured on 19 December 1978 from the Register of Deeds of Bacolod City TCT 106367 in his name. It was then that he discovered that improvements had been introduced on Lot 9 by Wilson Kee, who had taken possession thereof. After the discovery, Jardinico confronted Kee. The parties tried to reach an amicable settlement, but failed. On 30 January 1981, Jardinico's lawyer wrote Kee, demanding that the latter remove all improvements and vacate Lot 9. When Kee refused to vacate, Jardinico filed with the MTCC a complaint for ejectment with damages against Kee. Kee, in turn, filed a third-party complaint against Pleasantville and CTTEI. The MTCC held that the erroneous delivery of Lot

9 to Kee was attributable to CTTEI. The MTCC also found that Pleasantville had already rescinded its contract with Kee over Lot 8 for the latter's failure to pay the installments due, and that Kee had not contested the rescission. The rescission was effected in 1979, before the complaint was instituted. The MTCC concluded that Kee no longer had any right over the lot subject of the contract between him and Pleasantville. Consequently, Kee must pay reasonable rentals for the use of Lot 9, and, furthermore, he cannot claim reimbursement for the improvements he introduced on said lot. The MTCC thus ordered Kee to vacate Lot 9, to remove all structures and improvements he introduced thereon, and to pay the Jardinico rentals of P15.00 a day computed from the time the suit was filed on 12 March 1981 until he vacateds the premises; such amount bearing an interest of 12% per annum. The MTCC also ordered CTTI and Pleasantville to pay Jardinico in solidum for the amount of P3,000 as attorney s fees and P700 as cost and litigation expenses. On appeal, the RTC Bacolod City (Branch 48) ruled that Pleasantville and CTTEI were not at fault or were not negligent and found Kee a builder in bad faith. Thus, the appellate court affirmed the decision with respect to the order to vacate the premises of Lot 9, the removal of the structure and improvements introduced thereon at Kee s expense, and to pay a rental of P15.00 a day until he vacates the premises, with an interest of 12% per annum. The Court further rendered judgment against Kee to pay Jardinico the sum of P3,000.00 as attorney's fees, plus costs of litigation; dismissed the third-party complaint against Pleasantville CTTEI, and reversed the order Pleasantville and CTTEI to pay Jardinico attorney's fees and costs of litigation. Following the denial of his motion for reconsideration on 20 October 1986, Kee appealed directly to the Supreme Court, which referred the matter to the Court of Appeals. Pending resolution of the case before the Court of Appeals, Jardinico and Kee on 24 July 1987 entered into a deed of sale, wherein the former sold Lot 9 to Kee. Jardinico and Kee did not inform the Court of Appeals of such deal. The appellate court ruled that Kee was a builder in good faith (entitled to rights under Articles 448, 546 and 548 of the Civil Code), as he was unaware of the "mix-up" when he began construction of the improvements

on Lot 8. It further ruled that the erroneous delivery was due to the negligence of CTTEI, and that such wrong delivery was likewise imputable to its principal, Pleasantville; and thus ordered the CTTEI and Pleasantville to be solidarily liable for the demolition expenses and value of improvements destroyed or rendered useless in case Jardinico decides to appropriate the improvements and thereafter remove the structures; or for the amount representing the value of Lot 9 that Kee should pay to Jardinico if Jardinico chose to sell the land to Kee. The appellate court ordered CTTEI and Pleasantville to pay in solidum the amount of P3,000.00 to Jardinico as attorney's fees, as well as litigation expenses; and ruled that the award of rentals was without basis. Further, the appellate court remanded the case to the court of origin for the determination of the actual value of the improvements and the property (Lot 9). Pleasantville filed the petition for review on certiorari. The Supreme Court partially granted the petition, and modified the decision of the Court of Appeals by declaring Wilson Kee a builder in good faith; and that Pleasantville Development and C.T. Torres Enterprises solidarily liable for damages due to negligence (however, since the amount and/or extent of such damages was proven during the trial, the same cannot now be quantified and awarded). The Court also ordered Pleasantville Development and C.T. Torres Enterprises to pay in solidum the amount of P3,000.00 to Jardinico as attorney's fees, as well as litigation expenses. The Court dispensed with the award of rentals to Jardinico. 1. Kee a builder in good faith; Prudent acts to ascertain land to build on Under the Torrens system of land registration, Kee is presumed to have knowledge of the metes and bounds of the property with which he is dealing. But as Kee is a layman not versed in the technical description of his property, he had to find a way to ascertain that what was described in TCT 69561 matched Lot 8. Thus, he went to the subdivision developer's agent and applied and paid for the relocation of the lot, as well as for the production of a lot plan by CTTEI's geodetic engineer. Upon Kee's receipt of the map, his wife went to the subdivision site accompanied by CTTEI's employee, Octaviano, who authoritatively declared that the land she was pointing to was indeed Lot 8. Having full faith and confidence in the reputation of CTTEI, and because of the company's positive identification of the property, Kee saw no reason to suspect that there had been a

misdelivery. The steps Kee had taken to protect his interests were reasonable. There was no need for him to have acted ex-abundantia cautela. such as being present during the geodetic engineer's relocation survey or hiring an independent geodetic engineer to countercheck for errors, for the final delivery of subdivision lots to their owners is part of the regular course of everyday business of CTTEI. Because of CTTEI's blunder, what Kee had hoped to forestall did in fact transpire. Kee had acted in the manner of a prudent man in ascertaining the identity of his property. 2. Scenario of bad faith improbable; Good faith presumed It is thus highly improbable that a purchaser of a lot would knowingly and willingly build his residence on a lot owned by another, deliberately exposing himself and his family to the risk of being ejected from the land and losing all improvements thereon, not to mention the social humiliation that would follow. Good faith consists in the belief of the builder that the land he is building on is his and his ignorance of any defect or flaw in his title. And as good faith is presumed, the one alleging bad faith has the burden of proving bad faith. 3. Contractual breach cannot be the basis to negate the presumption of builder in good faith Violations of paragraphs 22 and 26 of the Contract of Sale on Installment have no bearing whatsoever on whether Kee was a builder in good faith, i.e. on his state of mind at the time he built the improvements on Lot 9. These alleged violations may give rise to Pleasantville's cause of action against Kee under the said contract (contractual breach), but may not be bases to negate the presumption that Kee was a builder in good faith. 4. Rescission does not negate the negligence of CTTEI The Contract of Sale on Installment covering Lot 8 between Pleasantville and Kee, which was rescinded long before the present action was instituted, has no relevance on the liability of Pleasantville, as such fact does not negate the negligence of its agent in pointing out the wrong lot to Kee. Such circumstance is relevant only as it gives Jardinico a cause of action for unlawful detainer against Kee. 5. Recovery of damages not waived Kee did not contracted away his right to recover damages resulting from Pleasantville's negligence. Such waiver would be contrary to public policy

and cannot be allowed. "Rights may be waived, unless the waiver is contrary to law, public order, public policy, morals, or good customs, or prejudicial to a third person with a right recognized by law." 6. Principal responsible for acts of agent if damaged caused to third persons; Agent is personally liable for damages if he exceeds his authority The principal is responsible for the acts of the agent, done within the scope of his authority, and should bear the damage caused to third persons. On the other hand, the agent who exceeds his authority is personally liable for the damages. In the present case, CTTEI was acting within its authority as the sole real estate representative of Pleasantville when it made the delivery to Kee. In acting within its scope of authority, it was, however, negligent. It is this negligence that is the basis of Pleasantville's liability, as principal of CTTEI, per Articles 1909 and 1910 of the Civil Code. 7. Deed of Sale between Kee and Jardinico merely regulates the reciprocal rights of the parties and has no effect on the liability of Pleasantville The deed of sale regulates the reciprocal rights of Kee and Jardinico; it stressed that they had reached an agreement independent of the outcome of the case. The "terms and conditions in the said deed of sale are strictly for the parties thereto" and that "there is no waiver made by either of the parties in said deed of whatever favorable judgment or award the Court of Appeals may make in Kee s and Jardinico s favor against Pleasantville and CTTEI. The deed of sale can have no effect on the liability of Pleasantville. Pleasantville's liability is grounded on the negligence of its agent. 8. Pleasantville supposedly liable for damages due to agents negligence; Due to lack of evidence, no damages due Pleasantville s liability lies in the negligence of its agent CTTEI. For such negligence, Pleasantville s should be held liable for damages. The extent and/or amount of damages to be awarded is a factual issue which should be determined after evidence is adduced. However, there is no showing that such evidence was actually presented in the trial court; hence no damages could be awarded. 9. Appellate court erred in modification of the application of the law on ground of equity The rights of Kee and Jardinico vis-a-vis each other, as builder in good faith and owner in good faith, respectively, are regulated by law (i.e., Arts. 448,

546 and 548 of the Civil Code). It was error for the Court of Appeals to make a "slight modification" in the application of such law, on the ground of "equity". At any rate, Kee and Jardinico have amicably settled through their deed of sale their rights and obligations with regards to Lot 9. Thus, the Court deleted the dispositive portion of the Court of Appeals' Decision holding Pleasantville and CTTEI solidarily liable for demolition expenses or the amount pertaining to the value of the lot, whichever is applicable in the exercise of the landowner s options. 10. Award of attorney s fees lies with the discretion of the court depending on the case s circumstances The award of attorney's fees lies within the discretion of the court and depends upon the circumstances of each case. The Supreme Court shall not interfere with the discretion of the Court of Appeals. Jardinico was compelled to litigate for the protection of his interests and for the recovery of damages sustained as a result of the negligence of Pleasantvile's agent. 11. No need to remand the case for the determination of the value and the land In view of the deed of sale entered into by Kee and Jardinico, which deed governs the rights of Jardinico and Kee as to each other, there is also no further need to remand the case to the court of origin "for determination of the actual value of the improvements and the property (Lot 9), as well as for further proceedings in conformity with Article 448 of the New Civil Code."

[G.R. No. 144291. April 20, 2001] EVADEL REALTY and DEVELOPMENT CORPORATION, petitioners, vs. SPOUSES ANTERO AND VIRGINIA SORIANO, respondents. DECISION KAPUNAN, J.:

This is an appeal by certiorari under Rule 45 of the Rules of Court of the decision of the Court of Appeals dated August 3, 2000 in CA-G.R. CV No. 60292 affirming the summary judgment rendered by the Regional Trial Court, Branch 88, Cavite City, in the case for accion reinvidicatoria filed by herein respondents Antero and Virginia Soriano against petitioner Evadel Realty and Development Corporation. The pertinent facts from which the present petition proceeds are as follows: On April 12, 1996, the spouses Antero and Virginia Soriano (respondent spouses), as sellers, entered into a Contract to Sell with Evadel Realty and Development Corporation (petitioner), as buyer, over a parcel of land denominated as Lot 5536-C of the Subdivision Plan of Lot 5536 covered by Transfer Certificate of Title No. 125062 which was part of a huge tract of land known as the Imus Estate. The pertinent portions of the Contract read: xxx WHEREAS : It is the desire of Party B to purchase a portion of a parcel of land owned by Party A and which portion consist of 28,958 sq.m. and specifically described as lot 5536-C of the Subdivision Plan of Lot 5536 of Imus Estate as surveyed for Antero Q. Soriano and covered by TCT 125062 issued by the Register of Deeds of the Province of Cavite and which portion is shown in Annex A hereof. xxx I. SUBJECT The subject of this agreement is the intended sale of 28,958 sq.m. which is a portion of TCT No. 125062 in the name of Party A to Party B and which portion is herewith shown in Annex A hereof. xxx III. Conditions to Govern Contract to Sell

1] The amount of Twenty Eight Million Nine Hundred Fifty Eight Thousand Pesos (P28,958,000.00) representing the first installment of the purchase price of the property shall be delivered by Party B to Party A upon the signing of this agreement. 2] The second and last installment of Twenty Eight Million Nine Hundred Fifty Eight Thousand Pesos (P28,958,000.00) shall be delivered by Party B to Party A simultaneously with the delivery of Party "A" to Party "B" of the Torrens Title to the lot specifically described as Lot No. 5536-C containing an area of 28,958 sq. m. and herewith shown in Annex A hereof; still in the name of Party A and the delivery of Party A to Party B of the Deed of Absolute Sale to the property in favor of Party B . Responsibility of the transfer of the Torrens Title from the name of Party A to Party B shall be the sole responsibility of Party B . Moreover, the balance in the amount of Twenty Eight Million Nine Hundred Fifty Eight Thousand Pesos(P28,958,000.00) shall be due and demandable immediately from the time Party B , thru its President or Vice-President receives either verbal or written notice that the Torrens Title to the segregated property and the Deed of Absolute Sale are already available for delivery to Party B . In the event of delay, however, Party B shall be charged with interest and penalty in the amount of 6% per month, compounded, for every month of delay or a fraction thereof in the event the delay does not exceed one month. xxx[1] Upon payment of the first installment, petitioner introduced improvements thereon and fenced off the property with concrete walls. Later, respondent spouses discovered that the area fenced off by petitioner exceeded the area subject of the contract to sell by 2,450 square meters. Upon verification by representatives of both parties, the area encroached upon was denominated as Lot 5536-D-1 of the subdivision plan of Lot 5536-D of Psd-04-092419 and was later on segregated from the mother title and issued a new transfer certificate of title, TCT No. 769166, in the name of respondent spouses. Respondent spouses successively sent demand letters to petitioner on February 14, March 7, and April 24, 1997, to vacate the encroached area.

Petitioner admitted receiving the demand letters but refused to vacate the said area. Thus, on May 23, 1997, a complaint for accion reinvindicatoria was filed by respondent spouses against petitioner with the Regional Trial Court, Branch 88 of Cavite City. In its Answer, petitioner admitted the encroachment but claimed that it was a builder in good faith since it merely relied on the boundaries pointed out by the representatives of respondent spouses. Petitioner also argued that there was a novation of contract because of the encroachment made by the national road on the property subject of the contract by 1,647 square meters. On March 19, 1998, respondents filed a Motion for Summary Judgment, alleging that there existed no genuine issue as to the material facts of the case due to the admissions made by petitioner in its Answer. The trial court granted the motion on June 11, 1998 and rendered judgment in favor of respondent spouses, the dispositive portion of which reads: WHEREFORE, in the light of the foregoing, this court hereby orders the defendant to remove without right of indemnity and at its expense, any or all improvements that it has introduced on the parcel of land covered by TCT No. T-769166 issued by the Register of Deeds of the Province of Cavite with an area of 2,450 square meters, more or less, in the name of plaintiffs spouses and to return to the plaintiffs the physical possession of the above-described parcel of land. Plaintiffs' and defendant s claim and counter-claim for damages and attorney s fees are dismissed. No pronouncement as to costs. SO ORDERED.[2] This prompted petitioner to appeal the matter to the Court of Appeals. On August 3, 2000, the Court of Appeals affirmed the order for summary judgment of the trial court. Hence, this petition ascribing the following errors:

I. XXX, THE HON. COURT OF APPEALS COMMITTED AN ERROR OF LAW IN AFFIRMING THAT UNDER THE FACTUAL CIRCUMSTANCES, A SUMMARY JUDGMENT COULD BE RENDERED BY THE COURT A QUO. II. XXX, THE HON. COURT OF APPEALS COMMITTED AN ERROR OF LAW IN ITS APPLICATION OF THE JURISPRUDENCE LAID DOWN IN THE CASE OF TERNATE v. COURT OF APPEALS (241 SCRA 254) AND NATIONAL IRRIGATION ADMINISTRATION v. GAMIT (215 SCRA 436) UNDER THE FACTUAL CONTENT OF THE CASE AT BAR. III. XXX, THE HON. COURT OF APPEALS COMMITTED AN ERROR OF LAW IN ITS APPLICATION OF THE JURISPRUDENCE LAID DOWN IN THE CASE OF J.M. TUASON & CO. INC. v. VDA. DE LUMANLAN (23 SCRA 230) UNDER THE FACTUAL CONTENT OF THE CASE AT BAR. IV. XXX, THE HON. COURT OF APPEALS COMMITTED AN ERROR OF LAW IN ITS APPLICATION OF THE JURISPRUDENCE LAID DOWN IN THE CASES OF MANILA BAY CLUB CORPORATION v. COURT OF APPEALS (245 SCRA 715) AND THE MARINE CULTURE INC. v. COURT OF APPEALS (219 SCRA 148) UNDER THE FACTUAL CONTENT OF THE CASE AT BAR. V. XXX, THE HON. COURT OF APPEALS COMMITTED AN ERROR OF LAW IN AFFIRMING THE DECISION OF THE COURT A QUO, THUS DEPRIVING THE PETITIONER OF ITS DAY IN COURT AND ITS CONSTITUTIONAL RIGHT TO DUE PROCESS OF LAW. Summarizing the aforecited issues, the basic issue posed for resolution is whether or not the trial court was in error in rendering summary judgment on the case. Petitioner claims that a summary judgment cannot be rendered on the case as there are genuine issues of fact which have to be threshed out during trial. It is alleged that in the original and amended complaint, private respondent spouses sought recovery of two thousand four hundred sixty two (2,462) square meters of land. This was, however, changed to 2,450 square meters in the second amended complaint. It is also argued that when petitioner entered upon the property in 1996, it relied on the metes and boundaries pointed out by respondents themselves and their surveyors. Moreover, title over the said area was obtained only after the

commencement of the complaint so petitioner could not have possibly disputed such title earlier. Therefore, petitioner maintains, the question of the exact area of the land allegedly encroached, whether 2,462 or 2,450 square meters; and the determination of whether its possession of the subject property was in good or bad faith, are genuine triable issues. Respondent spouses, on the other hand, maintain that there are no genuine issues of fact in the present case in view of the admission by petitioner of (1) the existence of the title over the subject property in the name of respondent spouses; and (2) its encroachment on the northern side of sold Lot 5536-C which is the area in dispute. It is claimed that such admissions are tantamount to an admission that respondents have a rightful claim of ownership to the subject property warranting a summary judgment in their favor. Prompt and expeditious resolution of cases have always been an underlying policy of the Court. For this reason, certain rules under the Rules of Court are designed to shorten the procedure in order to allow the speedy disposition of a case. Some of these are Rule 33 on Demurrer to Evidence, Rule 34 on Judgment on the Pleadings and Rule 35 on Summary Judgments. In all these instances, full-blown trial of a case is dispensed with and judgment is rendered on the basis of the pleadings, supporting affidavits, depositions and admissions of the parties. Under Rule 35 of the 1997 Rules of Civil Procedure, except as to the amount of damages, when there is no genuine issue as to any material fact and the moving party is entitled to a judgment as a matter of law, summary judgment may be allowed.[3] Summary or accelerated judgment is a procedural technique aimed at weeding out sham claims or defenses at an early stage of the litigation thereby avoiding the expense and loss of time involved in a trial.[4] The law itself determines when a summary judgment is proper. Under the rules, summary judgment is appropriate when there are no genuine issues of fact which call for the presentation of evidence in a full-blown trial. Even if on their face the pleadings appear to raise issues, when the affidavits, depositions and admissions show that such issues are not genuine, then summary judgment as prescribed by the rules must ensue as

a matter of law. What is crucial for determination, therefore, is the presence or absence of a genuine issue as to any material fact.[5] A genuine issue is an issue of fact which require the presentation of evidence as distinguished from a sham, fictitious, contrived or false claim. When the facts as pleaded appear uncontested or undisputed, then there is no real or genuine issue or question as to the facts, and summary judgment is called for. The party who moves for summary judgment has the burden of demonstrating clearly the absence of any genuine issue of fact, or that the issue posed in the complaint is patently unsubstantial so as not to constitute a genuine issue for trial.[6] Trial courts have limited authority to render summary judgments and may do so only when there is clearly no genuine issue as to any material fact. When the facts as pleaded by the parties are disputed or contested, proceedings for summary judgment cannot take the place of trial.[7] Applying these principles to the present case, we hold that the CA did not commit any reversible error in affirming the summary judgment rendered by the trial court. Hence, the instant petition must be denied. The case at bar is one for accion reinvindicatoria which is an action to recover ownership over real property. Respondent spouses (plaintiffs below) seek to recover a certain portion of land with a total area of 2,450 square meters from petitioner which portion was allegedly in excess of the total area of the property actually sold by them to the latter. In a reinvindicatory action, the basic issue for resolution is that of ownership and in the present case, the determination of ownership of the subject property is hinged on the following questions of fact - first, what was the total area of the lot sold to petitioner by respondent spouses as agreed upon and embodied in the contract to sell; and second, whether or not the area being occupied by the petitioner is in excess of the land which it actually bought from respondent spouses under the said contract. In its Answer to the Amended Complaint, petitioner admitted the existence and due execution of the Contract to Sell which contained the specific description of the property it bought from respondent spouses, to wit: xxx

WHEREAS : It is the desire of Party B to purchase a portion of a parcel of land owned by Party A and which portion consist of 28,958 sq.m. and specifically described as lot 5536-C of the Subdivision Plan of Lot 5536 of Imus Estate as surveyed for Antero Q. Soriano and covered by TCT 125062 issued by the Register of Deeds of the Province of Cavite and which portion is shown in Annex A hereof. xxx Equally significant is the fact that in the same Answer, petitioner likewise admitted that the relocation survey conducted by geodetic engineers of both parties disclosed that indeed there were two encroachments, i.e. 1) encroachment at the eastern frontage of Lot 5536-C by the national road; and 2) encroachment by defendant (petitioner) EVADEL on the northern side of sold Lot 5536-C. [8] and that the second area encroached upon was denominated as Lot 5536D-1 of the subdivision plan of Lot 5536-D of Psd-04-092419 and later on segregated from the mother title and issued a new transfer certificate of title, TCT No. 769166, during the pendency of the case before the trial court. With the foregoing admissions by petitioner, clearly, there is no genuine issue of fact as to ownership of the subject property because the said admissions made by petitioner in its Answer are tantamount to an admission that respondent spouses owned the property in question. The CA thus correctly affirmed the trial court as it summarily resolved the issue of ownership of the subject property in favor of respondent spouses. Petitioner, however, maintains that the issue of whether or not it was a builder in good faith should not have been peremptorily disposed of by the trial court. Petitioner decries the fact that it was not given an opportunity to submit evidence to establish good faith as regards the improvements it introduced on respondent spouses property.

Petitioner s contention is untenable. As correctly pointed out by the trial court and the CA, petitioner already admitted in its Amended Answer that the lot in dispute is covered by TCT No. T-769166 of respondent spouses. With this admission, petitioner can no longer claim that it was a builder in good faith. Good faith consists in the belief of the builder that the land he is building on is his and his ignorance of any defect or flaw in his title.[9] In this case, since petitioner, by its own admission, had knowledge of respondent spouses title over the subject lot, it was clearly in bad faith when it introduced improvements thereon. Further, the contract to sell[10] between petitioner and respondent spouses, the genuineness and due execution thereof was admitted by petitioner, clearly delineated the metes and bounds of the lot subject thereof. Attached to the said contract was a graphic illustration of the lot purchased by petitioner including a technical description thereof. Petitioner, as a real estate developer, is presumed to be experienced in its business and ought to have sufficient technical expertise to correctly determine the metes and bounds of the lands it acquires. Despite this, petitioner still introduced improvements on the lot not covered by the contract to sell. Petitioner s bad faith had been duly established by the pleadings and there was thus no need to further conduct any trial on the matter. Our ruling in Congregation of the Religious of the Virgin Mary vs. Court of Appeals[11]is particularly instructive: x x x As discussed earlier, petitioner has no right whatsoever to possess and construct permanent structures on the questioned land owned by respondents-spouses. Petitioner admits in its answer to the complaint that it introduced improvements on the subject lot without the consent and knowledge of respondents-spouses. It is thus a builder in bad faith. Again, we find no reversible error in the following ruling of the respondent court: "Which leads us to a discussion of whether or not appellant was in bad faith in introducing improvements on the subject land. It cannot be denied that appellant never gained title to the subject land as it admits to not having purchased the said lot (TSN, p. 81, November 9, 1992). Neither has appellant successfully shown any right to introduce improvements on the said land (its claim of grant of perpetual use of the same as a road lot

and its right to build on a right of way both having been rejected above). This being so, it follows that appellant was a builder in bad faith in that, knowing that the land did not belong to it and that it had no right to build thereon, it nevertheless caused the improvements in question to be erected."[12] Finally, petitioner s claim that there was a novation of contract because there was a second agreement between the parties due to the encroachment made by the national road on the property subject of the contract by 1,647 square meters, is unavailing. Novation, one of the modes of extinguishing an obligation, requires the concurrence of the following: (1) there is a valid previous obligation; (2) the parties concerned agree to a new contract; (3) the old contract is extinguished; and (4) there is valid new contract.[13] Novation may be express or implied. In order that an obligation may be extinguished by another which substitutes the same, it is imperative that it be so declared in unequivocal terms (express novation) or that the old and the new obligations be on every point incompatible with each other (implied novation).[14] In the instant case, there was no express novation because the second agreement was not even put in writing.[15] Neither was there implied novation since it was not shown that the two agreements were materially and substantially incompatible with each other. We quote with approval the following findings of the trial court: Since the alleged agreement between the plaintiffs [herein respondents] and defendant [herein petitioner] is not in writing and the alleged agreement pertains to the novation of the conditions of the contract to sell of the parcel of land subject of the instant litigation, ipso facto, novation is not applicable in this case since, as stated above, novation must be clearly proven by the proponent thereof and the defendant in this case is clearly barred by the Statute of Frauds from proving its claim.[16] In fine, the CA correctly affirmed the summary judgment rendered by the trial court. Considering the parties allegations and admissions in their respective pleadings filed with the court a quo, there existed no genuine issue as to any material fact so that respondent spouses as movants therein were entitled to a judgment as a matter of law.

WHEREFORE, premises considered, the instant Petition is hereby DENIED for lack of merit. The assailed Decision, dated August 3, 2000, of the Court of Appeals is AFFIRMED in toto. SO ORDERED.

TECHNOGAS PHIL. V. CA 268 SCRA 5

FACTS: The parties in this case are owners of adjoining lots in Paraaque, Metro Manila. It was discovered in a survey, that a portion of a building of petitioner, which was presumably constructed by its predecessor-ininterest, encroached on a portion of the lot owned by private respondent.Technogas owned property with buildings and walls. Uy bought an adjacent property. There was an agreement for Technogas to demolish the wall. Uy filed a complained but the case was dismissed. This prompted him to dig a hole along the wall, which led to the partial collapse of the wall. A case for malicious mischief was filed against Uy. ISSUE: Is petitioner considered a builder in bad faith because, as held by respondent Court, he is "presumed to know the metes and bounds of his property as described in his certificate of title"? Does petitioner succeed into the good faith or bad faith of his predecessor-in-interest which presumably constructed the building?

HELD: 1. Unless one is versed in the science of surveying, no one can determine the precise extent or location of the property by merely examining his proper title. 2. The supervening awareness of the encroachment by petitioner doesn't militate against its right to claim the status of builder in good faith. 3. Bad faith isn t imputable to a registered owner of a land when a part of his building encroaches upon a builder s land G.R. No. 140357 September 24, 2004

SPOUSES REYNALDO and EDITHA LOPEZ, petitioners, vs. MARGARITA SARABIA, respondent. DECISION CALLEJO, SR., J.: This is a petition for review on certiorari of the Decision1 of the Court of Appeals (CA) dated June 11, 1999 and the Resolution dated October 5, 1999 denying the motion for reconsideration thereof in CA-G.R. CV No. 50656 which affirmed with modification the Decision2 of the Regional Trial Court (RTC) of Kalibo, Aklan, Branch 6. Case for the Respondent Margarita Sarabia owned two (2) lots with a residential house built on one of the lots in Poblacion, Kalibo, Aklan. Spouses Reynaldo and Editha Lopez were renting the second floor of the house for P300.00 per month. On the other lot was a building rented by Dr. Nilda Tambong for P600.00 a month, with two (2) boarders upstairs paying P440.00 a month.3 Sometime in March 1984, the Spouses Lopez approached Margarita and asked her if they could construct additional rooms for their growing children. Margarita told them that she did not have the money for such construction project.

They then proposed that they could apply for a Pag-ibig Housing Loan from the Development Bank of the Philippines (DBP) and use Margarita s property as collateral. Margarita, however, informed them that her property had already been mortgaged to the Philippine National Bank (PNB) in 1978 in the amount of P20,000.00,4 and was, in fact, in danger of being foreclosed for non-payment of amortization. Her outstanding loan balance as of March 1984 had already ballooned to about P63,000.00. The Spouses Lopez tried to convince Margarita that it was better to transfer the mortgage to the DBP where interest rates were lower; Editha Lopez was a public school teacher and the monthly amortization could easily be deducted from her salary. They told Margarita that the PNB loan balance could be paid off from the proceeds of the loan from the DBP, and the excess could be used for the construction of the rooms. In order to facilitate the loan, it was, however, necessary that the property be in the name of the Spouses Lopez. Relying on the couple s good faith and assurances that they would religiously pay the amortization, Margarita agreed to their proposition. A document was thus executed denominated as "Assumption of Mortgage with Quitclaim."5 In said document, the Register of Deeds was authorized to cancel TCT No. T-4471 and TCT No. T-4474 over the two (2) parcels of land and issue new TCT s6 under the name of the Spouses Lopez covering the two lots. The Spouses Lopez then mortgaged the properties to DBP where they obtained a loan in the amount of P163,000.00. They paid the PNB, which then released the mortgage of Margarita. The Spouses Lopez ceased paying rentals to Margarita and even collected the rentals from the other lot as part of the payment of the monthly amortization. Sometime in October 1987, Reynaldo Lopez approached Margarita and informed her that he needed P30,000.00 to update their loan payments. Margarita gave him the amount as part of the refund to the payment of the PNB loan. She expected Reynaldo to give her an official receipt from the DBP, but did not receive any. Sensing something irregular, she went to the DBP to inquire about the status of the loan. She was aghast to find out that the loan amortization had not been paid and that her property was again in danger of being foreclosed.

Margarita was constrained to file an action with the RTC against the Spouses Lopez for annulment of document, specific performance and reconveyance with damages. The DBP was included as party-defendant. In her complaint, Margarita prayed for the following: a. Declaring the Assumption of Mortgage with Quitclaim null and void; b. Ordering the defendants Lopezes to redeem the parcels of land and residential house presently mortgaged to the Development Bank of the Philippines; c. Ordering the defendants to reconvey the certificates of title as well as the tax declarations of the said parcels of land and the house in favor of plaintiff; d. Ordering the defendants to pay the plaintiff the amount of P10,000.00 as attorney s fee; litigation expenses in the amount of P10,000.00, and as actual damage to the value of the property mentioned above to be determined by this Honorable Court, and monthly rental of P300.00 from 1984 up to actual payment.7 Case for the Petitioners In their Answer8 to the Complaint, the Spouses Lopez averred that it was Margarita who approached them to help her redeem her property from the PNB because it was going to be foreclosed. She was aware that the couple wanted to buy a house and lot of their own, and offered her property to them instead. The Spouses Lopez told her that they did not have the money to redeem the property, but if Margarita was certain in selling her house to them, they could arrange for a loan from the DBP, the proceeds of which the PNB loan could be paid in full and would form part of the purchase price. The balance would also be taken from the proceeds of the DBP loan. Pursuant to their mutual and verbal agreement, Margarita executed a Deed of Assumption of Mortgage with Quitclaim,9 authorizing the couple to assume her loan with the PNB over the two lots, together with all the improvements thereon and renouncing all her rights over the property. The same document authorized the Register of Deeds of Aklan to cancel TCT Nos. T-4471 and T-4474 and issue two (2) new certificates of

title in the name of the Spouses Reynaldo and Editha Lopez. In April 1984, Margarita asked for partial payment from the Spouses Lopez and was given the amount of P6,700.00 which the former acknowledged.10 On May 8, 1984, Margarita executed a Special Power of Attorney11 appointing the DBP to be her attorney-in-fact, where the latter would issue a check in favor of the PNB covering the amount of P63,307.34 as payment of the outstanding loan balance. Another check in the amount of P89,992.66 was also issued in the name of Margarita, as per the Distribution of Proceeds and Release Guide of the DBP.12 The couple has introduced improvements on the land since then, which cost them about P300,000.00. The Spouses Lopez claim and assert ownership over the subject properties, as evidenced by the TCTs issued in their names. On the part of DBP, it alleged in its answer with cross-claim that it had no knowledge of the agreement between Margarita and the Spouses Lopez. It granted a loan to the spouses in the amount of P163,500.00 and accepted the certificates of title presented to it by the Spouses Lopez over the two parcels of land as security/collateral. It had the right to rely on the certificates of title presented to it, which were free from all liens and encumbrances. The DBP was an innocent mortgagee for value. As crossclaim, DBP demanded payment from the Spouses Lopez the amount of the loan granted to them, plus damages for misrepresenting to the bank that they were the owners in fee simple of the subject properties which they mortgaged to the bank.13 The Findings of the RTC On November 29, 1994, the RTC rendered judgment in favor of Margarita. The trial court found that it was Margarita who sought the help of the Spouses Lopez so that she could redeem her property which was on the verge of being foreclosed by the PNB for non-payment of the loan amortization. By virtue of the documents executed by Margarita in favor of the Spouses Lopez, viz, Deed of Assumption and Quitclaim dated March 6, 1984,14 Offer to Sell dated March 20, 1984,15 and Release of Real Estate Mortgage dated May 9, 1984,16 the titles to the land were transferred and registered in the names of the latter. The Spouses Lopez applied for a Pagibig Housing Loan from the DBP using Margarita s property as security, and was granted thereof in the amount of P163,500.00. The PNB loan was paid

pursuant to the special power of attorney. Although another check in the amount of P87,000.00 was issued to Margarita and later endorsed by her for encashment, she testified that she never received the money.17 The Spouses Lopez ceased paying rentals and even collected the rentals of the other tenants which were supposed to be applied to the monthly amortization. The trial court found that the true intentions of the parties were not really embodied in the documents/instruments. The documentary, as well as parol evidence, clearly showed that Margarita did not really intend to convey her property to the petitioners. She merely agreed to lend her titles so that the Spouses Lopez could procure a bigger loan which she could not possibly obtain, considering her age and meager salary as Supervising Accounting Clerk in the Municipality of Kalibo. She agreed to sign the pertinent documents with the understanding that they were requirements of the bank in processing the loan applied for by the Spouses Lopez. The trial court continued to rule that the Spouses Lopez were in bad faith, so whatever improvements were made on the land were forfeited in favor of Margarita.18 The dispositive portion of the decision reads as follows: WHEREFORE, premises considered, judgment is hereby rendered as follows: 1. Declaring the Deed of Assumption and Quitclaim executed by the plaintiff in favor of the defendant spouses, Exhibit "G" of the plaintiff and Exhibit "4" of defendants a relatively simulated contract; 2. Declaring the conveyance of title in favor of the defendant spouses under TCT No. T-13472 and TCT No. T-13473, as a simulated or fictitious transfer, and therefore void; and that said spouses merely hold legal title in trust and for the benefit of the plaintiff; 3. Declaring the assumption by the defendant spouses of plaintiff s loan valid;

4. Declaring the loan obtained by the defendant spouses from the defendant bank valid and subsisting, but declaring the mortgage, giving the properties in question as a security for the payment thereof, null and void; 5. Ordering that the properties in question with all the existing improvements thereon, covered by TCT No. T-13472 and TCT No. T-13473 in the names of the defendant spouses, be conveyed in the name of the plaintiff upon payment of proper fees; and for the purpose, ordering the defendant bank to return the owner s duplicate of said certificates of title to the plaintiff; 6. Ordering the defendant spouses to vacate the premises and return possession over the same to the plaintiff; 7. Ordering the defendant spouses to pay Ten Thousand Pesos (P10,000.00) as attorney s fees, and litigation expenses, and to pay the costs.19 The Spouses Lopez appealed to the Court of Appeals. The CA affirmed the RTC finding that the nature of the transaction between Margarita and the Spouses Lopez was, verily, an equitable mortgage and not a sale. The CA, however, declared that the petitioners were builders in good faith. According to the CA, Margarita was aware and approved the construction/improvements undertaken by the Spouses Lopez; thus, forfeiture of the improvements in favor of Margarita was unwarranted. The fallo of the decision reads: WHEREFORE, the decision appealed from is AFFIRMED with the modification that, defendant-appellant Lopez spouses being considered builders in good faith, the improvements they introduced after the transaction in question be either purchased by plaintiffappellee Margarita Sarabia or removed at defendant-appellants own expense.20 The Spouses Lopez are now before the Court raising the following:

(1) THAT WHILE THE COURT OF APPEALS HAS CORRECTLY REVERSED THE FINDING OF THE TRIAL COURT THAT THE DEFENDANTS-APPELLANTS (HEREIN PETITIONERS) WERE NOT BUILDERS IN BAD FAITH AND CATEGORICALLY DECLARED THEM TO BE BUILDERS IN GOOD FAITH, IT FAILED TO APPLY CORRECTLY THE RULES ON BUILDER IN GOOD FAITH UNDER ART. 448 OF THE NEW CIVIL CODE ON THE OPTIONS OF THE OWNER OF THE LAND AND THE RIGHTS OF THE BUILDER IN GOOD FAITH; and (2) THAT WHILE THE HONORABLE COURT OF APPEALS HAS AFFIRMED THE RULING OF THE TRIAL COURT THAT THE REAL AGREEMENT BETWEEN THE PARTIES WAS A FORM OF EQUITABLE MORTGAGE AND NOT A SALE, IT FAILED TO DEFINE AND ADJUDICATE WITH CERTAINTY THE RELATIVE RIGHTS AND RECIPROCAL OBLIGATIONS OF THE PARTIES UNDER ART. 1616 OF THE NEW CIVIL CODE.21 Ruling of the Court There is no dispute that the transaction between the parties is one of equitable mortgage and not a sale as maintained by the petitioners. This was a finding correctly made by the trial court and the appellate court, which we find no cogent reason to disturb. No matter what nomenclature is given to a document, Article 1602 of the New Civil Code provides that the contract is presumed to be an equitable mortgage in any of the following cases: 1) When the price of a sale with right to repurchase is usually inadequate; 2) When the vendor remains in possession as lessee or otherwise; 3) When upon the expiration of the right to repurchase another instrument extending the period of redemption or granting a new period is executed; 4) When the purchaser retains for himself a part of the purchase price;

5) When the vendor binds himself to pay the taxes on the thing sold; 6) In any other case where it may be fairly inferred that the real intention of the parties is that the transaction shall secure the payment of a debt or the performance of any other obligation; In any of the foregoing cases, any money, fruits or other benefit to be received by the vendee as rent or otherwise shall be considered as interest which shall be subject to the usury laws. The pertinent document which is subject to scrutiny in this case is the Deed of Assumption of Mortgage with Quitclaim22 executed by Margarita in favor of the Spouses Lopez. The said document empowered the Spouses Lopez to assume the loan of Margarita with the PNB. And in consideration for such assumption of indebtedness, Margarita was considered to have waived all her rights and participation over the two parcels of land, together with all the improvements thereon, and that such titles were transferred to the Spouses Lopez. This document was followed by a Deed of Offer to Sell signed by Margarita bearing the value of the subject property which was P160,000.00, since the Deed of Assumption of Mortgage did not contain the amount of the purchase price of the property.23 In line with the basic requirement in our laws that the mortgagor be the absolute owner of the property sought to be mortgaged,24 it was, thus, made to appear that Margarita sold her property to the Spouses Lopez so that they could declare the same as collateral for the housing loan. While under the Deed of Assumption of Mortgage, Margarita allowed the transfer of title over the subject property in the name of the Spouses Lopez, the evidence showed that ownership thereof was not intended to be conveyed to them. Margarita was firm in her testimony that she merely allowed the Spouses Lopez to apply for a loan using her titles as collateral, so that the couple could help her redeem her property from PNB. She never made any offer to sell and never thought of such.25 Unfortunately, she signed the Deed of Assumption of Mortgage with Quitclaim and the Offer to Sell without actually reading and understanding the contents thereof.26 The real agreement was for the Spouses Lopez to apply for a loan in order to pay Margarita s indebtedness with the PNB. Margarita, in turn, would pay the Spouses Lopez by installment.27 The trial court correctly found that the Deed of Assumption of Mortgage did not

actually contain all the matters agreed upon by the parties prior to its execution.28 In the case of Lorbes v. Court of Appeals,29 the Court held that: The decisive factor in evaluating such agreement is the intention of the parties, as shown not necessarily by the terminology used in the contract but by all the surrounding circumstances, such as the relative situation of the parties at that time, the attitude, acts, conduct, declarations of the parties, the negotiations between them leading to the deed, and generally, all pertinent facts having a tendency to fix and determine the real nature of their design and understanding. As such, documentary and parol evidence may be submitted and admitted to prove the intention of the parties.30 The trial and appellate courts did not find the version of the petitioners credible, considering that the subsequent acts and conditions of the parties were more leaning to the presumption of an equitable mortgage and not of sale. First. The owner, Margarita, remained in possession of the house. If she really intended to sell her house, then she would have looked for another place to live. Second. It was inconceivable that Margarita would sell her house and the two lots just to pay the PNB loan. She would have necessarily retained one parcel of land which she could have called her own. Third. The acknowledgement receipt31 signed by Reynaldo Lopez showing that they were paid by Margarita the sum of P30,000.00 is quite telling. The said receipt states: This is to acknowledge from MISS MARGARITA SARABIA, of Mabini Street, Kalibo, Aklan, the amount of THIRTY THOUSAND PESOS (P30,000.00) as partial refund of the previous loan assumed by Engr. Reynaldo L. Lopez from the Philippine National Bank to be paid to the Development Bank of the Philippines.32

If it were a sale in favor of the couple, it behooved the Spouses Lopez to show why Margarita should pay them the amount, when it should be the other way around. Fourth. The Spouses Lopez never paid the monthly amortization. If they were truly the owner, then they would have protected their own property from being foreclosed. It bears stressing that the law favors the least transmission of rights and interests over a property in controversy. The purpose of the law is to prevent circumvention of the law on usury and the prohibition against a creditor appropriating the mortgaged property. Additionally, it is aimed to end unjust or oppressive transactions or violations in connection with the sale of the property. The wisdom of these provisions cannot be doubted, considering many cases of unlettered persons or even those with average intelligence invariably finding themselves in no position whatsoever to bargain fairly with their creditors.33 No doubt in this case, the Spouses Lopez took advantage of Margarita s advanced age and urgent necessity for money, which explains why she agreed to sign the documents without being fully aware of their meaning and contents. "Necessitous men are not, truly speaking, free men; but to answer a present emergency, will submit to any terms that the crafty may impose upon them."34 What was intended to be a mere loan so as to enjoin the foreclosure by the bank of her property, ended up as a transfer of property to the Spouses Lopez, which was not the real intention and agreement of the parties in the first place. This is a fact which the Spouses Lopez cannot deny. From all indications, the Spouses Lopez were quite dishonest in attempting to appropriate the property as their own when this was not their agreement with Margarita. Conceding that the transaction was not really a sale of the subject property, the Spouses Lopez now demand their rights for reimbursement for expenses and improvements made on the land under Articles 448 and 1616 of the Civil Code. This leads us to the pivotal question: Can the Spouses Lopez invoke Article 448 and claim the benefits of this provision as builders in good faith when they constructed improvements on the subject property?

The trial court found the Spouses Lopez in bad faith and ordered the forfeiture of the improvements in Margarita s favor. The CA disagreed with the trial court as it ruled: Construction of the improvements went on without the objections of Margarita. It can thus be safely concluded that, absent any objections, the Lopez spouses sincerely believed that as lessees, they had Margarita s approval to construct such improvements. Forfeiture of the improvements in Margarita s favor is thus not warranted.35 The petitioners allege that Article 448 applies in this case because they constructed the building on one of the lots in the concept of owner, after the title over the two lots had already been transferred in their names and out of the proceeds of their Pag-ibig loan. They believed that they have a right to build because they thought that they owned the land or believed themselves to have claim or title.36 The contention is untenable. Articles 44837 and 54638 of the New Civil Code, which allow full reimbursement of useful improvements and retention of the premises until reimbursement is made, apply only to a possessor in good faith, i.e., one who builds on land with the belief that he is the owner thereof. A builder in good faith is one who is unaware of any flaw in his title to the land at the time he builds on it.39 In this case, the petitioners cannot claim that they were not aware of any flaw in their title or were under the belief that they were owners of the subject properties. It was the agreement and intention that Margarita s titles would only be lent to them in order to secure the Pag-ibig Housing Loan, in which Margarita had a direct interest since the proceeds thereof were to be immediately applied to her mortgage obligation with the PNB. There was no agreement or intention to transfer ownership of the subject properties. The petitioners cannot claim to be owners. Hence, they cannot be considered builders in good faith. Article 448 is not applicable. More importantly, however, it must be remembered that the Spouses Lopez were lessees of Margarita who were renting the place for P300.00 a month. Such fact was never controverted. The CA unmistakably did not overlook this relationship but apparently erred in defining the rights of the

lessor and/or lessee with regard to indemnity for improvements made on the land. Article 448 does not apply to a case where one builds, plants, or sows on land where the only interest of the builder, planter, or sower is that of a holder, such as a tenant or a lessee.40 Thus, whether or not Margarita gave her consent to the construction so as to be considered builders in good faith, as ruled by the CA, is of no moment. As lessees, their right for reimbursement viz-a-viz the improvements made on the land is governed by Article 1678 of the New Civil Code which reads: Art. 1678. If the lessee makes, in good faith, useful improvements which are suitable to the use for which the lease is intended, without altering the form or substance of the property leased, the lessor upon the termination of the lease, shall pay the lessee one-half of the value of the improvements at that time. Should the lessor refuse to reimburse said amount, the lessee may remove the improvements, even though the principal thing may suffer damage thereby. He shall not, however, cause any more impairment upon the property leased than is necessary. With regard to ornamental expenses, the lessee shall not be entitled to any reimbursement, but he may remove the ornamental objects, provided no damage is caused to the principal thing, and the lessor does not choose to retain them by paying their value at the time the lease is extinguished. The petitioners reliance on Article 448 is, therefore, misplaced. Being mere lessees, the petitioners knew that their occupation of the premises would continue only for the life of the lease. Plainly, they cannot be considered as possessors nor builders in good faith.41 In the case of Sia v. Court of Appeals,42 we explained that: In the 1991 case of Cabangis v. Court of Appeals where the subject of the lease contract was also a parcel of land and the lessee s father constructed a family residential house thereon, and the lessee subsequently demanded indemnity for the improvements built on the lessor s land based on Articles 448 and 546 of the New Civil Code, we pointed out that reliance on said legal provisions was misplaced. "The reliance by the respondent Court of Appeals on Articles 448 and 546 of the Civil Code of the Philippines is misplaced.

These provisions have no application to a contract of lease which is the subject matter of this controversy. Instead, Article 1678 of the Civil Code applies. We quote: " Art. 1678. If the lessee makes, in good faith, useful improvements which are suitable to the use for which the lease is intended, without altering the form or substance of the property leased, the lessor upon termination of the lease, shall pay the lessee one-half of the value of the improvements at that time. Should the lessor refuse to reimburse said amount, the lessee may remove the improvements, even though the principal thing may suffer damage thereby. He shall not, however, cause any more impairment upon the property leased than is necessary. "

On the other hand, Article 448 governs the right of accession while Article 546 pertains to effects of possession. The very language of these two provisions clearly manifest their inapplicability to lease contracts. They provide:

The petitioners do not dispute the contention of the private respondent that her father Gaspar Devis, filled the leased parcel of land with truck loads of big stones or rocks (escumbro), and enclosed or walled the same with hollow blocks before constructing a residential house thereon. All these, being in the nature of expenses which augmented the value of the land, (Manresa, 270 cited in 2, A. Tolentino, Civil Code 110 [2nd ed., 1972) or increased the income from it, or improved its productivity, are useful improvements within the purview of the law (Alburo v. Villanueva, 7 Phil. 277 [1907]; Valencia v. Roxas, 13 Phil. 45 [1909]). But, it must be remembered, as in fact it is not controverted, that Gaspar Devis was a lessee by virtue of a lease contract between him and the City of Manila. As a mere lessee, he knew

that the parcel of land in question was not his but belonged to the latter. Even the respondent court conceded this fact when it stated that the private respondent was "not claiming prior possession much less ownership of the land as heir of her father." (Rollo, p. 16). Thus, the improvements that the private respondent s father had introduced in the leased premises were done at his own risk as lessee. The right to indemnity equivalent to one-half of the value of the said improvements the house, the filling materials, and the hollow block fence or wall is governed, as earlier adverted to, by the provisions of Art. 1678, first paragraph of the Civil Code above quoted. But this right to indemnity exists only if the lessor opts to appropriate the improvements (Alburo v. Villanueva, supra, note 10 at 279-280; Valencia v. Ayala de Roxas, supra, note 10 at 46). The refusal of the lessor to pay the lessee one-half of the value of the useful improvements gives rise to the right of removal. On this score, the commentary of Justice Paras is enlightening. "Note that under the 1st paragraph of Art. 1678, the law on the right of REMOVAL says that should the lessor refuse to reimburse said amount, the lessee may remove the improvements, even though the principal thing may suffer thereby. While the phrase even though implies that Art. 1678 always applies regardless of whether or not the improvements can be removed without injury to the leased premises, it is believed that application of the Article cannot always be done. The rule is evidently intended for cases where a true accession takes place as when part of the land leased is, say, converted into a fishpond; and certainly not where as easily removable thing (such as a wooden fence) has been introduced. There is no doubt that in a case involving such a detachable fence, the lessee can take the same away with him when the lease expires (5 E. Paras, Civil code of the Philippines Annotated 345 [11th ed., 1986])."43

The petitioners have made substantial improvements on the land for which they seek indemnity. Petitioner Reynaldo Lopez testified that there are now three buildings standing on the two parcels of land: the first building is where Margarita and they are presently residing, constructed wayback in 1970; the second building, with an estimated cost of P300,000.00, is the one the couple constructed after obtaining the loan from DBP, with an office at the ground floor and the second floor with three rooms also occupied by the Lopezes; and the third building is the old house where the first floor is being rented by Dr. Nilda Tambong with boarders on the second floor.44 It must be stressed that the right to indemnity under Article 1678 arises only if the lessor opts to appropriate the improvements. The respondent (Margarita) would become the owner of the building constructed by the petitioners by reimbursing to the couple one-half (1/2) of the value of the building at the time it was built. This option to pay such indemnity is given to herein respondent. On the other hand, the petitioners do not actually have the right to demand that they be paid therefor.45 Neither do they have the right to retain in the premises until reimbursement is made. If Margarita refuses to pay indemnity, the petitioners sole right then is to remove the improvements without causing anymore impairment upon the lot than is necessary.46 Notwithstanding the finding that the nature of the transaction is an equitable mortgage, the petitioners have no basis to invoke Article 1616.47 The petitioners attempt to seek reimbursement for whatever expenses have been incurred or resulted from this transaction with Margarita cannot prosper. It must be noted that after the transfer of title in the name of the petitioners, the latter ceased paying rentals to Margarita since 1984 and, in fact, collected the rentals from the other tenants. We find that the petitioners have benefited more than enough, having stayed in the premises without paying rentals therefor. On the other hand, Margarita was deprived of the fruits and enjoyment of her property. Thus, the petition has no merit. WHEREFORE, the petition is DENIED. The Decision of the Court of Appeals is AFFIRMED with the modification that respondent Margarita Sarabia is DIRECTED to exercise, within thirty (30) days from the finality of this

decision, her option of either paying one-half of the value of the improvements made on the land at that time they were made, or to demand the removal by the petitioners of the improvements made on the subject property at their expense. No costs. SO ORDERED. PROGRAMME INCORPORATED, Petitioner, vs. PROVINCE OF BATAAN,1 Respondent. DECISION CORONA, J.: In this petition filed under Rule 45 of the Rules of Court, petitioner Programme Incorporated contests the Court of Appeals (CA) decision2 and resolution3 upholding respondent Province of Bataan s ownership of Piazza Hotel and the land on which it stands. The assailed decision in CA-G.R. CV No. 49135 affirmed the decision of the Regional Trial Court (RTC), Branch 4, Balanga, Bataan in a suit for preliminary injunction and sum of money filed by petitioner against Bataan Shipyard and Engineering Co., Inc. (BASECO). The case was docketed as Civil Case No. 129-ML. The dispositive portion of the trial court decision read: WHEREFORE, in view of all the foregoing considerations, judgment is hereby rendered dismissing the complaint, without pronouncement as to costs. Similarly, [BASECO s] counterclaim is dismissed. On the complaint in intervention, judgment is hereby rendered ordering [petitioner] to pay [respondent] the rentals for the leased premises in question, namely, the Piazza Hotel and the Mariveles Lodge, situated at the Bataan Export Processing Zone (BEPZ) Compound in Mariveles, Bataan, at the rate of six thousand five hundred pesos (P6,500.00) per month for both establishments, starting in August 1989 with legal interest at 6% per annum, up to and until the legal arrearages shall have been fully paid, and to pay the succeeding rentals therefor at the same rate.

SO ORDERED.4 The controversy arose from the following facts. BASECO was the owner of Piazza Hotel and Mariveles Lodge, both located in Mariveles, Bataan. On May 14, 1986, BASECO granted petitioner a contract of lease over Piazza Hotel at a monthly rental of P6,500 for three years, i.e., from January 1, 1986 to January 1, 1989, subject to renewal by mutual agreement of the parties. After the expiration of the three-year lease period, petitioner was allowed to continue operating the hotel on monthly extensions of the lease. In April 1989, however, the Presidential Commission on Good Government (PCGG) issued a sequestration order against BASECO pursuant to Executive Order No. 1 of former President Corazon C. Aquino.5 Among the properties provisionally seized and taken over was the lot on which Piazza Hotel stood. On July 19, 1989, however, Piazza Hotel was sold at a public auction for non-payment of taxes to respondent Province of Bataan. The title of the property was transferred to respondent. BASECO s Transfer Certificate of Title (TCT) No. T-59631 was cancelled and a new one, TCT No. T-128456, was issued to the Province of Bataan. On July 21, 1989, petitioner filed a complaint for preliminary injunction and collection of sum of money against BASECO (Civil Case No. 129-ML).6 Respondent, as the new owner of the property, filed a motion for leave to intervene on November 22, 1990. After its motion was granted, respondent filed a complaint-in-intervention praying, inter alia, that petitioner be ordered to vacate Piazza Hotel and Mariveles Lodge for lack of legal interest. During the pre-trial of the complaint-in-intervention, the parties agreed that the case7 be tried on the sole issue of whether respondent province, as complainant-intervenor, was the legitimate owner of the Piazza Hotel and Mariveles Lodge.

On February 3, 1995, after trial on the merits, the trial court rendered judgment in favor of respondent.1avvphil.net On appeal, the CA addressed the issue of ownership of Piazza Hotel and Mariveles Lodge as follows: [W]e affirm the trial court s ruling that [respondent] Province of Bataan has established by preponderance of evidence its claim of ownership of Piazza Hotel and Mariveles Lodge. In fact, [petitioner] has not presented evidence proving its ownership of the said buildings[, whereas respondent presented] a tax declaration and certificate of title over the same properties, over which it now exercises full control and dominion. The fact that the subject properties were placed under sequestration is of no moment for the PCGG is not an owner but a conservator who can exercise only powers of administration over property sequestered, frozen or provisionally taken over. As the owner of said properties, [respondentintervenor] is entitled to the payment of the monthly rental in the sum of P6,500.00 as ruled by the trial court.8 (emphasis ours) We agree with the appellate court. Time and again, we have ruled that factual matters are best evaluated by trial courts which can scrutinize evidence and hear testimony presented and offered by the parties (in this case, on the issue of ownership of the subject property). All the more does this principle ring true in this petition since such factual determination by the RTC was upheld by the CA.9 Only questions of law are the proper subject of a petition for review on certiorari in this Court, unless any of the known exceptions is extant in this case.10 There is none. The evidence clearly established respondent s ownership of Piazza Hotel.11 First, the title of the land on which Piazza Hotel stands was in the name of respondent.12 Second, Tax Declaration No. 12782 was in the name of respondent as owner of Piazza Hotel.13 A note at the back of the tax declaration read: Transferred by virtue of a final bill of sale executed by the Provincial [Treasurer] of Bataan in favor of the Provincial Government on Feb. 13, 1989[, a] year after the expiration of the redemption period from date of

auction sale held on Feb. 12, 1988 of all real property declared in the name of [BASECO].14 (emphasis ours) Third, petitioner was doubtlessly just a lessee. In the lease contract annexed to the complaint, petitioner in fact admitted BASECO s (respondent s predecessor-in-interest) ownership then of the subject property. A stipulation in the contract read: WHEREAS, the lessor (BASECO) is the owner of the building PIAZZA HOTEL and its outlet MARIVELES LODGE located at BASECO, Mariveles, Bataan xxx15 (emphasis ours) The Rules of Court states that "[a]n admission, verbal or written, made by a party in the course of the proceedings in the same case, does not require proof. The admission may be contradicted only by showing that it was made through palpable mistake or that no such admission was made."16 [Such admissions] may be made in (a) the pleadings filed by the parties, (b) in the course of the trial either by verbal or written manifestations or stipulations, or (c) in other stages of the judicial proceeding, as in the pretrial of the case. Admissions obtained through depositions, written interrogatories or requests for admission are also considered judicial admissions.17 (emphasis ours) "To be considered as a judicial admission, the same must be made in the same case in which it is offered."18 In its own complaint19 for preliminary injunction and sum of money, petitioner acknowledged that it was not the owner of the property when it stated that "[BASECO] lease[d] to [petitioner] the building Piazza Hotel and its outlet Mariveles Lodge xxx for monthly rentals of P6,500.00."20 Petitioner could not possibly be the owner of a building merely leased to it.21 Furthermore, petitioner s reference to Article 44822 of the Civil Code to justify its supposed rights as "possessor in good faith" was erroneous.

The benefits granted to a possessor in good faith cannot be maintained by the lessee against the lessor because, such benefits are intended to apply only to a case where one builds or sows or plants on land which he believes himself to have a claim of title and not to lands wherein one s only interest is that of a tenant under a rental contract, otherwise, it would always be in the power of a tenant to improve his landlord out of his property. Besides, as between lessor and lessee, the Code applies specific provisions designed to cover their rights. Hence, the lessee cannot claim reimbursement, as a matter of right, for useful improvements he has made on the property, nor can he assert a right of retention until reimbursed. His only remedy is to remove the improvement if the lessor does not choose to pay its value; but the court cannot give him the right to buy the land.23 Petitioner s assertion that Piazza Hotel was constructed "at (its) expense" found no support in the records. Neither did any document or testimony prove this claim. At best, what was confirmed was that petitioner managed and operated the hotel. There was no evidence that petitioner was the one which spent for the construction or renovation of the property. And since petitioner s alleged expenditures were never proven, it could not even seek reimbursement of one-half of the value of the improvements upon termination of the lease under Article 167824 of the Civil Code. Finally, both the trial and appellate courts declared that the land as well as the improvement thereon (Piazza Hotel) belonged to respondent. We find no reason to overturn this factual conclusion. Since this petition for review on certiorari was clearly without legal and factual basis, petitioner s counsel should not have even filed this appeal. It is obvious that the intention was merely to delay the disposition of the case. WHEREFORE, the petition is hereby DENIED. The decision and resolution of the Court of Appeals in CA-G.R. CV No. 49135 are AFFIRMED. Costs against petitioner. Same costs against Atty. Benito R. Cuesta I, petitioner s counsel, for filing this flimsy appeal, payable within ten (10) days from finality of this decision.

SO ORDERED.

Geminiano v. CA [G.R. No. 120303. July 24, 1996.] Third Division, Davide Jr (J): 4 concur Facts: Lot 3765-B-1 (314 sq. m.) was originally owned by Paulina Amado vda. de Geminiano, the mother of Federico, Maria, Ernesto, Asuncion, Larry and Marlyn Geminiano. On a 12-sq. m. portion of that lot stood the Geminianos' unfinished bungalow, which the Geminianos sold in November 1978 to Dominador and Mary Nicolas for the sum of P6,000.00, with an alleged promise to sell to the latter that portion of the lot occupied by the house. Subsequently, Paulina Amado-Geminiano executed a contract of lease over a 126 sq. m. portion of the lot, including that portion on which the house stood, in favor of the Nicolas spouse for P40 per month for a period of 7 years commencing on 15 November 1978. The Nicolas spouses then introduced additional improvements and registered the house in their names. After the expiration of the lease contract in November 1985, however, the Paulina refused to accept the monthly rentals. It turned out that the lot in question was the subject of a suit, which resulted in its acquisition by one Maria Lee in 1972. In 1982, Lee sold the lot to Lily Salcedo, who in turn sold it in 1984 to the spouses Agustin and Ester Dionisio. On 14 February 1992, the Dionisio spouses executed a Deed of Quitclaim over the said property in favor of the Geminianos. As such, the lot was registered in the latter's names. On 9 February 1993, the Geminianos sent, via registered mail, a letter addressed to Mary Nicolas demanding that she vacate the premises and pay the rentals in arrears within 20 days from notice. Upon failure of the Nicolas spouses to heed the demand, the Geminianos filed with the MTCC of Dagupan City a complaint for unlawful detainer and damages. The trial court held that there was no lease to speak of to be renewed as the lot was acquired by Maria Lee in 1972, and that if indeed there is a legal lease existing, its renewal can only be made on a month-to-month pursuant to Article 1687 of the Civil Code; that the lessees were not

builders in good faith and the reimbursement as such are governed by Article 1678; and that the value of the house and improvements was P180,000 as there was controverting evidence presented. The Court thus ordered the Nicolas spouses to vacate the premises, to pay the Geminianos P40 a month as reasonable compensation for their stay thereon from the filing of the complaint on 14 April 1993 until they vacated, and to pay the sum of P1,000 as attorney's fees, plus costs. On appeal by the Nicolas spouses, the RTC Dagupan City reversed the trial court's decision and rendered a new judgment: (1) ordering the Geminianos to reimburse the Nicolas spouses for the value of the house and improvements in the amount of P180,000.00 and to pay the latter P10,000.00 as attorney's fees and P2,000.00 as litigation expenses; and (2) allowing the Nicolas spouses to remain in possession of the premises until they were fully reimbursed for the value of the house. It ruled that since the Nicolas spouses were assured by the Geminianos that the lot they leased would eventually be sold to them, they could be considered builders in good faith, and as such, were entitled to reimbursement of the value of the house and improvements with the right of retention until reimbursement had been made. On appeal, this time by Geminianos, the Court of Appeals affirmed the decision of the RTC and denied the Geminianos' motion for reconsideration. Hence, the petition for review on certiorari. The Supreme Court granted the petition; reversing and setting aside the decision of the Court of Appeals of 27 January 1995 in CA-GR SP 34337; and reinstating the decision of Branch 3 of the Municipal Trial Court in Cities of Dagupan City in Civil Case 9214; with costs against the Nicolas spouses. 1. Non-owner of the premises may lease property While the right to lease property is an incident of title and possession, a person may be a lessor and occupy the position of a landlord to the tenant although he is not the owner of the premises leased. After all, ownership of the property is not being transferred, only the temporary use and enjoyment thereof.

2. Nicolas spouses estopped; Estoppel applies even if lessor has no title, may be asserted not only by original lessor but also those who succeed to his title The Nicolas spouses came into possession of a 126 sq. m. portion of the said lot by virtue of a contract of lease executed by the Geminianos' mother in their favor. The juridical relation between the Geminianos' mother as lessor, and the Nicolas spouses as lessees, is therefore wellestablished, and carries with it a recognition of the lessor's title. The lessees who had undisturbed possession for the entire term under the lease, are then estopped to deny their landlord's title, or to assert a better title not only in themselves, but also in some third person while they remain in possession of the leased premises and until they surrender possession to the landlord. This estoppel applies even though the lessor had no title at the time the relation of lessor and lessee was created, and may be asserted not only by the original lessor, but also by those who succeed to his title. 3. Lessees not possessors not builders in good faith Being mere lessees, the Nicolas spouses knew that their occupation of the premises would continue only for the life of the lease. Plainly, they cannot be considered as possessors nor builders in good faith. 4. Article 448 in relation to Article 546 applies only to a possessor in good faith; does not apply to lessee Article 448 of the Civil Code, in relation to Article 546 of the same Code, which allows full reimbursement of useful improvements and retention of the premises until reimbursement is made, applies only to a possessor in good faith, i.e., one who builds on land with the belief that he is the owner thereof. It does not apply where one's only interest is that of a lessee under a rental contract; otherwise, it would always be in the power of the tenant to "improve" his landlord out of his property. 5. Alleged option to buy not supported by evidence; Promise unenforceable unless option is in writing Neither the deed of sale over the house nor the contract of lease contained an option in favor of the Nicolas spouses to purchase the said lot. The first thing that the spouses should have done was to reduce the alleged promise into writing, because under Article 1403 of the Civil Code, an

agreement for the sale of real property or an interest therein is unenforceable, unless some note or memorandum thereof be produced. Not having taken any steps in order that the alleged promise to sell may be enforced, the private respondents cannot bank on that promise and profess any claim nor color of title over the lot in question. 6. Option does not render the Nicolas spouses builders in good faith Even if the Germinianos indeed promised to sell, it would not make the spouses possessors or builders in good faith so as to be covered by the provisions of Article 448 of the Civil Code. The latter cannot raise the mere expectancy of ownership of the lot because the alleged promise to sell was not fulfilled nor its existence even proven. 7. Pecson v. CA does not apply; No forced co-ownership There is no need to apply by analogy the provisions of Article 448 on indemnity as was done in Pecson vs. Court of Appeals, because the situation sought to be avoided and which would justify the application of that provision, is not present in the present case. "A state of forced coownership" would not be created between the Germinianos and the Nicolas spouses. 8. Lessees governed by Article 1678 The rights of the lessees are governed by Article 1678 of the Civil Code which allows reimbursement to the extent of one-half of the value of the useful improvements. The right to indemnity under Article 1678 of the Civil Code, however, arises only if the lessor opts to appropriate the improvements. Since the Germinianos refused to exercise that option, the Nicolas spouses cannot compel them to reimburse the one-half value of the house and improvements. Neither can they retain the premises until reimbursement is made. The spouses sole right then is to remove the improvements without causing any more impairment upon the property leased than is necessary.

SULO SA NAYON, INC. and/or PHILIPPINE VILLAGE HOTEL, INC. and JOSE MARCEL E. PANLILIO, Petitioners,

G.R. No. 170923

Present: PUNO, C.J., Chairperson, CARPIO, CORONA, AZCUNA, and

- versus -

LEONARDO-DE CASTRO, JJ.

Promulgated: January 20, 2009 NAYONG PILIPINO FOUNDATION, Respondent. x---------------------------------------------------------- x

DECISION PUNO, C.J.: On appeal are the Court of Appeals (CA s) October 4, 2005 Decision in CA-G.R. SP No. 74631 and December 22, 2005 Resolution, reversing the

November 29, 2002 Decision of the Regional Trial Court (RTC) of Pasay City in Civil Case No. 02-0133. The RTC modified the Decision of the Metropolitan Trial Court (MeTC) of Pasay City which ruled against petitioners and ordered them to vacate the premises and pay their arrears. The RTC declared petitioners as builders in good faith and upheld their right to indemnity. The facts are as follows: Respondent Nayong Pilipino Foundation, a government-owned and controlled corporation, is the owner of a parcel of land in Pasay City, known as the Nayong Pilipino Complex. Petitioner Philippine Village Hotel, Inc. (PVHI), formerly called Sulo sa Nayon, Inc., is a domestic corporation duly organized and existing under Philippine laws. Petitioner Jose Marcel E. Panlilio is its Senior Executive Vice President. On June 1, 1975, respondent leased a portion of the Nayong Pilipino Complex, consisting of 36,289 square meters, to petitioner Sulo sa Nayon, Inc. for the construction and operation of a hotel building, to be known as the Philippine Village Hotel. The lease was for an initial period of 21 years, or until May 1996. It is renewable for a period of 25 years under the same terms and conditions upon due notice in writing to respondent of the intention to renew at least 6 months before its expiration. Thus, on March 7, 1995, petitioners sent respondent a letter notifying the latter of their intention to renew the contract for another 25 years. On July 4, 1995, the parties executed a Voluntary Addendum to the Lease Agreement. The

addendum was signed by petitioner Jose Marcel E. Panlilio in his official capacity as Senior Executive Vice President of the PVHI and by Chairman Alberto A. Lim of the Nayong Pilipino Foundation. They agreed to the renewal of the contract for another 25 years, or until 2021. Under the new agreement, petitioner PVHI was bound to pay the monthly rental on a per square meter basis at the rate of P20.00 per square meter, which shall be subject to an increase of 20% at the end of every 3-year period. At the time of the renewal of the lease contract, the monthly rental amounted to P725,780.00. Beginning January 2001, petitioners defaulted in the payment of their monthly rental. Respondent repeatedly demanded petitioners to pay the arrears and vacate the premises. The last demand letter was sent on March 26, 2001. On September 5, 2001, respondent filed a complaint for unlawful detainer before the MeTC of Pasay City. The complaint was docketed as Civil Case No. 708-01. Respondent computed the arrears of petitioners in the amount of twenty-six million one hundred eighty-three thousand two hundred twenty-five pesos and fourteen centavos (P26,183,225.14), as of July 31, 2001. On February 26, 2002, the MeTC rendered its decision in favor of respondent. It ruled, thus: . . . . The court is convinced by the evidence that indeed, defendants defaulted in the payment of their rentals. It is basic

that the lessee is obliged to pay the price of the lease according to the terms stipulated (Art. 1657, Civil Code). Upon the failure of the lessee to pay the stipulated rentals, the lessor may eject (sic) and treat the lease as rescinded and sue to eject the lessee (C. Vda[.] De Pamintuan v. Tiglao, 53 Phil. 1). For nonpayment of rentals, the lessor may rescind the lease, recover the back rentals and recover possession of the leased premises. .. xxx . . . . Improvements made by a lessee such as the defendants herein on leased premises are not valid reasons for their retention thereof. The Supreme Court has occasion to address a similar issue in which it ruled that: The fact that petitioners allegedly made repairs on the premises in question is not a reason for them to retain the possession of the premises. There is no provision of law which grants the lessee a right of retention over the leased premises on that ground. Article 448 of the Civil Code, in relation to Article 546, which provides for full reimbursement of useful improvements and retention of the premises until reimbursement is made, applies only to a possessor in good faith, i.e., one who builds on a land in the belief that he is the owner thereof. This right of retention does not apply to a mere lessee, like the petitioners, otherwise, it would always be in his power to improve his landlord out of the latter s property (Jose L. Chua and Co Sio Eng vs. Court of Appeals and Ramon Ibarra, G.R. No. 109840, January 21, 1999). Although the Contract of Lease stipulates that the building and all the improvements in the leased premises belong to the defendants herein, such will not defeat the right of the plaintiff to its property as the defendants failed to pay

their rentals in violation of the terms of the contract. At most, defendants can only invoke [their] right under Article 1678 of the New Civil Code which grants them the right to be reimbursed one-half of the value of the building upon the termination of the lease, or, in the alternative, to remove the improvements if the lessor refuses to make reimbursement.

The dispositive portion of the decision reads as follows: WHEREFORE, premises considered, judgment is hereby rendered in favor of Nayong Pilipino Foundation, and against the defendant Philippine Village Hotel, Inc[.], and all persons claiming rights under it, ordering the latter to: 1. 2. VACATE the subject premises surrender possession thereof to plaintiff; and

PAY plaintiff its rental arrearages in the sum of TWENTY SIX MILLION ONE HUNDRED EIGHTY THREE THOUSAND TWO HUNDRED TWENTY FIVE PESOS AND 14/100 (P26,183,225.14) incurred as of July 31, 2001; PAY plaintiff the sum of SEVEN HUNDRED TWENTY FIVE THOUSAND SEVEN HUNDRED EIGHTY PESOS (P725,780.00) per month starting from August 2001 and every month thereafter by way of reasonable compensation for the use and occupation of the premises; PAY plaintiff the sum of FIFTY THOUSAND PESOS (P50,000.00) by way of attorney s fees[; and] PAY the costs of suit.

3.

4.

5.

The complaint against defendant Jose Marcel E. Panlilio is hereby dismissed for lack of cause of action. The said defendant s counterclaim however is likewise dismissed as the complaint does not appear to be frivolous or maliciously instituted.

SO ORDERED. Petitioners appealed to the RTC which modified the ruling of the MeTC. It held that: . . . it is clear and undisputed that appellants-lessees were expressly required to construct a first-class hotel with complete facilities. The appellants were also unequivocally declared in the Lease Agreement as the owner of the improvements so constructed. They were even explicitly allowed to use the improvements and building as security or collateral on loans and credit accommodations that the Lessee may secure for the purpose of financing the construction of the building and other improvements (Section 2; pars. A to B, Lease Agreement). Moreover, a time frame was setforth (sic) with respect to the duration of the lease initially for 21 years and renewable for another 25 years in order to enable the appellants-lessees to recoup their huge money investments relative to the construction and maintenance of the improvements. xxx Considering therefore, the elements of permanency of the construction and substantial value of the improvements as well as the undispute[d] ownership over the land improvements, these, immensely engender the application of Art. 448 of the Civil Code. The only remaining and most crucial issue to be resolved is whether or not the appellants as builders have acted in good faith in order for Art. 448 in relation to Art. 546 of the Civil Code may apply with respect to their rights over improvements. xxx

. . . it is undeniable that the improvement of the hotel building of appellants (sic) PVHI was constructed with the written consent and knowledge of appellee. In fact, it was precisely the primary purpose for which they entered into an agreement. Thus, it could not be denied that appellants were builders in good faith. Accordingly, and pursuant to Article 448 in relation to Art. 546 of the Civil Code, plaintiff-appellee has the sole option or choice, either to appropriate the building, upon payment of proper indemnity consonant to Art. 546 or compel the appellants to purchase the land whereon the building was erected. Until such time that plaintiff-appellee has elected an option or choice, it has no right of removal or demolition against appellants unless after having selected a compulsory sale, appellants fail to pay for the land (Ignacio vs. Hilario; 76 Phil. 605). This, however, is without prejudice from the parties agreeing to adjust their rights in some other way as they may mutually deem fit and proper. The dispositive portion of the decision of the RTC reads as follows: WHEREFORE, and in view of the foregoing, judgment is hereby rendered modifying the decision of [the] MTC, Branch 45 of Pasay City rendered on February 26, 2002 as follows: 1. Ordering plaintiff-appellee to submit within thirty (30) days from receipt of a copy of this decision a written manifestation of the option or choice it selected, i.e., to appropriate the improvements upon payment of proper indemnity or compulsory sale of the land whereon the hotel building of PVHI and related improvements or facilities were erected;

2.

Directing the plaintiff-appellee to desist and/or refrain from doing acts in the furtherance or exercise of its rights and demolition against appellants unless and after having selected the option of compulsory sale and appellants failed to pay [and] purchase the land within a reasonable time or at such time as this court will direct; Ordering defendants-appellants to pay plaintiffappellee [their] arrears in rent incurred as of July 31, 2001 in the amount of P26,183,225.14; Ordering defendants-appellants to pay to plaintiffappellee the unpaid monthly rentals for the use and occupation of the premises pending this appeal from July to November 2002 only at P725,780.00 per month; The fourth and fifth directives in the dispositive portion of the trial court s decision including that the last paragraph thereof JME Panlilio s complaint is hereby affirmed; The parties are directed to adjust their respective rights in the interest of justice as they may deem fit and proper if necessary.

3.

4.

5.

6.

SO ORDERED. Respondent appealed to the CA which held that the RTC erroneously applied the rules on accession, as found in Articles 448 and 546 of the Civil Code when it held that petitioners were builders in good faith and, thus, have the right to indemnity. The CA held:

By and large, respondents are admittedly mere lessees of the subject premises and as such, cannot validly claim that they are builders in good faith in order to solicit the application of Articles 448 and 546 of the Civil Code in their favor. As it is, it is glaring error on the part of the RTC to apply the aforesaid legal provisions on the supposition that the improvements, which are of substantial value, had been introduced on the leased premises with the permission of the petitioner. To grant the respondents the right of retention and reimbursement as builders in good faith merely because of the valuable and substantial improvements that they introduced to the leased premises plainly contravenes the law and settled jurisprudential doctrines and would, as stated, allow the lessee to easily improve the lessor out of its property. . . . . Introduction of valuable improvements on the leased premises does not strip the petitioner of its right to avail of recourses under the law and the lease contract itself in case of breach thereof. Neither does it deprive the petitioner of its right under Article 1678 to exercise its option to acquire the improvements or to let the respondents remove the same. Petitioners Motion for Reconsideration was denied. Hence, this appeal. Petitioners assign the following errors: I THE HONORABLE COURT OF APPEALS COMMITTED A GRAVE REVERSIBLE ERROR IN NOT HOLDING THAT PETITIONERS WERE BUILDERS IN GOOD FAITH OVER THE

SUBSTANTIAL AND VALUABLE IMPROVEMENTS WHICH THEY HAD INTRODUCED ON THE SUBJECT PROPERTY, THUS COMPELLING THE APPLICATION OF ARTICLE 448 OF THE CIVIL CODE IN RELATION TO ARTICLE 546 OF THE SAME CODE, INSTEAD OF ARTICLE 1678 OF THE CIVIL CODE. II THE HONORABLE COURT OF APPEALS COMMITTED A SERIOUS REVERSIBLE ERROR WHEN IT DISREGARDED THE FACT THAT THE LEASE CONTRACT GOVERNS THE RELATIONSHIP OF THE PARTIES AND CONSEQUENTLY THE PARTIES MAY BE CONSIDERED TO HAVE IMPLIEDLY WAIVED THE APPLICATION OF ARTICLE 1678 OF THE CIVIL CODE TO THE INSTANT CASE. III ASSUMING ARGUENDO THAT THE PETITIONERS ARE NOT BUILDERS IN GOOD FAITH, THE HONORABLE COURT OF APPEALS COMMITTED A GRAVE REVERSIBLE ERROR WHEN IT OVERLOOKED THE FACT THAT RESPONDENT ALSO ACTED IN BAD FAITH WHEN IT DID NOT HONOR AND INSTEAD BREACHED THE LEASE CONTRACT BETWEEN THE PARTIES, THUS BOTH PARTIES ACTED AS IF THEY ARE IN GOOD FAITH. IV TO SANCTION THE APPLICATION OF ARTICLE 1678 OF THE CIVIL CODE INSTEAD OF ARTICLE 448 OF THE CIVIL CODE IN RELATION TO ARTICLE 546 OF THE SAME CODE WOULD NOT ONLY WREAK HAVOC AND CAUSE SUBSTANTIAL INJURY TO THE RIGHTS AND INTERESTS OF PETITIONER PHILIPPINE VILLAGE HOTEL, INC. WHILE RESPONDENT NAYONG PILIPINO FOUNDATION, IN COMPARISON THERETO, WOULD SUFFER ONLY SLIGHT OR INCONSEQUENTIAL INJURY

OR LOSS, BUT ALSO WOULD CONSTITUTE UNJUST ENRICHMENT ON THE PART OF RESPONDENT AT GREAT EXPENSE AND GRAVE PREJUDICE OF PETITIONERS. V THE HONORABLE COURT OF APPEALS COMMITTED A GRAVE REVERSIBLE ERROR IN NOT HOLDING THAT THE COURTS A QUO DID NOT ACQUIRE JURISDICTION OVER THE UNLAWFUL DETAINER CASE FOR NON-COMPLIANCE WITH JURISDICTIONAL REQUIREMENTS DUE TO THE ABSENCE OF A NOTICE TO VACATE UPON PETITIONERS. First, we settle the issue of jurisdiction. Petitioners argue that the MeTC did not acquire jurisdiction to hear and decide the ejectment case because they never received any demand from respondent to pay rentals and vacate the premises, since such demand is a jurisdictional requisite. We reiterate the ruling of the MeTC, RTC and CA. Contrary to the claim of petitioners, documentary evidence proved that a demand letter dated March 26, 2001 was sent by respondent through registered mail to petitioners, requesting them to pay the rental arrears or else it will be constrained to file the appropriate legal action and possess the leased premises. Further, petitioners argument that the demand letter is inadequate because it contained no demand to vacate the leased premises does not persuade. We have ruled that: . . . . The word vacate is not a talismanic word that must be employed in all notices. The alternatives in this case

are clear cut. The tenants must pay rentals which are fixed and which became payable in the past, failing which they must move out. There can be no other interpretation of the notice given to them. Hence, when the petitioners demanded that either he pays P18,000 in five days or a case of ejectment would be filed against him, he was placed on notice to move out if he does not pay. There was, in effect, a notice or demand to vacate.

In the case at bar, the language of the demand letter is plain and simple: respondent demanded payment of the rental arrears amounting to P26,183,225.14 within ten days from receipt by petitioners, or respondent will be constrained to file an appropriate legal action against petitioners to recover the said amount. The demand letter further stated that respondent will possess the leased premises in case of petitioners failure to pay the rental arrears within ten days. Thus, it is clear that the demand letter is intended as a notice to petitioners to pay the rental arrears, and a notice to vacate the premises in case of failure of petitioners to perform their obligation to pay. Second, we resolve the main issue of whether the rules on accession, as found in Articles 448 and 546 of the Civil Code, apply to the instant case. Article 448 and Article 546 provide:

Art. 448. The owner of the land on which anything has been built, sown or planted in good faith, shall have the right to appropriate as his own the works, sowing or planting, after payment of the indemnity provided for in Articles 546 and 548, or to oblige the one who built or planted to pay the price of the land, and the one who sowed, the proper rent. However, the builder or planter cannot be obliged to buy the land if its value is considerably more than that of the building or trees. In such case, he shall pay reasonable rent, if the owner of the land does not choose to appropriate the building or trees after proper indemnity. The parties shall agree upon the terms of the lease and in case of disagreement, the court shall fix the terms thereof. Art. 546. Necessary expenses shall be refunded to every possessor; but only the possessor in good faith may retain the thing until he has been reimbursed therefor. Useful expenses shall be refunded only to the possessor in good faith with the same right of retention, the person who has defeated him in the possession having the option of refunding the amount of the expenses or of paying the increase in value which the thing may have acquired by reason thereof. We uphold the ruling of the CA. The late Senator Arturo M. Tolentino, a leading expert in Civil Law, explains: This article [Article 448] is manifestly intended to apply only to a case where one builds, plants, or sows on land in which he believes himself to have a claim of title, and not to

lands where the only interest of the builder, planter or sower is that of a holder, such as a tenant. In the case at bar, petitioners have no adverse claim or title to the land. In fact, as lessees, they recognize that the respondent is the owner of the land. What petitioners insist is that because of the improvements, which are of substantial value, that they have introduced on the leased premises with the permission of respondent, they should be considered builders in good faith who have the right to retain possession of the property until reimbursement by respondent. We affirm the ruling of the CA that introduction of valuable improvements on the leased premises does not give the petitioners the right of retention and reimbursement which rightfully belongs to a builder in good faith. Otherwise, such a situation would allow the lessee to easily improve the lessor out of its property. We reiterate the doctrine that a lessee is neither a builder in good faith nor in bad faith that would call for the application of Articles 448 and 546 of the Civil Code. His rights are governed by Article 1678 of the Civil Code, which reads: Art. 1678. If the lessee makes, in good faith, useful improvements which are suitable to the use for which the lease is intended, without altering the form or substance of the property leased, the lessor upon the termination of the lease shall pay the lessee one-half of the value of the improvements at that time. Should the lessor refuse to reimburse said amount, the lessee may remove the improvements, even though the principal thing may suffer damage thereby. He shall not, however, cause any more impairment upon the property leased than is necessary.

With regard to ornamental expenses, the lessee shall not be entitled to any reimbursement, but he may remove the ornamental objects, provided no damage is caused to the principal thing, and the lessor does not choose to retain them by paying their value at the time the lease is extinguished.

Under Article 1678, the lessor has the option of paying one-half of the value of the improvements which the lessee made in good faith, which are suitable for the use for which the lease is intended, and which have not altered the form and substance of the land. On the other hand, the lessee may remove the improvements should the lessor refuse to reimburse. Petitioners argue that to apply Article 1678 to their case would result to sheer injustice, as it would amount to giving away the hotel and its other structures at virtually bargain prices. They allege that the value of the hotel and its appurtenant facilities amounts to more than two billion pesos, while the monetary claim of respondent against them only amounts to a little more than twenty six-million pesos. Thus, they contend that it is the lease contract that governs the relationship of the parties, and consequently, the parties may be considered to have impliedly waived the application of Article 1678. We cannot sustain this line of argument by petitioners. Basic is the doctrine that laws are deemed incorporated in each and every contract. Existing laws always form part of any contract. Further, the lease contract in

the case at bar shows no special kind of agreement between the parties as to how to proceed in cases of default or breach of the contract. Petitioners maintain that the lease contract contains a default provision which does not give respondent the right to appropriate the improvements nor evict petitioners in cases of cancellation or termination of the contract due to default or breach of its terms. They cite paragraph 10 of the lease contract, which provides that: 10. DEFAULT. - . . . Default shall automatically take place upon the failure of the LESSEE to pay or perform its obligation during the time fixed herein for such obligations without necessity of demand, or, if no time is fixed, after 90 days from the receipt of notice or demand from the LESSOR. . . In case of cancellation or termination of this contract due to the default or breach of its terms, the LESSEE will pay all reasonable attorney s fees, costs and expenses of litigation that may be incurred by the LESSOR in enforcing its rights under this contract or any of its provisions, as well as all unpaid rents, fees, charges, taxes, assessment and others which the LESSOR may be entitled to. Petitioners assert that respondent committed a breach of the lease contract when it filed the ejectment suit against them. However, we find nothing in the above quoted provision that prohibits respondent to proceed the way it did in enforcing its rights as lessor. It can rightfully file for ejectment to evict petitioners, as it did before the court a quo. IN VIEW WHEREOF, petitioners appeal is DENIED. The October 4, 2005 Decision of the Court of Appeals in CA-G.R. SP No. 74631 and its December 22, 2005 Resolution are AFFIRMED. Costs against petitioners.

SO ORDERED.

G.R. No. 167017

June 22, 2009

SERAFIN CHENG, Petitioner, vs. SPOUSES VITTORIO and MA. HELEN DONINI, Respondents. DECISION CORONA,J.: The subject of this petition is an oral lease agreement that went sour. Petitioner Serafin Cheng agreed to lease his property located at 479 Shaw Blvd., Mandaluyong City to respondents, Spouses Vittorio and Ma. Helen Donini, who intended to put up a restaurant thereon. They agreed to a monthly rental of P17,000, to commence in December 1990. Bearing an Interim Grant of Authority executed by petitioner, respondents proceeded to introduce improvements in the premises. The authority read: I, Serafin Cheng, of legal age and with office address at Room 310 Federation Center Building Muelle de Binondo, Manila, owner of the building/structure located at 479 Shaw Boulevard, Mandaluyong, Metro Manila, pursuant to a lease agreement now being finalized and to take effect December 1, 1990, hereby grants VITTORIO DONINI (Prospective Lessee) and all those acting under his orders to make all the necessary improvements on the prospective leased premises located at 479 Shaw Blvd., Mandaluyong, Metro Manila, and for this purpose, to enter said premises and perform, all such works and activities to make the leased premises operational as a restaurant or similar purpose. Manila, 31 October 1990.1 However, before respondents business could take off and before any final lease agreement could be drafted and signed, the parties began to have serious disagreements regarding its terms and conditions. Petitioner thus

wrote respondents on January 28, 1991, demanding payment of the deposit and rentals, and signifying that he had no intention to continue with the agreement should respondents fail to pay. Respondents, however, ignoring petitioner s demand, continued to occupy the premises until April 17, 1991 when their caretaker voluntarily surrendered the property to petitioner. Respondents then filed an action for specific performance and damages with a prayer for the issuance of a writ of preliminary injunction in the Regional Trial Court (RTC) of Pasig City, Branch 67, docketed as Civil Case No. 60769. Respondents prayed that petitioner be ordered to execute a written lease contract for five years, deducting from the deposit and rent the cost of repairs in the amount of P445,000, or to order petitioner to return their investment in the amount of P964,000 and compensate for their unearned net income of P200,000 with interest, plus attorney s fees.2 Petitioner, in his answer, denied respondents claims and sought the award of moral and exemplary damages, and attorney s fees.3 After trial, the RTC rendered its decision in favor of petitioner, the dispositive portion of which provided: WHEREFORE, in view of all the foregoing, this Court finds the preponderance of evidence in favor of the [petitioner] and hereby renders judgment as follows: 1. The Complaint is dismissed. 2. On the counterclaim, [respondents] are ordered, jointly and severally, to pay the [petitioner] P500,000.00 as moral damages; P100,000.00 as exemplary damages; and P50,000.00 as attorney s fees. 3. [Respondents] are likewise ordered to pay the costs. SO ORDERED.4 Respondents appealed to the Court of Appeals (CA) which, in its decision5 dated March 31, 2004, recalled and set aside the RTC decision, and

entered a new one ordering petitioner to pay respondents the amount of P964,000 representing the latter s expenses incurred for the repairs and improvements of the premises.6 Petitioner filed a motion for reconsideration on the ground that the award of reimbursement had no factual and legal bases,7 but this was denied by the CA in its resolution dated February 21, 2005.8 Hence, this petition for certiorari under Rule 45 of the Rules of Court, with petitioner arguing that: THE COURT OF APPEALS DECIDED THIS CASE NOT IN ACCORD WITH LAW AND WITH APPLICABLE DECISIONS OF THIS HONORABLE COURT. PUT OTHERWISE: A. BY ORDERING PETITIONER TO REIMBURSE RESPONDENTS THE FULL VALUE OF EXPENSES FOR THEIR ALLEGED REPAIRS AND IMPROVEMENTS OF THE LEASED PREMISES, THE COURT OF APPEALS ERRONEOUSLY CONSIDERED RESPONDENTS NOT AS MERE LESSEES BUT POSSESSORS IN GOOD FAITH UNDER ARTICLES 448 AND 546 OF THE CIVIL CODE. B. THE COURT OF APPEALS DECIDED THIS CASE NOT IN ACCORD WITH ARTICLE 1678 OF THE CIVIL CODE WHICH GIVES THE LESSOR THE OPTION TO REIMBURSE THE LESSEE ONE-HALF OF THE VALUE OF USEFUL IMPROVEMENTS OR, IF HE DOES NOT WANT TO, ALLOW THE LESSEE TO REMOVE THE IMPROVEMENTS. C. LIKEWISE, BY ORDERING PETITIONER TO REIMBURSE THE VALUE OF ORNAMENTAL EXPENSES, THE COURT OF APPEALS CONTRAVENED THE SECOND PARAGRAPH OF ARTICLE 1678. D. THE COURT OF APPEALS ERRED IN APPLYING THE PRINCIPLE OF EQUITY IN FAVOR OF THE RESPONDENTS. E. THE COURT OF APPEALS ERRED IN NOT AFFIRMING THE DECISION OF THE TRIAL COURT AWARDING DAMAGES TO PETITIONER.

F. THE COURT OF APPEALS SERIOUSLY ERRED AND/OR GRAVELY ABUSED ITS DISCRETION IN FIXING THE AMOUNT OF P961,000.009 CONTRARY TO RESPONDENTS OWN REPRESENTATION AND EVIDENCE.10 Respondents were required to file their comment on the petition but their counsel manifested that he could not file one since his clients whereabouts were unknown to him.11 Counsel also urged the Court to render a decision on the basis of the available records and documents.12 Per resolution dated August 30, 2006, copies of the resolutions requiring respondents to file their comment were sent to their last known address and were deemed served. The order requiring respondents counsel to file a comment in their behalf was reiterated.13 In their comment, respondents argued that they were possessors in good faith, hence, Articles 448 and 546 of the Civil Code applied and they should be indemnified for the improvements introduced on the leased premises. Respondents bewailed the fact that petitioner was going to benefit from these improvements, the cost of which amounted to P1.409 million, in contrast to respondents rental/deposit obligation amounting to only P34,000. Respondents also contended that petitioner s rescission of the agreement was in bad faith and they were thus entitled to a refund.14 In settling the appeal before it, the CA made the following findings and conclusions: 1. there was no agreement that the deposit and rentals accruing to petitioner would be deducted from the costs of repairs and renovation incurred by respondents; 2. respondents committed a breach in the terms and conditions of the agreement when they failed to pay the rentals; 3. there was no valid rescission on the part of petitioner; 4. respondents were entitled to reimbursement for the cost of improvements under the principle of equity and unjust enrichment; and

5. the award of damages in favor of petitioner had no basis in fact and law.15 As the correctness of the CA s ruling regarding (1) the lack of agreement on the deposit and rentals; (2) respondents breach of the terms of the verbal agreement and (3) the lack of valid rescission by petitioner was never put in issue, this decision will be confined only to the issues raised by petitioner, that is, the award of reimbursement and the deletion of the award of damages. It need not be stressed that an appellate court will not review errors that are not assigned before it, save in certain exceptional circumstances and those affecting jurisdiction over the subject matter as well as plain and clerical errors, none of which is present in this case.16 Remarkably, in ruling that respondents were entitled to reimbursement, the CA did not provide any statutory basis therefor and instead applied the principles of equity and unjust enrichment, stating: It would be inequitable to allow the defendant-appellee, as owner of the property to enjoy perpetually the improvements introduced by the plaintiffs-appellants without reimbursing them for the value of the said improvements. Well-settled is the rule that no one shall be unjustly enriched or benefitted at the expense of another.17 Petitioner, however, correctly argued that the principle of equity did not apply in this case. Equity, which has been aptly described as "justice outside legality," is applied only in the absence of, and never against, statutory law or judicial rules of procedure.18 Positive rules prevail over all abstract arguments based on equity contra legem.19 Neither is the principle of unjust enrichment applicable since petitioner (who was to benefit from it) had a valid claim.20 The relationship between petitioner and respondents was explicitly governed by the Civil Code provisions on lease, which clearly provide for the rule on reimbursement of useful improvements and ornamental expenses after termination of a lease agreement. Article 1678 states: If the lessee makes, in good faith, useful improvements which are suitable to the use for which the lease is intended, without altering the form or substance of the property leased, the lessor upon the termination of the

lease shall pay the lessee one-half of the value of the improvements at that time. Should the lessor refuse to reimburse said amount, the lessee may remove the improvements, even though the principal thing may suffer damage thereby. He shall not, however, cause any more impairment upon the property leased than is necessary. With regard to ornamental expenses, the lessee shall not be entitled to any reimbursement, but he may remove the ornamental objects, provided no damage is caused to the principal thing, and the lessor does not choose to retain them by paying their value at the time the lease is extinguished. Article 1678 modified the (old) Civil Code provision on reimbursement where the lessee had no right at all to be reimbursed for the improvements introduced on the leased property, he being entitled merely to the rights of a usufructuary the right of removal and set-off but not to reimbursement.21 Contrary to respondents position, Articles 448 and 546 of the Civil Code did not apply. Under these provisions, to be entitled to reimbursement for useful improvements introduced on the property, respondents must be considered builders in good faith. Articles 448 and 546, which allow full reimbursement of useful improvements and retention of the premises until reimbursement is made, apply only to a possessor in good faith or one who builds on land in the belief that he is the owner thereof. A builder in good faith is one who is unaware of any flaw in his title to the land at the time he builds on it. 22 But respondents cannot be considered possessors or builders in good faith. As early as 1956, in Lopez v. Philippine & Eastern Trading Co., Inc.,23 the Court clarified that a lessee is neither a builder nor a possessor in good faith x x x This principle of possessor in good faith naturally cannot apply to a lessee because as such lessee he knows that he is not the owner of the leased property. Neither can he deny the ownership or title of his lessor. Knowing that his occupation of the premises continues only during the life of the lease contract and that he must vacate the property upon termination of the lease or upon the violation by him of any of its terms, he introduces improvements on said property at his own risk in the sense that

he cannot recover their value from the lessor, much less retain the premises until such reimbursement. (Emphasis supplied) Being mere lessees, respondents knew that their right to occupy the premises existed only for the duration of the lease.24 Cortez v. Manimbo25 went further to state that: If the rule were otherwise, it would always be in the power of the tenant to improve his landlord out of his property. These principles have been consistently adhered to and applied by the Court in many cases.26 Under Article 1678 of the Civil Code, the lessor has the primary right (or the first move) to reimburse the lessee for 50% of the value of the improvements at the end of the lease. If the lessor refuses to make the reimbursement, the subsidiary right of the lessee to remove the improvements, even though the principal thing suffers damage, arises. Consequently, on petitioner rests the primary option to pay for one-half of the value of the useful improvements. It is only when petitioner as lessor refuses to make the reimbursement that respondents, as lessees, may remove the improvements. Should petitioner refuse to exercise the option of paying for one-half of the value of the improvements, he cannot be compelled to do so. It then lies on respondents to insist on their subsidiary right to remove the improvements even though the principal thing suffers damage but without causing any more impairment on the property leased than is necessary. As regards the ornamental expenses, respondents are not entitled to reimbursement. Article 1678 gives respondents the right to remove the ornaments without damage to the principal thing. But if petitioner appropriates and retains said ornaments, he shall pay for their value upon the termination of the lease. The fact that petitioner will benefit from the improvements introduced by respondents is beside the point. In the first place, respondents introduced these improvements at their own risk as lessees. Respondents were not forced or obliged to splurge on the leased premises as it was a matter of necessity as well as a business strategy.27 In fact, had respondents only

complied with their obligation to pay the deposit/rent, there would have been no dispute to begin with. If they were able to shell out more than a million pesos to improve the property, the measly P34,000 deposit demanded by petitioner was a mere "drop in the bucket," so to speak. More importantly, the unequivocal terms of Article 1678 of the Civil Code should be the foremost consideration. The Court notes that the CA pegged the total value of the improvements made on the leased premises at P964,000, which was apparently based on the allegation in respondents complaint that it was their total investment cost.28 The CA lumped together all of respondents expenses, which was a blatant error. A qualification should have been made as to how much was spent for useful improvements (or those which were suitable to the use for which the lease was intended) and how much was for ornamental expenses. Respondent Vittorio Donini testified that he spent P450,000 for necessary repairs, while P500,000 was spent for adornments.29 The evidence on record, however, showed respondents expenses for useful improvements to be as follows: Expense Electrical Roofing Labor Ceiling Labor Electrical (phase 2) Door Labor Water Amount P31,893.65 P14,856.00 P19,909.75 P65,712.00 P38,689.20 P76,539.10 P41,371.75 P25,126.00 P 8,031.00 Exh. "F", et seq. Exhibit "O"31 Exh. "P", et seq.32 Exh. "Q", et seq.33 Exh. "R", et seq.34 Exh. "S", et seq.35 Exh. "T", et seq.36 Exh. "U", et seq.37 Exhs. "W" & "W-1"38
30

Gutters Outside Wall Inside Wall Electrical (phase 3) Labor Total

P 35,550.05 P 24,744.00 P 22,186.10 P 88,698.30 P 19,995.00 P513,301.90

Exhs. "X" & "X-1"39 Exh. "X-2"40 Exh. "X-3"41 Exhs. "X-8" to "X-11"42 Exhibit "Y"43

Accordingly, the 50% value of the useful improvements to be reimbursed by petitioner, if he chose to do so, should be based on P513,301.90. Since petitioner did not exercise his option to retain these useful improvements, then respondents could have removed the same. This was the legal consequence of the application of Article 1678 under ordinary circumstances. The reality on the ground ought to be recognized. For one, as disclosed by respondents counsel, he no longer knows the exact whereabouts of his clients, only that they are now in Europe and he has no communication with them at all.44 For another, it appears that as soon as respondents vacated the premises, petitioner immediately reclaimed the property and barred respondents from entering it. Respondents also alleged, and petitioner did not deny, that the property subject of this case had already been leased to another entity since 1991.45 This is where considerations of equity should come into play. It is obviously no longer feasible for respondents to remove the improvements from the property, if they still exist. The only equitable alternative then, given the circumstances, is to order petitioner to pay respondents one-half of the value of the useful improvements (50% of P513,301.90) introduced on the property, or P256,650.95. To be off-set against this amount are respondents unpaid P17,000 monthly rentals for the period of December 1990 to April 1991,46 or P85,000. Petitioner should, therefore, indemnify respondents the amount of P171,650.95. This is in accord with the law s intent of preventing unjust enrichment of a lessor who now has to pay one-half of the value of the useful improvements at the end of the lease because the

lessee has already enjoyed the same, whereas the lessor can enjoy them indefinitely thereafter.47 Respondents are not entitled to reimbursement for the ornamental expenses under the express provision of Article 1678. Moreover, since they failed to remove these ornaments despite the opportunity to do so when they vacated the property, then they were deemed to have waived or abandoned their right of removal. The CA also erred when it deleted the awards of moral and exemplary damages and attorney s fees. Petitioner is entitled to moral damages but not in the amount of P500,000 awarded by the RTC, which the Court finds to be excessive. While trial courts are given discretion to determine` the amount of moral damages, it "should not be palpably and scandalously excessive."48 Moral damages are not meant to enrich a person at the expense of the other but are awarded only to allow the former to obtain means, diversion or amusements that will serve to alleviate the moral suffering he has undergone due to the other person s culpable action.49 It must always reasonably approximate the extent of injury and be proportional to the wrong committed.50 The award of P100,000 as moral damages is sufficient and reasonable under the circumstances. The award of P100,000 as exemplary damages is likewise excessive. Exemplary damages are imposed not to enrich one party or impoverish another but to serve as a deterrent against or as a negative incentive to curb socially deleterious actions.51 We think P50,000 is reasonable in this case.1avvphi1 Finally, Article 2208 of the Civil Code allows recovery of attorney's fees when exemplary damages are awarded or when the defendant's act or omission has compelled the plaintiff to litigate with third persons or to incur expenses to protect his interest.52 Petitioner is entitled to it since exemplary damages were awarded in this case and respondents act in filing Civil Case No. 60769 compelled him to litigate. The amount of P25,000 is in accord with prevailing jurisprudence.53

WHEREFORE, the petition is PARTIALLY GRANTED. The decision dated March 31, 2004 rendered by the Court of Appeals in CA-G.R. CV No. 54430 is hereby MODIFIED in that (1) petitioner Serafin Cheng is ORDERED to pay respondents, spouses Vittorio and Ma. Helen Donini, the amount of P171,650.95 as indemnity for the useful improvements; and (2) respondents, spouses Vittorio and Ma. Helen Donini, are ORDERED to pay petitioner Serafin Cheng the following sums: a) P100,000.00 moral damages; b) P50,000.00 exemplary damages and c) P25,000.00 attorney s fees. Let copies of this decision be furnished respondents, spouses Vittorio and Ma. Helen Donini, at their last known address, and their counsel of record. SO ORDERED. SPOUSES VIRGILIO and JOSIE JIMENEZ, petitioners, vs. PATRICIA, INC., respondent. DECISION BELLOSILLO, J.: The Joint Decision of the Court of Appeals (dismissing the petition for review filed by spouses Virgilio and Josie Jimenez in CA-G.R. SP No. 43185 and giving due course to the petition for review filed by Patricia, Inc., in CA-G.R. SP No. 43179), in effect reversing the decision of the Regional Trial Court and reinstating that of the Metropolitan Trial Court, is assailed in the instant petition. Petitioners Virgilio and Josie Jimenez, spouses, are sublessees of a lot and building located at 2853 Juan Luna Street, Tondo, Manila, owned by respondent Patricia Inc. (PATRICIA for brevity), a domestic corporation duly organized and existing under Philippine laws. The Jimenez spouses

subleased the property in 1980 from a certain Purisima Salazar who had been leasing the property from PATRICIA since 1970. Sometime in 1995 Purisima Salazar abandoned the property thus incurring back rentals dating back to January 1992. Hence, by reason of her nonpayment of the monthly rentals, her contract of lease with PATRICIA was terminated. On 29 March 1995 PATRICIA sent a letter to the Jimenez spouses informing them of the termination of the lease and demanding that they vacate the premises within fifteen (15) days from notice since they had no existing lease contract with it. But the spouses refused to leave. Thus, on 5 May 1995 PATRICIA filed a complaint for unlawful detainer against the Jimenez spouses alleging, among others, that the lessee Purisima Salazar subleased the premises to the Jimenezes; that Purisima Salazar no longer occupied the premises; that this notwithstanding, the Jimenez spouses continued to occupy the premises without any contract with PATRICIA, its owner, hence, their stay was merely being tolerated by the latter; and, that despite demands made upon them, they refused to vacate the premises thereby unlawfully and illegally withholding the property to the damage and prejudice of PATRICIA. In their Answer, the Jimenez spouses claimed that they occupied the premises as sublessees of Purisima Salazar with the knowledge of PATRICIA; that the building originally found on the lot was owned by Purisima Salazar which she sold to them in 1984 with notice and without any objection from PATRICIA; that, when the building was gutted by fire in 1987 they constructed a new house on the lot worth P1,500,000.00 with the knowledge and without any objection from PATRICIA; and, that PATRICIA never collected any rental for the land but they nevertheless voluntarily paid the amount of P23,537.25 as rent corresponding to the period of September 1979 to 31 December 1991. The MeTC ruled in favor of PATRICIA and ordered the Jimenez spouses to vacate the premises, to pay PATRICIA the sum of P3,000.00 a month as reasonable rental and/or compensation for the use of the premises beginning April 1995 until they finally vacated the premises, and to pay

PATRICIA the sum of P5,000.00 as reasonable attorney's fees, plus costs of suit. The Jimenez spouses appealed the MeTC decision to the RTC. On 2 January 1997 the RTC modified the decision in favor of the spouses holding that an implied new lease contract existed between the Jimenez spouses and PATRICIA in view of the latter's acceptance of rentals from the former. Thus the RTC extended the term of the lease between the parties for a period of one (1) year from date of decision, and ordered PATRICIA to reimburse the Jimenez spouses the expenses incurred in the construction of the house built on the property and/or for the Jimenez spouses to remove the improvements thereon. On 20 January 1997 PATRICIA filed a Motion for Clarificatory Judgment and later added a Supplement to the Motion for Clarificatory Judgment. On 27 January 1997 PATRICIA, without waiting for the resolution of its Motion for Clarificatory Judgment as well as its supplement thereto, filed a Petition for Review of the RTC decision with the Court of Appeals, docketed as CA-G.R. SP No. 43179. On 13 February 1997 the Jimenez spouses filed their own Petition for Review, docketed as CA-G.R. SP No. 43185. Subsequently, this petition was consolidated with PATRICIA's Petition for Review since it involved the same parties, facts, and issues. The Court of Appeals in due course rendered a Joint Decision dismissing the Petition for Review filed by the Jimenez spouses while giving due course to the petition of PATRICIA. The Court of Appeals held that there was no implied renewal of the lease contract between the parties since, to begin with, there was no lease contract between them; hence, the Jimenez spouses could not have tendered payment of rentals to PATRICIA. Instead, it declared the status of the Jimenez spouses as being analogous to that of a lessee or tenant whose lease has expired but whose occupancy has been continued by mere tolerance of the owner, and hence, bound by an implied promise that he would vacate the premises upon demand. Thus, the appellate court reversed and set aside the decision of the RTC and reinstated the decision of the MeTC which, among others, ordered the Jimenez spouses to vacate the premises.

Petitioners now assail the jurisdiction of the MeTC contending that the failure of the complaint to allege the character of the sublease or entry of the Jimenez spouses into the property, whether legal or illegal, automatically classified it into an accion publiciana or reinvindicatoria cognizable by the RTC and not by the MeTC; thus, the action should have been dismissed. The rule is settled that a question of jurisdiction may be raised at any time, even on appeal, provided that its application does not result in a mockery of the tenets of fair play. In the instant case, the jurisdictional issue was raised by petitioners for the first time only in the instant Petition for Review. However, it should be noted that they did so only after an adverse decision was rendered by the Court of Appeals. Despite several opportunities in the RTC, which ruled in their favor, and in the Court of Appeals, petitioners never advanced the question of jurisdiction of the MeTC. Additionally, petitioners participated actively in the proceedings before the MeTC and invoked its jurisdiction with the filing of their answer, in seeking affirmative relief from it, in subsequently filing a notice of appeal before the RTC, and later, a Petition for Review with the Court of Appeals. Upon these premises, petitioners cannot now be allowed belatedly to adopt an inconsistent posture by attacking the jurisdiction of the court to which they had submitted themselves voluntarily. Laches now bars them from doing so. Be that as it may, we find no error in the MeTC assuming jurisdiction over the subject matter. A complaint for unlawful detainer is sufficient if it alleges that the withholding of possession or the refusal to vacate is unlawful without necessarily employing the terminology of the law. As correctly found by the appellate court, to which we agree, the allegations in the complaint sufficiently established a cause of action for unlawful detainer. The complaint clearly stated how entry was effected and how and when dispossession started - petitioners were able to enter the subject premises as sublessees of Purisima Salazar who, despite the termination of her lease with respondent, continued to occupy the subject premises without any contract with it; thus, their stay was by tolerance of respondent.

The fact that the complaint failed to state that respondent was in prior possession of the property before it was unlawfully withheld by petitioner spouses is of no moment. Prior physical possession is indispensable only in actions for forcible entry but not in unlawful detainer. Petitioner spouses, as mere sublessees of Purisima Salazar, derive their right from the sublessor whose termination of contract with the lessor necessarily also ends the sublease contract. Thus, when the contract of lease of Purisima Salazar with respondent was terminated the contract of sublease of petitioners with the former also necessarily ended and petitioners cannot insist on staying on the premises. Petitioners can invoke no right superior to that of their sublessor. It is not correct to say that petitioners could not have occupied the property by tolerance of respondent as their entry into the premises was inceptively illegal, the sublease being entered into without the consent of the owner. Petitioners argue that tolerance is only available in cases where entry was lawful from the start and cannot be asserted where entry was illegal from the start. It appears however that respondent did not expressly and equivocally prohibit the subleasing of the property. Although the attached contracts of lease state that the lessee cannot sublease the property, none of those contracts pertain to the contract of lease between Purisima Salazar and respondent PATRICIA. In any event, the fact that PATRICIA sent a letter to the Jimenez spouses informing them of the termination of the lease of Purisima Salazar shows that they recognize and acknowledge their stay in the premises as sublessees of Salazar. However, after the termination of the contract of lease of Purisima Salazar with PATRICIA, any right of the Jimenez spouses to stay in the premises, although previously recognized, then and there ended. After the termination of the contract of lease of Salazar the continued stay of the Jimenez spouses thereat was merely by tolerance of PATRICIA and it became unlawful after they ignored the lessor's demand to leave. The status of petitioner spouses is akin to that of a lessee or a tenant whose term of lease has expired but whose occupancy has continued by tolerance of the owner. A person who occupies the land of another at the latter's forbearance or permission without any contract between them is necessarily bound by an implied promise that he will vacate upon demand

failing which a summary action for ejectment is the proper remedy against him. The present action being for unlawful detainer, it is well within the exclusive original jurisdiction of the metropolitan trial courts. Petitioners contend that respondent has no cause of action against them since, as proved by Transfer Certificate of Title No. T-44247, the property is in the name of the City of Manila and not of respondent PATRICIA. Records however show that this issue has not been raised in the proceedings below, hence, will not be ruled upon by this Court. Any issue raised for the first time on appeal and not timely raised in the proceedings in the lower court is barred by estoppel. Moreover, being mere sublessees of the property in question, petitioners cannot in an action involving possession of the leased premises controvert the title of PATRICIA, or assert any right adverse to its title. It is the Manila City Government, not the Jimenez spouses, that is the proper party to dispute the ownership of PATRICIA. Petitioners argue that the Petition for Review of respondent should have been dismissed for being premature in view of the pendency of its Motion for Clarificatory Judgment and Supplement to the Motion for Clarificatory Judgment which remained unresolved by the RTC. They assert that because of the pendency of its motion, there was no final judgment or decision that could properly be the subject of a petition for review before the Court of Appeals. We do not agree. The Petition for Review filed by respondent with the Court of Appeals was not prematurely filed. It should be borne in mind that a Motion for Clarificatory Judgment not being in the character of a motion for reconsideration does not toll the reglementary period for filing a petition for review with the Court of Appeals. Its filing will not bar the judgment from attaining finality, nor will its resolution amend the decision to be reviewed. Thus, when respondent filed a Petition for Review before the Court of Appeals, there was already a final judgment that could properly be the subject of a petition for review. Moreover, under the Rules on Summary Procedure, the decision of the RTC in civil cases governed by this Rule, including forcible entry and unlawful detainer, is immediately executory without prejudice to a further appeal

that may be taken therefrom. The judgment of the RTC being final and executory the filing of the Petition for Review was proper. As to the house built by petitioners on the property, this Court has previously ruled that lessees, much less, sublessees, are not possessors or builders in good faith over rented land because they know that their occupancy of the premises continues only during the life of the lease, or sublease as the case may be; and, they cannot as a matter of right recover the value of their improvements from the lessor, much less retain the premises until they are reimbursed. Instead, their rights are governed by Art. 1678 of the Civil Code which allows reimbursement of lessees up to one-half (1/2) of the value of their improvements if the lessor so elects: Art. 1678. If the lessee makes, in good faith, useful improvements which are suitable to the use for which the lease is intended, without altering the form or substance of the property leased, the lessor upon the termination of the lease shall pay the lessee one-half of the value of the improvements at that time. Should the lessor refuse to reimburse said amount, the lessee may remove the improvements, even though the principal thing may suffer damage thereby. He shall not, however, cause any more impairment upon the property leased than is necessary xxx (New Civil Code). Thus, applying the above rule, petitioners cannot recover full reimbursement of the value spent for the construction of the house, but is limited only to one-half (1/2) of its value at the election of the lessor. However, as PATRICIA has manifested its lack of intention to do so, the Jimenez spouses have no recourse but to remove the house at their own expense. WHEREFORE, the assailed Joint Decision of the Court of Appeals reversing and setting aside the decision of the Regional Trial Court and reinstating the decision of the Metropolitan Trial Court is AFFIRMED, with the MODIFICATION that petitioner spouses Virgilio and Josie Jimenez should also remove the house they have constructed on the lot at their own expense. Thus, petitioner spouses and all persons claiming title under them are ordered: (a) to vacate the premises described in the complaint located at 2853 Juan Luna Street, Tondo, Manila; (b) to remove at their own expense within sixty (60) days from finality of this Decision the house they have constructed thereon; (c) to pay respondent Patricia, Inc., the sum of

P3,000.00 a month as reasonable rental/compensation for the use of the premises beginning April 1995 until they finally vacate the premises; and, (d) to pay respondent Patricia, Inc., the sum of P5,000.00 as attorney's fees, plus costs of suit. SO ORDERED. G.R. No. 156437 March 1, 2004

NATIONAL HOUSING AUTHORITY, petitioner, vs. GRACE BAPTIST CHURCH and the COURT OF APPEALS, respondents. DECISION YNARES-SANTIAGO, J.: This is a petition for review under Rule 45 of the Rules of Court, seeking to reverse the Decision of the Court of Appeals dated February 26, 2001,1 and its Resolution dated November 8, 2002,2 which modified the decision of the Regional Trial Court of Quezon City, Branch 90, dated February 25, 1997.3 On June 13, 1986, respondent Grace Baptist Church (hereinafter, the Church) wrote a letter to petitioner National Housing Authority (NHA), manifesting its interest in acquiring Lots 4 and 17 of the General Mariano Alvarez Resettlement Project in Cavite.4 In its letter-reply dated July 9, 1986, petitioner informed respondent: In reference to your request letter dated 13 June 1986, regarding your application for Lots 4 and 17, Block C-3-CL, we are glad to inform you that your request was granted and you may now visit our Project Office at General Mariano Alvarez for processing of your application to purchase said lots. We hereby advise you also that prior to approval of such application and in accordance with our existing policies and guidelines, your other accounts with us shall be maintained in good standing.5 Respondent entered into possession of the lots and introduced improvements thereon.6

On February 22, 1991, the NHA s Board of Directors passed Resolution No. 2126, approving the sale of the subject lots to respondent Church at the price of P700.00 per square meter, or a total price of P430,500.00.7 The Church was duly informed of this Resolution through a letter sent by the NHA.8 On April 8, 1991, the Church tendered to the NHA a manager s check in the amount of P55,350.00, purportedly in full payment of the subject properties.9 The Church insisted that this was the price quoted to them by the NHA Field Office, as shown by an unsigned piece of paper with a handwritten computation scribbled thereon.10 Petitioner NHA returned the check, stating that the amount was insufficient considering that the price of the properties have changed. The Church made several demands on the NHA to accept their tender of payment, but the latter refused. Thus, the Church instituted a complaint for specific performance and damages against the NHA with the Regional Trial Court of Quezon City,11 where it was docketed as Civil Case No. Q-91-9148. On February 25, 1997, the trial court rendered its decision, the dispositive portion of which reads: WHEREFORE, premises considered, judgment is hereby rendered as follows: 1. Ordering the defendant to reimburse to the plaintiff the amount of P4,290.00 representing the overpayment made for Lots 1, 2, 3, 18, 19 and 20; 2. Declaring that there was no perfected contract of sale with respect to Lots 4 and 17 and ordering the plaintiff to return possession of the property to the defendant and to pay the latter reasonable rental for the use of the property at P200.00 per month computed from the time it took possession thereof until finally vacated. Costs against defendant. SO ORDERED.12 On appeal, the Court of Appeals, affirmed the trial court s finding that there was indeed no contract of sale between the parties. However, petitioner

was ordered to execute the sale of the lots to Grace Baptist Church at the price of P700.00 per square meter, with 6% interest per annum from March 1991. The dispositive portion of the Court of Appeals decision, dated February 26, 2001, reads: WHEREFORE, the appealed Decision is hereby AFFIRMED with the MODIFICATION that defendant-appellee NHA is hereby ordered to sell to plaintiff-appellant Grace Baptist Church Lots 4 and 17 at the price of P700.00 per square meter, or a total cost P430,000.00 with 6% interest per annum from March, 1991 until full payment in cash. SO ORDERED.13 The appellate court ruled that the NHA s Resolution No. 2126, which earlier approved the sale of the subject lots to Grace Baptist Church at the price of P700.00 per square meter, has not been revoked at any time and was therefore still in effect. As a result, the NHA was estopped from fixing a different price for the subject properties. Considering further that the Church had been occupying the subject lots and even introduced improvements thereon, the Court of Appeals ruled that, in the interest of equity, it should be allowed to purchase the subject properties.14 Petitioner NHA filed a Motion for Reconsideration which was denied in a Resolution dated November 8, 2002. Hence, the instant petition for review on the sole issue of: Can the NHA be compelled to sell the subject lots to Grace Baptist Church in the absence of any perfected contract of sale between the parties? Petitioner submits that the Court cannot compel it to sell the subject property to Grace Baptist Church without violating its freedom to contract.15 Moreover, it contends that equity should be applied only in the absence of any law governing the relationship between the parties, and that the law on sales and the law on contracts in general apply to the present case.16 We find merit in petitioner s submission. Petitioner NHA is not estopped from selling the subject lots at a price equal to their fair market value, even if it failed to expressly revoke Resolution

No. 2126. It is, after all, hornbook law that the principle of estoppel does not operate against the Government for the act of its agents,17 or, as in this case, their inaction. On the application of equity, it appears that the crux of the controversy involves the characterization of equity in the context of contract law. Preliminarily, we reiterate that this Court, while aware of its equity jurisdiction, is first and foremost, a court of law. While equity might tilt on the side of one party, the same cannot be enforced so as to overrule positive provisions of law in favor of the other.18 Thus, before we can pass upon the propriety of an application of equitable principles in the case at bar, we must first determine whether or not positive provisions of law govern. It is a fundamental rule that contracts, once perfected, bind both contracting parties, and obligations arising therefrom have the force of law between the parties and should be complied with in good faith.19 However, it must be understood that contracts are not the only source of law that govern the rights and obligations between the parties. More specifically, no contractual stipulation may contradict law, morals, good customs, public order or public policy.20 Verily, the mere inexistence of a contract, which would ordinarily serve as the law between the parties, does not automatically authorize disposing of a controversy based on equitable principles alone. Notwithstanding the absence of a perfected contract between the parties, their relationship may be governed by other existing laws which provide for their reciprocal rights and obligations. It must be remembered that contracts in which the Government is a party are subject to the same rules of contract law which govern the validity and sufficiency of contract between individuals. All the essential elements and characteristics of a contract in general must be present in order to create a binding and enforceable Government contract.21 It appearing that there is no dispute that this case involves an unperfected contract, the Civil Law principles governing contracts should apply. In Vda. de Urbano v. Government Service Insurance System,22 it was ruled that a qualified acceptance constitutes a counter-offer as expressly stated by Article 1319 of the Civil Code. In said case, petitioners offered to redeem mortgaged property and requested for an extension of the period of

redemption. However, the offer was not accepted by the GSIS. Instead, it made a counter-offer, which petitioners did not accept. Petitioners again offer to pay the redemption price on staggered basis. In deciding said case, it was held that when there is absolutely no acceptance of an offer or if the offer is expressly rejected, there is no meeting of the minds. Since petitioners offer was denied twice by GSIS, it was held that there was clearly no meeting of the minds and, thus, no perfected contract. All that is established was a counter-offer.23 In the case at bar, the offer of the NHA to sell the subject property, as embodied in Resolution No. 2126, was similarly not accepted by the respondent.24 Thus, the alleged contract involved in this case should be more accurately denominated as inexistent. There being no concurrence of the offer and acceptance, it did not pass the stage of generation to the point of perfection.25 As such, it is without force and effect from the very beginning or from its incipiency, as if it had never been entered into, and hence, cannot be validated either by lapse of time or ratification.26 Equity can not give validity to a void contract,27 and this rule should apply with equal force to inexistent contracts. We note from the records, however, that the Church, despite knowledge that its intended contract of sale with the NHA had not been perfected, proceeded to introduce improvements on the disputed land. On the other hand, the NHA knowingly granted the Church temporary use of the subject properties and did not prevent the Church from making improvements thereon. Thus, the Church and the NHA, who both acted in bad faith, shall be treated as if they were both in good faith.28 In this connection, Article 448 of the Civil Code provides: The owner of the land on which anything has been built, sown or planted in good faith, shall have the right to appropriate as his own the works, sowing or planting, after payment of the indemnity provided for in articles 546 and 548, or to oblige the one who built or planted to pay the price of the land, and the one who sowed, the proper rent. However, the builder or planter cannot be obliged to buy the land and if its value is considerably more than that of the building or trees. In such case, he shall pay reasonable rent, if the owner of the land does not choose to appropriate the building or trees after

proper indemnity. The parties shall agree upon the terms of the lease and in case of disagreement, the court shall fix the terms thereof. Pursuant to our ruling in Depra v. Dumlao,29 there is a need to remand this case to the trial court, which shall conduct the appropriate proceedings to assess the respective values of the improvements and of the land, as well as the amounts of reasonable rentals and indemnity, fix the terms of the lease if the parties so agree, and to determine other matters necessary for the proper application of Article 448, in relation to Articles 546 and 548, of the Civil Code. WHEREFORE, in view of the foregoing, the petition is GRANTED. The Court of Appeals Decision dated February 26, 2001 and Resolution dated November 8, 2002 are REVERSED and SET ASIDE. The Decision of the Regional Trial Court of Quezon City-Branch 90, dated February 25, 1997, is REINSTATED. This case is REMANDED to the Regional Trial Court of Quezon City, Branch 90, for further proceedings consistent with Articles 448 and 546 of the Civil Code. No costs. SO ORDERED. DIONISIA P. BAGAIPO, petitioner, vs. THE HON. COURT OF APPEALS and LEONOR LOZANO, respondents. QUISUMBING, J.: This petition assails the decision dated June 30, 1994 of the Court of Appeals affirming the dismissal by the Regional Trial Court of Davao City, Branch 8, in Civil Case No. 555-89, of petitioner s complaint for recovery of possession with prayer for preliminary mandatory injunction and damages. The undisputed facts of the case are as follows: Petitioner Dionisia P. Bagaipo is the registered owner of Lot No. 415, a 146,900 square meter agricultural land situated in Ma-a, Davao City under Transfer Certificate of Title No. T-15757 particularly described as follows:

Bounded on the NE., by Lots Nos. 419 and 416; on the SE by the Davao River; on the SE., (sic) by Lots Nos. 1092 and 1091; and on the NW., by Lots Nos. 413 and 418 Respondent Leonor Lozano is the owner of a registered parcel of land located across and opposite the southeast portion of petitioner s lot facing the Davao River. Lozano acquired and occupied her property in 1962 when his wife inherited the land from her father who died that year. On May 26, 1989, Bagaipo filed a complaint for Recovery of Possession with Mandatory Writ of Preliminary Injunction and Damages against Lozano for: (1) the surrender of possession by Lozano of a certain portion of land measuring 29,162 square meters which is supposedly included in the area belonging to Bagaipo under TCT No. T-15757; and (2) the recovery of a land area measuring 37,901 square meters which Bagaipo allegedly lost when the Davao River traversed her property. Bagaipo contended that as a result of a change in course of the said river, her property became divided into three lots, namely: Lots 415-A, 415-B and 415-C. In January 1988, Bagaipo commissioned a resurvey of Lot 415 and presented before the trial court a survey plan prepared by Geodetic Engineer Gersacio A. Magno. The survey plan allegedly showed that: a) the area presently occupied by Bagaipo, identified as Lot 415-A, now had an area of only 79,843 square meters; b) Lot 415-B, with an area measuring 37,901 square meters, which cut across Bagaipo s land was taken up by the new course of the Davao River; and c) an area of 29,162 square meters designated as Lot 415-C was illegally occupied by respondent Lozano. The combined area of the lots described by Engineer Magno in the survey plan tallied with the technical description of Bagaipo s land under TCT No. T-15757. Magno concluded that the land presently located across the river and parallel to Bagaipo s property still belonged to the latter and not to Lozano, who planted some 350 fruit-bearing trees on Lot 415-C and the old abandoned river bed. Bagaipo also presented Godofredo Corias, a former barangay captain and long-time resident of Ma-a to prove her claim that the Davao River had indeed changed its course. Corias testified that the occurrence was caused by a big flood in 1968 and a bamboo grove which used to indicate the position of the river was washed away. The river which flowed previously

in front of a chapel located 15 meters away from the riverbank within Bagaipo s property now flowed behind it. Corias was also present when Magno conducted the relocation survey in 1988. For his part, Lozano insisted that the land claimed by Bagaipo is actually an accretion to their titled property. He asserted that the Davao River did not change its course and that the reduction in Bagaipo s domain was caused by gradual erosion due to the current of the Davao River. Lozano added that it is also because of the river s natural action that silt slowly deposited and added to his land over a long period of time. He further averred that this accretion continues up to the present and that registration proceedings instituted by him over the alluvial formation could not be concluded precisely because it continued to increase in size. Lozano presented three witnesses: Atty. Pedro Castillo, his brother-in-law; Cabitunga Pasanday, a tenant of Atty. Castillo; and Alamin Catucag, a tenant of the Lozanos. Atty. Castillo testified that the land occupied by the Lozanos was transferred to his sister, Ramona when they extra-judicially partitioned their parents property upon his father s death. On September 9, 1973, Atty. Castillo filed a land registration case involving the accretion which formed on the property and submitted for this purpose, a survey plan approved by the Bureau of Lands as well as tax declarations covering the said accretion. An Order of General Default was already issued in the land registration case on November 5, 1975, but the case itself remained pending since the petition had to be amended to include the continuing addition to the land area. Mr. Cabitunga Pasanday testified that he has continuously worked on the land as tenant of the Castillos since 1925, tilling an area of about 3 hectares. However, the land he tilled located opposite the land of the Lozanos and adjacent to the Davao River has decreased over the years to its present size of about 1 hectare. He said the soil on the bank of the river, as well as coconut trees he planted would be carried away each time there was a flood. This similar erosion occurs on the properties of Bagaipo and a certain Dr. Rodriguez, since the elevation of the riverbank on their properties is higher than the elevation on Lozano s side.

Alamin Catucag testified that he has been a tenant of the Castillos since 1939 and that the portion he occupies was given to Ramona, Lozano s wife. It was only 1 hectare in 1939 but has increased to 3 hectares due to soil deposits from the mountains and river. Catucag said that Bagaipo s property was reduced to half since it is in the curve of the river and its soil erodes and gets carried away by river water. On April 5, 1991, the trial court conducted an ocular inspection. It concluded that the applicable law is Article 457. To the owners of lands adjoining the banks of rivers belong the accretion which they gradually receive from the effects of the current of the waters.7 of the New Civil Code and not Art. 461 The reduction in the land area of plaintiff was caused by erosion and not by a change in course of the Davao River. Conformably then, the trial court dismissed the complaint. On appeal, the Court of Appeals affirmed the decision of the trial court and decreed as follows: WHEREFORE, the decision appealed from is hereby affirmed, with costs against the plaintiff-appellant. Hence, this appeal. Petitioner asserts that the Court of Appeals erred in: ....NOT GIVING PROBATIVE VALUE TO THE RELOCATION SURVEY (EXHIBIT B ) PREPARED BY LICENSED GEODETIC ENGINEER GERSACIO MAGNO. THE CASE OF DIRECTOR OF LANDS VS. HEIRS OF JUANA CAROLINA 140 SCRA 396 CITED BY THE RESPONDENT COURT IN DISREGARDING EXHIBIT B IS NOT APPLICABLE TO THE CASE AT BAR. ....NOT FINDING THAT ASSUMING WITHOUT ADMITTING THAT THE QUESTIONED LOT 415-C (EXHIBIT B-1 ) OCCUPIED BY RESPONDENT LEONOR LOZANO WAS THE RESULT OF AN ACCRETION, THE PRINCIPLE OF ACCRETION CANNOT AND DOES NOT APPLY IN THE INSTANT CASE TO FAVOR SAID RESPONDENT BECAUSE SAID LOT 415-C IS WITHIN AND FORM PART OF PETITIONER S LAND DESCRIBED IN TCT NO. 15757 (EXHIBIT A )

....FINDING PETITIONER GUILTY OF LACHES WHEN SHE INSTITUTED THE SUIT. ....NOT ORDERING RESPONDENT LEONOR LOZANO TO VACATE AND SURRENDER LOT 415-C IN FAVOR OF PETITIONER AND FOR HIM TO PAY PETITIONER DAMAGES FOR ITS UNLAWFUL OCCUPATION THEREOF. ....NOT HOLDING PETITIONER ENTITLED TO THE ABANDONED RIVER BED. For this Court s resolution are the following issues: Did the trial court err in holding that there was no change in course of the Davao River such that petitioner owns the abandoned river bed pursuant to Article 461 of the Civil Code? Did private respondent own Lot 415-C in accordance with the principle of accretion under Article 457? Should the relocation survey prepared by a licensed geodetic engineer be disregarded since it was not approved by the Director of Lands? Is petitioner s claim barred by laches? On the first issue. The trial court and the appellate court both found that the decrease in land area was brought about by erosion and not a change in the river s course. This conclusion was reached after the trial judge observed during ocular inspection that the banks located on petitioner s land are sharp, craggy and very much higher than the land on the other side of the river. Additionally, the riverbank on respondent s side is lower and gently sloping. The lower land therefore naturally received the alluvial soil carried by the river current. These findings are factual, thus conclusive on this Court, unless there are strong and exceptional reasons, or they are unsupported by the evidence on record, or the judgment itself is based on a misapprehension of facts. These factual findings are based on an ocular inspection of the judge and convincing testimonies, and we find no convincing reason to disregard or disbelieve them. The decrease in petitioner s land area and the corresponding expansion of respondent s property were the combined effect of erosion and accretion respectively. Art. 461 of the Civil Code is inapplicable. Petitioner cannot claim ownership over the old abandoned riverbed because the same is inexistent. The riverbed s former location cannot even be pinpointed with

particularity since the movement of the Davao River took place gradually over an unspecified period of time, up to the present. The rule is well-settled that accretion benefits a riparian owner when the following requisites are present: 1) That the deposit be gradual and imperceptible; 2) That it resulted from the effects of the current of the water; and 3) That the land where accretion takes place is adjacent to the bank of the river. These requisites were sufficiently proven in favor of respondents. In the absence of evidence that the change in the course of the river was sudden or that it occurred through avulsion, the presumption is that the change was gradual and was caused by alluvium and erosion. As to Lot 415-C, which petitioner insists forms part of her property under TCT No. T-15757, it is well to recall our holding in C.N. Hodges vs. Garcia, 109 Phil. 133, 135: The fact that the accretion to his land used to pertain to plaintiff s estate, which is covered by a Torrens certificate of title, cannot preclude him (defendant) from being the owner thereof. Registration does not protect the riparian owner against the diminution of the area of his land through gradual changes in the course of the adjoining stream. Accretions which the banks of rivers may gradually receive from the effect of the current become the property of the owners of the banks (Art. 366 of the old Civil Code; Art. 457 of the new). Such accretions are natural incidents to land bordering on running streams and the provisions of the Civil Code in that respect are not affected by the Land Registration Act. Petitioner did not demonstrate that Lot 415-C allegedly comprising 29,162 square meters was within the boundaries of her titled property. The survey plan commissioned by petitioner which was not approved by the Director of Lands was properly discounted by the appellate court. In Titong vs. Court of Appeals we affirmed the trial court s refusal to give probative value to a private survey plan and held thus: the plan was not verified and approved by the Bureau of Lands in accordance with Sec. 28, paragraph 5 of Act No. 2259, the Cadastral Act, as amended by Sec. 1862 of Act No. 2711. Said law ordains that private surveyors send their original field notes, computations, reports, surveys, maps and plots regarding a piece of property to the Bureau of Lands for

verification and approval. A survey plan not verified and approved by said Bureau is nothing more than a private writing, the due execution and authenticity of which must be proven in accordance with Sec. 20 of Rule 132 of the Rules of Court. The circumstance that the plan was admitted in evidence without any objection as to its due execution and authenticity does not signify that the courts shall give probative value therefor. To admit evidence and not to believe it subsequently are not contradictory to each other In view of the foregoing, it is no longer necessary now to discuss the defense of laches. It is mooted by the disquisition on the foregoing issues. WHEREFORE, the assailed decision dated June 30, 1994, of the Court of Appeals in C.A.-G. R. CV No. 37615, sustaining the judgment of the court a quo, is AFFIRMED. Costs against petitioner. SO ORDERED. G.R. No. 68166 February 12, 1997 HEIRS OF EMILIANO NAVARRO, petitioner, vs. INTERMEDIATE APPELLATE COURT & HEIRS OF SINFOROSO PASCUAL, respondents.

HERMOSISIMA, JR., J.: Unique is the legal question visited upon the claim of an applicant in a Land Registration case by oppositors thereto, the Government and a Government lessee, involving as it does ownership of land formed by alluvium. The applicant owns the property immediately adjoining the land sought to be registered. His registered property is bounded on the east by the Talisay River, on the west by the Bulacan River, and on the north by the Manila Bay. The Talisay River and the Bulacan River flow down towards the Manila

Bay and act as boundaries of the applicant's registered land on the east and on the west. The land sought to be registered was formed at the northern tip of the applicant's land. Applicant's registered property is bounded on the north by the Manila Bay. The issue: May the land sought to be registered be deemed an accretion in the sense that it naturally accrues in favor of the riparian owner or should the land be considered as foreshore land? Before us is a petition for review of: (1) the decision 1 and (2) two subsequent resolutions 2 of the Intermediate Appellate Court 3 (now the Court of Appeals) in Land Registration Case No. N-84, 4 the application over which was filed by private respondents' predecessor-in-interest, Sinforoso Pascual, now deceased, before the Court of First Instance 5 (now the Regional Trial Court) of Balanga, Bataan. There is no dispute as to the following facts: On October 3, 1946, Sinforoso Pascual, now deceased, filed an application for foreshore lease covering a tract of foreshore land in Sibocon, Balanga, Bataan, having an area of approximately seventeen (17) hectares. This application was denied on January 15, 1953. So was his motion for reconsideration. Subsequently, petitioners' predecessor-in-interest, also now deceased, Emiliano Navarro, filed a fishpond application with the Bureau of Fisheries covering twenty five (25) hectares of foreshore land also in Sibocon, Balanga, Bataan. Initially, such application was denied by the Director of Fisheries on the ground that the property formed part of the public domain. Upon motion for reconsideration, the Director of Fisheries, on May 27, 1958, gave due course to his application but only to the extent of seven (7) hectares of the property as may be certified by the Bureau of Forestry as suitable for fishpond purposes. The Municipal Council of Balanga, Bataan, had opposed Emiliano Navarro's application. Aggrieved by the decision of the Director of Fisheries, it appealed to the Secretary of Natural Resources who, however, affirmed the

grant. The then Executive Secretary, acting in behalf of the President of the Philippines, similarly affirmed the grant. On the other hand, sometime in the early part of 1960, Sinforoso Pascual flied an application to register and confirm his title to a parcel of land, situated in Sibocon, Balanga, Bataan, described in Plan Psu-175181 and said to have an area of 146,611 square meters. Pascual claimed that this land is an accretion to his property, situated in Barrio Puerto Rivas, Balanga, Bataan, and covered by Original Certificate of Title No. 6830. It is bounded on the eastern side by the Talisay River, on the western side by the Bulacan River, and on the northern side by the Manila Bay. The Talisay River as well as the Bulacan River flow downstream and meet at the Manila Bay thereby depositing sand and silt on Pascual's property resulting in an accretion thereon. Sinforoso Pascual claimed the accretion as the riparian owner. On March 25, 1960, the Director of Lands, represented by the Assistant Solicitor General, filed an opposition thereto stating that neither Pascual nor his predecessors-in-interest possessed sufficient title to the subject property, the same being a portion of the public domain and, therefore, it belongs to the Republic of the Philippines. The Director of Forestry, through the Provincial Fiscal, similarly opposed Pascual's application for the same reason as that advanced by the Director of Lands. Later on, however, the Director of Lands withdrew his opposition. The Director of Forestry become the sole oppositor. On June 2, 1960, the court a quo issued an order of general default excepting the Director of Lands and the Director of Forestry. Upon motion of Emiliano Navarro, however, the order of general default was lifted and, on February 13, 1961, Navarro thereupon filed an opposition to Pascual's application. Navarro claimed that the land sought to be registered has always been part of the public domain, it being a part of the foreshore of Manila Bay; that he was a lessee and in possession of a part of the subject property by virtue of a fishpond permit issued by the Bureau of Fisheries and confirmed by the Office of the President; and that be bad already converted the area covered by the lease into a fishpond.

During the pendency of the land registration case, that is, on November 6, 1960, Sinforoso Pascual filed a complaint for ejectment against Emiliano Navarro, one Marcelo Lopez and their privies, alleged by Pascual to have unlawfully claimed and possessed, through stealth, force and strategy, a portion of the subject property covered by Plan Psu-175181. The defendants in the case were alleged to have built a provisional dike thereon: thus they have thereby deprived Pascual of the premises sought to be registered. This, notwithstanding repeated demands for defendants to vacate the property. The case was decided adversely against Pascual. Thus, Pascual appealed to the Court of First Instance (now Regional Trial Court) of Balanga, Bataan, the appeal having been docketed as Civil Case No. 2873. Because of the similarity of the parties and the subject matter, the appealed case for ejectment was consolidated with the land registration case and was jointly tried by the court a quo. During the pendency of the trial of the consolidated cases, Emiliano Navarro died on November 1, 1961 and was substituted by his heirs, the herein petitioners. Subsequently, on August 26, 1962, Pascual died and was substituted by his heirs, the herein private respondents. On November 10, 1975, the court a quo rendered judgment finding the subject property to be foreshore land and, being a part of the public domain, it cannot be the subject of land registration proceedings. The decision's dispositive portion reads: WHEREFORE, judgment is rendered: (1) Dismissing plaintiff [private respondent] Sinforoso Pascual's complaint for ejectment in Civil Case No. 2873; (2) Denying the application of Sinforoso Pascual for land registration over the land in question; and

(3) Directing said Sinforoso Pascual, through his heirs, as plaintiff in Civil Case No. 2873 and as applicant in Land Registration Case No. N-84 to pay costs in both instances."

The heirs of Pascual appealed and, before the respondent appellate court, assisted the following errors: 1. The lower court erred in not finding the land in question as an accretion by the action of the Talisay and Bulacan Rivers to the land admittedly owned by applicants-appellants [private respondents]. 2. The lower court erred in holding that the land in question is foreshore land. 3. The lower court erred in not ordering the registration of the land in controversy in favor of applicants-appellants [private respondents]. 4. The lower court erred in not finding that the applicantsappellants [private respondents] are entitled to eject the oppositor-appellee [petitioners]. 7 On appeal, the respondent court reversed the findings of the court a quo and granted the petition for registration of the subject property but excluding therefrom fifty (50) meters from corner 2 towards corner 1; and fifty meters (50) meters from corner 5 towards corner 6 of the Psu-175181. The respondent appellate court explained the reversal in this wise: The paramount issue to be resolved in this appeal as set forth by the parties in their respective briefs is whether or not the land sought to be registered is accretion or foreshore land, or, whether or not said land was formed by the action of the two rivers of Talisay and Bulacan or by the action of the Manila Bay. If formed by the action of the Talisay and Bulacan rivers, the subject land is accretion but if formed by the action of the Manila Bay then it is foreshore land.

xxx xxx xxx It is undisputed that applicants-appellants [private respondents] owned the land immediately adjoining the land sought to be registered. Their property which is covered by OCT No. 6830 is bounded on the east by the Talisay River, on the west by the Bulacan River, and on the north by the Manila Bay. The Talisay and Bulacan rivers come from inland flowing downstream towards the Manila Bay. In other words, between the Talisay River and the Bulacan River is the property of applicants with both rivers acting as the boundary to said land and the flow of both rivers meeting and emptying into the Manila Bay. The subject land was formed at the tip or apex of appellants' [private respondents'] land adding thereto the land now sought to be registered. This makes this case quite unique because while it is undisputed that the subject land is immediately attached to appellants' [private respondents'] land and forms the tip thereof, at the same time, said land immediately faces the Manila Bay which is part of the sea. We can understand therefore the confusion this case might have caused the lower court, faced as it was with the uneasy problem of deciding whether or not the subject land was formed by the action of the two rivers or by the action of the sea. Since the subject land is found at the shore of the Manila Bay facing appellants' [private respondents'] land, it would be quite easy to conclude that it is foreshore and therefore part of the patrimonial property of the State as the lower court did in fact rule . . . . xxx xxx xxx It is however undisputed that appellants' [private respondents'] land lies between these two rivers and it is precisely appellants' [private respondents'] land which acts as a barricade preventing these two rivers to meet. Thus, since the flow of the two rivers is downwards to the Manila Bay the sediments of sand and silt are deposited at their mouths.

It is, therefore, difficult to see how the Manila Bay could have been the cause of the deposit thereat for in the natural course of things, the waves of the sea eat the land on the shore, as they suge [sic] inland. It would not therefore add anything to the land but instead subtract from it due to the action of the waves and the wind. It is then more logical to believe that the two rivers flowing towards the bay emptied their cargo of sand, silt and clay at their mouths, thus causing appellants' [private respondents'] land to accumulate therein. However, our distinguished colleage [sic], Mr. Justice Serrano, do [sic] not seem to accept this theory and stated that the subject land arose only when . . . . Pascual planted "palapat" and "bakawan" trees thereat to serve as a boundary or strainer. But we do not see how this act of planting trees by Pascual would explain how the land mass came into being. Much less will it prove that the same came from the sea. Following Mr. Justice Serrano's argument that it were the few trees that acted as strainers or blocks, then the land that grew would have stopped at the place where the said trees were planted. But this is not so because the land mass went far beyond the boundary, or where the trees were planted. On the other hand, the picture-exhibits of appellants [private respondents] clearly show that the land that accumulated beyond the so- called boundary, as well as the entire area being applied for is dry land, above sea level, and bearing innumerable trees . . . The existence of vegetation on the land could only confirm that the soil thereat came from inland rather than from the sea, for what could the sea bring to the shore but sand, pebbles, stones, rocks and corrals? On the other hand, the two rivers would be bringing soil on their downward flow which they brought along from the eroded mountains, the lands along their path, and dumped them all on the northern portion of appellants' [private respondents'] land. In view of the foregoing, we have to deviate from the lower court's finding. While it is true that the subject land is found at

the shore of the Manila Bay fronting appellants' [private respondents'] land, said land is not foreshore but an accretion from the action of the Talisay and Bulacan rivers. In fact, this is exactly what the Bureau of Lands found out, as shown in the following report of the Acting Provincial Officer, Jesus M. Orozco, to wit: "Upon ocular inspection of the land subject of this registration made on June 11, 1960, it was found out that the said land is . . . . sandwitched [sic] by two big rivers . . . . These two rivers bring down considerable amount of soil and sediments during floods every year thus raising the soil of the land adjoining the private property of the applicant [private respondents]. About four-fifth [sic] of the area applied for is now dry land whereon are planted palapat trees thickly growing thereon. It is the natural action of these two rivers that has caused the formation of said land . . . . subject of this registration case. It has been formed, therefore, by accretion. And having been formed by accretion, the said land may be considered the private property of the riparian owner who is the applicant herein [private respondents] . . . . In view of the above, the opposition hereto filed by the government should be withdrawn, except for the portion recommended by the land investigator in his report dated May 2, 1960, to be excluded and considered foreshore. . . ." Because of this report, no less than the Solicitor General representing the Bureau of Lands withdrew his opposition dated March 25, 1960, and limited "the same to the northern portion of the land applied for, compromising a strip 50 meters wide along the Manila Bay, which should be declared public land as part of the foreshore" . . . . 8

Pursuant to the aforecited decision, the respondent appellate court ordered the issuance of the corresponding decree of registration in the name of private respondents and the reversion to private respondents of the possession of the portion of the subject property included in Navarro's fishpond permit. On December 20, 1978, petitioners filed a motion for reconsideration of the aforecited decision. The Director of Forestry also moved for the reconsideration of the same decision. Both motions were opposed by private respondents on January 27, 1979. On November 21, 1980, respondent appellate court promulgated a resolution denying the motion for reconsideration filed by the Director of Forestry. It, however, modified its decision, to read, viz: (3). Ordering private oppositors Heirs of Emiliano Navarro to vacate that portion included in their fishpond permit covered by Plan Psu-175181 and hand over possession of said portion to applicants-appellants, if the said portion is not within the strip of land fifty (50) meters wide along Manila Bay on the northern portion of the land subject of the registration proceedings and which area is more particularly referred to as fifty (50) meters from corner 2 towards corner 1; and fifty (50) meters from corner 5 towards corner 6 of Plan Psu-175181. . . . 9 On December 15, 1980, we granted the Solicitor General, acting as counsel for the Director of Forestry, an extension of time within which to file in this court, a petition for review of the decision dated November 29, 1978 of the respondent appellate court and of the aforecited resolution dated November 21, 1980. Thereafter, the Solicitor General, in behalf of the Director of Forestry, filed a petition for review entitled, "The Director of Forestry vs. the Court of Appeals." 10 We, however, denied the same in a minute resolution dated July 20, 1981, such petition having been prematurely filed at a time when the Court of Appeals was yet to resolve petitioners' pending motion to set aside the resolution dated November 21, 1980.

On October 9, 1981, respondent appellate court denied petitioners' motion for reconsideration of the decision dated November 29, 1978. On October 17, 1981, respondent appellate court made an entry of judgment stating that the decision dated November 29, 1978 had become final and executory as against herein petitioners as oppositors in L.R.C. Case No. N-84 and Civil Case No. 2873 of the Court of First Instance (now the Regional Trial Court) of Balanga, Bataan. On October 26, 1981, a second motion for reconsideration of the decision dated November 29, 1978 was filed by petitioners' new counsel. On March 26, 1982, respondent appellate court issued a resolution granting petitioners' request for leave to file a second motion for reconsideration. On July 13, 1984, after hearing, respondent appellate court denied petitioners' second motion for reconsideration on the ground that the same was filed out of time, citing Rule 52, Section 1 of the Rules of Court which provides that a motion for reconsideration shall be made ex-parte and filed within fifteen (15) days from the notice of the final order or judgment. Hence this petition where the respondent appellate court is imputed to have palpably erred in appreciating the fact of the case and to have gravely misapplied statutory and case law relating to accretion, specifically, Article 457 of the Civil Code. We find no merit in the petition. The disputed property was brought forth by both the withdrawal of the waters of Manila Bay and the accretion formed on the exposed foreshore land by the action of the sea which brought soil and sand sediments in turn trapped by the palapat and bakawan trees planted thereon by petitioner Sulpicio Pascual in 1948 Anchoring their claim of ownership on Article 457 of the Civil Code, petitioners vigorously argue that the disputed 14-hectare land is an accretion caused by the joint action of the Talisay and Bulacan Rivers

which run their course on the eastern and western boundaries, respectively, of petitioners' own tract of land. Accretion as a mode of acquiring property under said Article 457, requires the concurrence of the following requisites: (1) that the accumulation of soil or sediment be gradual and imperceptible; (2) that it be the result of the action of the waters of the river; and (3) that the land where the accretion takes place is adjacent to the bank of the river. 11 Accretion is the process whereby the soil is deposited, while alluvium is the soil deposited on the estate fronting the river bank 12; the owner of such estate is called the riparian owner. Riparian owners are, strictly speaking, distinct from littoral owners, the latter being owners of lands bordering the shore of the sea or lake or other tidal waters. 13 The alluvium, by mandate of Article 457 of the Civil Code, is automatically owned by the riparian owner from the moment the soil deposit can be seen 14 but is not automatically registered property, hence, subject to acquisition through prescription by third persons 15. Petitioners' claim of ownership over the disputed property under the principle of accretion, is misplaced. First, the title of petitioners' own tract of land reveals its northeastern boundary to be Manila Bay. Petitioners' land, therefore, used to adjoin, border or front the Manila Bay and not any of the two rivers whose torrential action, petitioners insist, is to account for the accretion on their land. In fact, one of the petitioners, Sulpicio Pascual, testified in open court that the waves of Manila Bay used to hit the disputed land being part of the bay's foreshore but, after he had planted palapat and bakawan trees thereon in 1948, the land began to rise. 16 Moreover, there is no dispute as to the location of: (a) the disputed land; (b) petitioners' own tract of land; (c) the Manila Bay; and, (d) the Talisay and Bulacan Rivers. Petitioners' own land lies between the Talisay and Bulacan Rivers; in front of their land on the northern side lies now the disputed land where before 1948, there lay the Manila Bay. If the accretion were to be attributed to the action of either or both of the Talisay and Bulacan Rivers, the alluvium should have been deposited on either or both of the eastern and western boundaries of petitioners' own tract of land, not

on the northern portion thereof which is adjacent to the Manila Bay. Clearly lacking, thus, is the third requisite of accretion, which is, that the alluvium is deposited on the portion of claimant's land which is adjacent to the river bank. Second, there is no dispute as to the fact that petitioners' own tract of land adjoins the Manila Bay. Manila Bay is obviously not a river, and jurisprudence is already settled as to what kind of body of water the Manila Bay is. It is to be remembered that we held that: Appellant next contends that . . . . Manila Bay cannot be considered as a sea. We find said contention untenable. A bay is part of the sea, being a mere indentatiom of the same: "Bay. An opening into the land where the water is shut in on all sides except at the entrance; an inlet of the sea; an arm of the sea, distinct from a river, a bending or curbing of the shore of the sea or of a lake. " 7 C.J. 1013-1014." 17 The disputed land, thus, is an accretion not on a river bank but on a sea bank, or on what used to be the foreshore of Manila Bay which adjoined petitioners' own tract of land on the northern side. As such, the applicable law is not Article 457 of to Civil Code but Article 4 of the Spanish Law of Waters of 1866. The process by which the disputed land was formed, is not difficult to discern from the facts of the case. As the trial court correctly observed: A perusal of the survey plan . . . . of the land subject matter of these cases shows that on the eastern side, the property is bounded by Talisay River, on the western side by Bulacan River, on the southern side by Lot 1436 and on the northern side by Manila Bay. It is not correct to state that the Talisay and Bulacan Rivers meet a certain portion because the two rivers both flow towards Manila Bay. The Talisay River is straight while the Bulacan River is a little bit meandering and there is no portion where the two rivers meet before they end up at Manila Bay. The land which is adjacent to the property

belonging to Pascual cannot be considered an accretion [caused by the action of the two rivers]. Applicant Pascual . . . . has not presented proofs to convince the Court that the land he has applied for registration is the result of the settling down on his registered land of soil, earth or other deposits so as to be rightfully be considered as an accretion [caused by the action of the two rivers]. Said Art. 457 finds no applicability where the accretion must have been caused by action of the bay. 18 The conclusion formed by the trial court on the basis of the aforegoing observation is that the disputed land is part of the foreshore of Manila Bay and therefore, part of the public domain. The respondent appellate court, however, perceived the fact that petitioners' own land lies between the Talisay and Bulacan Rivers, to be basis to conclude that the disputed land must be an accretion formed by the action of the two rivers because petitioners' own land acted as a barricade preventing the two rivers to meet and that the current of the two rivers carried sediments of sand and silt downwards to the Manila Bay which accumulated somehow to a 14hectare land. These conclusions, however, are fatally incongruous in the light of the one undisputed critical fact: the accretion was deposited, not on either the eastern or western portion of petitioners' land where a river each runs, but on the northern portion of petitioners' land which adjoins the Manila Bay. Worse, such conclusions are further eroded of their practical logic and consonance with natural experience in the light of Sulpicio Pascual's admission as to having planted palapat and bakawan trees on the northern boundary of their own land. In amplification of this, plainly more reasonable and valid are Justice Mariano Serrano's observations in his dissenting opinion when he stated that: As appellants' (titled) land . . . . acts as a barricade that prevents the two rivers to meet, and considering the wide expanse of the boundary between said land and the Manila Bay, measuring some 593.00 meters . . . . it is believed rather farfetched for the land in question to have been formed through "sediments of sand and salt [sic] . . . . deposited at their [rivers'] mouths." Moreover, if "since the flow of the two

rivers is downwards to the Manila Bay the sediments of sand and silt are deposited at their mouths," why then would the alleged cargo of sand, silt and clay accumulate at the northern portion of appellants' titled land facing Manila Bay instead of merely at the mouths and banks of these two rivers? That being the case, the accretion formed at said portion of appellants' titled [land] was not caused by the current of the two rivers but by the action of the sea (Manila Bay) into which the rivers empty. The conclusion . . . . is not supported by any reference to the evidence which, on the contrary, shows that the disputed land was formed by the action of the sea. Thus, no less than Sulpicio Pascual, one of the heirs of the original applicant, testified on cross-examination that the land in dispute was part of the shore and it was only in 1948 that he noticed that the land was beginning to get higher after he had planted trees thereon in 1948. . . . . . . . . it is established that before 1948 sea water from the Manila Bay at high tide could reach as far as the dike of appellants' fishpond within their titled property, which dike now separates this titled property from the land in question. Even in 1948 when appellants had already planted palapat and bakawan trees in the land involved, inasmuch as these trees were yet small, the waves of the sea could still reach the dike. This must be so because in . . . . the survey plan of the titled property approved in 1918, said titled land was bounded on the north by Manila Bay. So Manila Bay was adjacent to it on the north. It was only after the planting of the aforesaid trees in 1948 that the land in question began to rise or to get higher in elevation. The trees planted by appellants in 1948 became a sort of strainer of the sea water and at the same time a kind of block to the strained sediments from being carried back to the sea by the very waves that brought them to the former shore at the

end of the dike, which must have caused the shoreline to recede and dry up eventually raising the former shore leading to the formation of the land in question." 19 In other words, the combined and interactive effect of the planting of palapat and bakawan trees, the withdrawal of the waters of Manila Bay eventually resulting in the drying up of its former foreshore, and the regular torrential action of the waters of Manila Bay, is the formation of the disputed land on the northern boundary of petitioners' own tract of land. The disputed property is an accretion on a sea bank, Manila Bay being an inlet or an arm of the sea; as such, the disputed property is, under Article 4 of the Spanish Law of Waters of 1866, part of the public domain At the outset, there is a need to distinguish between Manila Bay and Laguna de Bay. While we held in the case of Ignacio v. Director of Lands and Valeriano 20 that Manila Bay is considered a sea for purposes of determining which law on accretion is to be applied in multifarious situations, we have ruled differently insofar as accretions on lands adjoining the Laguna de Bay are concerned. In the cases of Government of the P.I v. Colegio de San Jose 21, Republic v. Court of Appeals 22, Republic v. Alagad 23, and Meneses v. Court of Appeals 24, we categorically ruled that Laguna de Bay is a lake the accretion on which, by the mandate of Article 84 of the Spanish Law of Waters of 1866, belongs to the owner of the land contiguous thereto. The instant controversy, however, brings a situation calling for the application of Article 4 of the Spanish Law of Waters of 1866, the disputed land being an accretion on the foreshore of Manila Bay which is, for all legal purposes, considered a sea. Article 4 of the Spanish Law of Waters of August 3, 1866 provides as follows:

Lands added to the shores by accretions and alluvial deposits caused by the action of the sea, form part of the public domain. When they are no longer washed by the waters of the sea and are not necessary for purposes of public utility, or for the establishment of special industries, or for the coast-guard service, the Government shall declare them to be the property of the owners of the estates adjacent thereto and as increment thereof. In the light of the aforecited vintage but still valid law, unequivocal is the public nature of the disputed land in this controversy, the same being an accretion on a sea bank which, for all legal purposes, the foreshore of Manila Bay is. As part of the public domain, the herein disputed land is intended for public uses, and "so long as the land in litigation belongs to the national domain and is reserved for public uses, it is not capable of being appropriated by any private person, except through express authorization granted in due form by a competent authority." 25 Only the executive and possibly the legislative departments have the right and the power to make the declaration that the lands so gained by action of the sea is no longer necessary for purposes of public utility or for the cause of establishment of special industries or for coast guard services. 26 Petitioners utterly fail to show that either the executive or legislative department has already declared the disputed land as qualified, under Article 4 of the Spanish Law of Waters of 1866, to be the property of petitioners as owners of the estates adjacent thereto. WHEREFORE, the instant Petition for Review is hereby DENIED and DISMISSED. Costs against petitioners. SO ORDERED. [G.R. No. 98045. June 26, 1996] DESAMPARADO VDA. DE NAZARENO and LETICIA NAZARENO TAPIA, petitioners, vs. THE COURT OF APPEALS, MR. & MRS. JOSE SALASALAN, MR. & MRS. LEO RABAYA, AVELINO LABIS, HON. ROBERTO G. HILARIO,

ROLLEO I. IGNACIO, ALBERTO M. GILLERA and HON. ABELARDO G. PALAD, JR., in their official and/or private capacities, respondents. SYLLABUS 1. CIVIL LAW; OWNERSHIP; RIGHTS OF ACCESSION WITH RESPECT TO IMMOVABLE PROPERTY; ARTICLE 457; REQUISITES.- In the case of Meneses vs. CA, this Court held that accretion, as a mode of acquiring property under Art. 457 of the Civil Code, requires the concurrence of these requisites: (1) that the deposition of soil or sediment be gradual and imperceptible; (2) that it be the result of the action of the waters of the river (or sea); and (3) that the land where accretion takes place is adjacent to the banks of rivers (or the sea coast). These are called the rules on alluvion which if present in a case, give to the owners of lands adjoining the banks of rivers or streams any accretion gradually received from the effects of the current of waters. 2. ID.; ID.; ID.; ID.; ID.; NOT PRESENT IN CASE AT BAR.- Where the accretion was formed by the dumping of boulders, soil and other filling materials on portions of the Balacanas Creek and the Cagayan River bounding petitioner's land, it cannot be claimed that the accumulation was gradual and imperceptible, resulting from the action of the waters or the current of the creek and the river. In Hilario vs. City of Manila, this Court held that the word current indicates the participation of the body of water in the ebb and flow of waters due to high and low tide. Not having met the first and second requirements of the rules of alluvion, petitioners cannot claim the rights of a riparian owner. 3. ID.; ID.; ID.; ID.; ID.; THAT DEPOSIT IS DUE TO THE CURRENT OF THE RIVER, MANDATORY.- In Republic vs. CA, this Court ruled that the requirement that the deposit should be due to the effect of the current of the river is indispensable. This excludes from Art. 457 of the Civil Code all deposits caused by human intervention. Putting it differently, alluvion must be the exclusive work of nature. Thus, in Tiongco vs. Director of Lands, et al., where the land was not formed solely by the natural effect of the water current of the river bordering said land but is also the consequence of the direct and deliberate intervention of man, it was deemed a man-made accretion and, as such, part of the public domain. In the case at bar, the subject land was the direct result of the dumping of

sawdust by the Sun Valley Lumber Co. consequent to its sawmill operations. 4. ID.; PUBLIC LANDS; FINDINGS AS SUCH BY THE BUREAU OF LANDS, RESPECTED.- The mere filing of the Miscellaneous Sales Application constituted an admission that the land being applied for was public land, having been the subject of a Survey Plan wherein said land was described as an orchard. Furthermore, the Bureau of Lands classified the subject land as an accretion area which was formed by deposits of sawdust in the Balacanas Creek and the Cagayan river, in accordance with the ocular inspection conducted by the Bureau of Lands. This Court has often enough held that findings of administrative agencies which have acquired expertise because their jurisdiction is confined to specific matters are generally accorded not only respect but even finality. Again, when said factual findings are affirmed by the Court of Appeals, the same are conclusive on the parties and not reviewable by this Court. 5. ID.; PUBLIC LAND LAW; JURISDICTION OVER PUBLIC LANDS.- Having determined that the subject land is public land, a fortiori, the Bureau of Lands, as well as the Office of the Secretary of Agriculture and Natural Resources have jurisdiction over the same in accordance with the Public Land Law. Under Sections 3 and 4 thereof, the Director of Lands has jurisdiction, authority and control over public lands. Here respondent Palad as Director of Lands, is authorized to exercise executive control over any form of concession, disposition and management of the lands of the public domain. He may issue decisions and orders as he may see fit under the circumstances as long as they are based on the findings of fact. In the case of Calibo vs. Ballesteros, this Court held that where, in the disposition of public lands, the Director of Lands bases his decision on the evidence thus presented, he clearly acts within his jurisdiction, and if he errs in appraising the evidence, the error is one of judgment, but not an act of grave abuse of discretion annullable by certiorari. 6. ADMINISTRATIVE LAW; ADMINISTRATIVE REMEDIES; EXHAUSTED IN CASE AT BAR.- The administrative remedies have been exhausted. Petitioners could not have intended to appeal to respondent Ignacio as an Officer-In-Charge of the Bureau of Lands. The decision being appealed from was the decision of respondent Hilario who was the Regional Director

of the Bureau of Lands. Said decision was made "for and by authority of the Director of Lands." It would be incongruous to appeal the decision of the Regional Director of the Bureau of Lands acting for the Director of the Bureau of Lands to an Officer-In-Charge of the Bureau of Lands. In any case, respondent Ignacio's official designation was "Undersecretary of the Department of Agriculture and Natural Resources." He was only an "Officer-In-Charge" of the Bureau of Lands. When he acted on the late Antonio Nazareno's motion for reconsideration by affirming or adopting respondent Hilario's decision, he was acting on said motion as an Undersecretary on behalf of the Secretary of the Department. In the case of Hamoy vs. Secretary of Agriculture and Natural Resources, this Court held that the Undersecretary of Agriculture and Natural Resources may modify, adopt, or set aside the orders or decisions of the Director of Lands with respect to questions involving public lands under the administration and control of the Bureau of Lands and the Department of Agriculture and Natural Resources. He cannot, therefore, be said to have acted beyond the bounds of his jurisdiction under Sections 3, 4 and 5 of Commonwealth Act No. 141. APPEARANCES OF COUNSEL Manolo L. Tagarda, Sr. for petitioners. Arturo R. Legaspi for private respondents. DECISION ROMERO, J.: Petitioners Desamparado Vda. de Nazareno and Leticia Nazareno Tapia challenge the decision of the Court of Appeals which affirmed the dismissal of petitioners' complaint by the Regional Trial Court of Misamis Oriental, Branch 22. The complaint was for annulment of the verification, report and recommendation, decision and order of the Bureau of Lands regarding a parcel of public land. The only issue involved in this petition is whether or not petitioners exhausted administrative remedies before having recourse to the courts.

The subject of this controversy is a parcel of land situated in Telegrapo, Puntod, Cagayan de Oro City. Said land was formed as a result of sawdust dumped into the dried-up Balacanas Creek and along the banks of the Cagayan river. Sometime in 1979, private respondents Jose Salasalan and Leo Rabaya leased the subject lots on which their houses stood from one Antonio Nazareno, petitioners' predecessor-in-interest. In the latter part of 1982, private respondents allegedly stopped paying rentals. As a result, Antonio Nazareno and petitioners filed a case for ejectment with the Municipal Trial Court of Cagayan de Oro City, Branch 4. A decision was rendered against private respondents, which decision was affirmed by the Regional Trial Court of Misamis Oriental, Branch 20. The case was remanded to the municipal trial court for execution of judgment after the same became final and executory. Private respondents filed a case for annulment of judgment before the Regional Trial Court of Misamis Oriental, Branch 24 which dismissed the same. Antonio Nazareno and petitioners again moved for execution of judgment but private respondents filed another case for certiorari with prayer for restraining order and/or writ of preliminary injunction with the Regional Trial Court of Misamis Oriental, Branch 25 which was likewise dismissed. The decision of the lower court was finally enforced with the private respondents being ejected from portions of the subject lots they occupied. Before he died, Antonio Nazareno caused the approval by the Bureau of Lands of the survey plan designated as Plan Csd-106-00571 with a view to perfecting his title over the accretion area being claimed by him. Before the approved survey plan could be released to the applicant, however, it was protested by private respondents before the Bureau of Lands. In compliance with the order of respondent District Land Officer Alberto M. Gillera, respondent Land Investigator Avelino G. Labis conducted an investigation and rendered a report to the Regional Director recommending that Survey Plan No. MSI-10-06-000571-D (equivalent to Lot No. 36302, Cad. 237) in the name of Antonio Nazareno, be cancelled and that private respondents be directed to file appropriate public land applications.

Based on said report, respondent Regional Director of the Bureau of Lands Roberto Hilario rendered a decision ordering the amendment of the survey plan in the name of Antonio Nazareno by segregating therefrom the areas occupied by the private respondents who, if qualified, may file public land applications covering their respective portions. Antonio Nazareno filed a motion for reconsideration with respondent Rolleo Ignacio, Undersecretary of the Department of Natural Resources and Officer-in-Charge of the Bureau of Lands who denied the motion. Respondent Director of Lands Abelardo Palad then ordered him to vacate the portions adjudicated to private respondents and remove whatever improvements they have introduced thereon. He also ordered that private respondents be placed in possession thereof. Upon the denial of the late Antonio Nazareno's motion for reconsideration, petitioners Desamparado Vda. de Nazareno and Leticia Tapia Nazareno, filed a case before the RTC, Branch 22 for annulment of the following: order of investigation by respondent Gillera, report and recommendation by respondent Labis, decision by respondent Hilario, order by respondent Ignacio affirming the decision of respondent Hilario and order of execution by respondent Palad. The RTC dismissed the complaint for failure to exhaust administrative remedies which resulted in the finality of the administrative decision of the Bureau of Lands. On appeal, the Court of Appeals affirmed the decision of the RTC dismissing the complaint. Applying Section 4 of C.A. No. 141, as amended, it contended that the approval of the survey plan belongs exclusively to the Director of Lands. Hence, factual findings made by the Metropolitan Trial Court respecting the subject land cannot be held to be controlling as the preparation and approval of said survey plans belong to the Director of Lands and the same shall be conclusive when approved by the Secretary of Agriculture and Natural Resources.[1] Furthermore, the appellate court contended that the motion for reconsideration filed by Antonio Nazareno cannot be considered as an appeal to the Office of the Secretary of Agriculture and Natural Resources, as mandated by C.A. No. 141 inasmuch as the same had been acted upon by respondent Undersecretary Ignacio in his capacity as Officer-in-Charge of the Bureau of Lands and not as Undersecretary acting for the Secretary

of Agriculture and Natural Resources. For the failure of Antonio Nazareno to appeal to the Secretary of Agriculture and Natural Resources, the present case does not fall within the exception to the doctrine of exhaustion of administrative remedies. It also held that there was no showing of oppressiveness in the manner in which the orders were issued and executed. Hence, this petition. Petitioners assign the following errors: I. PUBLIC RESPONDENT COURT OF APPEALS IN A WHIMSICAL, ARBITRARY AND CAPRICIOUS MANNER AFFIRMED THE DECISION OF THE LOWER COURT WHICH IS CONTRARY TO THE PREVAILING FACTS AND THE LAW ON THE MATTER; II. PUBLIC RESPONDENT COURT OF APPEALS IN A WHIMSICAL, ARBITRARY AND CAPRICIOUS MANNER AFFIRMED THE DECISION OF THE LOWER COURT DISMISSING THE ORIGINAL CASE WHICH FAILED TO CONSIDER THAT THE EXECUTION ORDER OF PUBLIC RESPONDENT ABELARDO G. PALAD, JR., DIRECTOR OF LANDS, MANILA, PRACTICALLY CHANGED THE DECISION OF PUBLIC RESPONDENT ROBERTO HILARIO, REGIONAL DIRECTOR, BUREAU OF LANDS, REGION 10, THUS MAKING THE CASE PROPER SUBJECT FOR ANNULMENT WELL WITHIN THE JURISDICTION OF THE LOWER COURT. The resolution of the above issues, however, hinges on the question of whether or not the subject land is public land. Petitioners claim that the subject land is private land being an accretion to his titled property, applying Article 457 of the Civil Code which provides: "To the owners of lands adjoining the banks of rivers belong the accretion which they gradually receive from the effects of the current of the waters." In the case of Meneses v. CA,[2] this Court held that accretion, as a mode of acquiring property under Art. 457 of the Civil Code, requires the concurrence of these requisites: (1) that the deposition of soil or sediment be gradual and imperceptible; (2) that it be the result of the action of the waters of the river (or sea); and (3) that the land where accretion takes

place is adjacent to the banks or rivers (or the sea coast). These are called the rules on alluvion which if present in a case, give to the owners of lands adjoining the banks of rivers or streams any accretion gradually received from the effects of the current of waters. For petitioners to insist on the application of these rules on alluvion to their case, the above-mentioned requisites must be present. However, they admit that the accretion was formed by the dumping of boulders, soil and other filling materials on portions of the Balacanas Creek and the Cagayan River bounding their land.[3] It cannot be claimed, therefore, that the accumulation of such boulders, soil and other filling materials was gradual and imperceptible, resulting from the action of the waters or the current of the Balacanas Creek and the Cagayan River. In Hilario v. City of Manila,[4] this Court held that the word "current" indicates the participation of the body of water in the ebb and flow of waters due to high and low tide. Petitioners' submission not having met the first and second requirements of the rules on alluvion, they cannot claim the rights of a riparian owner. In any case, this court agrees with private respondents that petitioners are estopped from denying the public character of the subject land, as well as the jurisdiction of the Bureau of Lands when the late Antonio Nazareno filed his Miscellaneous Sales Application MSA (G-6) 571.[5] The mere filing of said Application constituted an admission that the land being applied for was public land, having been the subject of Survey Plan No. MSI-10-06000571-D (Equivalent to Lot No. 36302, Cad-237) which was conducted as a consequence of Antonio Nazareno's Miscellaneous Sales Application wherein said land was described as an orchard. Said description by Antonio Nazareno was, however, controverted by respondent Labis in his investigation report to respondent Hilario based on the findings of his ocular inspection that said land actually covers a dry portion of Balacanas Creek and a swampy portion of Cagayan River. The investigation report also states that except for the swampy portion which is fully planted to nipa palms, the whole area is fully occupied by a part of a big concrete bodega of petitioners and several residential houses made of light materials, including those of private respondents which were erected by themselves sometime in the early part of 1978.[6]

Furthermore, the Bureau of Lands classified the subject land as an accretion area which was formed by deposits of sawdust in the Balacanas Creek and the Cagayan river, in accordance with the ocular inspection conducted by the Bureau of Lands.[7] This Court has often enough held that findings of administrative agencies which have acquired expertise because their jurisdiction is confined to specific matters are generally accorded not only respect but even finality.[8] Again, when said factual findings are affirmed by the Court of Appeals, the same are conclusive on the parties and not reviewable by this Court.[9] It is this Court's irresistible conclusion, therefore, that the accretion was man-made or artificial. In Republic v. CA,[10] this Court ruled that the requirement that the deposit should be due to the effect of the current of the river is indispensable. This excludes from Art. 457 of the Civil Code all deposits caused by human intervention. Putting it differently, alluvion must be the exclusive work of nature. Thus, in Tiongco v. Director of Lands, et al.,[11] where the land was not formed solely by the natural effect of the water current of the river bordering said land but is also the consequence of the direct and deliberate intervention of man, it was deemed a man-made accretion and, as such, part of the public domain. In the case at bar, the subject land was the direct result of the dumping of sawdust by the Sun Valley Lumber Co. consequent to its sawmill operations.[12] Even if this Court were to take into consideration petitioners' submission that the accretion site was the result of the late Antonio Nazareno's labor consisting in the dumping of boulders, soil and other filling materials into the Balacanas Creek and Cagayan River bounding his land,[13] the same would still be part of the public domain. Having determined that the subject land is public land, a fortiori, the Bureau of Lands, as well as the Office of the Secretary of Agriculture and Natural Resources have Jurisdiction over the same in accordance with the Public Land Law. Accordingly, the court a quo dismissed petitioners' complaint for non-exhaustion of administrative remedies which ruling the Court of Appeals affirmed. However, this Court agrees with petitioners that administrative remedies have been exhausted. Petitioners could not have intended to appeal to respondent Ignacio as an Officer-in-Charge of the Bureau of Lands. The

decision being appealed from was the decision of respondent Hilario who was the Regional Director of The Bureau of Lands. Said decision was made "for and by authority of the Director of Lands."[14] It would be incongruous to appeal the decision of the Regional Director of the Bureau of Lands acting for the Director of the Bureau of Lands to an Officer-InCharge of the Bureau of Lands. In any case, respondent Rolleo Ignacio's official designation was "Undersecretary of the Department of Agriculture and Natural Resources." He was only an "Officer-In-Charge" of the Bureau of Lands. When he acted on the late Antonio Nazareno's motion for reconsideration by affirming or adopting respondent's Hilario's decision, he was acting on said motion as an Undersecretary on behalf of the Secretary of the Department. In the case of Hamoy v. Secretary of Agriculture and Natural Resources,[15] This Court held that the Undersecretary of Agriculture and Natural Resources may modify, adopt, or set aside the orders or decisions of the Director of Lands with respect to questions involving public lands under the administration and control of the Bureau of Lands and the Department of Agriculture and Natural Resources. He cannot therefore, be said to have acted beyond the bounds of his jurisdiction under Sections 3, 4 and 5 of Commonwealth Act No. 141.[16] As borne out by the administrative findings, the controverted land is public land, being an artificial accretion of sawdust. As such, the Director of Lands has jurisdiction, authority and control over the same, as mandated under Sections 3 and 4 of the Public Land Law (C.A. No. 141) which states, thus: "Sec. 3. The Secretary of Agriculture and Natural Resources shall be the exclusive officer charged with carrying out the provisions of this Act through the Director of Lands who shall act under his immediate control. Sec. 4. Subject to said control, the Director of Lands shall have direct executive control of the survey, classification, lease, sale or any other form of concession or disposition and management of the lands of the public domain, and his decisions as to questions of fact shall be conclusive when approved by the Secretary of Agriculture and Natural Resources."

In connection with the second issue, petitioners ascribe whim, arbitrariness or capriciousness in the execution order of public respondent Abelardo G. Palad, the Director of Lands. This Court finds otherwise since said decision was based on the conclusive finding that the subject land was public land. Thus, this Court agrees with the Court of Appeals that the Director of Lands acted within his rights when he issued the assailed execution order, as mandated by the aforecited provisions. Petitioners' allegation that respondent Palad's execution order directing them to vacate the subject land practically changed respondent Hilario's decision is baseless. It is incorrect for petitioners to assume that respondent Palad awarded portions of the subject land to private respondents Salasalans and Rayabas as they had not yet been issued patents or titles over the subject land. The execution order merely directed the segregation of petitioners' titled lot from the subject land which was actually being occupied by private respondents before they were ejected from it. Based on the finding that private respondents were actually in possession or were actually occupying the subject land instead of petitioners, respondent Palad, being the Director of Lands and in the exercise of this administrative discretion, directed petitioners to vacate the subject land on the ground that private respondents have a preferential right, being the occupants thereof. While private respondents may not have filed their application over the land occupied by them, they nevertheless filed their protest or opposition to petitioners' Miscellaneous Sales Application, the same being preparatory to the filing of an application as they were in fact directed to do so. In any case, respondent Palad's execution order merely implements respondent Hilario's order. It should be noted that petitioners' own application still has to be given due course.[17] As Director of lands, respondent Palad is authorized to exercise executive control over any form of concession, disposition and management of the lands of the public domain.[18] He may issue decisions and orders as he may see fit under the circumstances as long as they are based on the findings of fact. In the case of Calibo v. Ballesteros,[19] this Court held that where, in the disposition of public lands, the Director of Lands bases his decision on the

evidence thus presented, he clearly acts within his jurisdiction, and if he errs in appraising the evidence, the error is one of judgment, but not an act or grave abuse of discretion annullable by certiorari. Thus, except for the issue of non-exhaustion of administrative remedies, this Court finds no reversible error nor grave abuse of discretion in the decision of the Court of Appeals. WHEREFORE, the petition is DISMISSED for lack of merit. SO ORDERED JAGUALING V. CA 194 SCRA 607

FACTS: Eduave owned a parcel of land which later was eroded due to a typhoon and through the movement of land deposit. Eduave granted defendants to plant corn and bananas. She also hired a surveyor to put monuments. She also paid taxes. Here comes petitioner who opposes the claim of ownership claiming the typhoon caused the formation of island, the same they occupied for 15 years now.

HELD: The island formed belongs to the owner of the land with the nearest margin. If the riparian owner fails to assert his claim, it could be open to adverse possession. agualing v. CA

[G.R. No. 94283. March 4, 1991.] First Division, Gancayco (J): 4 concur Facts: A certain parcel of land is located in Sta. Cruz, Tagoloan, Misamis Oriental with an area of 16,452 sq. m., forming part of an island in a nonnavigable river, bounded by the Tagoloan river on the north, south, and east and by the portion belonging to Vicente Neri on the west. Janita Eduave claims that she inherited the land from her father, Felomino Factura, together with his co-heirs, Reneiro Factura and Aldenora Factura, and acquired sole ownership of the property by virtue of a Deed of Extra Judicial Partition with sale. The land is declared for tax purposes under Tax Declaration 26137 with an area of 16,452 sq. m. Since the death of her father on 5 May 1949, Eduave had been in possession of the property although the tax declaration remains in the name of the deceased father. The entire land had an area of 16,452 sq. m. appearing in the deed of extrajudicial partition, while in tax declaration the area is only 4,937 sq. m., and she reasoned out that she included the land that was under water. The land was eroded sometime in November 1964 due to typhoon Ineng, destroying the bigger portion and the improvements leaving only a coconut tree. In 1966 due to the movement of the river deposits on the land that was not eroded increased the area to almost half a hectare and in 1970 Eduave started to plant banana trees. In 1973, Maximo and Anuncita Jagualing asked her permission to plant corn and bananas provided that they prevent squatters to come to the area. Eduave engaged the services of a surveyor who conducted a survey and placed concrete monuments over the land. Eduave also paid taxes on the land in litigation, and mortgaged the land to the Luzon Surety and Co., for a consideration of P6,000.00. The land was the subject of a reconveyance case, in the CFI Misamis Oriental (Branch V, Cagayan de Oro City, Civil Case 5892), between Janita Eduave vs. Heirs of Antonio Factura, which was the subject of judgment by compromise in view of the amicable settlement of the parties, dated 31 May 1979. The heirs of Antonio Factura had ceded a portion of the land with an area of 1,289 sq. m., to Janita Eduave in a notarial document of conveyance, pursuant to the decision of the CFI, after a subdivision of the lot 62 Pls-799, and containing 1,289 sq. m. was designated as Lot 62-A, and the subdivision plan was approved as Pls-799Psd-10-001782. Eduave also applied for concession with the Bureau of Mines to extract 200 m3 of grave, and after an ocular inspection the permit

was granted. Eduave, after permit was granted, entered into an agreement with Tagoloan Aggregates to extract sand and gravel, which agreement was registered in the office of the Register of Deeds. Maximo and Anuncita Jagualing assert that they are the real owners of the land in litigation containing an area of 18,000 sq. m. During the typhoon Ineng in 1964 the river control was washed away causing the formation of an island. Jagualing started occupying the land in 1969, paid land taxes as evidenced by tax declaration 26380 and tax receipts, and tax clearances. Actual occupation of the land by Jagualing included improvements and the house. Rudygondo and Janita Eduave filed with the RTC Misamis Oriental an action to quiet title and/or remove a cloud over the property in question against Jagualing. On 17 July 1987 the trial court dismissed the complaint for failure of Eduave to establish by preponderance of evidence their claim of ownership over the land in litigation. The court found that the island is a delta forming part of the river bed which the government may use to reroute, redirect or control the course of the Tagoloan River. Accordingly, it held that it was outside the commerce of man and part of the public domain, citing Article 420 of the Civil Code. As such it cannot be registered under the land registration law or be acquired by prescription. The trial court, however, recognized the validity of Jagualing's possession and gave them preferential rights to use and enjoy the property. The trial court added that should the State allow the island to be the subject of private ownership, the Jagualings have rights better than that of Eduave. On appeal to the Court of Appeals, the court found that the island was formed by the branching off of the Tagoloan River and subsequent thereto the accumulation of alluvial deposits. Basing its ruling on Articles 463 and 465 of the Civil Code, the Court of Appeals reversed the decision of the trial court, declared Eduave as the lawful and true owners of the land subject of the case and ordered Jagualing to vacate the premises and deliver possession of the land to Eduave. Hence, the present petition. The Supreme Court found no error committed by the appellate court, denied the petition for lack of sufficient merit, and affirmed the decision of the Court of Appeals; without pronouncement as to costs. 1. Evidence not properly appreciated by trial court; CA properly applied Article 463

The appellate court reversed the decision of the trial court because it did not take into account the other pieces of evidence in favor of the private respondents. The complaint was dismissed by the trial court because it did not accept Eduave s explanation regarding the initial discrepancy as to the area they claimed (4937sq.m. v. 16452 sq.m.); because it favored the theory that Eduave became interested in the land only in 1979 not for agricultural purposes but in order to extract gravel and sand, which is belied by other circumstances tantamount to acts of ownership exercised by Eduave over the property prior to said year (e.g. the payment of land taxes thereon, the monuments placed by the surveyor whose services were engaged by Eduave, the agreement entered into by Eduave and Tagoloan Aggregates to extract gravel and sand, which agreement was duly registered with the Register of Deeds); because it disregarded the testimony of 2 disinterested witnesses (Gergorio Neri, as to metes and bounds of the property and the effect of the typhoon; and Candida Ehem, as to the caretaker agreement between her and Eduave) without explaining why it doubted their credibility. From the evidence thus submitted, the appellate court had sufficient basis for the finding that the property of Eduave actually existed and was identified prior to the branching off or division of the river. The Court of Appeals, therefore, properly applied Article 463 of the Civil Code which allows the ownership over a portion of land separated or isolated by river movement to be retained by the owner thereof prior to such separation or isolation. 2. Island formed in a non-navigable and non-floatable river; Article 465 The parcel of land is part of an island that formed in a non-navigable and non-flotable river; from a small mass of eroded or segregated outcrop of land, it increased to its present size due to the gradual and successive accumulation of alluvial deposits. The Court of Appeals did not err in applying Article 465 of the Civil Code. Under this provision, the island belongs to the owner of the land along the nearer margin as sole owner thereof; or more accurately, because the island is longer than the property of Eduave, they are deemed ipso jure to be the owners of that portion which corresponds to the length of their property along the margin of the river. 3. Land formed by accretion belongs to riparian owner, even without a specific act of possession over it; Land however may yield to adverse

possession of third party if riparian owner fails to assert claim Lands formed by accretion belong to the riparian owner. This preferential right is, under Article 465, also granted the owners of the land located in the margin nearest the formed island for the reason that they are in the best position to cultivate and attend to the exploitation of the same. In fact, no specific act of possession over the accretion is required. If, however, the riparian owner fails to assert his claim thereof, the same may yield to the adverse possession of third parties, as indeed even accretion to land titled under the torrens system must itself still be registered. 4. Doctrine of acquisitive prescription The property may be acquired by adverse possession for the required number of years under the doctrine of acquisitive prescription. Jagualing s possession cannot be considered in good faith, however, because they are presumed to have notice of the status of Eduave as riparian owners who have the preferential right to the island as recognized and accorded by law; they may claim ignorance of the law, specifically Article 465 of the Civil Code, but such is not, under Articles 3 and 526 of the same code, an adequate and valid defense to support their claim of good faith. Hence, not qualifying as possessors in good faith, they may acquire ownership over the island only through uninterrupted adverse possession for a period of thirty years. By their own admission, Jagualing have been in possession of the property for only about 15 years, and thus, the island cannot be adjudicated in their favor. 5. Origin of island not tackled as case is not between opposing riparian owners but between a riparian owner and one in possession of the land There is no need to make a final determination regarding the origins of the island, i.e., whether the island was initially formed by the branching off or division of the river and covered by Article 463 of the Civil Code, in which case there is strictly no accession because the original owner retains ownership, or whether it was due to the action of the river under Article 465, or whether it was caused by the abrupt segregation and washing away of the stockpile of the river control, which makes it a case of avulsion under Article 459, as the case is not between parties as opposing riparian owners contesting ownership over an accession but rather between a riparian owner and the one in possession of the island.

6. Quasi in Rem; Judgment conclusive upon the parties and does not bind the State and other riparian owners The Court is not prepared to concede that the island is a delta which should be outside the commerce of man and that it belongs to the State as property of the public domain in the absence of any showing that the legal requirements to establish such a status have been satisfied, which duty properly pertains to the State. Since the petition is an upshot of the action to quiet title brought by Eduave against Jagualing, it is thus not technically an action in rem or an action in personam, but characterized as quasi in rem, which is an action in personam concerning real property. Thus, the judgment in proceedings of this nature is conclusive only between the parties and does not bind the State or the other riparian owners who may have an interest over the island involved herein.

G.R. No. 147340

December 13, 2007

CYNTHIA CRUZ KHEMANI and SHANKER N. KHEMANI, petitioners, vs. THE HEIRS OF ANASTACIO TRINIDAD, represented by NAPOLEON and ROLANDO TRINIDAD, respondents. DECISION YNARES-SANTIAGO, J.: This petition for review on certiorari1 assails the July 31, 2000 Decision2 of the Court of Appeals in CA-G.R. SP No. 55581, which affirmed the May 24, 1999 Order3 of the Regional Trial Court, Branch 24, Koronadal, South Cotabato in Civil Case No. 1122, entitled "Heirs of Anastacio and Francisca Trinidad, et al. v. Heirs of Jose Pea, et al." Also assailed is the January 8, 2001 Resolution4 denying the motion for reconsideration. The factual antecedents are as follows: Petitioner Cynthia Cruz Khemani is the registered owner of Lot No. 107, Ts1032 (Lot No. 107), which is covered by Transfer Certificate of Title (TCT) No. 58976 issued on March 10, 1994.5 Khemani purchased the lot from the

heirs of Jose B. Pea (the Pea Heirs) on February 17, 1994. Shanker N. Khemani is her brother-in-law and duly authorized representative. Subject of the instant case is a 340 square meter portion (the Disputed Property) of Lot No. 107 over which respondents Heirs of Anastacio Trinidad, represented by Napoleon and Rolando Trinidad, are claiming ownership. Respondents allege that they and their predecessors-ininterest, Spouses Anastacio and Francisca Trinidad, have openly, peacefully, publicly and adversely possessed the Disputed Property in the concept of owner since 1950. Lot No. 107 and Lot Nos. 108 and 109, constitute Lot No. 355 which was part of the public domain. On July 10, 1950, Lot No. 355 with an original area of 1,500 square meters was awarded to Jesus M. Larrabaster by the National Land Settlement Administration (NLSA) who subsequently sold his rights and interests over the said property to Jose B. Pea (Pea) on June 29, 1956. Thereafter, the original area of Lot No. 355 which was 1,500 square meters increased to 3,616.93 square meters due to accretion. Pea then requested the Bureau of Lands (BOL) to adjust the area of the lot awarded to him but the BOL denied the request on the ground that the accretion belonged to the government. Aggrieved, Pea appealed to the Office of the President. The BOL recommended that Lot No. 355 be subdivided into three parts, to wit, Lot Nos. 107, 108 and 109, and that Lot No. 108 with an area of 1,500 square meters, be awarded to Pea, instead of the whole of Lot No. 355. Meanwhile, Lot Nos. 107 and 109 would be allocated to Basilio Mendoza (Mendoza) and Arturo Roxas, respectively. The Office of the President initially adopted the recommendation of the BOL. Upon reconsideration, however, it modified its decision and held that the entire area of Lot No. 355, including the accretion, belonged to Pea and not to the government. Thus, Lot Nos. 107, 108, and 109 were awarded to him. On January 27, 1970, Mendoza filed a special civil action for certiorari against the Assistant Executive Secretary for Legal Affairs of the Office of

the President, the BOL, the Director of Lands, and Pea before Branch 24 of the Court of First Instance of South Cotabato, which was docketed as Civil Case No. 98. Claiming that he was denied due process, Mendoza assailed the decision of the Office of the President awarding the entire area of Lot No. 355 to Pea. He asserted ownership over Lot No. 107 on the strength of a Miscellaneous Sales Application he allegedly filed with the BOL on November 6, 1962. On May 10, 1985, the trial court rendered a decision dismissing Mendoza s petition for certiorari but the same was reversed by the Court of Appeals on appeal. Hence, Mendoza filed a petition for review on certiorari before the Supreme Court. In the case of Assistant Executive Secretary for Legal Affairs of the Office of the President v. Court of Appeals6 which was decided on January 9, 1989, the Supreme Court rejected Mendoza s claim over Lot No. 107 and found the Miscellaneous Sales Application without legal force and effect since the object thereof was no longer public land. Thus, Pea s right of ownership over the entire area of Lot No. 355, which consists of Lot Nos. 107, 108 and 109, was affirmed. On September 20, 1993, the Pea Heirs were awarded a patent by the Department of Environment and Natural Resources (DENR), and on September 21, 1993, Original Certificate of Title No. P-336587 covering Lot No. 107 was issued in their name. On January 27, 1994, respondents filed with the Regional Trial Court, Branch 24, Koronadal, South Cotabato a verified complaint8 against the Pea Heirs,9 the DENR Region IX Office, and the BOL for "Review of Decree of Registration and/or Reconveyance with Prayer for Issuance of Writ of Preliminary Prohibitory Injunction and Temporary Restraining Order," which was docketed as Civil Case No. 1122. Respondents filed the complaint on the strength of their own and their predecessors open, peaceful, public and adverse possession of the Disputed Property in the concept of owner since 1950. Respondents also claimed that on July 16, 1976, their predecessor-ininterest, Anastacio, applied for a Miscellaneous Sales Application over the Disputed Property which was designated as a portion of Lot No. 107, Ts-

1032.10 On March 2, 1979, the BOL allegedly issued Certification No. 3445 certifying that the Disputed Property was awarded to Anastacio and that the transfer had been duly investigated and approved per Board Resolution No. 133, Series of 1979. Instead of an answer, the Pea Heirs filed a Motion to Dismiss11 alleging that the Regional Trial Court lacks jurisdiction over the nature of the action or the suit; that respondents have no legal capacity to sue as only the government may seek nullification of the land grant in their favor; and that the cause of action is barred by prior judgment or the statute of limitations. They asserted that the issue of ownership over the Disputed Property has long been settled in the Assistant Executive Secretary case. Further, they argued that respondents predecessor-in-interest, Anastacio, was a mere squatter who had been allowed by Mendoza to occupy a portion of Lot No. 107 sometime in 1960. In respondents Comment/Opposition,12 they claimed that the Disputed Property had long ceased to be public land by virtue of their open, public, continuous, adverse and exclusive possession in the concept of owner for more than 40 years, and that they were never parties in the Assistant Executive Secretary case involving Mendoza. On September 3, 1997, Judge Rodolfo C. Soledad (Judge Soledad) granted petitioner s motion to dismiss and held that respondents are bound by the ruling of this Court in the Assistant Executive Secretary case.13 Respondents filed a motion for reconsideration14 alleging that res judicata does not apply and that their action is not barred by the Assistant Executive Secretary case. They argued that neither they, nor Anastacio, were parties in the said case and that there is no identity of causes of action. In 1998, Judge Soledad died without resolving the motion for reconsideration filed by respondents. Judge Francisco S. Ampig (Judge Ampig) was designated Acting Judge. On May 24, 1999, Judge Ampig granted the motion for reconsideration, reinstated Civil Case No. 1122, and directed the Pea Heirs to file an answer.

The Pea Heirs, together with herein petitioner as the new owner of Lot No. 107, filed a petition for certiorari15 before the Court of Appeals which was docketed as CA-G.R. SP No. 55581. On July 31, 2000, the Court of Appeals rendered the assailed decision dismissing the petition. It ruled that a petition for certiorari is not the proper remedy against an order denying a motion to dismiss. Further, it held that there is no res judicata. Thus: Moreover, petitioners have plain, speedy and adequate remedy in the ordinary course of law. The remedy against an adverse interlocutory order, such as the assailed orders, is not certiorari but to continue with the case in due course and, when an unfavorable verdict is handed down, to take an appeal in the manner authorized by law. x x x With the denial of the motion to dismiss and reinstatement of the case, petitioners will still answer the complaint. Upon joinder of issues, the parties will enter into trial, after which, the lower court will render a verdict. And if adverse to them, petitioners may appeal the decision together with the assailed orders. The case at bench does not fall under any of the exceptional circumstances where the extraordinary writ of certiorari may be resorted to despite availability of appeal. xxxx Private respondents are not parties in the first action. Neither are they the successors-in-interest of any of the parties therein. The first action is in personam. The final judgment in said action is only binding and conclusive upon the parties therein and their successorsin-interest. xxxx Mendoza, the petitioner in the first action, laid claim in Lot 107 on the basis of his possession thereof and Miscellaneous Sales Application. On the other hand, private respondents interest in the contested property is anchored on their own possession and

Miscellaneous Sales Application. In other words, private respondents are not asserting rights under Mendoza. Consequently, they have no community of interests in the contested property; in fact, their interests are antagonistic to each other. On the other hand, "the test often used in determining whether causes of action are identical is to ascertain whether the same evidence which is necessary to sustain the second action would have been sufficient to authorize recovery in the first, even if the forms or nature of the two actions be different" (Carlet vs. Court of Appeals, 275 SCRA 97). Considering that the foundation of private respondents action is different from that of Mendoza, the evidence necessary to sustain the latter s claim in the first action would be separate and distinct from that required to establish private respondents cause of action. Since not all requisites of res judicata are present, respondent judge acted rightly in issuing the assailed orders. In short, he committed no abuse of discretion. WHEREFORE, the petition is DISMISSED for lack of merit. SO ORDERED.16 The motion for reconsideration of the foregoing decision was denied hence, this petition. Petitioner claims that the case of Assistant Executive Secretary bars the filing of Civil Case No. 1122, and that a petition for certiorari under Rule 65 of the Rules of Court is the proper remedy in assailing the order of the Regional Trial Court denying the motion to dismiss. Respondents argue that they have been in open, peaceful, public and adverse possession of the Disputed Property in the concept of owner since 1950; that the patent and original certificate of title were fraudulently issued in favor of the Pea Heirs; and that their action for review of decree of registration and/or reconveyance is not barred by the Court s ruling in Assistant Executive Secretary.

The issues for resolution are as follows: 1) whether a petition for certiorari under Rule 65 is the proper remedy in assailing an order denying a motion to dismiss; and 2) whether Judge Ampig committed grave abuse of discretion in denying petitioner s motion to dismiss and reinstating Civil Case No. 1122. The petition lacks merit. It has long been settled that an order denying a motion to dismiss is an interlocutory order. It neither terminates nor finally disposes of a case, as it leaves something to be done by the court before the case is finally decided on the merits. As such, the general rule is that the denial of a motion to dismiss cannot be questioned in a special civil action for certiorari.17 However, there are exceptions to the general rule. In Velarde v. Lopez, Jr.,18 the Court held that resort to a special civil action for certiorari is allowed when the ground for the motion to dismiss is improper venue, lack of jurisdiction, or res judicata as in the case at bar.19 Thus, petitioner did not commit a procedural error in filing a petition for certiorari before the Court of Appeals. Nevertheless, as to the substantive issue raised herein, the petition must fail. We find that Judge Ampig did not commit grave abuse of discretion in denying petitioner s motion to dismiss and reinstating Civil Case No. 1122. In Oropeza Marketing Corp. v. Allied Banking Corp.,20 we held that res judicata literally means "a matter adjudged; a thing judicially acted upon or decided; a thing or matter settled by judgment." It lays the rule that an existing final judgment or decree rendered on the merits, and without fraud or collusion, by a court of competent jurisdiction, upon any matter within its jurisdiction, is conclusive of the rights of the parties or their privies, in all other actions or suits in the same or any other judicial tribunal of concurrent jurisdiction on the points and matters in issue in the first suit.21 A case is barred by prior judgment or res judicata when the following requisites concur: (1) the former judgment is final; (2) it is rendered by a court having jurisdiction over the subject matter and the parties; (3) it is a judgment or an order on the merits; and (4) there is between the first

and the second actions action.22

identity of parties, subject matter, and causes of

In this case, it is not disputed that the first three elements are present. Likewise, there is no controversy regarding the identity of the subject matter. The question, therefore, is whether there is identity of parties and causes of action. We find that there is none. Civil Case No. 98 was a special civil action for certiorari filed by Mendoza against the Assistant Executive Secretary for Legal Affairs of the Office of the President, the BOL, the Director of Lands, and Pea. On the other hand, Civil Case No. 1122 is an action for review of decree of registration and/or reconveyance. The parties are respondents Trinidad, the Pea Heirs, the DENR Region IX Office, and the BOL. Mendoza s action in Civil Case No. 98 was based on alleged grave abuse of discretion of the Office of the President in awarding the entire area of Lot No. 355 to Pea. He claimed ownership over Lot No. 7 and in support thereof, presented the Miscellaneous Sales Application he filed with the BOL on November 6, 1962. Meanwhile, respondents action in Civil Case No. 1122 was based on their continued possession of the Disputed Property in the concept of owner for over 40 years, and the alleged fraudulent issuance of a patent and certificate of title to the Pea Heirs. True, res judicata does not require absolute but only substantial identity of parties. However, there is substantial identity only when the "additional" party acts in the same capacity or is in privity with the parties in the former action.23 This is not so in the present case. It must be emphasized that respondents are not asserting rights under Mendoza. Indeed, the records will show that the parties in the two cases have their own rights and interests in relation to the subject matter in litigation. Moreover, as correctly found by the Court of Appeals, the basis of respondents action was different from that of Mendoza; the evidence necessary to sustain the latter s claim is separate and distinct from that required to establish respondents cause of action.24 While Mendoza relied on the Miscellaneous Sales Application as evidence to support his claim, herein respondents would have to present proof of their alleged continuous possession of the Disputed Property as well as fraud in the issuance of the

patent and title in favor of the Pea Heirs. In Morato v. Court of Appeals,25 we held that the test of identity of causes of action lies not in the form of action but in whether the same facts or evidence would support and establish the former and present causes of action.26 Thus, res judicata does not apply in the instant case there being no identity of parties and causes of action. Nevertheless, the public policy underlying the principle of res judicata must be considered together with the policy that a party shall not be deprived of a fair adversary proceeding wherein to present his case.27 It bears stressing that respondents action for review of decree of registration is sanctioned under Section 32 of Presidential Decree No. 1529,28 which provides that a person deprived of his land through actual fraud may institute an action to reopen or review a decree of registration within one year from entry of such decree. It states: Section 32. Review of decree of registration; Innocent purchaser for value. The decree of registration shall not be reopened or revised by reason of absence, minority, or other disability of any person adversely affected thereby, nor by any proceeding in any court for reversing judgments, subject, however, to the right of any person, including the government and the branches thereof, deprived of land or of any estate or interest therein by such adjudication or confirmation of title obtained by actual fraud, to file in the proper Court of First Instance a petition for reopening and review of the decree of registration not later than one year from and after the date of the entry of such decree of registration, but in no case shall such petition be entertained by the court where an innocent purchaser for value has acquired the land or an interest therein, whose rights may be prejudiced. Whenever the phrase "innocent purchaser for value" or an equivalent phrase occurs in this Decree, it shall be deemed to include an innocent lessee, mortgagee, or other encumbrancer for value. The Court has repeatedly applied the foregoing provision of law to a patent issued by the Director of Lands, approved by the Secretary of Natural Resources, under the signature of the President of the Philippines. The date of the issuance of the patent corresponds to the date of the issuance of the decree in ordinary cases.29

In this case, the patent was issued in favor of the Pea Heirs on September 20, 1993. Respondents filed Civil Case No. 1122 for "Review of Decree of Registration and/or Reconveyance with Prayer for Issuance of Writ of Preliminary Prohibitory Injunction and Temporary Restraining Order" on January 27, 1994, or well within the prescribed one-year period. Likewise, records show that TCT No. 58976 under petitioner s name bears a Notice of Lis Pendens.30 Thus, it cannot be said that petitioner is an innocent purchaser for value as she was well aware of respondents claim over the Disputed Property. Further, even assuming arguendo that respondents filed their action after one year, they may still be entitled to relief. An aggrieved party may file an action for reconveyance based on implied or constructive trust, which prescribes in ten years from the date of the issuance of the certificate of title over the property provided that the property has not been acquired by an innocent purchaser for value.31 Respondents clearly asserted in their complaint that they and their predecessors-in-interest have long been the owners of the Disputed Property and that they were fraudulently deprived of ownership thereof when the Pea Heirs obtained a patent and certificate of title in their favor. These allegations certainly measure up to the requisite statement of facts to constitute an action for reconveyance.32 A final note. It appears from the records that after our ruling in the Assistant Executive Secretary case in 1989, the BOL issued a Patent on September 20, 1993 in favor of the Pea Heirs which became the basis for the issuance of OCT No. P-33658 covering Lot No. 107. However, as held in the Assistant Executive Secretary case, Lot No. 107 as accretions to the original lot (Lot No. 355) awarded to Larrabaster on July 10, 1950 "no longer belonged to the Government[,] the subdivision thereof by the Bureau of Lands into three lots (Lot No. 107, Lot No. 108 and Lot No. 109), as well as the allocation of said lots to two other individuals, was beyond the scope of its authority."33 As a result, while Lot No. 107 may no longer be acquired under the provisions of the Public Land Act, it does not absolutely foreclose the possibility that, as a private property, a portion thereof (the Disputed Property) may have been acquired by respondents through acquisitive prescription under the Civil Code. These matters,

however, are the proper subject of a separate action should one be filed subject, of course, to such claims and defenses that either party may have under relevant laws. All told, it would be premature to order the dismissal of respondents complaint as they have yet to be given an opportunity to substantiate their claims. We note that respondents are in actual physical possession of the Disputed Property up to this date, and the fact of their physical possession over many years is not disputed by petitioner. 34 Under the circumstances, it would be more in keeping with the standards of fairness to have a fullblown trial where the evidentiary matters are threshed out. WHEREFORE, the petition is DENIED. The July 31, 2000 Decision, and the January 8, 2001 Resolution of the Court of Appeals in CA-G.R. SP No. 55581 are AFFIRMED. The trial court is ORDERED to resume trial in Civil Case No. 1122 and to resolve the same with dispatch. SO ORDERED. Agustin v. IAC [G.R. Nos. 66075-76. July 5, 1990.] First Division, Grino-Aquino (J): 4 concur Facts: The Cagayan River separates the towns of Solana on the west and Tuguegarao on the east in the province of Cagayan. In 1919 the lands east of the river were covered by the Tuguegarao Cadastre. In 1925, OCT 5472 was issued for land east of the Cagayan River owned by Eulogio Agustin. As the years went by, the Cagayan River moved gradually eastward, depositing silt on the western bank. The shifting of the river and the siltation continued until 1968. In 1950, all lands west of the river were included in the Solana Cadastre. Among these occupying lands covered by the Solana Cadastre were Pablo Binayug and Maria Melad. Binayug was in possession since 1947 of Lots 3349, 7875 to 7879, 7881 to 7885, 7891 and 7892. It is has an area of 8 hectares planted to tobacco and corn and another 12 hectares overgrown with talahib. Binayug's Homestead Application W-79055 over this land was approved in 1959 and his possession recognized in the decision in Civil Case 101. On the other hand, as a result of Civil Case 343-T, Macario Melad, the predecessor-in-interest of Maria Melad and Timoteo Melad, was issued OCT P-5026 for Lot 3351 of

Cad. 293 on 1 June 1956. Through the years, the Cagayan River eroded lands of the Tuguegarao Cadastre on its eastern bank among which was Agustin's Lot 8457, depositing the alluvium as accretion on the land possessed by Binayug on the western bank. However, in 1968, after a big flood, the Cagayan River changed its course, returned to its 1919 bed, and, in the process, cut across the lands of Maria Melad, Timoteo Melad, and the spouses Pablo Binayug and Geronima Ubina whose lands were transferred on the eastern, or Tuguegarao, side of the river. To cultivate those lots they had to cross the river. In April 1969, while the Melads, Binayug, Urbina and their tenants were planting corn on their lots located on the eastern side of the Cagayan River, Agustin, the Heirs of Baldomero Langcay, Juan Langcay, and Arturo Balisi, accompanied by the mayor and some policemen of Tuguegarao, claimed the same lands as their own and drove away the Melads, Binayug and Urbina from the premises. On 21 April 1970, Maria and Timoteo Melad filed a complaint (Civil Case 343-T) to recover Lot 3351 with an area of 5 hectares and its 6.6-hectare accretion. On 24 April 1970, Pablo Binayug filed a separate complaint (Civil Case 344-T) to recover his lots and their accretions. On 16 June 1975, the trial court rendered a decision in Civil Case 343-T, ordering Eulogio Agustin, Gregorio Tuliao, Jacinto Buquel and Octavio Bancud, their representatives or agents to vacate Lot 3351 of Solana Cadastre together with its accretion consisting of portions of Lots 9463, 9462 and 9461 of Tuguegarao Cadastre and to restore ownership in favor of Maria Melad and Timoteo Melad who are the only interested heirs of Macario Melad. The trial court likewise ordered, in Civil Case 344-T, Justo Adduru, Andres Pastor, Teofilo Tagacay, Vicente Camilan, Nicanor Mora, Baldomero Cagurangan, Domingo Quilang, Cesar Cabalza, Elias Macababbad, Titong Macababbad, Arturo Balisi, Jose Allabun, Eulogio Agustin, Banong Aquino, Junior Cambri and Juan Langoay, their representatives or agents to vacate Lots 3349, 7875 to 7879, 7881 to 7885, 7891 and 7892, together with its accretion and to restore possession to Pablo Binayug and Geronimo Urbina. Without pronouncement as to damages which were not properly proven and to costs. Eulogio Agustin appealed the decision in Civil Case 343-T, while Eulogio Agustin, Baldomero Cagurangan (substituted by his heir), Arturo Balisi and Juan Langcay appealed the decision in Civil Case 344-T. But upon motion

of the Melads, Binayug and Urbina, the trial court ordered on 15 August 1975 the execution pending appeal of the judgment in Civil Case 344-T against Cagurangan, Balisi and Langcay on the ground that their appeal was dilatory as they had not presented evidence at the trial. On 29 November 1983, the Intermediate Appellate Court rendered a decision affirming in toto the judgment of the trial court, with costs against the Agustin, Cagurangan, Balisi and Langcay. Hence, the petition for review. The Supreme Court denied the petition for lack of merit, and affirmed the decision of the IAC, now CA; with costs against Agustin, et.al. 1. Findings of fact of the Court of Appeal conclusive with the Supreme Court The finding of the Court of Appeals that there had been accretions to the lots of the Melads, Binauyg and Urbina who did not lose the ownership of such accretions even after they were separated from the principal lots by the sudden change of course of the river, is a finding of fact which is conclusive on this Court. That finding is supported by Art. 457 of the New Civil Code which provides that "to the owners of lands adjoining the banks of rivers belong the accretion which they gradually receive from the effects of the current of the waters. (366)" 2. Conditions for accretion to benefit a riparian owner Accretion benefits a riparian owner when the following requisites are present: (1) that the deposit be gradual and imperceptible; (2) that it resulted from the effects of the current of the water; and (3) that the land where accretion takes place is adjacent to the bank of a river (Republic vs. CA, 132 SCRA 514). In the present case, the accretion on the western bank of the Cagayan River had been going on from 1919 up to 1968 or for a period of 49 years. It was gradual and imperceptible. Only when Lot 3351, with an original area of 5 hectares described in the free patent that was issued to Macario Melad in June 1956, was resurveyed in 1968 did it become known that 6.6 hectares had been added to it. Lot 3351, covered by a homestead patent issued in June 1950 to Pablo Binayug, grew from its original area of 18 hectares, by an additional 50 hectares through alluvium as the Cagayan River gradually moved to the east. These accretions belong to riparian owners upon whose lands the alluvial deposits

were made (Roxas vs. Tuason, 9 Phil. 408; Director of Lands vs. Rizal, 87 Phil. 806). 3. Reason for the principle of accretion benefiting a riparian owner The reason for the principle is because, if lands bordering on streams are exposed to floods and other damage due to the destructive force of the waters, and if by virtue of law they are subject to encumbrances and various kinds of easements, it is only just that such risks or dangers as may prejudice the owners thereof should in some way be compensated by the right of accretion (Cortes vs. City of Manila, 10 Phil. 567). 4. Ownership of accretion not lost upon sudden and abrupt change of the river The' ownership of the accretion to the lands was not lost upon the sudden and abrupt change of the course of the river (Cagayan River in 1968 or 1969 when it reverted to its old 1919 bed), and separated or transferred said accretions to the other side (or eastern bank) of the river. Articles 459 and 463 of the New Civil Code apply to this situation. Article 459 provides that whenever the current of a river, creek or torrent segregates from an estate on its bank a known portion of land and transfers it to another estate, the owner of the land to which the segregated portion belonged retains the ownership of it, provided that he removes the same within two years." Article 463 provides that whenever the current of a river divides itself into branches, leaving a piece of land or part thereof isolated, the owner of the land retains his ownership. He also retains it if a portion of land is separated from the estate by the current. Haystack: Ronquillo v. Court of Appeals (GR 43346, 20 March 1991) Ronquillo v. CA [G.R. No. 43346. March 20, 1991.] Second Division, Regalado (J): 4 concur Facts: Rosendo del Rosario was a registered owner of a parcel of land known as Lot 34, Block 9, Sulucan Subdivision, situated at Sampaloc, Manila and covered by TCT 34797 of the Registry of Deeds of Manila. Florencia and Amparo del Rosario were daughters of said Rosendo del Rosario. Adjoining said lot is a dried-up portion of the old Estero Calubcub occupied by Mario C. Ronquillo since 1945. Both del Rosario and Ronquillo

have filed with the Bureau of Lands miscellaneous sales application for the purchase of the abandoned river bed known as Estero Calubcub and their sales applications, dated 5 August 1958 and 13 October 1959, respectively, are still pending action before the Bureau of Lands. Del Rosario claims that long before 1930, when TCT 34797 over Lot 34 was issued in the name of Rosendo del Rosario, the latter had been in possession of said lot including the adjoining dried-up portion of the old Estero Calubcub, having bought the same from Arsenio Arzaga. Sometime in 1935, said titled lot was occupied by Isabel Roldan with the tolerance and consent of del Rosario on condition that the former will make improvements on the adjoining driedup portion of the Estero Calubcub. In the early part of 1945 defendant occupied the eastern portion of said titled lot as well as the dried-up portion of the old Estero Calubcub which abuts del Rosario's titled lot. After a relocation survey of the land in question sometime in 1960, del Rosario learned that Ronquillo was occupying a portion of their land and thus demanded Ronquillo to vacate said land when the latter refused to pay the reasonable rent for its occupancy. However, despite said demand Ronquillo refused to vacate. On the other hand, Ronquillo claims that sometime before 1945 he was living with his sister who was then residing or renting Del Rosario's titled lot. In 1945 he built his house on the disputed dried-up portion of the Estero Calubcub with a small portion thereof on the titled lot of del Rosario. Later in 1961, said house was destroyed by a fire which prompted him to rebuild the same but, this time it was built only on the dried-up portion of the old Estero Calubcub without touching any part of del Rosario s titled land. He further claims that said dried-up portion is a land of public domain. Rosendo, Amparo and Florencia del Rosario lodged a complaint with the CFI Manila praying, among others, that they be declared the rightful owners of the dried-up portion of Estero Calubcub. Ronquillo filed a motion to dismiss the complaint on the ground that the trial court had no jurisdiction over the case since the dried-up portion of Estero Calubcub is public land and, thus, subject to the disposition of the Director of Lands. The Del Rosarios opposed the motion arguing that since they are claiming title to the dried-up portion of Estero Calubcub as riparian owners, the trial court has jurisdiction. The resolution of the motion to dismiss was deferred until after trial on the merits. On 26 December 1962, the trial court rendered judgment ordering Ronquillo to deliver to del Rosario the portion

of the land covered by TCT 34797 which is occupied by him and to pay for the use and occupation of said portion of land at the rate of P5 a month from the date of the filing of the complaint until such time as he surrenders the same to del Rosario and declaring Del Rosario to be the owners of the dried-up portion of estero Calubcub which is abutting del Rosario' property; with costs against Ronquillo. On appeal (CA-GR 32479-R), the Court of Appeals affirmed the decision of the trial court on 25 September 1975 and declared that since Estero Calubcub had already dried-up way back in 1930 due to the natural change in the course of the waters, under Article 370 of the old Civil Code which it considers applicable to the present case, the abandoned river bed belongs to the Del Rosarios as riparian owners. Consequently, respondent court opines, the dried-up river bed is private land and does not form part of the land of the public domain. It stated further that even assuming for the sake of argument that said estero did not change its course but merely dried up or disappeared, said dried-up estero would still belong to the riparian owner, citing its ruling in the case of Pinzon vs. Rama. Upon motion of Ronquillo, respondent court modified its decision on 28 January 1976 by setting aside the first portion of the trial court's decision ordering Ronquillo to surrender to the Del Rosarios that portion of land covered by TCT 34797 occupied by the former, based on the former's representation that he had already vacated the same prior to the commencement of this case. However, the appellate court upheld its declaration that the Del Rosarios are the rightful owners of the dried-up river bed. Hence, the petition for review. On 17 May 1976, the Supreme Court issued a resolution requiring the Solicitor General to comment on the petition in behalf of the Director of Lands as an indispensable party in representation of the Republic of the Philippines, and who, not having been impleaded, was subsequently considered impleaded as such in the Court s resolution of 10 September 1976. In his Motion to Admit Comment, the Solicitor General manifested that pursuant to a request made by this office with the Bureau of Lands to conduct an investigation, the Chief of the Legal Division of the Bureau sent a communication informing him that the records of his office do not show

that Mario Ronquillo, Rosendo del Rosario, Amparo del Rosario or Florencia del Rosario has filed any public land application covering parcels of land situated at Estero Calubcub, Manila as verified by its Records Division. The position taken by the Director of Lands (in his comment on 3 September 1978, in the 4 May 1989 reply, and 17 August 1989 comment) explicates that Article 370 of the old Code does not apply as the abandoned riverbed as such was abandoned not by the natural change in the course of the river but by the drying up of the bed caused by human activity. The Director of Lands also added that the del Rosario and Ronquillo have claimed pending sales applications over the dried portion of the estero (admitting thus that it is public land under the authority of the Bureau of Lands), which were rejected as the Manila City Engineer s Office needed the dried portion of the estero for drainage purposes. On 29 June 1979, Florencia del Rosario manifested to this Court that Rosendo, Amparo and Casiano del Rosario have all died, and that she is the only one still alive among the private respondents in the case. In a resolution dated 20 January 1988, the Court required Ronquillo to implead one Benjamin Diaz pursuant to the former's manifestation that the land adjacent to the driedup river bed has already been sold to the latter, and the Solicitor General was also required to inquire into the status of the investigation being conducted by the Bureau of Lands. In compliance therewith, the Solicitor General presented a letter from the Director of Lands to the effect that neither of the parties involved in the present case has filed any public land application. On 3 April 1989, Ronquillo filed an Amended Petition for Certiorari, this time impleading the Development Bank of the Philippines (DBP) which subsequently bought the property adjacent to the dried-up river bed from Benjamin Diaz. In its resolution dated 10 January 1990, the Court ordered that DBP be impleaded as a party respondent. On 13 September 1990, DBP filed a Manifestation/ Compliance stating that DBP's interest over TCT 139215 issued in its name (formerly TCT 34797 of the Del Rosarios and TCT 135170 of Benjamin Diaz) has been transferred to Spouses Victoriano and Pacita A. Tolentino pursuant to a Deed of Sale dated 11 September 1990. The Supreme Court reversed and set aside the remaining effective portion of the appealed decision which declares Del Rosario as riparian owner of the dried-up portion of Estero Calubcub.

1. Findings of appellate court conclusive to the Supreme Court; Exceptions The jurisdiction of the Supreme Court in cases brought to it from the Court of Appeals in a petition for certiorari under Rule 45 of the Rules of Court is limited to the review of errors of law, and that said appellate court's finding of fact is conclusive upon this Court. However, there are certain exceptions, such as (1) when the conclusion is a finding grounded entirely on speculation, surmises or conjectures; (2) when the inference made is manifestly absurd, mistaken or impossible; (3) when there is grave abuse of discretion in the appreciation of facts; (4) when the judgment is premised on a misapprehension of facts; (5) when the findings of fact are conflicting; and (6) when the Court of Appeals in making its findings went beyond the issues of the case and the same is contrary to the admissions of both appellant and appellee. 2. Evidence reveal change in the course of river not caused by natural forces A careful perusal of the evidence presented by both parties in the case at bar will reveal that the change in the course of Estero Calubcub was caused, not by natural forces, but due to the dumping of garbage therein by the people of the surrounding neighborhood. There is nothing in the testimony of lone witness Florencia del Rosario nor in said relocation plan which would indicate that the change in the course of the estero was due to the ebb and flow of the waters. On the contrary, the testimony of the witness belies such fact, while the relocation plan is absolutely silent on the matter. The inescapable conclusion is that the dried-up portion of Estero Calubcub was occasioned, not by a natural change in the course of the waters, but through the active intervention of man. 3. Article 370 of the old Civil Code applies only to natural change in the course of the waters; Law clear, no room for interpretation Article 370 of the old Civil Code which provides that "the beds of rivers, which are abandoned because of a natural change in the course of the waters, belong to the owners of the riparian lands throughout the respective length of each. If the abandoned bed divided tenements belonging to different owners the new dividing line shall be equidistant from one and the other." The law is clear and unambiguous; and leaves no room for interpretation. Article 370 applies only if there is a natural change in the course of the waters. The rules on alluvion do not apply to man-

made or artificial accretions 23 nor to accretions to lands that adjoin canals or esteros or artificial drainage systems. Considering the finding that the dried-up portion of Estero Calubcub was actually caused by the active intervention of man, it follows that Article 370 does not apply to the present case and, hence, the Del Rosarios cannot be entitled thereto supposedly as riparian owners. 4. Dried up portion of Estero Calubcub belongs to public domain; Land used for drainage purposes cannot be subject of a miscellaneous sales application The dried-up portion of Estero Calubcub should thus be considered as forming part of the land of the public domain which cannot be subject to acquisition by private ownership. This is made more evident in the letter, dated 28 April 1989, of the Chief of the Legal Division of the Bureau of Lands, stating that the alleged application filed by Ronquillo no longer exists in its records as it must have already been disposed of as a rejected application for the reason that other applications covering Estero Calubcub, Sampaloc, Manila for areas other than that contested in the presented case, were all rejected by the office because of the objection interposed by the City Engineer's office that they need the same land for drainage purposes. Since the land is to be used for drainage purposes the same cannot be the subject of a miscellaneous sales application. 5. Del Rosario and Ronquillo estopped from claiming land is not public land The fact that Ronquillo and del Rosario filed their sales applications with the Bureau of Lands covering the subject dried-up portion of Estero Calubcub cannot but be deemed as outright admissions by them that the same is public land. They are now estopped from claiming otherwise. Hilario v. City of Manila [GR No. L-19570 April 27, 1967] Bengzon JP (J): 8 concur Facts: Dr. Jose Hilario was the registered owner of a large tract of land around 49 hectares in area (Barrio Guinayang, San Mateo, Rizal). Upon his death this property was inherited by his son, Jose Hilario, Jr., to whom a new certificate of title was issued. During the lifetime of plaintiff's father, the Hilario estate was bounded on the western side by the San Mateo River.3 To prevent its entry into the land, a bamboo and lumber post dike

or ditch was constructed on the northwestern side. This was further fortified by a stonewall built on the northern side. For years, these safeguards served their purpose. However, in 1937, a great and extraordinary flood occurred which inundated the entire place including the neighboring barrios and municipalities. The River destroyed the dike on the northwest, left its original bed and meandered into the Hilario estate, segregating from the rest thereof a lenticular piece of land. The disputed area is on the eastern side of this lenticular strip which now stands between the old riverbed site and the new course. In 1945, the US Army opened a sand and gravel plant within the premises, and started scraping, excavating and extracting soil, gravel and sand from the nearby areas along the River. The operations eventually extended northward into the strip of land. Consequently, a claim for damages was filed with the US War Department by Luis Hidalgo, the then administrator of Dr. Hilario's estate. The US Army paid. In 1947, the plant was turned over to herein defendants-appellants and appellee who took over its operations. On 22 October 22, 1949, plaintiff filed his complaint for injunction and damages against the defendants City Engineer of Manila, District Engineer of Rizal, the Director of Public Works, and Engr. Busuego, the Engineer-incharge of the plant. Subsequently, the Bureau of Mines and Atty. Maximo Calalang were respectively allowed to join the litigation as intervenors; as per issue of fees and penalties for materials (sand and gravel) extracted. On 14 March 1954, defendants filed a petition for injunction against plaintiff and intervenor Calalang in the same case, alleging that the latter have fenced off the disputed area in contravention of an agreement had between the latter and the Director of Public Works wherein the defendants were allowed to continue their operations but subject to the final outcome of the pending suit. On 13 May 1954, plaintiff amended his complaint and impleaded as additional defendants the City of Manila, the Provincial Treasurer of Rizal, and Engr. Eulogio Sese, the new Engineer-incharge of the plant. Plaintiff also converted his claim to one purely for damages directed against the City of Manila and the Director of Public Works, solidarily, in the amount of P1,000,000.00, as the cost of materials taken since 1949, as well as those to be extracted therefrom until defendants stop their operations.

On 21 December 1956, the lower court rendered its decision, ordering the City of Manila and Director of Public Works to pay Hilario in solidum the sum of P376,989.60 as cost of gravel and sand extracted from the plaintiff s land, plus costs; and ordering the Provincial Treasurer of Rizal to reimburse intervenor Calalang of P36.80 representing gravel fees illegally collected. None of the parties litigants seemed satisfied with this decision and they all sought a reconsideration of the same. On August 30, 1957, the lower court resolved the motions to reconsider with an order, holding that the 2/5 portion of the area in controversy to Hilario, and dismissing the case against the Bureau of Public Works insofar as money claims are concerned without prejudice to Hilario taking action against proper party in such claim. Hilario and Calalang filed a second motion for reconsideration, which the lower court denied. Hence, the appeal. The Supreme Court set aside the decision and orders appealed from, and entered another judgment to the effect that the City of Manila and the Director of Public Works, and his agent and employees, are absolved of liability from extracting materials from subject property (of public domain); and the portion within the strip of land question declared not part of public domain and confirmed as part of Hilario s private property. No Costs. 1. Old Civil Code and Law of Waters of 1866 controlling law Since the change in the course of the River took place in 1937, long before the present Civil Code took effect, the question should be determined in accordance with the provisions of the old Civil Code and those of the Law of Waters of 3 August 1866. 2. All riverbanks, as part of the riverbeds, are of public ownership Under the old Civil Law and the Law of Waters, all riverbanks are of public ownership, including those formed when a river leaves its old bed and opens a new course through a private estate. Artcile 339 of the old Civil Code is very clear. Without any qualifications, it provides that that devoted to public use, such as roads, canals, rivers, torrents, ports and bridges constructed by the State, riverbanks, shores, roadsteads, and that of a similar character are property of public ownership. Further, the riverbank is part of the riverbed. Article 73 of the Law of Waters which provides that the phrase "banks of a river" is understood those lateral strips of zones of its beds which are washed by the stream only during such high floods as

do not cause inundations. The use of the words "of its bed [de sus alveos] " clearly indicates the intent of the law to consider the banks for all legal purposes, as part of the riverbed. Thus, the banks of the River are part of its bed. Since undeniably all beds of river are of public ownership, it follows that the banks, which form part of them, are also of public ownership. 3. Natural bed or channel of a creek or river defined The natural bed or channel of a creek or river is the ground covered by its waters during the highest [ordinary] floods (Article 70 of the Law of the Waters). 4. New bed, when river changes course, is of public ownership; Means to recover Article 372 of the old Civil Code which provides that "whenever a navigable or floatable river changes its course from natural causes and opens a new bed through a private estate, the new bed shall be of public ownership, but the owner of the estate shall recover it in the event that the waters leave it dry again either naturally or as the result of any work legally authorized for this purpose." Banks are not mentioned in the provision, as the nature of banks follows that of the bed and the running water of the river. 5. A river is a compound concept consisting of running waters, bed, and banks A river is a compound concept consisting of three elements; (1) the running waters, (2) the bed and (3) the banks. All these constitute the river. American authorities are in accord with this view, as that "' River' consists of water, bed and banks"; and that "A 'river' consists of water, a bed and banks, these several parts constituting the river, the whole river. It is a compound idea; it cannot exist without all its parts. Evaporate the water, and you have a dry hollow. If you could sink the bed, instead of a river you would have a fathomless gulf. Remove the banks, and you have, a boundless flood" 6. River is of public ownership, elements follow same nature of ownership; Law explicit Since a river is but one compound concept, it should have only one nature, i.e., it should either be totally public or completely private. Since rivers are of public ownership, it is implicit that all the three component elements be of the same nature also. Still, the law expressly makes all three elements

public. Thus, riverbanks and beds are public under Artciles 339 and 407, respectively, of the Code, while the flowing waters are declared so under Articles 33, par. 2 of the Law of Waters of 1866. 7. Natural is not synonymous to original or prior condition "Natural" is not made synonymous to "original" or "prior condition". On the contrary, even if a river should leave its original bed so long as it is due to the force of nature, the new course would still fall within the scope of the definition provided by the Diccionario de La Real Academia Espaola. Hence, the law must have used the word "natural" only because it is in keeping with the ordinary nature and concept of a river always to have a bed and banks. Diccionario De La Real Academia Espaola: "NATURAL-perteneciente a la naturaleza o conforme a la calidad o propriedad de las cosas; -nativa, origivario de un pueblo o imcio'n; hecho con verdad, ni artificio, mezela ni compocision alguna, ingenuo y sin doblez en su modo de proceder; di cese tambien de las cosas que imitar a, la naturaleza con propiedad; regular y que comumnente sucede, y par eso, facilmente creible; que se produce por solas las fuerzas de la naturaleza, canio contraVuesto a sobre natural y milagroso." 8. Article 553 of the old Civil Code does not intend to authorize private acquisition of river banks but recognizes vested rights of riparian owners; History of ownership of River Banks Article 553 was never intended to authorize the private acquisition of river banks, as this would conflict with clear legislative policy enunciated in Article 339 of the Code that all riverbanks were of public ownership. The article merely recognized and preserved the vested rights of riparian owners who, because of prior law or custom, were able to acquire ownership over the banks. This was possible under the Siete Partidas which was promulgated in 1834. Under Law 6, Title 28, Partida 3, the banks of rivers belonged to the riparian owners, following the Roman Law rule. But subsequent legislation radically changed this rule. By the Law of Waters of 3 August 1866, riverbanks became of public ownership, albeit impliedly only because considered part of the bed which was public, by statutory definition. This law, while expressly repealing all prior inconsistent laws, left undisturbed

all vested rights then existing. Article 73 of the Law of Waters of 1866 is the reconciliation effected between the private ownership of the banks and the policy of the law to devote all banks to public Use. The easement would preserve the private ownership of the banks and still effectuate the policy of the law. So, the easement in Article 73 only recognized and preserved existing privately owned banks; it did not authorize future private appropriation of riverbanks. Subsequently, the Law of Waters of 13 June 1879 reenacted Article 73 of the Law of Waters of 1866 and affirmed the public ownership of rivers and their beds and the treatment of the banks as part of the bed. But nowhere in the law was there any provision authorizing the private appropriation of the banks. The public nature of riverbanks are obtained only by implication until the promulgation of the Civil Code of 1899, which was explicit in Article 339 that riverbanks were declared public property since they were destined for public use. Since the first paragraph of Article 36 of the Law of Waters if 1879 was reenacted in Article 553 of the Code, this article must also be understood not as authorizing the private acquisition of riverbanks but only as recognizing the vested titles of riparian owners who already owned the banks. In the present case, since the new banks were formed when the river changed its course in 1937, the banks cannot be subjected to the provisions of the Siete Partidas, to claim private ownership of the banks, as such was already superceded by then. 9. Legal definition applies with the legal order, distinction due to physical order cannot prevail The conclusion made by the lower court that only the northern 2/5 of the disputed area remained as plaintiff's private property is predicated from the findings that the portion where rice and corn were found in the ocular inspection of 15 June 1951, was on the northern 2/5 of the disputed area; that this cannot be a part of the bed because of the existence of vegetation which could not have grown underwater, and that this portion is manmade. This is bereft of evidence, as the unexcavated portion of the land is the southwestern . Further, American cases cannot be applied as these do not involve a similar statutory provision, unlike in the Law of Waters, which defined "beds" and "banks" and considered the latter as part of the former. That plants can and do grow on the banks which otherwise could not have grown on the bed which is constantly subjected to the flow of the

waters proves the distinction between "beds" and "banks" in the physical order. However, in dealing with the legal order, legal definitions prevail. 10. Limits of banks of rivers Article 73 of the Law of Waters which defines the limits of banks of rivers "By the phrase 'banks of a river' is understood those lateral strips or zones of its bed which are washed by the stream only during such high floods as do not cause inundations. The farthest extremity of the bank on the west side would, therefore, be that lateral line or strip which is reached by the waters during those high floods that do not cause inundations. In other words, the extent reached by the waters when the River is at high tide. 11. Banks of river different in topography There is a difference between the topography of the two sides immediately adjoining the River. The line indicated as "primary bank," which is on the east, is about 3 meters high and has a steep grade right at the edge where it drops almost vertically to the watercourse level. The opposite side, on the other hand, has no such steep acclivity. The bank near the water edge, is about 30 to 50 cms. high only, and gradually slopes up to a height of about 2 to 2-1/2 meters along the line indicated as "secondary bank", which is quite far from the waterline. Considering the peculiar characteristics of the two sides banking the river, the rise in the waterlevel would not have the same effect on the two sides. Thus, on the east, the water would rise vertically, until the top of the "primary bank" is reached, but on the west, there would be a low angled inclined rise, the water covering more ground until the "secondary bank" line is reached. In other words, while the water expansion on the east is vertical, that on the west is more or less lateral, or horizontal. 12. Ordinary and extraordinary flood There are two types of floods in the area during the rainy season. One is the so-called "ordinary" flood, when the river is swollen but the flowing water is kept within the confines of the "primary" and "secondary" banks. This occurs annually, about three to four times during the period. Then there is the "extraordinary" flood, when the waters overflow beyond the said banks, and even inundate the surrounding areas. However, this flood does not happen regularly. From 1947 to 1955, there were only three such floods.

13. Movement of the river, west bank, from 1945-1955 From 1945 to 1949, the west bank of the River extended westward up to the "secondary bank" line; from 1950 to 1952, this bank had moved, with the River, to the east, its lateral borders running along a line just 20 meters west of the camachile tree; and from 1953 to 1955, the extremities of the west bank further receded eastward beyond the camachile tree, until they lay just about 20 meters east of said tree. 14. Floodings not accidental as they are annual; Government v. Colegio de San Jose does not apply Evidence shows that the River floods with annual regularity during the rainy season. These floods can hardly be called "accidental". The Colegio de San Jose case is not exactly in point. What was mainly considered there was Article 74 of the Law of Waters relating to lakes, ponds and pools. In the present case, none of these is involved. 15. Movement of the river not due to excavation and extraction of materials The excavations and extractions of materials, even from the American period, have been made only on the strip of land west of the River. Under the "following-the nature-of-things" argument advanced by plaintiff, the River should have moved westward, where the level of the ground had been lowered. But the movement has been in the opposite direction instead. Therefore, it cannot be attributed to defendants' operations. Moreover, Hilario's own evidence indicates that the movement eastward was all due to natural causes. The movement eastward of the channel by as much as 31 meters, from 1950 to 1953, was due to two typhoons which caused the erosion of the east bank and the depositing of materials on the west side which increased its level from as much as .93 to 2 meters. 16. River of different width; claim of unnatural widening unfounded Reliance is made on the finding by the lower court that in 1943, the river was only 60 meters wide, whereas in 1950, it was already 140 meters wide. Such area sampled shows only the width of the River near the southwestern boundary of the Hilario estate. It does not indicate how wide it was in the other parts, especially up north. 17. Extraction confined on the banks of the river and not beyond limits of the west bank to invade his private estate; Hilario cannot recover damages

from defendants From 1947 to the early part of 1949, the defendants conducted their operations only in the New Accretion Area along a narrow longitudinal zone contiguous to the watercourse then. This zone, City Engineer Manila, is about 1 km. long and extends northward up to pt. 50.35. However, no extractions nor excavations were undertaken west of this zone, i.e., above the "temporary bank" line. This line is located east of the "secondary bank" line, the lateral extremity of the west bank then. In the latter part of 1949, plaintiff prohibited the defendants from extracting along the New Accretion Area and constructed a fence across the same. This forced the defendants to go southeast of the "Excavated Area". From 1954 to 1955, defendants' area of operation was still farther east of the New Accretion Area. They were. working within a confined area along the west waterline, the northern and western boundaries of which were 20 meters away east from the camachile tree. It appears sufficiently established, therefore, that defendants have not gone beyond the receding western extremities of the west riverbank. They have confined their extraction of gravel and sand only from which the banks of the River, which constitute part of the public domain wherein they had the right to operate. Plaintiff has not presented sufficient evidence that defendants have gone beyond the limits of the west bank, as previously established, and have invaded his private estate. He cannot, therefore, recover from them. 18. Plaintiff not denied of property without just compensation The Court does not declare that the entire channel, i.e., all that space between the "secondary bank" line and the "primary bank" line, has permanently become part of the riverbed. What is held is that at the time the defendants made their extractions, the excavations were within the confines of the riverbanks then. All that space to the west of said receding line" would still be part of plaintiff's property and also whatever portion adjoining the river is, at present, no longer reached by the non-inundating ordinary floods. Further, it is not correct to say that plaintiff would be deprived of his property without any compensation at all. Under Article 370 of the old Civil Code, the abandoned bed of the old river belongs to the riparian owners either fully or in part with the other riparian owners. And had the change occurred under the Civil Code of the Philippines, plaintiff would even be entitled to all of the old bed in proportion to the area he has lost.

19. Defendants did not unjustly profit at plaintiff s expense as they are not responsible for the shifting of the river Defendants cannot be accused of unjustly profiting at plaintiff's expense. They were not responsible for the shifting of the river. It was due to natural causes for which no one can be blamed. Furher, defendants were extracting from public property then, under proper authorization. The government, through the defendants, may have been enriched by chance, but not unjustly. G.R. No. 154270 March 9, 2010

TEOFISTO OO, PRECY O. NAMBATAC, VICTORIA O. MANUGAS and POLOR O. CONSOLACION, Petitioners, vs. VICENTE N. LIM, Respondent. DECISION BERSAMIN, J.: The subject of controversy is Lot No. 943 of the Balamban Cadastre in Cebu City, covered by Original Certificate of Title (OCT) No. RO-9969-(O20449), over which the contending parties in this action for quieting of title, initiated by respondent Vicente N. Lim (Lim) in the Regional Trial Court (RTC) in Cebu City, assert exclusive ownership, to the exclusion of the other. In its decision dated July 30, 1996,1 the RTC favored Lim, and ordered the cancellation of OCT No. RO-9969-(O-20449) and the issuance of a new certificate of title in the name of Luisa Narvios-Lim (Luisa), Lim s deceased mother and predecessor-in-interest. On appeal (CA-GR CV No. 57823), the Court of Appeals (CA) affirmed the RTC on January 28, 2002.2 It later denied the petitioners motion for reconsideration through the resolution dated June 17, 2002.3 Hence, this appeal via petition for review on certiorari. Antecedents

On October 23, 1992, Lim filed in the RTC in Cebu City a petition for the reconstitution of the owner s duplicate copy of OCT No. RO-9969-(O20449), alleging that said OCT had been lost during World War II by his mother, Luisa;4 that Lot No. 943 of the Balamban Cadastre in Cebu City covered by said OCT had been sold in 1937 to Luisa by Spouses Diego Oo and Estefania Apas (Spouses Oo), the lot s registered owners; and that although the deed evidencing the sale had been lost without being registered, Antonio Oo (Antonio), the only legitimate heir of Spouses Oo, had executed on April 23, 1961 in favor of Luisa a notarized document denominated as confirmation of sale,5 which was duly filed in the Provincial Assessor s Office of Cebu. Zosimo Oo and petitioner Teofisto Oo (Oos) opposed Lim s petition, contending that they had the certificate of title in their possession as the successors-in-interest of Spouses Oo. On account of the Oos opposition, and upon order of the RTC, Lim converted the petition for reconstitution into a complaint for quieting of title,6 averring additionally that he and his predecessor-in-interest had been in actual possession of the property since 1937, cultivating and developing it, enjoying its fruits, and paying the taxes corresponding to it. He prayed, inter alia, that the Oos be ordered to surrender the reconstituted owner s duplicate copy of OCT No. RO-9969-(O-20449), and that said OCT be cancelled and a new certificate of title be issued in the name of Luisa in lieu of said OCT. In their answer,7 the Oos claimed that their predecessors-in-interest, Spouses Oo, never sold Lot No. 943 to Luisa; and that the confirmation of sale purportedly executed by Antonio was fabricated, his signature thereon not being authentic. RTC Ruling On July 30, 1996, after trial, the RTC rendered its decision,8 viz: WHEREFORE, premises considered, judgment is hereby rendered quieting plaintiff's title to Lot No. 943 of the Balamban (Cebu) Cadastre, and directing the Register of Deeds of Cebu

(1) To register the aforestated April 23, 1961 Confirmation of Sale of Lot No. 943 of the Balamban, Cebu Cadastre by Antonio Oo in favor of Luisa Narvios-Lim; (2) To cancel the original certificate of title covering the said Lot No. 943 of the Balamban, Cebu Cadastre; and, (3) To issue in the name of Luisa Narvios-Lim, a new duplicate certificate of title No. RO-9969 (O-20449) of the Register of Deeds of Cebu, which shall contain a memorandum of the fact that it is issued in place of the lost duplicate certificate of title, and shall in all respects be entitled to like faith and credit as the original certificate, and shall be regarded as such for all purposes of this decree, pursuant to the last paragraph of Section 109, Presidential Decree No. 1529. Without special pronouncement as to costs. SO ORDERED.9 The RTC found that the Lims had been in peaceful possession of the land since 1937; that their possession had never been disturbed by the Oos, except on two occasions in 1993 when the Oos seized the harvested copra from the Lims caretaker; that the Lims had since declared the lot in their name for taxation purposes, and had paid the taxes corresponding to the lot; that the signature of Antonio on the confirmation of sale was genuine, thereby giving more weight to the testimony of the notary public who had notarized the document and affirmatively testified that Antonio and Luisa had both appeared before him to acknowledge the instrument as true than to the testimony of the expert witness who attested that Antonio s signature was a forgery. CA Ruling On appeal, the Oos maintained that the confirmation of sale was spurious; that the property, being a titled one, could not be acquired by the Lims through prescription; that their (the Oos) action to claim the property could not be barred by laches; and that the action instituted by

the Lims constituted a collateral attack against their registered title.1avvphi1 The CA affirmed the RTC, however, and found that Spouses Oo had sold Lot No. 943 to Luisa; and that such sale had been confirmed by their son Antonio. The CA ruled that the action for quieting of title was not a collateral, but a direct attack on the title; and that the Lims undisturbed possession had given them a continuing right to seek the aid of the courts to determine the nature of the adverse claim of a third party and its effect on their own title. Nonetheless, the CA corrected the RTC, by ordering that the Office of the Register of Deeds of Cebu City issue a new duplicate certificate of title in the name of Luisa, considering that the owner s duplicate was still intact in the possession of the Oos. The decree of the CA decision was as follows: WHEREFORE, the appeal is DISMISSED for lack of merit. However, the dispositive portion of the decision appealed from is CORRECTED as follows: (1) Within five (5) days from finality of the decision, defendantsappellants are directed to present the owner's duplicate copy of OCT No. RO-9969 (O-20449) to the Register of Deeds who shall thereupon register the "Confirmation of Sale" of Lot No. 943, Balamban Cadastre, Cebu, executed on April 23, 1961 by Antonio Oo in favor of Luisa Narvios-Lim, and issue a new transfer certificate of title to and in the name of the latter upon cancellation of the outstanding original and owner's duplicate certificate of title. (2) In the event defendants-appellants neglect or refuse to present the owner's copy of the title to the Register of Deeds as herein directed, the said title, by force of this decision, shall be deemed annulled, and the Register of Deeds shall make a memorandum of such fact in the record and in the new transfer certificate of title to be issued to Luisa Narvios-Lim. (3) Defendants-appellants shall pay the costs.

SO ORDERED.10 The CA denied the Oos motion for reconsideration11 on June 17, 2002.12 Hence, this appeal. Issues The petitioners raise the following issues: 1. Whether or not the validity of the OCT could be collaterally attacked through an ordinary civil action to quiet title; 2. Whether or not the ownership over registered land could be lost by prescription, laches, or adverse possession; 3. Whether or not there was a deed of sale executed by Spouses Oo in favor of Luisa and whether or not said deed was lost during World War II; 4. Whether or not the confirmation of sale executed by Antonio in favor of Luisa existed; and 5. Whether or not the signature purportedly of Antonio in that confirmation of sale was genuine. Ruling of the Court The petition has no merit. A. Action for cancellation of title is not an attack on the title The petitioners contend that this action for quieting of title should be disallowed because it constituted a collateral attack on OCT No. RO-9969(O-20449), citing Section 48 of Presidential Decree No. 1529, viz:

Section 48. Certificate not subject to collateral attack. A certificate of title shall not be subject to collateral attack. It cannot be altered, modified, or cancelled except in a direct proceeding in accordance with law. The petitioners contention is not well taken. An action or proceeding is deemed an attack on a title when its objective is to nullify the title, thereby challenging the judgment pursuant to which the title was decreed.13 The attack is direct when the objective is to annul or set aside such judgment, or enjoin its enforcement. On the other hand, the attack is indirect or collateral when, in an action to obtain a different relief, an attack on the judgment is nevertheless made as an incident thereof.14 Quieting of title is a common law remedy for the removal of any cloud, doubt, or uncertainty affecting title to real property.15 Whenever there is a cloud on title to real property or any interest in real property by reason of any instrument, record, claim, encumbrance, or proceeding that is apparently valid or effective, but is, in truth and in fact, invalid, ineffective, voidable, or unenforceable, and may be prejudicial to said title, an action may be brought to remove such cloud or to quiet the title.16 In such action, the competent court is tasked to determine the respective rights of the complainant and the other claimants, not only to place things in their proper places, and to make the claimant, who has no rights to said immovable, respect and not disturb the one so entitled, but also for the benefit of both, so that whoever has the right will see every cloud of doubt over the property dissipated, and he can thereafter fearlessly introduce the improvements he may desire, as well as use, and even abuse the property as he deems fit.17 Lim s complaint pertinently alleged: 18. If indeed, the genuine original of the Owner's Duplicate of the Reconstituted Original Certificate of Title No. RO-9699 (O-20449) for Lot 943, Balamban Cadastre xxx is in Defendant's (Oo s) possession, then VNL submits the following PROPOSITIONS: xxx

18.2. Therefore, the Original of Owner s Duplicate Certificate (which Respondents [Defendants Oos] claim in their Opposition is in their possession) must be surrendered to VNL upon order of this Court, after the Court shall have determined VNL's mother's acquisition of the attributes of ownership over said Lot 943, in this action, in accordance with Section 107, P.D. 1529, Property Registration Decree xxx xxx [t]hat OCT 20449 be cancelled and new title for Lot 943 be issued directly in favor of LUISA NARVIOS, to complete her title to said Lot;18 The averments readily show that the action was neither a direct nor a collateral attack on OCT No. RO-9969-(O-20449), for Lim was asserting only that the existing title registered in the name of the petitioners predecessors had become inoperative due to the conveyance in favor of Lim s mother, and resultantly should be cancelled. Lim did not thereby assail the validity of OCT No. RO-9969-(O-20449), or challenge the judgment by which the title of the lot involved had been decreed. In other words, the action sought the removal of a cloud from Lim s title, and the confirmation of Lim s ownership over the disputed property as the successor-in-interest of Luisa. B. Prescription was not relevant The petitioners assert that the lot, being titled in the name of their predecessors-in-interest, could not be acquired by prescription or adverse possession. The assertion is unwarranted. Prescription, in general, is a mode of acquiring or losing ownership and other real rights through the lapse of time in the manner and under the conditions laid down by law.19 However, prescription was not relevant to the determination of the dispute herein, considering that Lim did not base his right of ownership on an adverse possession over a certain period. He insisted herein, instead, that title to the land had been voluntarily

transferred by the registered owners themselves to Luisa, his predecessorin-interest. Lim showed that his mother had derived a just title to the property by virtue of sale; that from the time Luisa had acquired the property in 1937, she had taken over its possession in the concept of an owner, and had performed her obligation by paying real property taxes on the property, as evidenced by tax declarations issued in her name;20 and that in view of the delivery of the property, coupled with Luisa s actual occupation of it, all that remained to be done was the issuance of a new transfer certificate of title in her name. C. Forgery, being a question of fact, could not be dealt with now The petitioners submit that Lim s evidence did not preponderantly show that the ownership of the lot had been transferred to Luisa; and that both the trial and the appellate courts disregarded their showing that Antonio s signature on the confirmation of sale was a forgery. Clearly, the petitioners hereby seek a review of the evaluation and appreciation of the evidence presented by the parties. The Court cannot anymore review the evaluation and appreciation of the evidence, because the Court is not a trier of facts.21 Although this rule admits of certain exceptions, viz: (1) when the conclusion is a finding grounded entirely on speculation, surmises, or conjecture; (2) when the inference made is manifestly mistaken; (3) where there is a grave abuse of discretion; (4) when the judgment is based on a misapprehension of facts; (5) when the findings of fact are conflicting; (6) when the Court of Appeals, in making its findings, went beyond the issues of the case, and the findings are contrary to the admissions of both appellant and appellee; (7) when the findings of the Court of Appeals are contrary to those of the trial court; (8) when the findings of fact are conclusions without specific evidence on which they are based; (9) when the facts set forth in the petition as well in the petitioners main and reply briefs are not disputed by the respondents; and, (10) when the findings of fact of the Court of Appeals are premised on the supposed absence of evidence and are

contradicted by the evidence on record,22 it does not appear now that any of the exceptions is present herein. We thus apply the rule without hesitation, and reject the appeal for that reason. It is emphasized, too, that the CA upheld the conclusion arrived at by the RTC that the signature of Antonio had not been simulated or forged. The CA ruled that the testimony of the notary public who had notarized the confirmation of sale to the effect that Antonio and Luisa had appeared before him prevailed over that of the petitioners expert witness. The concurrence of their conclusion on the genuineness of Antonio s signature now binds the Court.23 In civil cases, the party having the burden of proof must establish his case by a preponderance of evidence. Preponderance of evidence is the weight, credit, and value of the aggregate evidence on either side, and is usually considered to be synonymous with the term greater weight of the evidence or greater weight of the credible evidence. Preponderance of evidence is a phrase that means, in the last analysis, probability of the truth.24 It is evidence that is more convincing to the court as worthy of belief than that which is offered in opposition thereto. Lim successfully discharged his burden of proof as the plaintiff. He established by preponderant evidence that he had a superior right and title to the property. In contrast, the petitioners did not present any proof of their better title other than their copy of the reconstituted certificate of title. Such proof was not enough, because the registration of a piece of land under the Torrens system did not create or vest title, such registration not being a mode of acquiring ownership. The petitioners need to be reminded that a certificate of title is merely an evidence of ownership or title over the particular property described therein. Its issuance in favor of a particular person does not foreclose the possibility that the real property may be co-owned with persons not named in the certificate, or that it may be held in trust for another person by the registered owner.25 WHEREFORE, the petition for review on certiorari is denied, and the decision dated January 28, 2002 is affirmed. The petitioners are ordered to pay the costs of suit.

SO ORDERED. [G.R. No. 176929, July 04, 2008] INOCENCIO Y. LUCASAN FOR HIMSELF AND AS THE JUDICIAL ADMINISTRATOR OF THE INTESTATE ESTATE OF THE LATE JULIANITA SORBITO LUCASAN, PETITIONER, VS. PHILIPPINE DEPOSIT INSURANCE CORPORATION (PDIC) AS RECEIVER AND LIQUIDATOR OF THE DEFUNCT PACIFIC BANKING CORPORATION, RESPONDENT. DECISION NACHURA, J.: On appeal is the March 23, 2006 Decision[1] of the Court of Appeals (CA) in CA-G.R. CV No. 81518, affirming the July 24, 2003 Order[2] of the Regional Trial Court (RTC) of Bacolod City, Branch 43, granting respondent's motion to dismiss, as well as its subsequent Resolution[3] denying petitioner's motion for reconsideration. The factual antecedents are as follows. Petitioner Inocencio Y. Lucasan (Lucasan) and his wife Julianita Sorbito (now deceased) were the owners of Lot Nos. 1500-A and 229-E situated in Bacolod City, respectively covered by TCT Nos. T-68115 and T-13816. On August 3, 1972, Pacific Banking Corporation (PBC) extended a P5,000.00 loan to Lucasan, with Carlos Benares as his co-maker. Lucasan and Benares failed to pay the loan when it became due and demandable. Consequently, PBC filed a collection case with the RTC of Bacolod City, docketed as Civil Case No. 12188. On April 30, 1979, the RTC rendered a decision ordering Lucasan and Benares to jointly and severally pay PBC P7,199.99 with interest at 14% per annum computed from February 7, 1979, until the full payment of the obligation. Lucasan failed to pay the monetary award; thus, to satisfy the judgment, the RTC issued a writ of execution directing the sheriff to effect a levy on the properties owned by Lucasan and sell the same at public

auction. In compliance with the writ, the City Sheriff of Bacolod issued a Notice of Embargo on January 8, 1981, which was annotated on Lucasan's TCT Nos. T-68115 and T-13816 as Entry No. 110107. Annotated as prior encumbrances on the same titles were the mortgages in favor of Philippine National Bank (PNB) and Republic Planter's Bank (RPB) executed to secure Lucasan's loans with the banks. On May 13, 1981, the lots were sold at public auction and were awarded to PBC as the highest bidder. A certificate of sale was executed in its favor and was registered and annotated on TCT Nos. T- 68115 and T-13816 as Entry No. 112552 on June 5, 1981. Neither PNB nor RPB, the mortgagees, assailed the auction sale. Lucasan, as well as the mortgagee banks, PNB and RPB, did not redeem the properties within the redemption period. Nevertheless, PBC did not file a petition for consolidation of ownership. In January 1997, Lucasan, through counsel, wrote a letter to the Philippine Deposit Insurance Corporation (PDIC), PBC's receiver and liquidator seeking the cancellation of the certificate of sale and offering to pay PBC's claim against Lucasan.[4] Not long thereafter, Lucasan paid his loans with the PNB and RPB. Consequently, the mortgagee banks executed their respective releases of mortgage, resulting in the cancellation of the prior encumbrances in favor of PNB and RPB. On August 13, 2001, PDIC denied Lucasan's request for the cancellation of the certificate of sale stating: Please be informed that based on our records, TCT Nos. T-68115 and T13816 have already become part of the acquired assets of Pacific Banking Corporation by virtue of a Certificate of Sale dated May 13, 1981 executed by the City Sheriff of Bacolod. Subsequently, this document was registered on the titles on June 5, 1981 so that the last day of the redemption period was June 5, 1982.

With regard to your request, we regret to inform you that reacquisition of the subject properties have to be through sale following PDIC's policy on disposal. Accordingly, these properties can be disposed through public bidding using the latest appraised value in the total amount of P2,900,300.00 as of March 29, 2000 as a minimum bid. If you are still interested to acquire the properties, please get in touch with our Asset Management Group x x x.[5] Lucasan then filed a petition denominated as declaratory relief with the RTC of Bacolod City docketed as Civil Case No. 02-11874.[6] He sought confirmation of his rights provided in the second paragraph of Section 1, Rule 63 of the Rules of Court in relation to Section 75 of Presidential Decree (P.D.) No. 1529. Lucasan also pleaded for the lifting and/or cancellation of the notice of embargo and the certificate of sale annotated on TCT Nos. T-68115 and T-13816, and offered to pay P100,000.00 or such amount as may be determined by the RTC, as consideration for the cancellation. PDIC moved to dismiss the complaint for lack of cause of action. It averred that an action to quiet title under Section 1 of Rule 63 may only be brought when there is a cloud on, or to prevent a cloud from being cast upon, the title to real property. It asseverated that a cloud on the title is an outstanding instrument record, claim, encumbrance or proceeding which is actually invalid or inoperative, but which may nevertheless impair or affect injuriously the title to property. PDIC claimed that the notice of embargo was issued pursuant to a writ of execution in Civil Case No. 12188, while the certificate of sale was executed as a result of a public bidding. Thus, their annotations on the titles were valid, operative or effective. PDIC asserted that Lucasan's petition is nothing but a disguised attempt to compel PDIC to resell the properties at a reduced price of P100,000.00. Accordingly, it prayed for the dismissal of the petition.[7] Lucasan opposed the motion.[8] He countered that the subject properties were still in his possession, and neither PBC nor PDIC instituted an action for consolidation of ownership. Since the certificate of title was still in his name, he contended that he could pursue all legal and equitable remedies, including those provided for in Section 1, Rule 63 of the Rules of Court to reacquire the properties. He also claimed that PDIC's policy of disposing the subject properties through public bidding at the appraised value of

P2,900,300.00 was unjust, capricious and arbitrary, considering that the judgment debt amounted only to P7,199.99 with interest at 14% per annum. Lucasan urged the RTC to apply the liberal construction of the redemption laws stressed in Cometa v. Court of Appeals.[9] In its Order[10] dated July 24, 2003, the RTC granted PDIC's motion to dismiss, thus: The clouds contemplated by the provision of law under Article 476 of the Civil Code is one where the instrument, record, claim, encumbrance or proceeding is apparently valid or effective on its face that nothing appears to be wrong, but in reality, is null and void. Hence, the petition filed by [Lucasan] pursuant to the said article is equivalent to questioning the validity of the subsequent annotation of Entry No. 110107 and Entry No. 112522 in TCT Nos. T-13816 and T-68115. Records disclose that Entry No. 110107 which is a Notice of Embargo was issued by virtue of a valid judgment rendered in Civil Case No. 12188 entitled "Pacific Banking Corporation vs. [Inocencio] Lucasan, et al.," whereby the Court found [Lucasan] liable in favor of [PBC] the sum of P7,199.99 with 14% interest per annum to be computed from February 7, 1979 until fully paid. As mandated in Sec. 12, Rule 39 of the Revised Rules of Court, such levy on execution create a lien in favor of [PBC] over the right, title and interest of [Lucasan] over the two (2) subject parcels of land covered by TCT Nos. T-13816 and T-68115, subject to liens and encumbrances then existing. The fact that [Lucasan] has redeemed the mortgage properties from the first mortgages (sic), PNB and PNB (sic) Republic Bank, does not vest him any title free from the lien of [PBC]. While the law requires that the judgment debtor, [Lucasan] must be served with a notice of levy and even if not served therewith, the defect is cured by service on him of the notice of sale prior to the sale, nowhere in the petition which alleges that [Lusasan] refutes the validity of the execution sale. Thus, he is deemed to have received and recognized the same. As support for his thesis, [Lucasan] cites the case of Balanga vs. Ca., et al. (supra). However this Court is unable to agree that it is applicable to the

present case. As correctly argued by [PDIC], in that case the proceedings under execution suffered infirmity from the very start as the levy and sale made by the sheriff of the land of petitioner Balanga included the house erected on the land [and] constituted as a family home which, under the law, exempt from execution. In the case at bar, no objection was interposed by [Lucasan] as a valid levy has been made pursuant to Sec. 7, Rule 57 of the Revised Rules of Court, as a consequence of which, the sale made pursuant to Sec. 11 of the same rule is also valid and effective.[11] The dispositive portion of the RTC Order reads: WHEREFORE, finding the claim of any cloud over the titles of [Lucasan] to be bereft of basis in fact and in law, the Motion to Dismiss filed by [PDIC] is granted. Accordingly, this is hereby ordered DISMISSED SO ORDERED.[12] Lucasan filed a motion for reconsideration, but the RTC denied it on October 20, 2003.[13] On appeal, the CA affirmed in toto the RTC ruling. It declared that Lucasan already lost his right to redeem the properties when he failed to exercise it within the prescribed period. The effect of such failure was to vest in PBC absolute ownership over the subject properties.[14] The CA disposed, thus: WHEREFORE, in view of all the foregoing premises, the appeal is hereby DENIED. Accordingly, the assailed Order of the Regional Trial Court of Bacolod City, Branch 43 dated 24 July 2003 dismissing [Lucasan's] Petition for Declaratory Relief and the subsequent Order of the same Court dated 20 October 2003 denying [Lucasan's] motion for reconsideration from the Order of Denial (sic) are hereby affirmed in toto. No costs. SO ORDERED.[15] Lucasan sought a reconsideration of the CA Decision, but the same was denied on February 7, 2007.[16] Before us, Lucasan impugns the CA Decision on the following grounds: 1- THE COURT OF APPEALS ERRED AND GRAVELY ABUSED ITS DISCRETION IN AFFIRMING THE ORDER OF DISMISSAL OF THE PETITIONER'S PETITION IN THE REGIONAL TRIAL COURT WHEN IT

DISREGARDED THE CLEAR PROVISION OF SECTION 75 OF PRESIDENTIAL DECREE NO. 1529 AND PUT TO NAUGHT THE APPLICABLE JURISPRUDENCE IN ZACARIAS COMETA x x x AND THE CASES CITED THEREIN, INSPITE (sic) OF THE CLEAR AND OUTSTANDING SIMILARITY OF FACTS WITH THE CASE UNDER CONSIDERATION. 2- THE COURT OF APPEALS ALSO ERRED AND GRAVELY ABUSED ITS DISCRETION WHEN IT FAILED TO CONSIDER THAT THE NOTICE OF EMBARGO AND CERTIFICATE OF SALE ISSUED BY THE CITY SHERIFF WERE ONLY LEVY ON THE INTEREST OF THE PETITIONER ON THE TWO (2) SUBJECT LOTS, AS DECREED IN QUEZON BEARING & PARTS CORPORATION, x x x, WHICH IS LIKEWISE APPLICABLE TO THE CASE AT BAR.[17] Lucasan posits that he has sufficient cause of action against PDIC; thus, he chides the RTC for dismissing his complaint, and the CA for affirming the dismissal. In support of his thesis, he cites Section 75 of Presidential Decree (PD) No. 1529, or the Property Registration Decree[18] and Cometa v. Court of Appeals.[19] As gleaned from the averments of the complaint, Lucasan's action was one for quieting of title under Rule 63 of the Rules of Court. Essentially, he sought the cancellation of the notice of embargo and the certificate of sale annotated on TCT Nos. T-68115 and T-13816 claiming that the said annotations beclouded the validity and efficacy of his title. The RTC, however, dismissed his complaint for lack of cause of action which was affirmed by the CA in its assailed Decision. Thus, the key issue for our consideration is whether the dismissal of Lucasan's complaint was proper. Quieting of title is a common law remedy for the removal of any cloud of doubt or uncertainty with respect to real property. The Civil Code authorizes the said remedy in the following language: ART. 476. Whenever there is a cloud on title to real property or any interest therein, by reason of any instrument, record, claim, encumbrance or proceeding which is apparently valid or effective but is in truth and in fact invalid, ineffective, voidable, or unenforceable, and may be prejudicial to said title, an action may be brought to remove such cloud or to quiet the title.

An action may also be brought to prevent a cloud from being cast upon title to real property or any interest therein. ART. 477. The plaintiff must have legal or equitable title to, or interest in the real property which is the subject-matter of the action. He need not be in possession of said property. To avail of the remedy of quieting of title, two (2) indispensable requisites must concur, namely: (1) the plaintiff or complainant has a legal or an equitable title to or interest in the real property subject of the action; and (2) the deed, claim, encumbrance or proceeding claimed to be casting a cloud on his title must be shown to be in fact invalid or inoperative despite its prima facie appearance of validity or legal efficacy.[20] Stated differently, the plaintiff must show that he has a legal or at least an equitable title over the real property in dispute, and that some deed or proceeding beclouds its validity or efficacy. Unfortunately, the foregoing requisites are wanting in this case. Admittedly, the subject parcels of land were levied upon by virtue of a writ of execution issued in Civil Case No. 12188. On May 13, 1981, a public auction of the subject parcels of land was held and the lots were awarded to PBC as the highest bidder. A certificate of sale in favor of PBC was issued on the same day, and was registered and annotated on TCT Nos. T68115 and T-13816 as Entry No. 112552 on June 5, 1981. Under the 1964 Rules of Court, which were in effect at that time, the judgment debtor or redemptioner had the right to redeem the property from PBC within twelve (12) months from the registration of the certificate of sale.[21] With the expiration of the twelve-month period of redemption and no redemption having been made, as in this case, the judgment debtor or the redemptioner lost whatever right he had over the land in question.[22] Lucasan admitted that he failed to redeem the properties within the redemption period, on account of his then limited financial situation. [23] It was only in January 1997 or fifteen (15) years later that he manifested his desire to reacquire the properties. Clearly thus, he had lost whatever right he had over Lot Nos. 1500-A and 229-E.

The payment of loans made by Lucasan to PNB and RPB in 1997 cannot, in any way, operate to restore whatever rights he had over the subject properties. Such payment only extinguished his loan obligations to the mortgagee banks and the liens which Lucasan claimed were subsisting at the time of the registration of the notice of embargo and certificate of sale. Neither can Lucasan capitalize on PBC's failure to file a petition for consolidation of ownership after the expiration of the redemption period. As we explained in Calacala v. Republic:[24] [P]etitioners' predecessors-in-interest lost whatever right they had over [the] land in question from the very moment they failed to redeem it during the 1-year period of redemption. Certainly, the Republic's failure to execute the acts referred to by the petitioners within ten (10) years from the registration of the Certificate of Sale cannot, in any way, operate to restore whatever rights petitioners' predecessors-in-interest had over the same. For sure, petitioners have yet to cite any provision of law or rule of jurisprudence, and we are not aware of any, to the effect that the failure of a buyer in a foreclosure sale to secure a Certificate of Final Sale, execute an Affidavit of Consolidation of Ownership and obtain a writ of possession over the property thus acquired, within ten (10) years from the registration of the Certificate of Sale will operate to bring ownership back to him whose property has been previously foreclosed and sold. xxxx Moreover, with the rule that the expiration of the 1-year redemption period forecloses the obligor's right to redeem and that the sale thereby becomes absolute, the issuance thereafter of a final deed of sale is at best a mere formality and mere confirmation of the title that is already vested in the purchaser. As this Court has said in Manuel vs. Philippine National Bank, et al.: Note must be taken of the fact that under the Rules of Court the expiration of that one-year period forecloses the owner's right to redeem, thus making the sheriff's sale absolute. The issuance thereafter of a final deed of sale becomes a mere formality, an act merely confirmatory of the title that is already in the purchaser and constituting official evidence of that fact.(Emphasis supplied.)

Certainly, Lucasan no longer possess any legal or equitable title to or interest over the subject parcels of land; hence, he cannot validly maintain an action for quieting of title. Furthermore, Lucasan failed to demonstrate that the notice of embargo and the certificate of sale are invalid or inoperative. In fact, he never put in issue the validity of the levy on execution and of the certificate of sale duly registered on June 5, 1981. It is clear, therefore, that the second requisite for an action to quiet title is, likewise, absent. Concededly, Lucasan can pursue all the legal and equitable remedies to impeach or annul the execution sale prior to the issuance of a new certificate of title in favor of PBC. Unfortunately, the remedy he had chosen cannot prosper because he failed to satisfy the requisites provided for by law for an action to quiet title. Hence, the RTC rightfully dismissed Lucasan's complaint. Lucasan tries to find solace in our ruling in Cometa v. Court of Appeals. Sadly for him, that case is not on all fours with his case, for it was not for quieting of title but a petition for issuance of a writ of possession and cancellation of lis pendens. Likewise, in Cometa the registered owner assailed the validity of the levy and sale, which Lucasan failed to do. Undoubtedly, Lucasan's right to redeem the subject properties had elapsed on June 5, 1982. His offer to redeem the same in 1997 or long after the expiration of the redemption period is not really one for redemption but for repurchase. Thus, PBC and PDIC, its receiver and liquidator, are no longer bound by the bid price. It is entirely within their discretion to set a higher price. As we explained in De Robles v. Court of Appeals:[25] The right to redeem becomes functus officio on the date of its expiry, and its exercise after the period is not really one of redemption but a repurchase. Distinction must be made because redemption is by force of law; the purchaser at public auction is bound to accept redemption. Repurchase however of foreclosed property, after redemption period, imposes no such obligation. After expiry, the purchaser may or may not resell the property but no law will compel him to do so. And, he is not bound by the bid price; it is entirely within his discretion to set a higher price, for after all, the property already belongs to him as owner.

Accordingly, the condition imposed by the PDIC for the re-acquisition of the property cannot be considered unjust or unreasonable. Verily, in several cases,[26] this Court allowed redemption even after the lapse of the redemption period. But in those cases a valid tender was made by the original owners within the redemption period. Even in Cometa, the redemption was allowed beyond the redemption period because a valid tender of payment was made within the redemption period. The same is not true in the case before us. In fine, we find that the RTC correctly dismissed Lucasan's complaint for quieting of title. Thus, the CA committed no reversible error in sustaining the RTC. WHEREFORE, the petition is DENIED. The Decision and Resolution of the Court of Appeals in CA-G.R. CV No. 81518, are AFFIRMED. Costs against the petitioner. SO ORDERED.

You might also like